You are on page 1of 163

http://mathkas.ici.ma http://mathkas.9e.

cc
http://mathkas.ici.ma
éJ“ AK P Ð ñÊ« ¼AK.2 2009 ñJKñK èPð X HA
 J“AK QË@ ú¯ úæ£ñË@ àAjJÓB@ iJj’


éK 
Q.m.Ì '@ HAJ J.Ë@ : ÈðB@ áK QÒJË@
( Ã ! )
x y  ÒjÖÏ @
F = M (x, y) =
0
1 ∈ M2 (R)/(x, y) ∈ R∗ ×R : é«ñ . F áºJË
x
(M2 (R), ×) áÓ Q®. J‚Ó ZQk. F à@ áJ.JË ( @ - 1
AJK YË F áÓ M (a, b) ð M (x, y) áºJË
à ! à !  y 
x y a b xa xb +
M (x, y) × M (a, b) = 1 × 1 = a 
0 0 1
x a 0
xa
y
= M (xa, xb + )
a
áÓ Q®. J‚Ó ZQk. F éJÓ ð
(M2 (R), ×)

éJ ËXAJ.K Q« èQÓP (F, ×) à @ áJ.JË (H.


  
F ú¯ ùªJÒm.' × àA¯ M2 (R) ú¯ ùªJÒm.' × ð (M2 (R), ×) áÓ Q®J‚Ó ZQk. F à @ AÜß. (1)

F ú¯ × È éJ.‚ËAK. F ú¯ YK AjÖÏ @ Qå”JªË@ I = M (1, 0) ∈ F AJK YË (2)
 
F ú¯ × È éJ.‚ËAK. éË ÉKAÜØ M ( , −y) ÉJ.®K F áÓ M (x, y) Qå”J« É¿ (3)
1
x
: XA’Ó ÈAJÓ AJK YË , F ú¯ úÍXAJ.K Q « × (4)
 
µ ¶ Ã ! 7
1 1 3  2 2
M (1, 1) × M (2, 3) = 1 = × 2 
0 1
0 1 
2 0
à ! µ à 2!
2 3 ¶ 2 5
1 1
M (2, 3) × M (1, 1) =
0
1 ×
0 1
=
0
1 ð
2 2
M (1, 1) × M (2, 3) 6= M (2, 3) × M (1, 1) éJÓ ð

(F, ×) áÓ éJ K Qk. èQÓP G à @ áJ.JË G = {M (x, 0) ∈ F/x ∈ R∗ } áºJË - 2

I∈G à B G 6= ∅ AJK YË (1)


1 x
M (x, 0) × M (a, 0)−1 = M (x, 0) × M ( , 0) = M ( , 0) :
a a
AJK YË R∗ áÓ a ð x áºJË (2)

áºJË - 3 E = R∗ × R.
y  
∀(x, y) ∈ E, ∀(a, b) ∈ E : (x, y)⊥(a, b) = (xa, xb + ) ⊥ úÎg@YË@ I
a . J»QË@ àñKA®K. E Xð QK
φ : (F, ×) 7→ (E, ⊥) 
: ‡JJ.¢JË@ Q.JªK
M (x, y) 7→ (x, y)
7
(1, 1)⊥(2, 3) = (2, ) ð (2, 3)⊥(1, 1) = (2, 5) ( @
2 
AJK YË ; F áÓ áK Qå”J« M (a, b) ð M (x, y) áºJË : É¿A‚  φ à@ áJ.JË (1) (H.
( @ (1 H. @ñm.Ì '@ I.‚k : φ(M (x, y) × M (a, b)) = φ(M (xa, xb + ay ))

éJKAK  éêk. áÓ ½Ë Y» AJK YËð
y y
éJÓð : φ(M (xa, xb + )) = (xa, xb + ) = (x, y)⊥(a, b) = φ(M (x, y)⊥φ(M (a, b))
a a
φ(M (x, y) × M (a, b)) = φ(M (x, y)⊥φ(M (a, b))
éJ“ AK P Ð ñÊ« ¼AK.2 2009 ñJKñK èPð X HA
 J“AK QË@ ú¯ úæ£ñË@ àAjJÓB@ iJj’

ÉK. A® K φ à@ áJ.JË (2)


:
φ(M (x, y)) = (x, y) IJ  m'. F áÓ M (x, y) èYJkð é¯ñ®’Ó XYm' E áÓ (x, y) h. ð P É¿
èQÓP (E, ⊥) à X@ , éK QmÌ '@ éJ JË@ ®K AÒêË (F, ×) ð (E, ⊥) à@ iJJ‚  
.. . .  ‡K. A‚Ë@ È@ñ‚Ë@ áÓ (h.
éJ ËXAJ.K Q«

.
 Y®ªË@
éK  X@Y« B@ : úGAJË@ áK QÒJË@

m 6= 1 IJ  X Y« m
 m'. ø Y®«
 Ï@ 
: (E) : z 2 − (1 − i)(1 + m)z − i(m2 + 1) = 0 éËXAªÖ C é«ñÒj. ÖÏ @ ú¯ Q.JªK −I
∆ = [(1 + i)(m − 1)]2
 JJË( @
à @ áÓ ‡ ®j −1
∆ = [(1 − i)(1 + m)]2 + 4i(m2 + 1) AJK YË
àA¯ (1 + i)2 = 2i ð (1 − i)2 = −2i à @AÜß.
∆ = 2i(m2 − 2m + 1) = (1 + i)2 (m − 1)2 = [(1 + i)(m − 1)]2
 Ï @ C ú¯ ÉjJË (H
éËXAªÖ
(E) .
ð z1 =
(1 − i)(1 + m) − (1 + i)(m − 1)
2
= 1 − im : AÒë áÊg ÉJ.® K E
(1 − i)(1 + m) + (1 + i)(m − 1)
z2 = =m−i
2
z1 z2 = 1 ⇔ (1 − im)(m − i) = 1 ⇔ −i(m2 + 1) = 1 (h.
⇔ m2 + 1 = i ⇔ m2 = −1 + i
: éJ ËAJË@ (S)
 ¢JË@
éÒ Zú¯A¾K iJ.’ m2 = −1 + i éK ðA‚Ö Ï @
m = x + iy ©’
 √
 2 
 2−1
 x − y2 = √ −1  x2 =

àA¯ èPAƒB
 @ ®K AÒêË y ð x à @AÜß. (S) ⇔ x2 + y2 = 2 ⇔ √ 2
2+1
  2
y =
2xy = 1 
 2

2xy = 1
r√ r√ r√ r√
2−1 2+1 2−1 2+1
m = −[ +i ] ð@ m = +i
2 2 2 2

: AJK YË ,
π 
< θ < π ð m = eiθ éËAg ú¯ −2
2
π θ π θ π θ π θ π
i(θ− ) i( − ) −i( − ) i( − ) θ π i( − )
ð z1 = 1 − im = 1 + e 2 = e 2 4 (e 2 4 + e 2 4 ) = 2 cos( − )e 2 4
2 4
θ π π π
éJÓð 0< − <
2 4 2
àA¯ <θ<π à @AÜß.
µ2 ¶
θ π θ π θ π
z1 = 2 cos( − ) cos( − ) + i sin( − )
2 4 2 4 2 4
 
π θ π θ π θ π
−i i( − ) i( + ) −i( + )
z2 = m − i = eiθ + e 2 = e 2 4 e 2 4 + e 2 4 
éJ“ AK P Ð ñÊ« ¼AK.2 2009 ñJKñK èPð X HA
 J“AK QË@ ú¯ úæ£ñË@ àAjJÓB@ iJj’

π θ π π
éJÓð 2
< + <π
2 4
àA¯ 2
<θ<π à @AÜß. ð
θ 3π µ ¶
i( + ) 3π θ 3π θ θ 3π θ 3π
z2 = 2e 2 4 cos( − ) = 2 cos( − ) cos( + ) + i sin( + )
4 2 4 2 2 4 2 4

M2 (z2 ) ð M1 (z1 ) , M (m) ¡® JË@ Q.JªK II

ø@ 1 − im − m
−i
∈R ø@ z1 − m
z2 − m
∈R àA¿ @ X@ ¡® ¯ð @ X@ éJ Ò® J‚Ó M2 , M1 , M ¡® JË@ àñºK 1
i + m − im ∈ R
M2 , M1 , M IJ  m'. M ¡® JË@ é«ñ
 Òm× éJÓð (x, y ∈ R, (x, y) 6= (1, 0)) ©Ó m = x + iy ©’
.
éJËXAªÓ Õæ® J‚Ó ø @ 1 + y − x = 0 ø @ i + x + iy − i(x + iy) ∈ R ‡ ® m' úæË@
 ùë éJ Ò® J‚Ó
 
A(1, 0) 颮JË@ áÓ ÐðQm× 1 + y − x = 0

z 0 = 1 − iz  m'. M 0 (z 0 ) ð M (z) áºJË ( @ −2


IJ
éJÓð ω = 1 − iω ⇔ ω =
1

2 2
i
‡ ® m' ω
 ® K ÉJ®K R ÉK ñjJË@ AJK YË
Aê®m Ì èYÓA“ é¢ .
z0
−ω
∀z ∈ C, z 6= ω,
z−ω
= −i. éJÓ ð z 0 = 1 − iz ⇔ z 0 − ω = −i(z − ω)
³−−→ −−→´
  i    ® JË@ Ω áºJË
àX@ π
ΩM , ΩM 0 ≡ − [2Π]ΩM 0 = ΩM.
2
àA i.JJ‚ ω=
1

2 2
‡jÊË@  X é¢
H@
− éJK ð@P €AJ¯
π  ð Ω èQ»QÓ à@P ðX R
2
m = x + iy, ©’ (H.
z2 − z1 m − i − 1 + im
∈ IR ⇔ ∈ IR ⇔ i(x + iy − i − 1 + ix − y) ∈ IR
z2 − m m−i−m
⇔ −y + 1 − x = 0 ⇔ Re(m) + Im(m) = 1
èQK @X Yg ñK éJÓð Ω ú¯ éK ð@QË@ Õç' A¯ ΩM M1 IÊ JÖÏ @ éJÓð z1 = 1 − im ⇒ R(M ) = M1 AJK YË (h.
.
. ΩM M1 IÊ
 JjÖÏ @ èQK @YË@ ùë ð , AîD¯
 JÖÏ AK. é¢ Q¢¯ [M M1 ] IJ  m'. Ω ð M, M1 áÓ QÖß èYJkð (C)
ú¯ éK ð@QË@ Õç' A¯ M2 M M1 Z ú¯A¾K M2 ∈ (C) Z ú¯A¾K èPð@YJÓ M2 ð Ω, M, M1 ¡® JË@ àñºK éJÓ ð
z2 − z1 z2 − z1 π ³−−−→ −−−−→´ π
z2 − m
∈ IR Z ú¯A¾K arg(
z2 − m
) ≡ ± [2π] Z ú¯A¾K M2 M , M2 M1 ≡ ± [2Π] Z ú¯A¾K M2
2 2

Re(m) + Im(m) = 1 ‡K. A‚Ë@ È@ñ‚Ë@ I
. ‚k Z ú¯A¾K
éJËXAªÓ ø YË@ Õæ® J‚ÖÏ @ ùë èPð@YJÓ M2 ð Ω, M, M1 ‡ ® m' úæÊ Ë@ M (m) ¡® JË@ é«ñÒm
 × à X@
.
 
A(1, 0) 颮JË@ áÓ ÐðQm× : x + y − 1 = 0

.
 JË@ áK QÒJË@
 K. A‚mÌ '@ : IËA
HAJ

an = 2n + 3n + 6n − 1 : ©’ N∗ áÓ n ɾË
úk. ð P X Y« an N∗ áÓ n
 ®j
É¾Ë éK@ áÓ ‡  JJË ( @ 1
2 ≡ 0[2] ⇒ ∀n ∈ N∗ , 2n ≡ 0[2]
3 ≡ 1[2] ⇒ ∀n ∈ N∗ , 3n ≡ 1[2]

http://mathkas.ici.ma
éJ“ AK P Ð ñÊ« ¼AK.2 2009 ñJKñK èPð X HA
 J“AK QË@ ú¯ úæ£ñË@ àAjJÓB@ iJj’

6 ≡ 0[2] ⇒ ∀n ∈ N∗ , 6n ≡ 0[2]
∀n ∈ N∗ , an = 2n + 3n + 6n − 1 ≡ 1 − 1 ≡ 0[2] éJÓð
úk. ð P X Y« áÓ n
an N∗ É¾Ë à X@
2 ≡ −1[3] ⇒ ∀n ∈ N∗ , 2n ≡ (−1)n [3] an ≡ 0[3]  n Õæ¯ XYjJË (H
AêÊg. @ áÓ àñºK úæË@ .
3 ≡ 0[3] ⇒ ∀n ∈ N∗ , 3n ≡ 0[3]
6 ≡ 0[0] ⇒ ∀n ∈ N∗ , 6n ≡ 0[3]
úk. ð P X Y« n àA¿ @ X@ ¡® ¯ð @ X@ ∀n ∈ N∗ , an = 2n + 3n + 6n − 1 ≡ (−1)n − 1 ≡ 0[3] éJÓð

p>3  m'. AJËð @ @ X Y«


IJ p áºJË −2

(F ermat) éJëQ.Ó I.‚k éKA¯ pZ6=1 ð pZ2=pZ3=1 àA¯ úÍð @ p ð p > 3 à @ AÜß. ( @
6p−1 ≡ 1[p] ð 2p−1 ≡ 1[p], 3p−1 ≡ 1[p]

à@  
i.JJ‚ ‡K . A‚Ë@ È@ñ‚Ë@ áÓ 6ap−2 = 3 × 2p−1 + 2 × 3p−1 + 6p−1 − 6 AJK YÊ(K.
6ap−2 = 32p−1 + 23p−1 + 6p−1 − 6 ≡ 3 + 2 + 1 − 6 ≡ 0[p]

p/ap−2 (GAU SS) éJëQ.Ó I.‚k àA¯ pZ6=1 à @ AÜß. ð p/6ap−2 éJÓð

áºJË (h. AJËð @ @ X Y« q



n = k éJÓð ak Z q = q ð ∀kN∗ , q/ak ( @ −1 È@ñ‚Ë@ áÓ q = 2 : I éËAmÌ '@

n = 2k éJÓð a2k Z q = q ð ∀kN∗ , q/a2k (H. −1 È@ñ‚Ë@ áÓ q = 3 : II éËAmÌ '@

n = q − 2 éJÓð aq−2 Z q = q ð q/aq−2 (H . −2 È@ñ‚Ë@ áÓ q > 3 : III éËAmÌ '@

ÉJÊjJË@ : éË A‚Ó

àA¿ @ X@ : fn (x) = x(1 − ln(x))n úÎK AÜß. é¯QªÖÏ @ fn : [0, +∞[7→ R


 QJªK
éË@YË@ . N∗ áºJËáÓ n
fn (0) = 0 ð x > 0
(O;~i, ~j) 
Ñ¢JÜØ YÓAªJÓ ÕÎªÓ ú¯ AëAJjJÓ (Cn )
È ðB@ ZQm.Ì '@
Q®’Ë@ ú¯ áÒJË@ úΫ fn ÈA’@ ( @ −1
fn (0) = lim fn (x) à@ áJ.JË
x→0+
úΫ ɒm' tn = x ©“ñK.
lim tln(t) = 0 àB lim fn (x) = lim tn (1 − nln(t))n = lim (t − ntln(t))n = 0 = fn (0)
t→0+ x→0+ t→0+ t→0+
éJ“ AK P Ð ñÊ« ¼AK.2 2009 ñJKñK èPð X HA
 J“AK QË@ ú¯ úæ£ñË@ àAjJÓB@ iJj’

Q®’Ë@ ú¯ áÒJË@ úΫ fn †A ® Jƒ@ éJÊK. A¯ (H.


fn (x) − fn (0)
lim
x−0
I.‚jJË
x→0+
Q « fn (x) − fn (0)
fn à X@ lim −ln(t) = +∞ àB lim
x−0
= lim (1 − ln(x))n = +∞ AJK YË
t→0+ x→0+ x→0+
Q®’Ë@ ú¯ áÒJË@ úΫ †A ® JƒC  A¯
 Ë éÊK.
f1 (x)
x→+∞
lim
x
= lim (1 − ln(x)) = −∞
x→+∞
lim f1 (x) = lim x(1 − ln(x)) = −∞
x→+∞ x→+∞
(h.
f2 (x)
lim = lim (1 − ln(x))2 = +∞ lim f2 (x) = lim x(1 − ln(x))2 = +∞
x→+∞ x x→+∞ x→+∞ x→+∞

f1  QªK ( @ −2
H@
∀x > 0, f10 (x) = 1 − ln(x) − 1 = −ln(x) ð R∗+ úΫ †A ® JƒC  A¯ f1 AJK YË
 Ë éÊK.
f1 H@ ªK ÈðYg.
 Q
x 0 1 +∞
f10 (x) k + 0 −
f1 (x) 0 % 1 & −∞
f2  QªK (H.
H@ −2
ð R∗+ ú Ϋ †A ® JƒC
 Ë  A¯
éÊK
. f2 AJK YË
1
∀x > 0, f20 (x) = (1 − ln(x))2 − 2x (1 − ln(x)) = (ln(x) − 1)(1 + ln(x))
x
x 0 e−1 e +∞
0
f2 (x) k + 0 − 0 +
f2 (x) 0 % 4e−1 & 0 % +∞
: (C2 ) ð (C1 ) È úæ.‚Ë@ ©“ñË@  PX
éƒ@ (@ −3
∀x ∈ R∗+ , f1 (x) − f2 (x) = x(1 − ln(x)(1 − 1 + ln(x)) = xln(x)(1 − ln(x))
x 0 1 e +∞
f1 (x) − f2 (x) 0 − 0 + 0 −
úæ.‚Ë@ ©“ñË @ C1
 ¯ C2
†ñ C2
 ¯ C1
†ñ C1
 ¯ C2
†ñ
éJ“ AK P Ð ñÊ« ¼AK.2 2009  J“AK QË@ ú¯ úæ£ñË@ àAjJÓB@ iJj’
ñJKñK èPð X HA

úGAJË@ ZQm.Ì '@


Z 1
 QJªK −1
 m'. F éË@YË@
IJ f1 (t)
∀x ≤ 0, F (x) =
e x 1 + t2 .
dt :

ð ] − ∞, 0[ úΫ †A ® JƒC  A¯ u : x 7→ ex


 Ë éÊK
.
 ð [0, +∞[ úΫ éʒ
éË@YË@  JÓ t :7→ f1 (t) éË@YË
1 + t2
 @ (@
à@ ð ] − ∞, 0[ úΫ †A ® JƒC  Ë  A¯ F éJÓ ð u (] − ∞, 0[) @ [0, +∞[
éÊK
.
(f (ex )) ex ex (1 − ln(ex )) e2x (x − 1)
∀x < 0, F 0 (x) = −ex = − =
1 + e2x 1 + e2x 1 + e2x
] − ∞, 0[ úΫ éJ ’¯A JK F (H.

éJÓð 0 ≤ f1 (t) ð ∀t ∈ [ex , 1], 2 ≥ 1 + t2 ≥ 1 + e2x AJK YË


] − ∞, 0[ áÓ x áºJË ( @ −2
1 1 1
éJÓ ð ∀t ∈ [ex , 1], f1 (t) ≤
2 1 + t
f (t) ≤
2 1 1 + e2x
f1 (t)
 Z 1 Z 1 Z 1
1 1 1
ø@ f1 (t)dt ≤ 2
f1 (t)dt ≤
e2x
f1 (t)dt
ex 2 Z 1 ex 1 + t ex 1 + Z
1
1 1
f1 (t)dt ≤ F (x) ≤ f1 (t)dt
2 ex 1 + e2x ex
µ ¶
3 ln(x) 0
2
x ( − ) = f1 (x) AJK YË ] − ∞, 0[ áÓ x É¾Ë (H . −2
4 2
Z 1 · ¸
3 x 3 ln(t) 1
àA¯ lim e2x ( − ) = 0
x→−∞ 4 2
à@ AÜß. ð x f1 (t)dt = t ( 4 − 2 ) = 43 − e2x ( 34 − x2 ) (h.
2
e ex Z 1
3
lim f1 (t)dt =
x→−∞ ex 4
3 3   (H
à@ i.JJ‚ È@ñ‚Ë@ áÓ
8
≤l≤
4 . −2 −3

 JË@ ZQm.Ì '@


IËA
Z e
un = fn (t)dt ©’ N∗ áÓ n ɾË
1
Z e
un = fn (t)dt ≥ 0 éJÓð ∀x ∈ [1, e], fn (x) ≥ 0 AJK YË N∗ áÓ n áºJË ( @ −1
1

AJK YË [1, e] áÓ x áºJË (H.


AJK YË ð fn+1 (x) − fn (x) = x(1 − ln(x))n (1 − ln(x) − 1) = −xln(x)(1 − ln(x))n

 0 ≤ ln(x)
1≤x≤e⇒ 0 ≤ 1 − ln(x)

0 ≤ (1 − ln(x))n
∀x ∈ [1, e], fn+1 (x) − fn (x) ≤ 0 éJÓð

úΫ ɒm' [1, e] ú Ϋ fn+1 − fn ≤ 0


 ß. ‡K A‚Ë@ È@ñ‚Ë@ áÓ
éKðA®JÖÏ @ éÊÓA¾Ö . n ∈ N∗ áºJÊ(k.
un+1 ≤ un

((h. −1 ) I.‚k éJ ’¯A JK ð (( @ −1 ) I.‚k Q®’ËAK. èPñª’Ó (un ) (X

http://mathkas.ici.ma
éJ“ AK P Ð ñÊ« ¼AK.2 2009 ñJKñK èPð X HA
 J“AK QË@ ú¯ úæ£ñË@ àAjJÓB@ iJj’

u0 (x) = x, v(x) = (1 − ln(x))n+1 ©’ n ∈ N∗ áºJË (@ −2


x2 (1 − ln(x))n
u(x) = , v 0 (x) = −(n + 1)
Z 2 · ¸e Z x
e e
x2 (n + 1)
éJÓð un+1 = x(1 − ln(x))n dx =
2
(1 − ln(x))n+1 +
2
x(1 − lnx)n dx
1 1 1
1 n+1
∀n ≥ 1, un+1 = − + un
2 2
úΫ AÒîDJËXAªÓ áK YË@ áÒJ® J‚ÖÏ @ ð C2 ð C1

áK. Pñ’jÖÏ @ ø ñJ‚ÖÏ @ Qg ékA‚Ó A áºJË (H .
Q   
. ÒJ‚ËAK. x = e ð x = 1 úÍ@ñJË@
Z e
A= |f2 (x) − f1 (x)|dx × 4cm2 = u1 − u2 × 4cm2
1
1
A = 2cm2 éJÓð u1 − u2 =
2
(( @ −2 ) I.‚k AJK YËð
1
) I.‚k à @ AÜß.ð un −
n+1
= (n + 1)un − 1 = 2un+1 (( @ −2 ) I. ‚k n ≥ 2 É¾Ë ( @ −3
1 1
un ≥
n+1
à X@ un −
n+1
≥ 0 àA¯ un+1 ≥ 0 (( @ −1

àA¯ éJ ’¯A JK (un ) à @ AÜß. ð 2un+1 + 1 = (n + 1)un (( @ −2 ) I.‚k n ≥ 2 É¾Ë AJK YËð
úÍAJËAK. ð un ≤ n −1 1 éJÓð (n + 1)un ≤ 2un + 1 éJÓð 2un+1 + 1 ≤ 2un + 1
1 1
≤ un ≤
n+1 n−1
  ‡K A‚Ë@ È@ñ‚Ë@ áÓ (H
i.JJ‚
lim nun = 1, lim un = 0
n→+∞ n→+∞
. .
∀n ≥ 1, dn = |vn − un | ð 1 n+1
∀n ∈ N∗ , vn+1 = − + vn , v1 = a; a 6= u1  m'.
IJ a ∈ R −4
2 2

∀n ≥ 1, dn = à @ ©k. QËAK. áJ.JË ( @


n!
d1
2n−1 
d1 = d1 : n = 1 Ég. @ áÓ AJK YË

AJK YË dn =
2n−1
n!
d1 à@ Q®K n≥1 áºJË
(n + 1) (n + 1) (n + 1)!
dn+1 = |vn+1 − un+1 | = |vn − un | = dn = d1
2 2 2n

éJÓð ∀n ≥ 1 :
dn+1
=
n+1
à@  
i.JJ‚ ‡K . A‚Ë@ È@ñ‚Ë@ áÓ (H.
dn 2
à @AÖß.ð dn ≥ 2(n−3) d3 úΫ ɒm' ©k. QËAK. ð n+1
2
≥ 2 ⇒,
∀n ≥ 3 ⇒
dn+1
dn
≥2
lim dn = +∞ àA¯ lim 2(n−3) d3 = +∞
n→+∞ n→+∞

ð éK . PA® JÓ àñºJƒ (dn ) àA¯ dn = |vn − un | ð éK . PA® JÓ (un ) à @ AÒÊ« ð éK . PA® JÓ (vn ) I KA¿ X@ (h.
‘¯A JK @ Yë

http://mathkas.ici.ma
))ÉK[„J))C))bKTj¦J ))a[†„J)r„J)ÉK[T†¤J)\[nT
))

))’pKc)‡„t)Kc„K‚R)’KV„J
))UJiJicR)kaKj„J)a†[†)’„‰MT„J)’KV„J 2008 Kc„K‚R„„)’aKu„J)‘ca„J

( ‫ ﻧﻘﻄﺔ‬3,25 ) C)…¦J)Éhhhhhhhhhhhhhhhhhhhhhhhhhhhhhhhhhhc†T„J

.‫ ( ﺣﻠﻘﺔ واﺣﺪﻳﺔ‬M2 ( \ ) , +, ×) •
.‫ ( ﻓﻀﺎء ﻣﺘﺠﻬﻲ ﺣﻘﻴﻘﻲ‬M2 ( \ ) , +,.) •
.‫^ ( ﺝﺴﻢ ﺕﺒﺎدﻝﻲ‬, +, ×) •
⎧ ⎛ a 3b ⎞ ⎫ ⎛ 0 3⎞
⎪ ⎜ ⎟ ⎪ ⎜ ⎟ ⎛1 0⎞
(
. E = ⎨M a,b = ⎜ 1 ) a ⎟⎟
/ (a,b ) \2 ⎬ ‫ و‬J = ⎜ 1 ∈ ⎟ ‫ و‬I = ⎜ ⎟ : ‫• ﻧﻀﻊ‬
⎪ ⎜− 3b ⎪ ⎜ −
3
0 ⎟ ⎝ 0 1 ⎠
⎩ ⎝ ⎠ ⎭ ⎝ ⎠
. J = M ( 0,1) ‫ و‬I = M (1,0 ) : ‫ﻻﺣﻆ أن‬
: ‫ ﻝﺪﻳﻨﺎ‬-‫ أ‬.1
⎛ 0 0⎞
. M ( 0, 0 ) = ⎜ ⎟ ∈ E : ‫ ﻷن‬، E ≠ ∅ 9
⎝ 0 0⎠
. E ⊂ M2 ( \ ) 9
∃( a , b ) ∈ \ 2 / A = M a , b ( : ‫ إذن‬.) (α , β ) ∈\ 2 ‫وﻝﻴﻜﻦ‬ E ‫ﻋﻨﺼﺮﻳﻦ ﻣﻦ‬ B ‫و‬ A ‫ﻝﻴﻜﻦ‬ 9
∃( a , b ) ∈ \ 2 / B = M c , d ( )
⎛ a 3b ⎞ c 3d α a +⎛ β c 3 ( b + dα) ⎞ β
αA + βB = α ⎜⎜ 1 ⎟
+β 1 = 1 ⎜
⎜ ⎟
⎜− 3b a ⎟⎟ − d c (αb βd ) a cα ⎟−β
⎝ ⎠ 3 3 ⎜⎝ ⎟

α A + β B = M (α a + β c ,αb + d ) ∈ βE
. ( M ( \ ) , +,.) ‫ ( ﻓﻀﺎء ﻣﺘﺠﻬﻲ ﺝﺰﺋﻲ ﻣﻦ اﻝﻔﻀﺎء اﻝﻤﺘﺠﻬﻲ اﻝﺤﻘﻴﻘﻲ‬E , +,.) ‫وﺏﺎﻝﺘﺎﻝﻲ ﻓﺈن‬
2

: ( E , +,.) ‫ ( أﺱﺎس ﻓﻲ اﻝﻔﻀﺎء اﻝﻤﺘﺠﻬﻲ اﻝﺤﻘﻴﻘﻲ‬I , J ) ‫ ﻝﻨﺒﻴﻦ أن‬-‫ب‬



0 3 ⎞⎟
⎛1 0 + b ⎜⎜

‫ ( أﺱﺮة ﻣﻮﻝﺪة‬I , J ) ‫ إذن‬. M ( )
a,b = a ⎜
⎜0 1 ⎟⎠ ⎜ − 1
⎟ ⎟ aI bJ : ‫ ﻝﺪﻳﻨﺎ‬، \ 2=‫( ﻣﻦ‬a ,b ) ‫ ﻝﻜﻞ‬9
+
⎝ ⎜
0 ⎟

⎝ 3 ⎠
. E ‫ﻝﻠﻔﻀﺎء اﻝﺤﻘﻴﻖ‬
: ‫ ﻝﺪﻳﻨﺎ‬، \ ‫( ﻣﻦ‬a , b ) ‫ ﻝﻜﻞ‬: ‫ ﻷن‬، E ‫ ( أﺱﺮة ﺣﺮة ﻓﻲ‬I , J ) 9
2

⎛ ⎞
⎜ a 3b ⎟ ⎛ 0 0⎞
aI + bJ = 0 M (a⇒
,b ) = 0 ⎜ ⇒ ⎟=
⎜ 1 ⎜
− b a ⎟⎟ ⎝ 0

0⎠
a b 0


⎝ 3 ⎟

.E = {aI + bJ / ( a, b ) ∈ \ 2 } ‫ و‬، ( E , +,.) ‫ ( أﺱﺎس ﻓﻲ اﻝﻔﻀﺎء اﻝﻤﺘﺠﻬﻲ اﻝﺤﻘﻴﻘﻲ‬I , J ) ‫وﺏﺎﻝﺘﺎﻝﻲ ﻓﺈن‬

)))))))))))))))’hhhhhhhhhhhhpKc)‡„t)Kc„K‚R)’KV„J 1 ’[}n„J
‫ﺏﻤﺎ ﻳﻠﻲ ‪:‬‬ ‫*‪E‬‬ ‫ﻧﺤﻮ‬ ‫‪ f‬اﻝﻤﻌﺮف ﻣﻦ *^‬ ‫‪ .‬ﻧﻌﺘﺒﺮ اﻝﺘﻄﺒﻴﻖ‬ ‫‪ .2‬ﻝﻴﻜﻦ })‪E * = E \ {M ( 0,0‬‬
‫‪f‬‬ ‫‪:‬‬ ‫→ *^‬ ‫*‪E‬‬
‫) ‪a + ib 6 M (a,b‬‬
‫ﻋﻨﺼﺮﻳﻦ ﻣﻦ ‪. E‬‬ ‫أ‪ -‬ﻧﻌﺘﺒﺮ ‪ A = aI + bJ‬و ‪B = cI + dJ‬‬
‫‪⎛ 0‬‬ ‫⎞‪3‬‬ ‫‪0‬‬ ‫‪3‬‬ ‫⎛‬ ‫⎞‬
‫⎜‬ ‫⎟‬ ‫⎞‪⎛ 0 −⎜ 1‬‬ ‫⎟‬
‫‪J2‬‬ ‫‪=⎜ 1‬‬ ‫×‬ ‫‪1‬‬ ‫⎜‬ ‫⎟ ⎜‬ ‫‪I‬‬ ‫=‬ ‫ﻝﺪﻳﻨﺎ ‪:‬‬
‫‪⎜− 3‬‬ ‫‪0 ⎟⎟ −‬‬ ‫‪0‬‬ ‫⎠ ‪⎝ −1 ⎜0‬‬
‫⎟‬
‫⎟‬
‫⎝‬ ‫⎠‬ ‫‪3‬‬ ‫⎝‬ ‫⎠‬
‫‪. A ×B‬‬ ‫‪= (aI + bJ ) × cI dJ‬‬ ‫‪+ ( (ad =bc ) J bdJ‬‬
‫‪acI‬‬ ‫‪)+ 2‬‬ ‫‪(ac‬‬ ‫‪ −‬إذن ‪+bd ) I + (ad+ bc J ∈ E= ) :‬‬
‫وﺏﺎﻝﺘﺎﻝﻲ ﻓﺈن ‪ E‬ﺝﺰء ﻣﺴﺘﻘﺮ ﻣﻦ )×‪. ( M2 ( \ ) ,‬‬
‫) ‪M (a,b ) × M (c , d ) = M (ac bd , ad bc‬‬ ‫ﻣﻼﺣﻈﺔ ‪− :‬‬ ‫‪+‬‬
‫ﺕﻄﺒﻴﻖ ﻣﻌﺮف‪.‬‬ ‫‪f‬‬ ‫‪ .‬وﻣﻨﻪ ﻓﺈن‬ ‫‪ ، z = a + ib ∈ ^* /‬ﻓﺈن ‪f ( z ) = M (a,b ) ∈ E * :‬‬ ‫) ‪(a,b‬‬ ‫∈ ‪\2‬‬
‫ب‪ -‬إذا آﺎن‬
‫‪ . z ′ = a′ + ib‬ﻝﺪﻳﻨﺎ ‪:‬‬ ‫‪^* ∈′/‬‬ ‫*^ ∈ ‪′ ∈z = a′ + ib‬و ‪(a ,b ) \ 2‬‬ ‫‪/‬‬ ‫) ‪(a,b‬‬ ‫‪∈ 9‬‬
‫ﻝﻴﻜﻦ ‪\ 2‬‬
‫‪f ( z × z ′) = f‬‬ ‫‪((aa′ bb ) i (ab− a b )) +‬‬ ‫‪′‬‬ ‫‪+‬‬ ‫‪′‬‬
‫= )‪f ( z × z ′‬‬ ‫) ‪M (aa′ bb , ab −a b‬‬ ‫‪+‬‬ ‫‪′‬‬ ‫‪′‬‬
‫= )‪f ( z × z ′‬‬ ‫×) ‪M a, a′ M (b( ,b‬‬ ‫)‬ ‫‪′‬‬
‫= )‪f ( z × z ′‬‬ ‫)‪f ( z ) f ( z ′‬‬ ‫×‬
‫(‬ ‫)‬ ‫(‬
‫ﺕﻄﺒﻴﻖ ﺕﺸﺎآﻠﻲ ﻣﻦ ×‪ ^* ,‬ﻧﺤﻮ × ‪. E * ,‬‬ ‫)‬ ‫‪f‬‬ ‫وﻣﻨﻪ ﻓﺈن‬
‫ﻓﺈن ‪:‬‬ ‫ﻧﻌﺘﺒﺮ * ‪ . A ∈ E‬ﺏﻤﺎ أن ) ‪ ( I , J‬أﺱﺎس ﻓﻲ اﻝﻔﻀﺎء اﻝﻤﺘﺠﻬﻲ اﻝﺤﻘﻴﻘﻲ )‪( E , +,.‬‬ ‫‪9‬‬
‫‪.‬‬ ‫‪ .‬إذن *^∈ ‪a + ib‬‬ ‫‪ . ∃!(a,b ) ∈ \ 2 / A = aI bJ M a,b = f( a +) ib‬وﻝﺪﻳﻨﺎ * ‪A ∈ E‬‬ ‫‪(+‬‬
‫∈) ‪ . ∀A ∈ E * : ∃z ^* / f ( z‬إذن ‪ f‬ﺕﻄﺒﻴﻖ ﺕﻘﺎﺏﻠﻲ‪.‬‬ ‫وﺏﺎﻝﺘﺎﻝﻲ ﻓﺈن ‪= A :‬‬
‫‪.‬‬ ‫‪ f‬ﺕﺸﺎآﻞ ﺕﻘﺎﺏﻠﻲ ﻣﻦ )×‪ ( ^ ,‬ﻧﺤﻮ )×‪( E ,‬‬
‫*‬ ‫*‬
‫ﺧﻼﺻﺔ ‪:‬‬
‫‪ .3‬ﻝﺪﻳﻨﺎ ‪:‬‬
‫‪ ( E , +,.) 9‬ﻓﻀﺎء ﻣﺘﺠﻬﻲ ﺣﻘﻴﻘﻲ ‪ .‬إذن ‪ ( E , + ) :‬زﻣﺮة ﺗﺒﺎدﻟﻴﺔ ‪.‬‬
‫‪ E‬ﺝﺰء ﻣﺴﺘﻘﺮ ﻣﻦ )×‪ ( M2 ( \ ) ,‬و )× ‪ ( M2 ( \ ) , +,‬ﺣﻠﻘﺔ واﺣﺪﻳﺔ ‪ .‬وﻣﻨﻪ ﻧﺴﺘﻨﺘﺞ أن ‪:‬‬ ‫‪9‬‬
‫اﻝﻘﺎﻧﻮن × ﺕﺠﻤﻴﻌﻲ ﻓﻲ ‪. E‬‬
‫اﻝﻘﺎﻧﻮن × ﺕﻮزﻳﻌﻲ ﻋﻠﻰ اﻝﻘﺎﻧﻮن ‪ +‬ﻓﻲ ‪. E‬‬
‫‪ I‬هﻮ اﻝﻌﻨﺼﺮ اﻝﻤﺤﺎﻳﺪ ﺏﺎﻝﻨﺴﺒﺔ ﻝﻠﻘﺎﻧﻮن × ﻓﻲ ‪. E‬‬
‫وهﺬا ﻳﺪل ﻋﻠﻰ أن )× ‪ ( E , +,‬ﺣﻠﻘﺔ واﺣﺪیﺔ‪.‬‬

‫(‬ ‫)‬ ‫(‬


‫ﺕﺸﺎآﻞ ﺕﻘﺎﺏﻠﻲ ﻣﻦ ×‪ ^* ,‬ﻧﺤﻮ × ‪ ، E * ,‬إذن ‪:‬‬ ‫)‬ ‫‪f‬‬ ‫‪ 9‬ﻝﺪﻳﻨﺎ‬
‫‪. ∀A ∈ E‬‬ ‫وﻝﺪﻳﻨﺎ ‪: A 0 ×0 A= 0‬‬
‫اﻝﻘﺎﻧﻮن × ﺕﺒﺎدﻝﻲ ﻓﻲ *=‪× ، E‬‬ ‫اﻝﻘﺎﻧﻮن × ﺕﺒﺎدﻝﻲ ﻓﻲ *^ ‪ ،‬ﻳﺴﺘﻠﺰم أن‬
‫إذن × ﻗﺎﻧﻮن ﺗﺒﺎدﻟﻲ ﻓﻲ ‪. E‬‬
‫(‬ ‫)‬ ‫(‬ ‫)‬ ‫(‬
‫ﺕﺸﺎآﻞ ﺕﻘﺎﺏﻠﻲ ﻣﻦ ×‪ ^* ,‬ﻧﺤﻮ × ‪ ، E * ,‬ﻓﺈن × ‪ E * ,‬زﻣﺮة‪.‬‬ ‫)‬ ‫‪f‬‬ ‫(‬
‫‪ 9‬ﺏﻤﺎ أن ×‪ ^* ,‬زﻣﺮة ﺕﺒﺎدﻝﻴﺔ و‬ ‫)‬
‫ﺟﺴﻢ ﺗﺒﺎدﻟﻲ ‪.‬‬ ‫وﺏﺎﻝﺘﺎﻝﻲ ﻓﺈن ‪( E , +, ×) :‬‬

‫‪)))))))))))))))’hhhhhhhhhhhhpKc)‡„t)Kc„K‚R)’KV„J‬‬ ‫‪2‬‬ ‫‪’[}n„J‬‬


: ‫ ﻝﺪﻳﻨﺎ‬. J ×X 3 = I ‫ اﻝﻤﻌﺎدﻝﺔ‬E ‫ ﻝﻨﺤﻞ ﻓﻲ‬.4
J ×X 3 = I ⇔ f −1
(J × X ) = f ( I )
3 −1

( J ) × ( f X () = f) ( I )
3
J ×X 3 = I ⇔ f −1 −1 1 −

. f −1
(I ) = f −1
( M (1, 0)) = 1 0 i 1 ‫و‬f −1
(J ) = f (M + ( 0,1) ) = 0 + 1
−1
i× ×i : ‫وﺏﻤﺎ=أن‬ =
: ‫ ﻓﺈن اﻝﻤﻌﺎدﻝﺔ اﻝﺴﺎﺏﻘﺔ ﺕﺼﺒﺢ‬، f −1
(X ) = z ‫و‬
J ×X 3 = I iz⇔ 3
1 =
⇔ z = −i
3

⇔ z 3 =i3
3
⎛z ⎞
⇔ ⎜ i ⎟ =1
⎝ ⎠
J ×X 3 = I
z
i
⇔ {
1, j , j } ∈
1 3 1 3
: ‫ إذن‬. j = − −i ‫ و‬j = − +i ‫ﺣﻴﺚ‬
2 2 2 2
J ×X 3 = I ⇔ z {i , ij , i j } ∈

J ×X 3 = I f (z )
X ⇔ {f (=i ) , f ( )}
ij ( , f ∈)i j
⎛ 0 3⎞
⎜ ⎟
f ( i ) = M ( 0,1) = J ⎜ 1 = : ‫وﻝﺪﻳﻨﺎ‬
⎜− 3 0 ⎟⎟
⎝ ⎠
⎛ 3 3⎞
⎛ ⎜− ⎛− ⎟
3 1 ⎞ 3 1 ⎜ −2 2 ⎟ ⎞
f ( ij ) = f ⎜ − i ⎟ −M =, ⎜ 1
⎜ − :‫و‬ = ⎟
⎜ 2 2 ⎟⎠ 2 2 ⎜ 3⎟ ⎟
⎝ ⎜⎜ ⎝− ⎟ ⎠
⎝2 3 2 ⎟⎠
⎛ 3 3⎞
⎛ 3 ⎜ − ⎟
1 ⎞ 3 1 ⎜ 2 ⎛ ⎞
( )
f i j =f ⎜

− i⎟ M

⎝ 2 2 ⎠ 2
, = ⎜
2 ⎜ 1 ⎜⎝ 3 ⎟
2 ⎟

:‫و‬ ⎟


=
⎜⎜ ⎟⎟
⎝ 2 3 2 ⎠
: ‫ هﻲ‬E ‫ ﻓﻲ‬J ×X 3 = I ‫وﺏﺎﻝﺘﺎﻝﻲ ﻓﺈن ﻣﺠﻤﻮﻋﺔ ﺣﻠﻮل اﻝﻤﻌﺎدﻝﺔ‬
⎧ ⎛ 3 3⎞ 3 3 ⎫ ⎛
⎪⎛ 0 3⎞ ⎜− − ⎟ ⎪ − ⎜
S = ⎪⎨⎜⎜ 1 ⎟
, ⎜ 2 2 ⎟
,
2 2 ⎪
⎬ ⎜
⎪⎜ − 0 ⎟⎟ ⎜ 1 3⎟ 1 3 ⎪ ⎜
⎜ − ⎟ ⎜
⎪⎩⎝ 3 ⎠
⎝2 3 2 ⎠ 2 3 2 ⎭⎪ ⎝

)))))))))))))))))’hhhhhhhhhhhhpKc)‡„t)Kc„K‚R)’KV„J 3 ’[}n„J
))( ‫ ﻧﻘﻄﺔ‬3,75 ) C)hhhhhhhhhhhhhhhhhhhhhKV„J)Éhhhhhhhhhhhhhhhhhhhhhc†T„J

. a∈^* ‫ﻝﻴﻜﻦ‬
. (G ) : iz 2
(
+ a + a − i z a −iaa −0 ) : =‫اﻝﻤﻌﺎدﻝﺔ‬ ^ ‫ ﻧﻌﺘﺒﺮ ﻓﻲ اﻝﻤﺠﻤﻮﻋﺔ‬.I
: ‫هﻮ‬ (G ) ‫ ﻣﻤﻴﺰ اﻝﻤﻌﺎدﻝﺔ‬-‫ أ‬.1
∆ = b 2 − 4ac = (a ) ( )
2
a i +4i a − iaa − − −

∆ = ( (a − i ) )
2
a 4+a (a i ) − −

∆ = (a − i )2 2a (a+ i ) a − 4a (a + i )
2
− −
∆ = (a − i )2 2a (a− i ) a −
2
+

∆ = (a − i )
2
a −

∆ = (a − a i)
2

∆ = (a − i )
2
: ‫( ﺣﻠﻴﻦ ﻋﻘﺪﻳﻴﻦ هﻤﺎ‬G ) ‫ إذن ﻝﻠﻤﻌﺎدﻝﺔ‬. a − (G ) ‫ ﻣﻤﻴﺰ اﻝﻤﻌﺎدﻝﺔ‬-‫ب‬
‫هﻮ‬

.z 2 =
(
− a +a i ) −(a a −i ) = −−2a + 2−i 1 + ia ‫و‬ z1 =
(
− a +a i ) −(a a +i ) = −−2a −
=i a
2i 2i 2i 2i
{ }
. S = i a,1 + ia : ‫( هﻲ‬G ) ‫وﻣﻨﻪ ﻓﺈن ﻣﺠﻤﻮﻋﺔ ﺣﻠﻮل اﻝﻤﻌﺎدﻝﺔ‬

(G ) ‫ ﺣﻞ ﻝﻠﻤﻌﺎدﻝﺔ‬a ⇔ a = i a ‫ أو‬a 1 =ia + .2

⇔ ℜe (a ) + i ℑm a (=) mℑ a i ( +e) a ℜ ‫أو‬ a( =


) 1 −1 i
⇔ ℜe (a ) = ℑm a ( ‫ ) أو‬a = 1 (1 +i )
2
⇔ ℜe (a ) = ℑm (a )
JJG J G
. ℜe (a ) ≠ ℑm a ( : ‫ ﻧﻔﺘﺮض )أن‬. (O ,u ,v ) ‫ اﻝﻤﺴﺘﻮى اﻝﻌﻘﺪي ﻣﻨﺴﻮب إﻝﻰ ﻣﻌﻠﻢ ﻣﺘﻌﺎﻣﺪ ﻣﻤﻨﻈﻢ ﻣﺒﺎﺵﺮ‬.II
. 1+ ia ‫ و‬i a ‫ و‬a ‫ ﻧﻘﻂ أﻝﺤﺎﻗﻬﺎ ﻋﻠﻰ اﻝﺘﻮاﻝﻲ‬C ‫ و‬B ‫ و‬A

. Z =
(1 + ia ) − a : ‫ ﻧﻀﻊ‬.1
(i a ) − a
.Z
( )
⎛ 1 + ia − a ⎞ 1 − i a − a i
= ⎜⎜ ⎟= =
( ) ⎢⎣ (
⎡ −i − a i a+⎤
) ⎥⎦ a ( i 1) i − −
: ‫ ﻝﺪﻳﻨﺎ‬-‫= أ‬
⎝ ( )
⎜ i a − a ⎟⎟

−ia − a ( ) i ⎡⎣ −a + i a ⎤⎦ i a −a
zc −z A
. ‫ ﻧﻘﻂ ﻣﺴﺘﻘﻴﻤﻴﺔ‬C ‫ و‬B ‫ و‬A ⇔ ∈\ -‫ب‬
zB −zA
⇔ Z ∈\
⇔Z Z =
))))))))))))))’hhhhhhhhhhhhpKc)‡„t)Kc„K‚R)’KV„J 4 ’[}n„J

(1 + ia ) − a = (i −1) a i −
(i a ) − a i a −a
⇔ (1 + ia ) − a i 1 =a i ( − )−
⇔ 1 + a ( i − 1) i 1 a− (i 0 − ) + =
⇔ (i −1) (a a ) (1 i )− = − +
1+ i
⇔ a −a =
1− i
(1 + i )
2

⇔ 2i ℑm (a ) =
2
⇔ 2i ℑm (a ) i =
1
⇔ ℑm (a ) =
2
1
. ℑm (a ) ≠
: ‫ ﻧﻔﺘﺮض ﻓﻲ هﺬا اﻝﺴﺆال أن‬.2
2
.‫ ﻧﻘﻄﺘﻴﻦ ﻣﻦ اﻝﻤﺴﺘﻮى اﻝﻌﻘﺪي‬M ′ ( z ′) ‫ و‬M ( z ) ‫ وﻝﺘﻜﻦ‬. θ ‫ وزاوﻳﺘﻪ‬Ω (ω ) ‫ اﻝﺪوران اﻝﺬي ﻣﺮآﺰﻩ‬R ( Ω,θ ) ‫ ﻝﻴﻜﻦ‬: ‫ﺗﺬآﻴﺮ‬

R ( Ω,θ ) ( M ) = M ′ ⇔ z ′ = e i θ z + 1− e i θ ω ( )
.
π ‫ وزاوﻳﺘﻪ‬A ‫اﻝﺪوران اﻝﺬي ﻣﺮآﺰﻩ‬ R2 ‫ و‬− π ‫ وزاوﻳﺘﻪ‬A ‫اﻝﺪوران اﻝﺬي ﻣﺮآﺰﻩ‬ R1 ‫ﻧﻌﺘﺒﺮ‬
2 2
. ⎡⎣ BC ⎤⎦ ‫ ﻣﻨﺘﺼﻒ اﻝﻘﻄﻌﺔ‬E ‫ ﻝﺘﻜﻦ اﻝﻨﻘﻄﺔ‬. R 2 (C ) = C ′ ‫ و‬R1 ( B ) = B ′ : ‫ﻧﻀﻊ‬
−i π⎛ −i π ⎞
. b ′ = e 2b
+ 1 e 2 ⎟a

⎜ ⎟ ( ) (1 i )a =(1+ i )a +−a
i −i a : ‫ إذن‬،+R1 ( B ) = B +
′ :‫ ﻝﺪﻳﻨﺎ‬-‫أ‬
⎝ ⎠
iπ ⎛ iπ ⎞
c ′ = e c + 1 e 2 ⎟ a i −1
2 ⎜
⎜ ⎟ ( ia ) (1 i =)a i a +a ia i −+ia R 2 (C ) = C ′ : =
: ‫ إذن‬، − ‫وﻝﺪﻳﻨﺎ‬ −
⎝ ⎠
b +c i a + 1 + ia
. e = aff ( E ) = : ‫ إذن‬.=⎡⎣ BC ⎤⎦ ‫ ﻣﻨﺘﺼﻒ اﻝﻘﻄﻌﺔ‬E ‫ ﻝﺪﻳﻨﺎ‬-‫ب‬
2 2
JJJJGJJJJJJG ⎛ c ′ −b ′ ⎞
. ⎛⎜ AE , B ′C ′ ⎞⎟ ≡ arg ⎜ ⎟ ⎡⎣ 2π ⎤⎦ : ‫إذن‬
⎝ ⎠ ⎝ e −a ⎠

.
c ′ − b ′ = ( i − ia ) (a− ia +a ) =+ i − 2ia − a a 2i−(1 ia i a 2+a ) +2i ⎡ 2, π−⎤ : ‫=وﻝﺪﻳﻨﺎ‬
e −a i a + 1 + ia 1 ia+ i a + 2a − 1 ia i a+ 2a + − ⎢⎣ 2 ⎥⎦
−a
2 2
B ′C ′ c − b′ ′ JJJJG JJJJJJG π
. B ′C ′ = 2AE ‫و‬ ( B ′C ′) ⊥ AE (
: ‫إذن‬.
AE
= )
e −a
= 2 ‫ ⎜⎛ و‬AE , B ′C ′ ⎞⎟ ≡ ⎡⎣ 2π ⎤⎦ :‫إذن‬
⎝ ⎠ 2

))))))))))))))’hhhhhhhhhhhhpKc)‡„t)Kc„K‚R)’KV„J 5 ’[}n„J
‫) ‪ 3‬ﻧﻘﻄﺔ ())‬ ‫‪C)W„KV„J)Éhhhhhhhhhhhhhhhhhhhhhc†T„J‬‬

‫‪.‬‬ ‫‪( E ) : 35u − 96v =1‬‬ ‫‪ .I‬ﻧﻌﺘﺒﺮ ﻓﻲ اﻝﻤﺠﻤﻮﻋﺔ ‪ ]2‬اﻝﻤﻌﺎدﻝﺔ اﻝﺘﺎﻝﻴﺔ ‪:‬‬

‫‪.‬‬ ‫‪ .1‬ﻝﺪﻳﻨﺎ ‪ . 35×11− 96× 4 385= 384 1−:‬إذن ) ‪ (11, 4‬ﺣﻞ ﺧﺎص ﻝﻠﻤﻌﺎدﻝﺔ ) ‪( E‬‬ ‫=‬
‫‪.‬‬ ‫‪ . 35×11− 96× 4 1‬ﺣﺴﺐ ‪ ،Bezout‬ﻧﺴﺘﻨﺘﺞ أن ‪ 96‬و ‪ 35‬أوﻝﻴﺎن ﻓﻴﻤﺎ ﺏﻴﻨﻬﻤﺎ ‪35 ∧ 96 =1 :‬‬ ‫‪= .2‬‬
‫ﻝﺪﻳﻨﺎ ‪:‬‬

‫) ‪⇔ 35 (u − 11) = 96 (v − 4‬‬ ‫) ‪(i‬‬


‫) ‪⇒ 35 / 96 (v − 4‬‬
‫‪⇒ 35/v − 4‬‬
‫‪Gauss‬‬
‫) ‪⇒ ∃k ∈] / v −4 =35k ( ii‬‬
‫‪⇒ ∃k ∈ ] / v = 4 + 35k‬‬
‫ﻧﻌﻮض ﻧﺘﻴﺠﺔ اﻝﻌﻼﻗﺔ ) ‪ ( ii‬ﻓﻲ اﻝﻌﻼﻗﺔ ) ‪ ، ( i‬ﻓﻨﺠﺪ ‪ . 35 (u − 11) = 96 × 35k :‬أي ‪ u − 11 = 96k :‬ﻳﻜﺎﻓﺊ ‪u = 11 + 96k‬‬
‫وﺏﻤﺎ أن اﻷزواج ) ‪ ، (11 + 96k ,4 + 35k‬ﺣﻴﺚ ] ∈ ‪ ، k‬ﺕﺤﻘﻖ اﻝﻤﻌﺎدﻝﺔ ) ‪ ، ( E‬ﻓﺈن ﻣﺠﻤﻮﻋﺔ ﺣﻠﻮل اﻝﻤﻌﺎدﻝﺔ ) ‪ ( E‬هﻲ ‪:‬‬

‫{‬
‫‪S = (11 + 96k ,4 35k ) /+ k‬‬ ‫]‬‫}‬ ‫∈‬
‫) ‪. (F‬‬ ‫⎦⎤ ‪: x 35 ≡ 2 ⎡⎣97‬‬ ‫اﻝﻤﻌﺎدﻝﺔ اﻝﺘﺎﻝﻴﺔ ‪:‬‬‫‪ .II‬ﻧﻌﺘﺒﺮ ﻓﻲ اﻝﻤﺠﻤﻮﻋﺔ `‬
‫‪ .1‬ﻝﻴﻜﻦ ‪ x‬ﺣﻼ ﻝﻠﻤﻌﺎدﻝﺔ ) ‪. ( F‬‬
‫أ‪ -‬ﻝﺪﻳﻨﺎ ‪:‬‬

‫(‬ ‫‪q<p‬‬ ‫أو‬ ‫‪p‬‬ ‫‪2‬‬


‫) ﻧﺘﻮﻗﻒ إذا آﺎن ‪> 97‬‬ ‫إذن ‪ 97‬ﻋﺪد أوﻝﻲ ‪.‬‬
‫‪.‬‬ ‫‪ . 97 ∧ x = d‬إذن ‪ d /97‬و ‪ 97‬ﻋﺪد أوﻝﻲ ‪ ،‬وﻣﻨﻪ ﻓﺈن ‪ d = 1 :‬أو ‪ . d = 97‬ﻧﻔﺘﺮض أن ‪d = 97 :‬‬ ‫ﻝﺒﻜﻦ‬
‫‪ . 97 ∧ x = 97‬وﻋﻠﻴﻪ ﻓﺈن ‪ 97/ x :‬أي ‪ x ≡ 0 ⎡⎣79⎤⎦ :‬وهﺬا ﻳﺴﺘﻠﺰم ‪ . x 35 ≡ 0 ⎡⎣97 ⎤⎦ :‬وﻝﺪﻳﻨﺎ‪:‬‬ ‫إذن ‪:‬‬

‫∈ ‪ . 0,2‬وهﺬا ﻳﺸﻴﺮ إﻝﻰ أن ‪ . 0 = 2‬وهﺬا ﺕﻨﺎﻗﺾ‪.‬‬ ‫⎦⎥⎤‪ . x 35 ≡ 2 ⎡⎢⎣97‬إذن ‪ 0 ≡ 2 ⎡⎢⎣97⎤⎥⎦ :‬و }‪{0,1,2,...,97‬‬
‫أوﻝﻴﺎن ﻓﻴﻤﺎ ﺏﻴﻨﻬﻤﺎ‪.‬‬ ‫‪x‬‬ ‫و‬ ‫‪97 . 97 ∧ x =1‬‬ ‫وﺏﺎﻝﺘﺎﻝﻲ ﻓﺈن ‪:‬‬

‫‪))))))))))))))’hhhhhhhhhhhhpKc)‡„t)Kc„K‚R)’KV„J‬‬ ‫‪6‬‬ ‫‪’[}n„J‬‬


‫‪.‬‬ ‫ب‪ -‬ﻝﺪﻳﻨﺎ ‪ 97 ∧ x =1 :‬و ‪ 97‬ﻋﺪد أوﻝﻲ ‪ .‬ﺣﺴﺐ ﻣﺒﺮهﻨﺔ ﻓﻴﺮﻣﺎ ‪ ،‬ﻝﺪﻳﻨﺎ ‪x 96 ≡1⎡⎢⎣97⎤⎥⎦ :‬‬
‫) (‬ ‫ﺏﻤﺎ أن ‪ ، x 35 ≡ 2 ⎡⎢97⎤⎥ :‬ﻓﺈن ‪ . ( x 35 ) ≡ 2 ⎡⎢97⎤⎥ :‬أي ‪x 385 ≡ 211 ⎡⎢97⎤⎥ :‬‬
‫‪11‬‬
‫‪11‬‬
‫‪. i‬‬ ‫‪:‬‬ ‫ﺝـ‪-‬‬
‫⎣‬ ‫⎦‬ ‫⎣‬ ‫⎦‬ ‫⎣‬ ‫⎦‬

‫) (‬
‫‪4‬‬
‫‪x 385=96×4 1 =+ x 96 × x‬‬ ‫‪ .‬إذن ‪:‬‬ ‫وﻝﺪﻳﻨﺎ ‪385=96× 4+1 :‬‬
‫) (‬ ‫) (‬
‫‪4‬‬ ‫‪4‬‬
‫‪. x 96 ≡ 1 ⎡⎣97 ⎤⎦ ⇒ x 96‬‬ ‫≡‪14 97 ⎡⎣ x‬‬
‫‪⎤⎦ 96‬‬ ‫‪x‬‬ ‫⇒‬
‫‪x 97‬‬ ‫‪x 385‬‬ ‫×‬
‫‪⎡⎣ ⎤⎦x 97‬‬ ‫و ⎦⎤ﻝﺪﻳﻨﺎ ‪(ii ) ≡ ⎡⎣:‬‬ ‫⇒‬
‫‪.‬‬ ‫⎦⎤ ‪x ≡ 211 ⎡⎣97‬‬ ‫ﻣﻦ ) ‪ ( i‬و ) ‪ ( ii‬ﻧﺴﺘﻨﺘﺞ أن ‪:‬‬

‫وﻣﻨﻪ ﻓﺈن ⎤ ‪ . x 35 ≡ 2385 ⎡97‬أي ‪:‬‬


‫⎦ ⎣‬
‫⎦⎥⎤‪x 35 ≡ 211×35 ⎡⎢⎣97‬‬ ‫⎦⎤ ‪ . x ≡ 211 ⎡⎣97‬إذن ‪:‬‬ ‫ﻋﺪدا ﺹﺤﻴﺤﺎ ﻃﺒﻴﻌﻴﺎ ﺏﺤﻴﺚ ‪:‬‬ ‫‪x‬‬ ‫‪ .2‬ﻝﻴﻜﻦ‬

‫⎦⎤‪ . x 35 ≡ ( 296 ) × 2 ⎡⎣97‬وﺏﻤﺎ أن ‪ 97 :‬أوﻝﻲ و ‪97 ∧ 2 =1‬‬


‫‪4‬‬
‫‪. 296‬‬ ‫⎦⎤ ‪≡ 1⎡⎣97‬‬ ‫‪ ،‬ﻓﺈﻧﻪ ‪ ،‬ﺣﺴﺐ ﻣﺒﺮهﻨﺔ ﻓﻴﺮﻣﺎ ‪ ،‬ﻝﺪﻳﻨﺎ ‪:‬‬

‫‪ x‬ﺣﻞ ﻝﻠﻤﻌﺎدﻝﺔ ) ‪( F‬‬ ‫⎦⎤‪x 35 ≡ 2 ⎡⎣97‬‬ ‫) ‪( 296‬‬


‫‪4‬‬
‫‪.‬‬ ‫‪ .‬أي ‪:‬‬ ‫وﻣﻨﻪ ﻧﺴﺘﻨﺘﺞ أن ‪:‬‬ ‫إذن ‪≡ 1⎡⎣97 ⎤⎦ :‬‬
‫‪⇔x‬‬
‫‪ . ( F ) +‬وﺏﻤﺎ×أن ‪ ، 211 = 2048 = 97 21 11 :‬ﻓﺈن ‪. 211 ≡ 11⎡97 ⎤ :‬‬
‫⎦ ⎣‬
‫‪ .3‬ﻝﺪﻳﻨﺎ ‪≡ 211 ⎡⎣97⎤⎦ :‬‬
‫⎤ ‪. ( F ) ⇔ x ≡ 11 ⎡97‬‬ ‫= ‪k⇔ ` ∃/ x ∈11 97k‬‬ ‫‪+‬‬
‫وﺏﻨﺎء ﻋﻠﻴﻪ ﻓﺈن ‪:‬‬
‫⎦ ⎣‬

‫ﻧﻘﻂ (‬ ‫) ‪10‬‬ ‫‪C)vhhhhhhhhhhhhhhhRJc„J)Éhhhhhhhhhhhhhhhhhhhhhhhhhhhhhhhhhhc†T„J‬‬

‫‪. f ( x ) = 2x − e − x‬‬
‫‪2‬‬
‫‪ .I‬ﻝﺘﻜﻦ ‪ f‬اﻝﺪاﻝﺔ اﻝﻌﺪدﻳﺔ ﻝﻠﻤﺘﻐﻴﺮ اﻝﺤﻘﻴﻘﻲ ‪ x‬اﻝﻤﻌﺮﻓﺔ ﻋﻠﻰ ‪ \ +‬ﺏﻤﺎ ﻳﻠﻲ ‪:‬‬

‫‪JG JG J‬‬
‫(‬ ‫)‬
‫‪ (C‬اﻝﻤﻨﺤﻨﻰ اﻝﻤﻤﺜﻞ ﻝﻠﺪاﻝﺔ ‪ f‬ﻓﻲ ﻣﻌﻠﻢ ﻣﺘﻌﺎﻣﺪ ﻣﻤﻨﻈﻢ ‪. O , i , j‬‬ ‫)‬ ‫وﻝﻴﻜﻦ‬

‫‪lim ( f ( x ) − 2x ) = xlim‬‬ ‫‪lim − e‬‬


‫‪2‬‬
‫ﺣﻴﺚ ‪ t = − x 2 :‬و ∞ ‪. t → −‬‬ ‫‪− e −x‬‬ ‫‪t‬‬
‫‪ .1‬أ‪0= -‬‬
‫∞‪x →+‬‬ ‫∞‪x →+‬‬ ‫∞‪→+‬‬ ‫∞‪t →−‬‬
‫‪. y = 2x‬‬ ‫وﻣﻦ ﻧﺴﺘﻨﺘﺞ أن ) ‪ (C‬ﻳﻘﺒﻞ ﻣﻘﺎرﺏﺎ ﻣﺎﺋﻼ ‪ ،‬ﺏﺠﻮار ∞‪ ، +‬ﻣﻌﺎدﻝﺘﻪ‬

‫‪.‬‬ ‫(‬
‫‪f ′ ( x ) = 2x − e − x‬‬
‫‪2‬‬
‫‪)′‬‬ ‫‪2 2xe −=x‬‬
‫‪2‬‬
‫‪2 1+ e‬‬ ‫‪x2‬‬
‫(‬‫> ب‪− -‬ﻝﻴﻜﻦ ‪+ +‬‬
‫\ ∈ ‪ . x‬ﻝﺪﻳﻨﺎ=‪0 :‬‬ ‫)‬

‫ﻣﻌﺮﻓﺔ ﻣﻦ اﻝﻤﺠﺎل‬ ‫‪f‬‬ ‫‪−1‬‬


‫ﺝـ‪ -‬ﻝﺪﻳﻨﺎ ‪ f :‬ﻣﺘﺼﻠﺔ وﺕﺰاﻳﺪﻳﺔ ﻗﻄﻌﺎ ﻋﻠﻰ اﻝﻤﺠﺎل ⎣⎡ ∞‪ . ⎡⎣0, +‬إذن ‪ f :‬ﺕﻘﺒﻞ داﻝﺔ ﻋﻜﺴﻴﺔ‬

‫(‬ ‫⎢⎣‬ ‫∞‪x →+‬‬‫)‬


‫‪ f ⎡⎣0, +∞ ⎡⎣ = ⎡ f ( 0 ) , lim f ( x ) ⎡ = ⎡⎣ −1,0‬ﻧﺤﻮ اﻝﻤﺠﺎل ⎣⎡ ∞‪. ⎡⎣0, +‬‬
‫⎢⎣‬
‫ﻓﻲ اﻝﻤﺠﺎل ⎣⎡ ∞‪. ⎡⎣0, +‬‬ ‫وﺏﻤﺎ أن ⎣⎡ ‪ ، 0 ∈ ⎡⎣ −1,0‬ﻓﺈن اﻝﻤﻌﺎدﻝﺔ ‪ f ( x ) = 0‬ﺕﻘﺒﻞ ﺣﻼ وﺣﻴﺪا ‪α‬‬

‫‪))))))))))))))))’hhhhhhhhhhhhpKc)‡„t)Kc„K‚R)’KV„J‬‬ ‫‪7‬‬ ‫‪’[}n„J‬‬


‫‪e −1‬‬
‫‪ . f‬وﺣﺴﺐ ﻣﺒﺮهﻨﺔ اﻝﻘﻴﻢ اﻝﻮﺱﻴﻄﻴﺔ ‪ ،‬ﻧﺴﺘﻨﺘﺞ‬ ‫‪( 0) × f (1) < 0‬‬ ‫= ‪ . f (1) = 1 − e −1‬إذن ‪:‬‬
‫‪e‬‬
‫وﻝﺪﻳﻨﺎ ‪ f >( 0 ) = −1 <0 :‬و ‪0‬‬

‫‪.‬‬ ‫‪0 < α <1‬‬ ‫أن ‪:‬‬


‫د‪ f -‬ﺕﺰاﻳﺪﻳﺔ ﻋﻠﻰ اﻝﻤﺠﺎل ⎦⎤‪ . ⎡⎣ 0,1‬إذن ‪ :‬ﻝﻜﻞ ⎦⎤‪ ، x ∈ ⎡⎣0,1‬ﻝﺪﻳﻨﺎ ‪:‬‬
‫‪x ∈ ⎡⎣0,α ⎡⎣ ⇒ x α‬‬ ‫⇒ ‪<f ( x ) f‬‬
‫) ‪(α‬‬ ‫‪f (<x ) 0‬‬ ‫⇒‬ ‫<‬
‫‪x ∈ ⎤⎦α ,1⎤⎦ ⇒ α < x‬‬ ‫⇒( ‪f‬‬
‫⇒‪α ) f ( x ) < 0 f (x‬‬
‫)‬ ‫<‬
‫‪f (α ) = 0‬‬
‫‪∀x ∈ ⎡⎣0,α ⎤⎦ : f ( x ) ≤ 0‬‬ ‫وﺏﺎﻝﺘﺎﻝﻲ ﻓﺈن ‪:‬‬
‫‪∀x ∈ ⎡⎣α ,1⎤⎦ : f ( x ) ≥ 0‬‬
‫‪α ≈ 0,4‬‬ ‫‪ .2‬إﻧﺸﺎء اﻝﻤﻨﺤﻨﻰ ) ‪: (C‬‬

‫‪))))))))))))))))’hhhhhhhhhhhhpKc)‡„t)Kc„K‚R)’KV„J‬‬ ‫‪8‬‬ ‫‪’[}n„J‬‬


‫اﻝﻤﻌﺮﻓﺘﻴﻦ ﻋﻠﻰ ‪ \ +‬ﺏﻤﺎ ﻳﻠﻲ ‪:‬‬ ‫‪x‬‬ ‫ﻝﻠﻤﺘﻐﻴﺮ اﻝﺤﻘﻴﻘﻲ‬ ‫‪ .II‬ﻧﻌﺘﺒﺮ اﻝﺪاﻝﺘﻴﻦ اﻝﻌﺪدﻳﺘﻴﻦ ‪ ϕ‬و ‪g‬‬
‫⎧‬ ‫‪1 x −t 2‬‬
‫‪⎪ϕ ( x ) = ∫0 e dt‬‬ ‫‪; x >0‬‬
‫‪g ( x ) = x 2 − ∫0 e −t dt‬‬
‫‪x‬‬ ‫‪2‬‬
‫⎨‬ ‫‪x‬‬ ‫و‬
‫⎪‬
‫⎩‬ ‫‪ϕ‬‬ ‫‪(0) = 1‬‬
‫‪ .1‬أ‪ -‬اﻟﻄﺮﻗﺔ اﻷوﻟﻰ ‪:‬‬
‫‪ . ζ : x 6 e‬ﻝﺪﻳﻨﺎ ‪ ζ :‬داﻝﺔ ﻣﺘﺼﻠﺔ ﻋﻠﻰ اﻝﻤﺠﺎل ⎦⎤ ‪ . ⎡⎣0,x‬ﺣﺴﺐ ﺧﺎﺻﻴﺔ اﻟﻘﻴﻤﺔ اﻟﻤﺘﻮﺱﻄﺔ‪،‬‬ ‫ﻝﻴﻜﻦ ‪ . x > 0‬ﻧﻌﺘﺒﺮ اﻝﺪاﻝﺔ‬ ‫‪−x 2‬‬

‫‪1 x −t 2‬‬ ‫‪1 x −t 2‬‬


‫‪. ∃c ∈ ⎤ 0, x ⎡ /‬‬ ‫‪−c 2‬‬
‫= ) ‪ . ∃c ∈ ⎤⎦ 0, x ⎡⎣ / ζ (c‬أي ‪:‬‬
‫‪x ∫0‬‬
‫=‬
‫‪x − 0 ∫0‬‬
‫⎦‬ ‫⎣‬ ‫‪e‬‬ ‫‪dt‬‬ ‫‪e‬‬ ‫ﻝﺪﻳﻨﺎ ‪e dt :‬‬
‫اﻟﻄﺮیﻘﺔ اﻟﺜﺎﻧﻴﺔ ‪:‬‬
‫‪∈ \+ : F ( x ) = ∫0 e −t dt‬‬
‫‪x‬‬
‫‪. ∀x‬‬
‫‪2‬‬
‫ﻧﻀﻊ ‪:‬‬
‫‪ F‬داﻝﺔ ﻣﺘﺼﻠﺔ ﻋﻠﻰ ‪ \ +‬وﻗﺎﺏﻠﺔ ﻝﻼﺵﺘﻘﺎق ﻋﻠﻰ *‪ . \ +‬ﻝﻴﻜﻦ *‪ . x ∈ \ +‬ﻝﺪﻳﻨﺎ ‪ F :‬داﻝﺔ ﻣﺘﺼﻠﺔ ﻋﻠﻰ ⎦⎤ ‪ ⎡⎣0,x‬وﻗﺎﺏﻠﺔ ﻝﻼﺵﺘﻘﺎق‬
‫ﻋﻠﻰ ⎣⎡ ‪ . ⎤⎦ 0,x‬ﺣﺴﺐ ﻣﺒﺮ هﻨﺔ اﻝﺘﺰاﻳﺪات اﻝﻤﻨﺘﻬﻴﺔ ‪−،‬ﻧﺴﺘﻨﺘﺞ أن ‪. ∃c ∈ ⎤⎦ 0, x ⎡⎣ / F ( x ) − F ( 0 ) = F ′ (c ) ( x 0 ) :‬‬

‫‪. F ( x ) = e −c‬أي ‪:‬‬ ‫‪ . F ′ (c ) = e −c‬وﻣﻨﻪ ﻓﺈن ‪:‬‬ ‫‪∀u ∈\ +* : F ′ (u ) = e −u‬‬


‫‪2‬‬ ‫‪2‬‬ ‫‪2‬‬
‫‪x‬‬ ‫‪ .‬إذن ‪:‬‬ ‫وﻝﺪﻳﻨﺎ ‪:‬‬
‫*‬ ‫‪1 x −t 2‬‬ ‫‪c2‬‬
‫‪x‬‬
‫‪∀x ∈ \ +‬‬
‫‪x ∫0‬‬ ‫‪∫0‬‬
‫‪2‬‬ ‫‪2‬‬
‫‪.‬‬ ‫‪,‬‬ ‫‪c ∃⎤⎦ 0, x∈⎡⎣ :‬‬ ‫‪e dt e‬‬ ‫‪ .‬وﺏﺎﻝﺘﺎﻝﻲ ﻓﺈن ‪=:‬‬ ‫‪e −t dt = e −c x‬‬
‫‪1‬‬
‫‪∫0 e‬‬ ‫ب‪ -‬ﻣﻦ أﺝﻞ ‪ ، x = 1‬ﺣﺴﺐ اﻝﺴﺆال اﻝﺴﺎﺏﻖ ‪ ،‬ﻝﺪﻳﻨﺎ ‪e −=c :‬‬
‫‪−t 2‬‬ ‫‪2‬‬
‫‪ . ∃c ∈ ⎤⎦ 0,1⎡⎣ /‬وﻝﺪﻳﻨﺎ ‪:‬‬ ‫‪dt‬‬
‫‪1‬‬
‫‪∫0e‬‬
‫‪−t 2‬‬
‫‪ . 0 < c < 1 ⇒ c 2−< 0 ⇒ e −c‬إذن ‪dt < 1 :‬‬
‫‪2‬‬
‫‪.‬‬ ‫< ‪1‬‬

‫(‬ ‫)‬
‫‪α‬‬ ‫‪α‬‬ ‫‪α‬‬ ‫‪α‬‬ ‫‪α‬‬
‫‪g (α ) = ∫0 f‬‬ ‫‪t dt( . ∫0) f (t ) dt = ∫0 2t − e −t dt‬‬
‫‪2‬‬
‫⎤ ‪⎡t 2‬‬
‫‪⎣ ⎦0‬‬ ‫‪0‬‬
‫‪−t‬‬
‫=‪e‬‬ ‫‪dt‬‬
‫‪2‬‬
‫‪α2‬‬
‫‪0‬‬
‫‪−‬‬ ‫∫‬
‫‪e t dt‬‬
‫‪2‬‬
‫=ﻝﺪﻳﻨﺎ‪g (α ) :‬‬ ‫‪ .2‬أ ‪-‬‬ ‫‪−‬‬
‫‪x‬‬
‫و‬ ‫‪ g‬ﻗﺎﺏﻠﺔ ﻝﻼﺵﺘﻘﺎق ﻋﻠﻰ ‪\ +‬‬ ‫داﻝﺔ ﻣﺘﺼﻠﺔ ﻋﻠﻰ ‪ . \ +‬إذن ‪:‬‬ ‫‪f‬‬ ‫‪ . ∀x‬وﻝﺪﻳﻨﺎ‬ ‫(‪∈ \+ : g ( x ) = ∫0 f t dt‬‬ ‫)‬
‫ب‪ -‬ﻝﺪﻳﻨﺎ ‪:‬‬

‫‪. ∀x‬‬ ‫) ‪∈ \+ : g ′ ( x ) = ( ∫0 f (t ) dt ) ' = f ( x‬‬


‫‪x‬‬
‫ﻝﺪﻳﻨﺎ ‪:‬‬

‫‪ ،‬إذن ‪ g‬ﺕﺰاﻳﺪﻳﺔ ﻗﻄﻌﺎ‬ ‫ﺝـ‪ -‬ﻧﻌﻠﻢ أن ‪ g‬داﻝﺔ ﻣﺘﺼﻠﺔ ﻋﻠﻰ اﻝﻤﺠﺎل ⎦⎤‪ ⎡⎣α ,1‬و أن ‪∀x ∈ ⎤⎦α ,1⎡⎣ : g ′ ( x ) = f ( x ) > 0 :‬‬
‫‪α‬‬
‫ﻋﻠﻰ اﻝﻤﺠﺎل ⎤‪ . ⎡α ,1‬وﻝﺪﻳﻨﺎ‪ ، g (1) = 12 − ∫ e −t dt 1 ∫=e −t dt− > 0 :‬ﻷن‪ ،)∫0 e −t dt < 1 :‬و (‪g (α ) = ∫ f t dt‬‬
‫‪1‬‬ ‫‪2‬‬ ‫‪1‬‬ ‫‪2‬‬ ‫‪1‬‬ ‫‪2‬‬

‫‪0‬‬ ‫⎣‬ ‫⎦‬‫‪0‬‬ ‫‪0‬‬

‫‪.‬‬ ‫‪g (α ) < 0‬‬ ‫‪ ، ∀t ∈ ⎡⎣0,α ⎡⎣ : f‬ﻓﺈن ‪:‬‬ ‫‪(t ) < 0‬‬ ‫وﺣﻴﺚ أن ‪:‬‬
‫ﻓﻲ اﻝﻤﺠﺎل ⎣⎡‪. ⎤⎦α ,1‬‬ ‫‪ g ( x ) = 0‬ﺕﻘﺒﻞ ﺣﻼ وﺣﻴﺪا ‪β‬‬ ‫ﺣﺴﺐ ﻣﺒﺮهﻨﺔ اﻝﻘﻴﻢ اﻝﻮﺱﻴﻄﻴﺔ‪ ،‬ﻧﺴﺘﻨﺘﺞ أن اﻝﻤﻌﺎدﻝﺔ‬
‫‪ .‬ﻝﺪﻳﻨﺎ ‪:‬‬ ‫‪ .3‬أ‪ -‬ﻝﻴﻜﻦ ‪ x > 0‬و ⎦⎤ ‪t ∈ ⎡⎣0,x‬‬
‫‪0 <t < x‬‬ ‫⇒‪−‬‬ ‫⇒‪e − x‬‬ ‫⇒‬ ‫‪xe< − x − <⇒ e t dt< x‬‬
‫‪x‬‬ ‫‪x‬‬ ‫‪2‬‬ ‫‪x‬‬
‫‪∫<0 e‬‬ ‫‪dt < ∫ e −t dt x‬‬ ‫∫‬
‫‪2‬‬ ‫‪2‬‬ ‫‪2‬‬ ‫‪2‬‬ ‫‪2‬‬
‫‪x2 < t2‬‬ ‫‪0−‬‬ ‫‪< e −t‬‬ ‫‪1‬‬ ‫‪x‬‬
‫‪0‬‬ ‫‪0‬‬
‫‪1 x −t 2‬‬
‫‪2‬‬
‫‪e <−x 2 <⇒ϕ ( x ) < 1‬‬
‫‪x ∫0‬‬
‫‪−x‬‬
‫‪0 <t < x‬‬ ‫⇒‪e‬‬ ‫‪<e dt‬‬ ‫‪1‬‬

‫ﻣﺘﺼﻠﺔ ﻋﻠﻰ اﻝﻴﻤﻴﻦ ﻓﻲ ‪. 0‬‬ ‫‪. xlim‬وﻣﻨﻪ ﻓﺈن ‪ϕ‬‬


‫‪→0‬‬
‫)‪ϕ ( x ) = 1 = ϕ ( 0‬‬
‫‪+‬‬
‫‪ ،‬ﻓﺈن ‪:‬‬ ‫ﺏﻤﺎ أن ‪ lim e −x = 1 :‬و ‪lim 1 = 1‬‬
‫‪x →0‬‬ ‫‪+‬‬
‫‪x →0‬‬ ‫‪+‬‬
‫‪2‬‬

‫ب‪ -‬ﻝﻴﻜﻦ ‪x > 0‬‬


‫‪2‬‬
‫‪.‬اﻝﺪاﻝﺘﺎن ‪ t 6 t‬و ‪ t 6 e −t‬ﻣﺘﺼﻠﺘﺎن وﻗﺎﺏﻠﺘﺎن ﻝﻼﺵﺘﻘﺎق ﻋﻠﻰ اﻝﻤﺠﺎل ⎦⎤ ‪ ⎡⎣0,x‬و داﻝﺘﺎهﻤﺎ اﻝﻤﺸﺘﻘﺘﺎن ‪ t 6 1‬و‬
‫‪2‬‬
‫ﻣﺘﺼﻠﺘﺎن ﻋﻠﻰ اﻝﻤﺠﺎل ⎣⎡ ‪ . ⎤⎦ 0,x‬ﺣﺴﺐ ﺕﻘﻨﻴﺔ اﻝﻤﻜﺎﻣﻠﺔ ﺏﺎﻷﺝﺰاء ‪ ،‬ﻝﺪﻳﻨﺎ ‪:‬‬ ‫‪t 6 −2te −t‬‬

‫‪)))))))))))))))))’hhhhhhhhhhhhpKc)‡„t)Kc„K‚R)’KV„J‬‬ ‫‪9‬‬ ‫‪’[}n„J‬‬


1 x −t 2 1 x 1 ⎛⎡
( )dt ⎞⎟⎠
x
ϕ (x ) = t2 ⎤
x
∫ e dt = ∫ t ′ e −t dt ∫
2 t2 −
te ×t 2te =
x 0 x 0 x ⎜⎝ ⎣⎢ ⎦⎥ 0 0

1
ϕ ( x ) = ⎛⎜ xe − x + ∫ t 2e −t dt ⎞⎟ e
x 2 x t 2e t 2dt= −
x ∫0
2 2 x2 −
+
x⎝ 0 ⎠

: ϕ ( x ) = e −x +
2 2 x t 2e −t 2dt
x ∫0
*
∀x ∈ \ + : ‫وﺏﺎﻝﺘﺎﻝﻲ ﻓﺈن‬
* x
∫0
2 2
: ‫ وﻝﺪﻳﻨﺎ‬، \ + ‫ ﻗﺎﺏﻠﺔ ﻝﻼﺵﺘﻘﺎق ﻋﻠﻰ‬x 6 t 2e −t dt ‫ ﻓﺈن اﻝﺪاﻝﺔ‬، \ + ‫ داﻝﺔ ﻣﺘﺼﻠﺔ ﻋﻠﻰ‬t 6 t 2e −t ‫ ﺏﻤﺎ أن‬-‫ﺝـ‬

: ⎛⎜ t 2e −t dt ⎞⎟
x
∫0
* 2 x2
∀x ∈ \ + x 2=e −
⎝ ⎠
* * * 2
ϕ ‫ ﻓﺈن اﻝﺪاﻝﺔ‬، \ + ‫ ﻗﺎﺏﻠﺘﻴﻦ ﻝﻼﺵﺘﻘﺎق ﻋﻠﻰ‬x 6 e − x ‫ و‬x 6
2
، \ + ‫ ﻣﻦ‬x ‫وﻝﻜﻞ‬ \+ ‫ﻗﺎﺏﻠﺔ ﻝﻼﺵﺘﻘﺎق ﻋﻠﻰ‬ ‫وﺏﻤﺎ أن اﻝﺪاﻝﺘﻴﻦ‬
x
⎞′ ⎛ 2 ⎞′ x 2 −t 2 ⎞′
ϕ ′ ( x ) = ⎜e −x + 2 ∫0 t 2e −t dt ⎟
⎛ x 2 ⎛ x 2 −t 2

2

2 2
2xe − x = t −
e dt + t+ e dt : ‫ﻝﺪﻳﻨﺎ‬
⎝ x ⎠
⎜x ⎟ 0
⎝ ⎠ x ⎜⎝ 0 ⎟

ϕ ′ ( x ) = −2xe −x − 22 ∫ t 2e −t dt +2 ⎛⎜ x 2e −x ⎞⎟ = − 2 x 2 −t 2
2 x 2 2

x 2 ∫0
t e dt
x 0 x ⎝ ⎠

∀x ∈ \+ : ϕ ′( x ) = 2− x t 2e −t 2dt

*
: ‫وﺏﺎﻝﺘﺎﻝﻲ ﻓﺈن‬
x2 0
=22 ∫− t 2e −t dt
x
: ϕ′(x )
2
ϕ ‫ إذن‬، ∀x ∈ \ + ‫<⎣⎡ وأن‬0,1⎤⎦ ‫داﻝﺔ ﻣﺘﺼﻠﺔ ﻋﻠﻰ اﻝﻤﺠﺎل‬
*
‫ﺕﻨﺎﻗﺼﻴﺔ‬ 0 ϕ ‫ ﻧﻌﻠﻢ أن‬-‫د‬
x 0
ϕ (1) > e −1 > 0 ‫ ﻷن‬ϕ ( ⎡⎣0,1⎤⎦ ) = ⎡ϕ (1) ,ϕ ( 0 ) ⎤ = (1) ,1 ϕ 0,1 :⎡⎣ ‫⎦⎤ إذن‬.⎡⎡⎣0,1⎤⎦ ‫⊂ ﻋﻠﻰ اﻝﻤﺠﺎل‬
‫⎤ ﻗﻄﻌﺎ‬
2
( -‫ أ‬-3 :‫)أﻧﻈﺮ‬
⎣ ⎦ ⎣ ⎦

ϕ ⎛⎜ ⎡⎣0,1⎤⎦ ⎞⎟ ⊂ 0,1 :⎡ ‫ﺧﻼﺹﺔ‬





⎝ ⎠
: ‫ ﻝﺪﻳﻨﺎ‬. t ∈ ⎡⎣0, x ⎤⎦ ‫ وﻝﻴﻜﻦ‬. x ∈ \ + ‫ ﻝﻴﻜﻦ‬-‫ أ‬.4
x
x x x ⎡t 3 ⎤
−t ≤ 0 ⇒ e
2 −t 2
1 ≤t e⇒ t 2 −t 2 2
∫≤0 t
2
e ⇒
dt t2
∫0 t
2
dt ≤t e 2 −t 2
dt ⇒⎢ ⎥ ∫ ≤
0 ⎢⎣ 3 ⎥⎦ 0
x
2e −t 2dt x3
⇒ ∫0 t ≤
3
x x3 +* 2 x 2 −t 2
( )
2
. ∀x ∈ \+ : ∫0 t 2e −t dt
3
≤ ‫و‬ ∀ x ∈ \ : ϕ ′ x
x 2 ∫0
= t e dt : ‫ ﻝﺪﻳﻨﺎ‬-‫ب‬
2 x3 2 2
. ∀x ∈ ⎤⎦ 0,1⎡⎣ : ϕ ′ ( x ) ≤ × ≤ x≤ : ‫إذن‬
x 2
3 3 3
1
. ϕ (x ) = x ⇔ 2
= 0 ⇔−g ( x ) = 0 : ∫‫ ﻝﺪﻳﻨﺎ‬.− x ∈ \+* ‫ ﻝﻴﻜﻦ‬-‫ﺝـ‬
x x x
∫ x =∫ ⇔
2 2 2
e −t dt
e −t dt x 2 x = e t⇔
dt

x 0 0 0

: ϕ (x ) = x ( 0)
*
. ∀x ∈ \ + g x : ‫وﺏﺎﻝﺘﺎﻝﻲ ﻓﺈن‬ ⇔

))))))))))))))’hhhhhhhhhhhhpKc)‡„t)Kc„K‚R)’KV„J 10 ’[}n„J
‫⎧‬ ‫‪2‬‬ ‫اﻝﻤﻌﺮﻓﺔ ﺏﻤﺎ ﻳﻠﻲ ‪:‬‬ ‫`∈‪(u n )n‬‬ ‫‪ .5‬ﻧﻌﺘﺒﺮ اﻝﻤﺘﺘﺎﻝﻴﺔ اﻝﻌﺪدﻳﺔ‬
‫= ‪⎪ u0‬‬
‫⎨‬ ‫‪3‬‬
‫‪⎪u‬‬
‫‪⎩ n +1‬‬
‫) ‪= ϕ (u n‬‬ ‫`∈ ‪; n‬‬
‫‪.‬‬ ‫‪ . u 0 = 2‬إذن ‪0 ≤ u 0 ≤ 1 :‬‬ ‫ﻣﻦ أﺝﻞ ‪ ، n = 0‬ﻝﺪﻳﻨﺎ‬ ‫أ‪-‬‬
‫‪3‬‬
‫‪:‬‬ ‫` ∈ ‪ . n‬ﻧﻔﺘﺮض أن ‪ 0 ≤ u n ≤ 1‬وﻧﺒﻴﻦ أن ‪0 ≤ u n +1 ≤ 1‬‬ ‫‪ 9‬ﻝﻴﻜﻦ‬
‫‪ ،‬ﻓﺈن ‪. 0 ≤ u n +1 ≤ 1‬‬ ‫⎦⎤‪ . ϕ ( ⎡⎣0,1⎤⎦ ) ⊂ ⎡⎣0,1‬إذن ‪ . ∀x ∈ ⎡⎣0,1⎤⎦ : 0 ≤ ϕ ( x ) ≤ 1 :‬وﺏﻤﺎ أن ‪0 ≤ u n ≤ 1‬‬ ‫ﻝﺪﻳﻨﺎ ‪:‬‬

‫‪.‬‬ ‫‪∀n ∈ ` : 0 ≤ u n ≤ 1‬‬ ‫‪ 9‬وﺏﺎﻝﺘﺎﻝﻲ ﻓﺈﻧﻪ ﺣﺴﺐ ﻣﺒﺪأ اﻝﺘﺮﺝﻊ ‪ ،‬ﻝﺪﻳﻨﺎ ‪:‬‬

‫داﻝﺔ ﻣﺘﺼﻠﺔ ﻋﻠﻰ‬ ‫‪ϕ‬‬‫ب‪ -‬ﻝﻴﻜﻦ ‪ x‬ﻣﻦ اﻝﻤﺠﺎل ⎤‪ ، ⎡0,1‬ﻧﻌﺘﺒﺮ اﻝﻤﺠﺎل ⎦⎤ ‪ ⎡⎣a,b‬ﺣﻴﺚ ) ‪ a = min ( x , β‬و ) ‪ . b = max ( x , β‬ﻝﺪﻳﻨﺎ ‪:‬‬
‫⎣‬ ‫⎦‬
‫اﻝﻤﺠﺎل ⎦⎤ ‪ ⎡⎣a,b‬وﻗﺎﺏﻠﺔ ﻝﻼﺵﺘﻘﺎق ﻋﻠﻰ اﻝﻤﺠﺎل ⎣⎡ ‪ . ⎤⎦a,b‬ﺣﺴﺐ ﻣﺒﺮهﻨﺔ اﻝﺘﺰاﻳﺪات اﻝﻤﻨﺘﻬﻴﺔ ‪ ،‬ﻳﻮﺝﺪ ﻋﻠﻰ اﻷﻗﻞ ﻋﺪد ﺣﻘﻴﻘﻲ ‪ c‬ﺏﺤﻴﺚ ‪:‬‬
‫‪.ϕ ( x‬‬ ‫‪ a < c < b‬و ) ‪ .ϕ ( x ) − ϕ ( β ) = ϕ ′ (c ) ( x − β‬أي ‪) − ϕ ( β ) = ϕ ′ (c ) ( x − β ) :‬‬
‫وﻝﺪﻳﻨﺎ ‪ . g ( β ) = 0 :‬إذن ‪ . ϕ ( β ) = β :‬وﻣﻨﻪ ﻓﺈن ‪. ϕ ( x ) − β = ϕ ′ (c ) ( x − β ) :‬‬
‫‪2‬‬ ‫‪2‬‬
‫‪. ϕ (x‬‬ ‫‪) − β = ϕ ′ (c ) x − β‬‬ ‫‪3‬‬
‫وﺏﻤﺎ أن ‪ ، 0 < a < c < b < 1 :‬ﻓﺈن ‪ . ϕ ′ (c ) ≤ :‬ﺙﻢ ﻧﺤﺼﻞ ﻋﻠﻰ ‪x − β :‬‬
‫≤‬
‫‪3‬‬
‫‪2‬‬
‫‪. ∀x ∈ ⎡⎣0,1⎤⎦ : ϕ ( x ) − β ≤ x − β‬‬ ‫إذن ‪:‬‬
‫‪3‬‬
‫‪2‬‬ ‫‪2‬‬
‫ﻝﻴﻜﻦ ` ∈ ‪ . n‬ﻝﺪﻳﻨﺎ ‪ . u n ∈ ⎡⎣0,1⎤⎦ :‬إذن ‪. ϕ (u n ) − β ≤ u n − β :‬أي ‪. u n +1 − β ≤ u n − β :‬‬
‫‪3‬‬ ‫‪3‬‬
‫‪2‬‬
‫وﻣﻨﻪ ﻓﺈن ‪ . ∀n ∈ ` : u n +1 − β ≤ u n − β :‬وﻝﺪﻳﻨﺎ ‪:‬‬
‫‪3‬‬

‫وذﻝﻚ ﺏﻌﺪ ﺽﺮب ﻃﺮﻓﻲ اﻝﻤﺘﻔﺎوﺕﺎت اﻝﺴﺎﺏﻘﺔ‪ ،‬ﻋﻠﻰ اﻝﺘﻮاﻝﻲ‪ ،‬ﻃﺮﻓﺎ ﺏﻄﺮف ‪ ،‬وﺏﻌﺪ اﻻﺧﺘﺰال‬

‫‪ÉKhhhhhhhhhhhhhhhh[„J)ahhhhhhhhhhh†[†)))))))))))))))))’hhhhhhhhhhhhpKc)‡„t)Kc„K‚R)’KV„J‬‬ ‫‪11‬‬ ‫‪’[}n„J‬‬


‫‪u 0 − β = 2 − β ≤ 1− 0 ≤ 1‬‬ ‫وﻝﺪﻳﻨﺎ ‪:‬‬
‫‪3‬‬

‫‪n‬‬
‫⎞‪⎛2‬‬
‫⎟ ⎜ ≤ ‪∀n ∈ ` : u n − β‬‬ ‫وﺏﺎﻝﺘﺎﻝـــــــﻲ ﻓﺈن ‪:‬‬
‫⎠‪⎝3‬‬
‫و ﻧﺘﺤﻘﻖ ﻣﻦ هﺬﻩ اﻝﻌﺒﺎرة ﺏﺎﻝﺘﺮﺝﻊ‪.‬‬
‫‪n‬‬ ‫‪n‬‬
‫⎞‪⎛2‬‬ ‫‪2‬‬ ‫⎞‪⎛2‬‬
‫‪ . nlim‬وﺣﺴﺐ ﻣﺼﺎدﻳﻖ اﻝﺘﻘﺎرب‪ ،‬ﻓﺈن `∈‪(u n )n‬‬ ‫⎜‬ ‫⎟‬ ‫ﺝـ‪ -‬ﻝﺪﻳﻨﺎ ‪ ∀n ∈ ` : u n − β ≤ ⎜ ⎟ :‬و ‪ . −1 < < 1‬إذن ‪= 0‬‬
‫‪→+∞ 3‬‬
‫⎠ ⎝‬ ‫‪3‬‬ ‫⎠‪⎝3‬‬
‫‪.‬‬ ‫‪lim u‬‬
‫‪n →+∞ n‬‬
‫‪=β‬‬ ‫‪.‬‬ ‫‪β‬‬ ‫ﻣﺘﺘﺎﻝﻴﺔ ﻣﺘﻘﺎرﺏﺔ ﻧﻬﺎﻳﺘﻬﺎ‬

‫‪ÉKhhhhhhhhhhhhhhhh[„J)ahhhhhhhhhhh†[†)))))))))))))))))’hhhhhhhhhhhhpKc)‡„t)Kc„K‚R)’KV„J‬‬ ‫‪12‬‬ ‫‪’[}n„J‬‬


‫اﻟﺜﺎﻧﻴﺔ ﻋﻠﻮم رﻳﺎﺿﻴﺔ‬ ‫ﺗﺼﺤﻴﺢ اﻻﻣﺘﺤﺎن اﻟﻮﻃﻨﻲ اﻟﻤﻮﺣﺪ‬ ‫ﺛﺎﻧﻮﻳﺔ ﻣﺤﻤﺪ اﻟﺨﺎﻣﺲ – اﻟﻘﻨﻴﻄﺮة ‪-‬‬
‫أوب‬ ‫ﻟﻠﺒـﺎآـﻠﻮرﻳــﺎ‬ ‫اﻷﺳﺘﺎذ ﻣﺤﻤﺪ ﻏﺮﻳﺰ‬
‫اﻟﺪورة ‪ -‬اﻻﺳﺘﺪراآﻴﺔ ‪2008 -‬‬ ‫‪http://mathkas.ici.ma‬‬

‫اﻟﺘﻤﺮﻳﻦ اﻷول ‪:‬‬


‫‪1 + 3i‬‬ ‫‪3+i‬‬
‫= ‪z1‬‬ ‫‪z+‬‬ ‫‪ - 1‬ﻟﻴﻜﻦ ‪ r‬اﻟﺘﻄﺒﻴﻖ اﻟﺬي ﻳﺮﺑﻂ آﻞ ﻧﻘﻄﺔ )‪ M(z‬ﺑﺎﻟﻨﻘﻄﺔ ) ‪ M 1 ( z1‬ﺣﻴﺚ ‪:‬‬
‫‪2‬‬ ‫‪2‬‬
‫‪1 + 3i‬‬ ‫‪1 + 3i‬‬
‫و ‪≠1‬‬ ‫‪=1‬‬ ‫ﻟﺪﻳﻨﺎ ‪:‬‬
‫‪2‬‬ ‫‪2‬‬
‫‪3+i‬‬
‫)‪( 3 + i)(1 + i 3‬‬ ‫‪3 +i‬‬ ‫‪2‬‬
‫=‪ω‬‬ ‫أي ‪=i‬‬ ‫=‪ω‬‬ ‫أي‬ ‫=‪ω‬‬ ‫اذن ‪ r‬دوران ﻣﺮآﺰﻩ ‪ Ω‬ذات اﻟﻠﺤﻖ‬
‫‪4‬‬ ‫‪1− i 3‬‬ ‫‪1 + 3i‬‬
‫‪1−‬‬
‫‪2‬‬
‫‪1 + 3i‬‬
‫(‪θ ≡ arg‬‬ ‫و زاوﻳﺘﻪ ]‪) [ 2π‬‬
‫‪2‬‬
‫‪π‬‬
‫]‪θ ≡ [ 2 π‬‬
‫‪3‬‬
‫ﻟﻴﻜﻦ ‪ h‬اﻟﺘﻄﺒﻴﻖ اﻟﺬي ﻳﺮﺑﻂ اﻟﻨﻘﻄﺔ )‪ M(z‬ﺑﺎﻟﻨﻘﻄﺔ ) ‪ M 2 ( z 2‬ﺣﻴﺚ ‪z 2 = −2z + 3i‬‬
‫ﻟﺪﻳﻨﺎ }‪ −2 ∈ \ * − {1‬اذن ‪ h‬ﺗﺤﺎآﻲ ﻣﺮآﺰﻩ )‪ Ω(i‬وﻧﺴﺒﺘﻪ ‪-2‬‬
‫‪ – 2‬أ ‪ -‬ﻟﺘﻜﻦ )’‪ M’(z‬هﻲ ﺻﻮرة )‪ M(z‬ﺑﺎﻟﺘﻄﺒﻴﻖ ‪F‬‬
‫ﻟﺪﻳﻨﺎ ‪F=hor‬‬
‫‪M ‬‬‫‪r‬‬
‫‪→ M1 h → M ' ‬‬
‫‪z ‬‬
‫‪→ z1  → z ' ‬‬
‫‪1 + 3i‬‬ ‫‪3+i‬‬ ‫‪1 + 3i‬‬ ‫‪3+i‬‬
‫((‪z ' = −2‬‬ ‫‪)z +‬‬ ‫‪) +‬‬
‫اذن ‪3i‬‬ ‫‪ z1‬و ‪z ' = −2z1 + 3i‬‬ ‫‪+‬‬ ‫ﻟﺪﻳﻨﺎ‬
‫‪2‬‬ ‫‪2‬‬ ‫‪2‬‬ ‫‪2‬‬
‫أي ‪z '− i = −(1 +i 3)z − 3 i+‬‬ ‫‪z ' = −(1 +i 3)z −‬‬‫اذن ‪( 3 i)+ 3i+‬‬
‫‪z '− i = −(1 +i 3)(z −‬‬
‫)‪i‬‬ ‫أي‬ ‫‪z '− i = (1− i 3)z‬‬
‫‪−‬أي ‪+ i( 1 −i 3)−‬‬
‫‪4π‬‬
‫‪i‬‬
‫)‪−(1 + i 3‬‬ ‫و ﺑﻤﺎ أن ‪2e=3‬‬
‫‪4π‬‬
‫‪i‬‬
‫‪z '− i = 2e‬‬ ‫‪3‬‬
‫ﻓﺈن ‪(z i)−‬‬
‫‪4π‬‬
‫‪i‬‬
‫ب – ﻟﺪﻳﻨﺎ‪z '− i = 2e 3 (z i)−‬‬
‫ﻣﻦ أﺟﻞ ‪ z=i‬ﻟﺪﻳﻨﺎ ‪z’=i‬‬
‫اذن ‪ Ω‬هﻲ اﻟﻨﻘﻄﺔ اﻟﻮﺣﻴﺪة اﻟﺘﻲ ﺗﺤﻘﻖ ‪F(Ω) = Ω‬‬
‫‪ – 3‬أ – ﻟﺪﻳﻨﺎ )‪B=F(A‬‬
‫‪4π‬‬ ‫‪4π‬‬ ‫‪4π‬‬
‫‪i‬‬ ‫‪i‬‬ ‫‪i‬‬
‫‪b = 2e 3 a − 2ie‬‬ ‫‪3‬‬
‫أي ‪+ i‬‬ ‫‪b − i = 2e‬‬ ‫‪3‬‬
‫اذن ‪(a i)−‬‬
‫‪4π‬‬ ‫‪5π‬‬ ‫‪π‬‬
‫‪i‬‬ ‫‪i‬‬ ‫‪−i‬‬
‫‪b = 2e 3 a + 2e‬‬ ‫‪6‬‬
‫ﻓﺎن ‪+ i‬‬ ‫و ﺑﻤﺎ أن ‪−i = e 2‬‬
‫)‪C=F(B‬‬ ‫ﻟﺪﻳﻨﺎ‬
‫‪4π‬‬ ‫‪4π‬‬ ‫‪5‬‬ ‫‪π‬‬
‫‪i‬‬ ‫‪i‬‬ ‫‪i‬‬
‫‪c − i = 2e‬‬ ‫‪3‬‬
‫اذن ) ‪(2e 3 a 2e+ 6‬‬
‫‪2π‬‬ ‫‪π‬‬
‫‪i‬‬ ‫‪i‬‬
‫‪c = 4e 3 a + 4e 6‬‬ ‫اذن ‪i‬‬
D=F(C) ‫ﻟﺪﻳﻨﺎ‬
4π 2π i π
i i
d − i = 2e 3
(4e 3
+ 4e 6 ) ‫اذن‬

i
d = 8ei2 π a + 8e + i ‫أي‬ 2

d = 8a − 7i ‫و ﻣﻨﻪ‬ d = 8a − 8i + i ‫أي‬
.‫ ﻣﺴﺘﻘﻴﻤﻴﺔ‬D ‫ و‬A‫ و‬Ω ‫ب – ﻟﻨﺒﻴﻦ أن اﻟﻨﻘﻂ‬
d − i 8a − 7i − i 8(a − i)
= = ‫ﻟﺪﻳﻨﺎ‬
a −i a −i a −i
d −i
= 8 ‫اذن‬
a −i
d −i
.‫ ﻣﺴﺘﻘﻴﻤﻴﺔ‬D ‫ و‬A‫ و‬Ω ‫ﻋﺪد ﺣﻘﻴﻘﻲ ﻓﺎن اﻟﻨﻘﻂ‬ ‫ﺑﻤﺎ أن‬
a −i
{(B, 4);(C, 2);(D,1)} ‫ ﻣﺮﺟﺢ اﻟﻨﻈﻤﺔ اﻟﻤﺘﺰﻧﺔ‬Ω ‫ج – ﻟﻨﺒﻴﻦ أن‬
JJJG JJJG JJJG G
4ΩB + 2ΩC + ΩD = 0 ‫أي‬
4π 5π 2π π
i i i i
4(b − i) + 2(c − i) + d − i = 8e a + 8e
3 6
+ 8e a + 8e + 8a − 8i
3 6
‫ﻟﺪﻳﻨﺎ‬
2π 4π 5π π
i i i i
= 8a(1 + e 3
+e 3
) + 8(e 6
+ e − i) 6

2π 2π π π π
i −i i i −i
= 8a(1 + e + e ) + 8(e (e + e ) − i)
3 3 2 3 3

2π π
= 8a(1 + 2 cos ) + 8(i.2 cos − i)
3 3
1 1
= 8a(1 + 2(− )) + 8(2i − i) = 0
2 2
{(B, 4);(C, 2);(D,1)} ‫ هﻮ ﻣﺮﺟﺢ اﻟﻨﻈﻤﺔ اﻟﻤﺘﺰﻧﺔ‬Ω ‫و ﺑﺎﻟﺘﺎﻟﻲ‬
G
D ∈ (O, u) ⇔ d = d -‫د‬
⇔ 8a − 7i = 8a + 7i
⇔ 8(a − a) − 14i = 0
⇔ 16 Im ai − 14i = 0
7
⇔ Im a =
8
7
y= ‫ هﻲ اﻟﻤﺴﺘﻘﻴﻢ ذو اﻟﻤﻌﺎدﻟﺔ‬A(a) ‫إذن ﻣﺠﻤﻮﻋﺔ اﻟﻨﻘﻂ‬
8

: ‫اﻟﺘﻤﺮﻳﻦ اﻟﺜﺎﻧﻲ‬
∀(x, y) ∈ \ 2
x * y = x + y − 3xy
(1 − 3x)(1 − 3y) = 1 − 3(x * y) ‫ ﻟﻨﺒﻴﻦ أن‬- ‫ – أ‬1
3(x * y) = 1 − (1 − 3x)(1 − 3y) ‫أي‬
3(x * y) = 3x + 3y − 9xy ‫اذن‬ x * y = x + y − 3xy ‫ﻟﺪﻳﻨﺎ‬
3(x * y) = 1 − (1 − 3x − 3y + 9xy) ‫أي‬
(1 − 3x)(1 − 3y) = 1 − 3(x * y) ‫ وﻣﻨﻪ‬1 − 3x − 3y + 9xy = 1 − 3(x * y) ‫أي‬
∀(x, y) ∈ \ 2 x * y = x + y − 3xy = y + x − 3yx = y * x - ‫ب‬
. ‫اذن * ﺗﺒﺎدﻟﻲ‬
∀(x, y, z) ∈ \ 3
x *(y * z) = (x * y)* z
. ‫اذن * ﺗﺠﻤﻴﻌﻲ‬
(∀x ∈ \ ) x *e = x ⇔ x + e − 3xe = x
⇔ ( ∀x ∈ \ ) e(1 − 3x) = 0
⇔ e = 0 ‫ أو‬1 − 3x = 0
‫ن‬ ‫‪ 1-3x =0‬ﻏﻴﺮ ﻣﻤﻜﻦ ﻟﻜﻞ ‪ x‬ﻣﻦ \ إذن ‪ 0‬هﻮ اﻟﻌﻨﺼﺮ اﻟﻤﺤﺎﻳﺪ ﻟﻠﻘﺎﻧﻮن *‪.‬‬
‫‪ x‬ﻳﻘﺒﻞ ﻣﻤﺎﺛﻼ ﻓﻲ )*‪ ( \ −   ,‬ﻳﻌﻨﻲ ﻳﻮﺟﺪ ’‪ x‬ﻓﻲ ‪ \ −  ‬ﺑﺤﻴﺚ ‪x*x’ =0‬‬
‫‪1‬‬ ‫‪1‬‬
‫‪3‬‬ ‫‪3‬‬
‫‪x * x ' = 0 ⇔ x + x '− 3xx ' = 0‬‬
‫‪⇔ x '(1 − 3x) = − x‬‬
‫‪−x‬‬ ‫‪1‬‬
‫='‪⇔ x‬‬ ‫)ﻻن ≠ ‪( x‬‬
‫‪1 − 3x‬‬ ‫‪3‬‬
‫‪1 ‬‬
‫و ﺑﺎﻟﺘﺎﻟﻲ )*‪ ( \ −   ,‬زﻣﺮة ﺗﺒﺎدﻟﻴﺔ‬
‫‪3‬‬
‫‪ – 2‬أ ‪ϕ(x) = 1 − 3x -‬‬
‫‪1 ‬‬
‫‪∀(x, y) ∈ ( \ −  ) 2‬‬ ‫)‪ϕ(x * y) = 1 − 3(x * y‬‬ ‫ﻟﺪﻳﻨﺎ‬
‫‪3‬‬
‫اذن )‪= (1 − 3x)(1 − 3y‬‬
‫)‪ϕ(x * y) = ϕ(x) × ϕ(y‬‬ ‫أي‬
‫‪1 ‬‬
‫اذن ‪ ϕ‬ﺗﺸﺎآﻞ ﻣﻦ )*‪ ( \ −   ,‬ﻧﺤﻮ )× ‪( \ * ,‬‬
‫‪3‬‬
‫‪1 ‬‬
‫) * \ ∈ ‪(∀y‬‬ ‫? )‪(∃!x ∈ \ −  ) / y = ϕ(x‬‬
‫‪3‬‬
‫‪y = ϕ(x) ⇔ y = 1 − 3x‬‬
‫‪1− y‬‬
‫=‪⇔x‬‬
‫‪3‬‬
‫‪1‬‬
‫≠‪x‬‬ ‫ﻟﻨﺒﻴﻦ أن‬
‫‪3‬‬
‫‪1‬‬
‫=‪x‬‬ ‫ﻧﻔﺘﺮض أن‬
‫‪3‬‬
‫‪1− y 1‬‬
‫‪= ⇔y=0‬‬ ‫) ﺗﻨﺎﻗﺾ ﻣﻊ آﻮن ‪( y ≠ 0‬‬
‫‪3‬‬ ‫‪3‬‬
‫‪1 ‬‬
‫إذن ‪ ϕ‬ﺗﻘﺎﺑﻞ ﻣﻦ )*‪ ( \ −   ,‬ﻧﺤﻮ )× ‪( \ * ,‬‬
‫‪3‬‬
‫‪‬‬ ‫‪1‬‬ ‫‪‬‬ ‫‪1‬‬
‫ب ‪ϕ−1 ( \*+ ) =  −∞,  ⇔ ϕ( −∞, ) = \*+ -‬‬
‫‪‬‬ ‫‪3‬‬ ‫‪‬‬ ‫‪3‬‬
‫‪‬‬ ‫‪1‬‬ ‫‪1‬‬
‫‪∀x ∈  −∞, ‬‬ ‫) (‪ϕ(x) > ϕ‬‬ ‫) ﻻن ‪ ϕ‬ﺗﻨﺎﻗﺼﻴﺔ(‬
‫‪‬‬ ‫‪3‬‬ ‫‪3‬‬
‫‪ϕ(x) ≥ 0‬‬
‫‪‬‬ ‫‪1‬‬
‫إذن ‪(1) ϕ( −∞, ) ⊂ \ *+‬‬
‫‪‬‬ ‫‪3‬‬
‫‪*+‬‬
‫ﻟﻴﻜﻦ ‪ y‬ﻋﻨﺼﺮا ﻣﻦ \‬
‫‪‬‬ ‫‪1‬‬
‫هﻞ ﻳﻮﺟﺪ ‪ x‬ﻣﻦ ‪  −∞, ‬ﺑﺤﻴﺚ ‪ ϕ(x) = y :‬؟‬
‫‪‬‬ ‫‪3‬‬
‫‪y = ϕ(x) ⇔ y = 1 − 3x‬‬
‫‪1− y‬‬
‫=‪⇔x‬‬
‫‪3‬‬
‫اذن ‪1 − y < 1‬‬ ‫ا ذن ‪− y < 0‬‬ ‫ﻟﺪﻳﻨﺎ ‪y > 0‬‬
‫‪1‬‬ ‫‪1− y 1‬‬
‫أي < ‪x‬‬ ‫أي <‬
‫‪3‬‬ ‫‪3‬‬ ‫‪3‬‬
‫‪‬‬ ‫‪1‬‬
‫اذن )‪(2) \ *+ ⊂ ϕ( −∞, ‬‬
‫‪‬‬ ‫‪3‬‬
 1  1
ϕ−1 ( \*+ ) =  −∞,  ‫وﺑﺎﻟﺘﺎﻟﻲ‬ ϕ( −∞, ) = \ *+ ‫( ﻧﺴﺘﻨﺘﺞ أن‬2) ‫( و‬1) ‫ﻣﻦ‬
 3  3
 1 1 
 −∞, 3  ⊂ \ −  3  ‫ج – ﻟﺪﻳﻨﺎ‬
 1  1
0 ∈  −∞,  ‫ ﻻن‬ −∞,  ≠ ∅ ‫و‬
 3  3
 1
 −∞, 3  ‫ ﻋﻨﺼﺮان ﻣﻦ‬y ‫ و‬x ‫ﻟﻴﻜﻦ‬
−y
x * y ' = x *( ) ‫ﻟﺪﻳﻨﺎ‬
1 − 3y
y −y
=x− − 3x( ) ‫اذن‬
1 − 3y 1 − 3y
x − 3xy − y + 3xy
= ‫أي‬
1 − 3y
x−y
x*y' = ‫أي‬
1 − 3y
1 x − y 1 − 3y − 3x + 3y 1 − 3x
− = = ‫ﻟﺪﻳﻨﺎ‬
3 1 − 3y 3(1 − 3y) 3(1 − 3y)
1 1
1 − 3y > 0 ‫ و‬1 − 3x > 0 ‫ﻓﺎن‬ y < ‫ و‬x < ‫و ﺑﻤﺎ أن‬
3 3
 1 x−y 1 1 − 3x
x * y ' ∈  −∞,  ‫اذن‬ < ‫أي‬ > 0 ‫اذن‬
 3 1 − 3y 3 3(1 − 3y)
1   1
( \ −   ,*) ‫( زﻣﺮة ﺟﺰﺋﻴﺔ ﻟﻠﺰﻣﺮة‬ −∞,  ,*) ‫و ﺑﺎﻟﺘﺎﻟﻲ‬
3  3
(ϕ(x)) = 1 ‫و‬
0
ϕ(x ) = ϕ(0) = 1 ‫ﻟﺪﻳﻨﺎ‬
(0)
n=0 ‫ – أ – ﻣﻦ أﺟﻞ‬3
ϕ(x (0) ) = (ϕ(x))0 ‫اذن‬
ϕ(x (n ) ) = (ϕ(x)) n ‫ﻧﻔﺘﺮض أن‬
(n +1) n +1
ϕ(x ) = (ϕ(x)) ‫و ﻧﺒﻴﻦ أن‬
(n +1)
ϕ(x ) = ϕ(x ) * x)n
‫ﻟﺪﻳﻨﺎ‬
= ϕ(x ) × ϕ(x) ( ‫ ﺗﺸﺎآﻞ‬ϕ ‫) ﻻن‬
n

= (ϕ(x)) n × ϕ(x) ‫أي‬


 1  −y (n +1) n +1
ϕ(x ) = (ϕ(x))
 −∞, 3  1 − 3y )
‫و ﻣﻨﻪ‬
−1
ϕ(x ) = (ϕ(x)) ⇔ x = ϕ ((ϕ(x) ) - ‫ب‬
(n ) n (n ) n

⇔ x (n ) = ϕ−1 ((1 − 3x) n )


1 − (1 − 3x) n
⇔ x (n ) =
3
1
(∀(x, y) ∈ \ 2 ) xTy = x + y − : ‫ ﻗﺎﻧﻮن اﻟﺘﺮآﻴﺐ اﻟﺪاﺧﻠﻲ اﻟﻤﻌﺮف ﻋﻠﻰ \ ﺑﻤﺎ ﻳﻠﻲ‬T ‫ – ﻟﻴﻜﻦ‬4
3
1
∀(x, y) ∈ \ 2 xTy = x + y − -‫أ‬
3
1
= y+x−
3
xTy = yTx
.‫ ﺗﺒﺎدﻟﻲ‬T ‫إذن‬
‫‪1‬‬
‫‪∀(x, y, z) ∈ \ 3‬‬ ‫ﻟﺪﻳﻨﺎ ‪(xTy)Tz = (x + y − )Tz‬‬
‫‪3‬‬
‫‪2‬‬
‫‪= x+y+z−‬‬ ‫اذن‬
‫‪3‬‬
‫‪1‬‬
‫ﻟﺪﻳﻨﺎ ) ‪xT(yTz) = xT(y + z −‬‬
‫‪3‬‬
‫‪2‬‬
‫‪= x+y+z−‬‬ ‫اذن‬
‫‪3‬‬
‫اذن )‪ (xTy)Tz = xT(yTz‬ﻟﻜﻞ )‪ (x, y, z‬ﻣﻦ ‪\ 3‬‬
‫اذن ‪ T‬ﺗﺠﻤﻴﻌﻲ ‪.‬‬
‫‪1‬‬ ‫‪1 1‬‬
‫) \ ∈ ‪(∀x‬‬ ‫ﻟﺪﻳﻨﺎ ‪xT = x + − = x‬‬
‫‪3‬‬ ‫‪3 3‬‬
‫‪1‬‬
‫هﻮ اﻟﻌﻨﺼﺮ اﻟﻤﺤﺎﻳﺪ ﻟﻠﻘﺎﻧﻮن ‪T‬‬ ‫اذن‬
‫‪3‬‬
‫‪ x‬ﻳﻘﺒﻞ ﻣﻤﺎﺛﻼ ﻓﻲ )‪ ( \, T‬ﻳﻌﻨﻲ ﻳﻮﺟﺪ ’‪ x‬ﻓﻲ ‪ E‬ﺑﺤﻴﺚ ‪:‬‬
‫‪1‬‬
‫= ' ‪ ) xTx‬هﺬﻩ اﻟﻌﻼﻗﺔ آﺎﻓﻴﺔ ﻻن ‪ T‬ﺗﺒﺎدﻟﻲ (‬
‫‪3‬‬
‫‪1‬‬ ‫‪1 1‬‬
‫= ‪xTx ' = ⇔ x + x '−‬‬
‫‪3‬‬ ‫‪3 3‬‬
‫‪2‬‬
‫‪⇔ x'= −x‬‬
‫‪3‬‬
‫اذن آﻞ ﻋﻨﺼﺮ ﻣﻦ \ ﻟﻪ ﻣﻤﺎﺛﻞ ﻓﻲ )‪ . ( \, T‬و ﺑﺎﻟﺘﺎﻟﻲ ‪ ( \, T) :‬زﻣﺮة ﺗﺒﺎدﻟﻴﺔ‪.‬‬
‫‪1 ‬‬
‫ب – ﻟﺪﻳﻨﺎ )‪ ( \, T‬زﻣﺮة ﺗﺒﺎدﻟﻴﺔ و )*‪ ( \ −   ,‬زﻣﺮة ﺗﺒﺎدﻟﻴﺔ‬
‫‪3‬‬
‫ﻟﻨﺒﻴﻦ أن * ﺗﻮزﻳﻌﻲ ﺑﺎﻟﻨﺴﺒﺔ ﻟﻠﻘﺎﻧﻮن ‪T‬‬
‫) هﺬﻩ اﻟﻌﻼﻗﺔ آﺎﻓﻴﺔ ﻻن * ﺗﺒﺎدﻟﻲ (‬ ‫\ ∈ )‪∀(x, y, z‬‬ ‫‪3‬‬
‫أي )‪x *(yTz) = (x * y)T(x * z‬‬
‫‪1‬‬
‫) ‪x *(yTz) = x *(y + z −‬‬ ‫ﻟﺪﻳﻨﺎ‬
‫‪3‬‬
‫‪1‬‬ ‫‪1‬‬
‫اذن ) ‪= x + y + z − − 3x(y + z −‬‬
‫‪3‬‬ ‫‪3‬‬
‫‪1‬‬
‫‪(1) x *(yTz) = 2x + y + z − 3xy − 3xz −‬‬ ‫اذن‬
‫‪3‬‬
‫ﻟﺪﻳﻨﺎ )‪(x * y)T(x * z) = (x + y − 3xy)T(x + z − 3xz‬‬
‫‪1‬‬
‫‪(x * y)T(x * z) = x + y − 3xy + x + z − 3xz −‬‬ ‫اذن‬
‫‪3‬‬
‫‪1‬‬
‫‪(2) (x * y)T(x * z) = 2x + y + z − 3xy − 3xz −‬‬
‫‪3‬‬
‫ﻣﻦ )‪ (1‬و )‪ (2‬ﻧﺴﺘﻨﺘﺞ أن * ﺗﻮزﻳﻌﻲ ﺑﺎﻟﻨﺴﺒﺔ ﻟﻠﻘﺎﻧﻮن ‪. T‬‬
‫و ﺑﺎﻟﺘﺎﻟﻲ ‪ ( \, T,*) :‬ﺟﺴﻢ ﺗﺒﺎدﻟﻲ‪.‬‬
‫اﻟﺘﻤﺮﻳﻦ اﻟﺜﺎﻟﺚ ‪:‬‬
‫ﻟﺪﻳﻨﺎ آﺮة ﺑﻴﻀﺎء و ﺛﻼث آﺮات ﺣﻤﺮاء‬
‫ﻧﺴﺤﺐ ﻋﺸﻮاﺋﻴﺎ آﺮة ﻣﻦ اﻟﺼﻨﺪوق ﻧﺴﺠﻞ ﻟﻮﻧﻬﺎ ﺛﻢ ﻧﻌﻴﺪهﺎ اﻟﻰ اﻟﺼﻨﺪوق‬
‫‪ = X‬رﺗﺒﺔ اﻟﺴﺤﺒﺔ اﻟﺘﻲ ﺗﻮﻗﻘﺖ ﻓﻴﻬﺎ اﻟﺘﺠﺮﺑﺔ‬
‫‪ (X=2) – 1‬ﻳﻌﻨﻲ ﺳﺤﺐ آﺮﺗﻴﻦ ﺑﻴﻀﺎوﺗﻴﻦ أو آﺮﺗﻴﻦ ﺣﻤﺮاوﻳﻦ أي ‪ BB‬أو ‪RR‬‬
‫‪1 1‬‬ ‫‪3 3‬‬
‫اذن ) × ( ‪p(X = 2) = ( × ) +‬‬
‫‪4 4‬‬ ‫‪4 4‬‬
1 9
p(X = 2) = +
16 16
5
p(X = 2) = ‫اذن‬
8
RBB ‫ أو‬BRR ‫( ﻳﻌﻨﻲ ﺳﺤﺐ‬X=3)
1 3 3 3 1 1
p(X = 3) = ( × × ) + ( × × ) ‫إذن‬
4 4 4 4 4 4
9 3
p(X = 3) = +
64 64
3
p(X = 3) =
16
k ∈ `* ‫ – ﻟﻴﻜﻦ‬2
(BRBRB................BRBB) ‫( ﻳﻌﻨﻲ ﺳﺤﺐ‬X = 2k) - ‫أ‬

2k-2 2k-1 2k
(RBRB................RBRR) ‫أو‬

2k-2 2k-1 2k
1 k −1 3 k −1 1 2 3 k −1 1 k −1 3 2
p 2k = ( ) .( ) .( ) + ( ) .( ) .( ) ‫إذن‬
4 4 4 4 4 4
1 k +1 3 k −1 3 k +1 1 k −1
= ( ) .( ) + ( ) .( ) ‫أي‬
4 4 4 4
1 3 1 3
= ( ) k −1.( ) k −1 (( ) 2 + ( ) 2 ) ‫أي‬
4 4 4 4
3 10
= ( ) k −1 × ‫إذن‬
16 16
5 3 k −1
p 2k = ( ) ‫وﻣﻨﻪ‬
8 16
: ‫( ﻳﻌﻨﻲ ﺳﺤﺐ‬X=2k+1) –‫ب‬
(BRBRB................BRBRR )

2k-2 2k-1 2k 2k+1


(RBRB................BRBB) ‫أو‬

2k-2 2k-1 2k 2k+1


1 k 3 k −1 3 2 3 k 1 k −1 1 k
p 2k +1 = ( ) ( ) ( ) + ( ) ( ) ( ) ‫اذن‬
4 4 4 4 4 4
1 k 3 k +1 3 k 1 k +1
p 2k +1 = ( ) ( ) + ( ) ( ) ‫أي‬
4 4 4 4
1 3 3 1
= ( )k ( )k ( + )
4 4 4 4
1 3
= ( )k ( )k
4 4
3
p 2k +1 = ( ) k
16
‫اﻟﺘﻤﺮﻳﻦ اﻟﺮاﺑﻊ ‪:‬‬
‫‪‬‬ ‫)‪ln(1 + 2x‬‬
‫= )‪f (x‬‬ ‫‪;x ≠ 0‬‬
‫ﻟﺘﻜﻦ ‪ f‬اﻟﺪاﻟﺔ اﻟﻤﻌﺮﻓﺔ ﻋﻠﻰ ‪ I =  − , +∞ ‬ﺑﻤﺎ ﻳﻠﻲ ‪:‬‬
‫‪1‬‬
‫‪‬‬ ‫‪x‬‬
‫‪f (0) = 2‬‬ ‫‪ 2‬‬ ‫‪‬‬
‫)‪ln(1 + 2x‬‬
‫‪lim f (x) = lim‬‬ ‫‪.2‬‬ ‫‪-1‬‬
‫‪x →0‬‬ ‫‪x →0‬‬ ‫‪2x‬‬
‫)‪= 1× 2 = 2 = f (0‬‬
‫إذن ‪ f‬ﻣﺘﺼﻠﺔ ﻓﻲ ‪x 0 = 0‬‬
‫‪ 1‬‬ ‫‪‬‬
‫)‪(a ≠ 0‬‬ ‫‪a ∈  − , +∞  - 2‬‬
‫‪ 2‬‬ ‫‪‬‬
‫أ – ﻟﺘﻜﻦ ‪h a (x) = (ln(1 + 2a) − 2a)x − (ln(1 + 2x) − 2x)a‬‬
‫‪2‬‬ ‫‪2‬‬

‫ﻟﺪﻳﻨﺎ ‪ h a (0) = 0‬و ‪h a (a) = 0‬‬


‫‪ h a‬ﻣﺘﺼﻠﺔ ﻋﻠﻰ اﻟﻤﺠﺎل اﻟﻤﻐﻠﻮق اﻟﺬي ﻃﺮﻓﺎﻩ ‪ 0‬و ‪a‬‬
‫و ق ‪ .‬ش ﻋﻠﻰ اﻟﻤﺠﺎل اﻟﻤﻔﺘﻮح اﻟﺬي ﻃﺮﻓﺎﻩ ‪ 0‬و ‪a‬‬
‫اذن ﺣﺴﺐ ﻣﺒﺮهﻨﺔ رول ‪ Rolle‬ﻳﻮﺟﺪ ﻋﺪد ﺣﻘﻴﻘﻲ ‪ b‬ﻣﺤﺼﻮر ﺑﻴﻦ ‪ 0‬و ‪ a‬ﺑﺤﻴﺚ ‪h a '(b) = 0 :‬‬
‫‪2‬‬
‫( ‪h 'a (x) = 2(ln(1 + 2a) − 2a)x −‬‬ ‫ﻟﺪﻳﻨﺎ ‪− 2)a 2‬‬
‫‪1 + 2x‬‬
‫‪1‬‬
‫( = ‪h 'a (b) = 0 ⇔ (ln(1 + 2a) − 2a)b‬‬ ‫‪− 1)a 2‬‬
‫‪1 + 2b‬‬
‫‪ln(1 + 2a) − 2a‬‬ ‫‪−2b‬‬
‫⇔‬ ‫=‬
‫‪a2‬‬ ‫)‪b(1 + 2b‬‬
‫‪ln(1 + 2a) − 2a‬‬ ‫‪−2‬‬
‫⇔‬ ‫=‬
‫‪a‬‬ ‫‪2‬‬
‫‪1 + 2b‬‬
‫)‪f (x) − f (0‬‬ ‫‪ln(1 + 2x) − 2x‬‬
‫‪lim‬‬ ‫‪= lim‬‬ ‫ب‪-‬‬
‫‪x →0‬‬ ‫‪x‬‬ ‫‪x →0‬‬ ‫‪x2‬‬
‫‪ln(1 + 2x) − 2x‬‬ ‫‪−2‬‬
‫ﺑﺤﻴﺚ ‪ b :‬ﻣﺤﺼﻮر ﺑﻴﻦ ‪ 0‬و ‪x‬‬ ‫=‬ ‫ﺣﺴﺐ س ‪ -‬أ ‪-‬‬
‫‪x‬‬ ‫‪2‬‬
‫‪1 + 2b‬‬
‫ﻓﺎن ‪0 < b < x‬‬ ‫اذا آﺎن )‪(x > 0‬‬
‫‪ln(1 + 2x) − 2x‬‬ ‫‪−2‬‬
‫‪lim+‬‬ ‫‪= lim+‬‬ ‫إذن ‪= −2‬‬
‫‪b → 0 1 + 2b‬‬
‫‪2‬‬
‫‪x →0‬‬ ‫‪x‬‬
‫اذن ‪ f‬ق ‪ .‬ش ﻋﻠﻰ ﻳﻤﻴﻦ ‪ 0‬و ‪f 'd (0) = −2‬‬
‫ﻓﺎن ‪x < b < 0‬‬ ‫اذا آﺎن )‪(x < 0‬‬
‫)‪f (x) − f (0‬‬ ‫‪−2‬‬
‫‪lim‬‬ ‫‪= lim−‬‬ ‫اذن ‪= −2‬‬
‫‪x → 0−‬‬ ‫‪x‬‬ ‫‪b‬‬ ‫→‬ ‫‪0‬‬ ‫‪1 + 2b‬‬
‫اذن ‪ f‬ق ‪ .‬ش ﻋﻠﻰ ﻳﺴﺎر ‪ 0‬و ‪f 'g (0) = −2‬‬
‫و ﺑﺎﻟﺘﺎﻟﻲ ‪ f‬ق ‪ .‬ش ﻓﻲ ‪ 0‬و ‪f '(0) = −2‬‬
‫‪ – 3‬أ – اﻟﺪاﻟﺔ ‪ f‬ق ‪.‬ش ﻋﻠﻰ }‪ I − {0‬ﻷﻧﻬﺎ ﻣﺮآﺐ و ﺧﺎرج داﻟﺘﻴﻦ ق ش ﻋﻠﻰ }‪I − {0‬‬
‫‪2‬‬
‫)‪x − ln(1 + 2x‬‬
‫)}‪(∀x ∈ I − {0‬‬ ‫‪f '(x) = 1 + 2x‬‬ ‫ﻟﺪﻳﻨﺎ‬
‫‪x2‬‬
‫)‪2x − (1 + 2x) ln(1 + 2x‬‬
‫= )‪f '(x‬‬ ‫اذن‬
‫)‪x 2 (1 + 2x‬‬
‫)‪g(x‬‬
‫‪f '(x) = 2‬‬ ‫أي‬
‫)‪x (1 + 2x‬‬
‫ﺑﺤﻴﺚ ‪g(x) = 2x − (1 + 2x) ln(1 + 2x) :‬‬
‫‪(1 + 2x)2‬‬
‫)‪(∀x ∈ I‬‬ ‫‪g '(x) = 2 − 2 ln(1 + 2x) −‬‬ ‫ب – ﻟﺪﻳﻨﺎ‬
‫)‪(1 + 2x‬‬
‫)‪g '(x) = −2 ln(1 + 2x‬‬ ‫إذن‬
‫‪g '(x) = 0 ⇔ 1 + 2x = 1‬‬
‫‪⇔x=0‬‬
‫‪x > 0 ⇔ 1 + 2x > 1‬‬
‫‪⇔ ln(1 + 2x) > 0‬‬
‫‪⇔ − ln(1 + 2x) < 0‬‬
‫‪⇔ g '(x) < 0‬‬

‫‪X‬‬ ‫‪-1/2‬‬ ‫‪0‬‬ ‫∞‪+‬‬


‫)‪g’(x‬‬ ‫‪+‬‬ ‫‪0‬‬ ‫‪-‬‬

‫‪0‬‬
‫)‪g(x‬‬

‫ﻣﻦ ﺧﻼل ﺟﺪول ﺗﻐﻴﺮات اﻟﺪاﻟﺔ ‪ g‬ﻧﺴﺘﻨﺘﺞ أن ‪ g :‬ﺗﻘﺒﻞ ﻗﻴﻤﺔ ﻗﺼﻮى ﻋﻨﺪ ‪ 0‬و هﻲ ‪g(0)=0‬‬
‫)}‪(∀x ∈ I − {0‬‬ ‫اذن ‪g(x) < 0‬‬
‫‪1+2x >0‬‬ ‫ج ‪ -‬ﻟﻜﻞ ‪ x‬ﻣﻦ ‪: I‬‬
‫اذن اﺷﺎرة )‪ f’(x‬هﻲ اﺷﺎرة )‪g(x‬‬
‫و ﺑﻤﺎ أن ‪ g(x)<0‬ﻟﻜﻞ ‪ x‬ﻣﻦ }‪I − {0‬‬
‫ﻓﺎن ‪ f’(x)<0‬اذن ‪ f‬ﺗﻨﺎﻗﺼﻴﺔ ﻗﻄﻌﺎ ﻋﻠﻰ ‪I‬‬
‫)‪ln(1 + 2x‬‬
‫‪lim f (x) = lim‬‬ ‫‪-4‬أ‪-‬‬
‫‪x →−‬‬
‫‪1‬‬
‫‪x →−‬‬
‫‪1‬‬ ‫‪x‬‬
‫‪2‬‬ ‫‪2‬‬
‫‪1‬‬ ‫‪1‬‬
‫‪x >−‬‬ ‫‪x >−‬‬
‫‪2‬‬ ‫‪2‬‬
‫∞‪−‬‬
‫=‬
‫∞‪= +‬‬
‫‪1‬‬
‫‪−‬‬
‫‪2‬‬
‫‪1‬‬
‫اذن اﻟﻤﺴﺘﻘﻴﻢ ذو اﻟﻤﻌﺎدﻟﺔ ‪ x = −‬ﻣﻘﺎرب ﻟﻠﻤﻨﺤﻨﻰ )‪(C‬‬
‫‪2‬‬
‫)‪ln(1 + 2x‬‬
‫‪lim f (x) = lim‬‬ ‫ﻟﺪﻳﻨﺎ‬
‫∞‪x →+‬‬ ‫∞‪x →+‬‬ ‫‪x‬‬
‫)‪ln(1 + 2x) (1 + 2x‬‬
‫‪= lim‬‬ ‫×‬ ‫اذن‬
‫)‪x →+∞ (1 + 2x‬‬ ‫‪x‬‬
‫‪= 0× 2 = 0‬‬
‫اذن اﻟﻤﺴﺘﻘﻴﻢ ذو اﻟﻤﻌﺎدﻟﺔ ‪ y=0‬ﻣﻘﺎرب ﻟﻠﻤﻨﺤﻨﻰ )‪ (C‬ﺑﺠﻮار )∞‪(+‬‬
‫ب – ﻟﺘﻜﻦ ‪h(x) = f (x) − 1‬‬
‫‪ h‬ﻣﺘﺼﻠﺔ ﻋﻠﻰ ]‪[1, 2‬‬
‫)]‪(∀x ∈ [1, 2‬‬ ‫‪h '(x) = f '(x) < 0‬‬
‫اذن ‪ h‬ﺗﻨﺎﻗﺼﻴﺔ ﻗﻄﻌﺎ ﻋﻠﻰ ]‪[1, 2‬‬
‫‪ln 5‬‬
‫= )‪h(2‬‬ ‫ﻟﺪﻳﻨﺎ ‪ h(1) = f (1) − 1‬و ‪− 1 < 0‬‬
‫‪2‬‬
‫‪= ln 3 − 1 > 0‬‬
‫اذن ‪h(1) × h(2) < 0‬‬
‫اذن ﺣﺴﺐ ﻣﺒﺮهﻨﺔ اﻟﻘﻴﻢ اﻟﻮﺳﻴﻄﻴﺔ ﻳﻮﺟﺪ ﻋﺪد ﺣﻘﻴﻘﻲ وﺣﻴﺪ ‪ α‬ﻣﻦ ]‪ [1, 2‬ﺑﺤﻴﺚ ‪ h(α) = 0 :‬أي ‪f (α) = 1‬‬
‫ج – اﻧﺸﺎء اﻟﻤﻨﺤﻨﻰ )‪(C‬‬

‫)‪(∀x ∈ I‬‬ ‫‪ - 1 ( II‬ﻧﻀﻊ ] ‪ J = [1, α‬و )‪ϕ(x) = ln(1 + 2x‬‬


‫أ – اﻟﺪاﻟﺔ ‪ ϕ‬ق ش ﻋﻠﻰ اﻟﻤﺠﺎل ‪ I‬ﻷﻧﻬﺎ ﻣﺮآﺐ داﻟﺘﻴﻦ ق ش ﻋﻠﻰ اﻟﻤﺠﺎل ‪I‬‬
‫‪2‬‬
‫)‪(∀x ∈ I‬‬ ‫= )‪ϕ '(x‬‬
‫‪1 + 2x‬‬
‫‪x ≥ 1 ⇒ 1 + 2x ≥ 3‬‬
‫‪1‬‬ ‫‪1‬‬
‫<‪⇒0‬‬ ‫≤‬
‫‪1 + 2x 3‬‬
‫‪2‬‬ ‫‪2‬‬
‫<‪⇒0‬‬ ‫≤‬
‫‪1 + 2x 3‬‬
‫‪2‬‬
‫≤ )‪⇒ 0 < ϕ '(x‬‬
‫‪3‬‬
‫) ‪ln(1 + 2α‬‬
‫⇔ ‪f (α ) = 1‬‬ ‫‪=1‬‬ ‫ب‪-‬‬
‫‪α‬‬
‫‪⇔ ln(1 + 2α ) = α‬‬
‫‪⇔ ϕ(α) = α‬‬
‫‪2‬‬
‫≤ )‪ 0 < ϕ '(x‬اذن ‪ ϕ‬ﺗﺰاﻳﺪﻳﺔ ﻋﻠﻰ ‪I‬‬ ‫ﻟﺪﻳﻨﺎ‬
‫‪3‬‬
‫)‪1 ≤ x ≤ α ⇒ ϕ(1) ≤ ϕ(x) ≤ ϕ(α‬‬
‫‪⇒ 1 < ln 3 ≤ ϕ(x) ≤ α‬‬
‫)‪(∀x ∈ J‬‬ ‫ﻟﺪﻳﻨﺎ ‪ ϕ‬ﻣﺘﺼﻠﺔ و ‪1 < ϕ(x) ≤ α‬‬
‫إذن ‪ϕ(J) ⊂ J‬‬
‫‪U0 = 1‬‬
‫‪‬‬ ‫‪-2‬‬
‫)‪ U n +1 = ln(1 + 2U n )(n ≥ 0‬‬
‫أ ‪ -‬ﻟﺪﻳﻨﺎ ) ‪U n +1 = ϕ(U n‬‬
‫ﻟﻨﺒﻴﻦ أن ‪ U n ∈ J‬أي ‪ 1 ≤ U n ≤ α‬ﻟﻜﻞ ‪n ≥ 0‬‬
‫ﻣﻦ أﺟﻞ ‪ n = 0‬ﻟﺪﻳﻨﺎ ‪ U 0 = 1‬اذن ‪U 0 ∈ J‬‬
‫ﻧﻔﺘﺮض أن ‪ U n ∈ J‬ﻟﻜﻞ ‪ n ≥ 0‬و ﻧﺒﻴﻦ أن ‪U n +1 ∈ J‬‬
‫ﻟﺪﻳﻨﺎ ‪ U n ∈ J‬وﺣﺴﺐ س ‪ -1‬ب ‪ϕ(J) ⊂ J -‬‬
‫اذن ‪ ϕ(U n ) ∈ J‬أي ‪U n +1 ∈ J‬‬
‫و ﺑﺎﻟﺘﺎﻟﻲ ‪ U n ∈ J‬ﻟﻜﻞ ‪n ≥ 0‬‬
‫ب – ﻟﺪﻳﻨﺎ ‪1 ≤ α ≤ 2‬‬
‫أي ‪−1 ≤ 1 − α ≤ 0 < 1‬‬ ‫اذن ‪−2 ≤ −α ≤ −1‬‬
‫‪2‬‬
‫اذن ‪U 0 − α ≤ ( )0‬‬
‫‪3‬‬
‫‪2‬‬
‫ﻧﻔﺘﺮض أن ‪ U n − α ≤ ( ) n‬ﻟﻜﻞ ‪n ≥ 0‬‬
‫‪3‬‬
‫‪2 n +1‬‬
‫و ﻧﺒﻴﻦ أن ) ( ≤ ‪U n +1 − α‬‬
‫‪3‬‬
‫ﻟﺪﻳﻨﺎ ‪ ϕ‬ﻣﺘﺼﻠﺔ ﻋﻠﻰ اﻟﻤﺠﺎل اﻟﻤﻐﻠﻮق اﻟﺬي ﻃﺮﻓﺎﻩ ‪ α‬و ‪ U n‬و ق ش ﻋﻠﻰ اﻟﻤﺠﺎل اﻟﻤﻔﺘﻮح اﻟﺬي ﻃﺮﻓﺎﻩ ‪ α‬و ‪U n‬‬
‫اذن ﺣﺴﺐ ﻣﺒﺮهﻨﺔ اﻟﺘﺰاﻳﺪات اﻟﻤﻨﺘﻬﻴﺔ ﻳﻮﺟﺪ ‪ c‬ﻣﺤﺼﻮر ﺑﻴﻦ ‪ α‬و ‪ U n‬ﺑﺤﻴﺚ ‪:‬‬
‫‪ϕ(U n ) − ϕ(α) = ϕ '(c) U n − α‬‬
‫‪2‬‬ ‫‪2‬‬
‫≤ )‪ϕ '(c‬‬ ‫ﻓﺎن‬ ‫≤ )‪ 0 < ϕ '(x‬ﻟﻜﻞ )‪(x ≥ 1‬‬ ‫و ﺑﻤﺎ أن‬
‫‪3‬‬ ‫‪3‬‬
‫و ﻟﺪﻳﻨﺎ ) ‪U n +1 = ϕ(U n‬‬
‫و ﺣﺴﺐ س ‪ – 1‬ب ‪ϕ(α) = α -‬‬
‫‪2‬‬
‫اذن ‪U n +1 − α ≤ U n − α‬‬
‫‪3‬‬
‫‪2‬‬
‫ﺣﺴﺐ اﻓﺘﺮاض اﻟﺘﺮﺟﻊ ﻟﺪﻳﻨﺎ ‪U n − α ≤ ( ) n‬‬
‫‪3‬‬
‫‪2‬‬
‫اذن ‪U n +1 − α ≤ ( ) n +1‬‬
‫‪3‬‬
‫‪2 n‬‬
‫و ﺑﺎﻟﺘﺎﻟﻲ ) ( ≤ ‪ U n − α‬ﻟﻜﻞ )‪(n ≥ 0‬‬
‫‪3‬‬
‫‪2‬‬ ‫‪2‬‬
‫ج – اﻟﻤﺘﺘﺎﻟﻴﺔ ‪ ( ) n‬ﻣﺘﻘﺎرﺑﺔ و ﻧﻬﺎﻳﺘﻬﺎ ‪ ) 0‬ﻻن ‪(−1 < < 1‬‬
‫‪3‬‬ ‫‪3‬‬
‫اذن ﺣﺴﺐ ﻣﺼﺎدﻳﻖ اﻟﺘﻘﺎرب اﻟﻤﺘﺘﺎﻟﻴﺔ ) ‪ (U n‬ﻣﺘﻘﺎرﺑﺔ و ‪lim U n = α‬‬
‫∞‪n →+‬‬
‫‪x‬‬
‫‪ ( III‬ﻟﺘﻜﻦ ‪ F‬اﻟﺪاﻟﺔ اﻟﻤﻌﺮﻓﺔ ﻋﻠﻰ ‪ I‬ﺑﻤﺎ ﻳﻠﻲ ‪F(x) = ∫ f (t)dt :‬‬
‫‪0‬‬

‫‪ - 1‬أ – ‪ f‬ﻣﺘﺼﻠﺔ ﻋﻠﻰ ‪ I‬اذن ﺗﻘﺒﻞ داﻟﺔ أﺻﻠﻴﺔ ‪ ψ‬ﻋﻠﻰ ‪I‬‬


‫)‪(∀x ∈ I‬‬ ‫ﺑﺤﻴﺚ ‪ ψ‬ق ش ﻋﻠﻰ ‪ I‬و )‪ψ '(x) = f (x‬‬
‫ﻟﺪﻳﻨﺎ ‪F(x) = [ ψ (t) ]0‬‬
‫‪x‬‬

‫)‪= ψ (x) − ψ (0‬‬


‫اذن ‪ F‬ق ش ﻋﻠﻰ ‪ I‬ﻻﻧﻬﺎ ﻣﺠﻤﻮع داﻟﺘﻴﻦ ق ش ﻋﻠﻰ ‪I‬‬
‫)‪(∀x ∈ I‬‬ ‫)‪F '(x) = ψ '(x‬‬
‫)‪F'(x) = f (x‬‬
‫)‪(∀x ∈ I‬‬ ‫ب – ﻣﻦ ﺧﻼل ﻣﺒﻴﺎن اﻟﺪاﻟﺔ ‪ f‬ﻧﺴﺘﻨﺘﺞ أن ‪f(x)>0‬‬
‫اذن ‪ F’(x)>0‬اذن ‪ F‬ﺗﺰاﻳﺪﻳﺔ ﻗﻄﻌﺎ ﻋﻠﻰ ‪I‬‬
‫‪x‬‬
‫‪ – 2‬أ – ﻟﺪﻳﻨﺎ ‪F(x) = ∫ f (t)dt‬‬
‫‪0‬‬
‫‪1‬‬ ‫‪x‬‬
‫اذن ‪F(x) = ∫ f (t)dt + ∫ f (t)dt‬‬
‫‪0‬‬ ‫‪1‬‬
‫‪x‬‬
‫‪x ≥ 1 ⇒ F(x) ≥ ∫ f (t)dt‬‬
‫‪1‬‬
‫‪x‬‬
‫)‪ln(1 + 2t‬‬
‫∫ ≥ )‪⇒ F(x‬‬ ‫‪dt‬‬
‫‪1‬‬
‫‪t‬‬
‫‪1‬‬ ‫‪1‬‬ ‫)‪ln(1 + 2t) ln(1 + 2t‬‬
‫≥ ⇒ ‪t ≤ 1 + 2t‬‬ ‫⇒‬ ‫≥‬
‫‪t 1 + 2t‬‬ ‫‪t‬‬ ‫‪1 + 2t‬‬
‫‪x‬‬ ‫‪x‬‬
‫)‪ln(1 + 2t‬‬ ‫)‪ln(1 + 2t‬‬
‫∫⇒‬ ‫∫ ≥ ‪dt‬‬ ‫‪dt‬‬
‫‪1‬‬
‫‪t‬‬ ‫‪1‬‬
‫‪1 + 2t‬‬
‫‪x‬‬
‫)‪ln(1 + 2t‬‬
‫∫ ≥ )‪F(x‬‬ ‫و ﺑﻤﺎ أن ‪dt‬‬
‫‪1‬‬
‫‪t‬‬
‫‪x‬‬
‫)‪ln(1 + 2t‬‬
‫∫ ≥ )‪F(x‬‬ ‫ﻓﺎن ‪dt‬‬
‫‪1‬‬
‫‪1 + 2t‬‬
‫‪x‬‬ ‫‪x‬‬
‫)‪ln(1 + 2t‬‬ ‫‪1‬‬
‫ب – ﻟﺪﻳﻨﺎ ‪∫1 1 + 2t dt = ∫1 ln(1 + 2t) 2 ln'(1 + 2t)dt‬‬
‫‪x‬‬ ‫‪x‬‬
‫)‪ln(1 + 2t‬‬ ‫‪1‬‬ ‫‪2‬‬
‫‪∫1 1 + 2t dt =  4 (ln(1 + 2t)) 1‬‬ ‫اذن‬

‫‪1‬‬ ‫‪1‬‬
‫أي ‪= (ln(1 + 2x)) 2 − (ln 3) 2‬‬
‫‪4‬‬ ‫‪4‬‬
‫‪1‬‬ ‫‪1‬‬
‫اذن ‪F(x) ≥ (ln(1 + 2x)) 2 − (ln 3) 2‬‬
‫‪4‬‬ ‫‪4‬‬
‫‪1‬‬ ‫‪1‬‬
‫و ﺑﻤﺎ أن ∞‪lim (ln(1 + 2x))2 − (ln 3) 2 = +‬‬
‫‪x →+∞ 4‬‬ ‫‪4‬‬
‫ﻓﺎن ∞‪lim F(x) = +‬‬
‫∞‪x →+‬‬

‫‪1‬‬
‫‪−‬‬ ‫‪ – 3‬ﻧﻔﺘﺮض أن اﻟﺪاﻟﺔ ‪ F‬ﺗﻘﺒﻞ ﻧﻬﺎﻳﺔ ﻣﻨﺘﻬﻴﺔ ‪ A‬ﻋﻠﻰ اﻟﻴﻤﻴﻦ ﻓﻲ‬
‫‪2‬‬
‫‪1‬‬
‫اذن ‪ F‬ﺗﻘﺒﻞ ﺗﻤﺪﻳﺪا ﺑﺎﻻﺗﺼﺎل ﻋﻠﻰ اﻟﻴﻤﻴﻦ ﻓﻲ ‪−‬‬
‫‪2‬‬
‫أ – ﻟﺘﻜﻦ ‪ F‬اﻟﺪاﻟﺔ اﻟﻤﻌﺮﻓﺔ ﻋﻠﻰ ‪  − 1 , +∞ ‬ﺑﻤﺎ ﻳﻠﻲ ‪:‬‬
‫‪ 2‬‬ ‫‪‬‬
‫‬
‫)‪F(x‬‬ ‫‪= F(x); x ∈ I‬‬
‫‪‬‬
‫‪ 1‬‬
‫‪F(− ) = A‬‬
‫‪‬‬ ‫‪2‬‬
‫‪1‬‬ ‫‬
‫‪−‬‬ ‫‪ F‬هﻲ اﻟﺘﻤﺪﻳﺪ ﺑﺎﻻﺗﺼﺎل ﻟﻠﺪاﻟﺔ ‪ F‬ﻋﻠﻰ اﻟﻴﻤﻴﻦ ﻓﻲ‬
‫‪2‬‬
‫‪ F‬ﻣﺘﺼﻠﺔ ﻋﻠﻰ ‪ 1 ‬‬ ‫ﻟﺪﻳﻨﺎ ‪ :‬‬
‫‪− ,x‬‬
‫‪ 2‬‬ ‫‪‬‬
‫‪ 1 ‬‬
‫و ق ش ﻋﻠﻰ ‪ − 2 , x ‬‬
‫اذن ﺣﺴﺐ ﻣﺒﺮهﻨﺔ اﻟﺘﺰاﻳﺪات اﻟﻤﻨﺘﻬﻴﺔ ‪T.A.F‬‬
‫(‪ − F‬‬
‫)‪F(x‬‬ ‫)‪ − 1‬‬
‫‪‬‬ ‫‪1‬‬ ‫‪‬‬ ‫)‪2 = F'(c‬‬
‫‬
‫‪∃c ∈  − , x  :‬‬
‫‪ 2 ‬‬ ‫‪1‬‬
‫‪x+‬‬
‫‪2‬‬
‫‪1‬‬
‫اذن ) ‪F(x) − A = F'(c)(x +‬‬
‫‪2‬‬
‫‪1‬‬
‫) ‪F(x) − A = f (c)(x +‬‬
‫‪2‬‬
‫‪1‬‬
‫ﻟﺪﻳﻨﺎ ‪− < c < x :‬‬
‫‪2‬‬
‫و ﺑﻤﺎ أن ‪ f‬ﺗﻨﺎﻗﺼﻴﺔ ﻋﻠﻰ ‪ I‬ﻓﺎن )‪f (c) ≥ f (x‬‬
‫‪1‬‬
‫) ‪F(x) − A ≥ f (x)(x +‬‬ ‫اذن‬
‫‪2‬‬
‫(‪ − F‬‬
‫)‪F(x‬‬ ‫)‪ − 1‬‬
‫‪lim‬‬ ‫ب ‪2 = lim F(x) − A -‬‬
‫‪1‬‬ ‫‪1‬‬ ‫‪1‬‬ ‫‪1‬‬
‫‪x →−‬‬
‫‪2‬‬ ‫‪x‬‬ ‫‪+‬‬ ‫‪x →−‬‬
‫‪2 x+‬‬
‫‪x >−‬‬
‫‪1‬‬ ‫‪2‬‬ ‫‪x >−‬‬
‫‪1‬‬ ‫‪2‬‬
‫‪2‬‬ ‫‪2‬‬

‫)‪≥ lim f (x‬‬


‫‪1‬‬
‫‪x →−‬‬
‫‪2‬‬
‫‪1‬‬
‫‪x >−‬‬
‫‪2‬‬

‫و ﺑﻤﺎ أن ∞‪lim f (x) = +‬‬


‫‪1‬‬
‫‪x →−‬‬
‫‪2‬‬
‫‪1‬‬
‫‪x >−‬‬
‫‪2‬‬

‫‬
‫)‪F(x‬‬ ‫)‪ − 1‬‬
‫(‪− F‬‬
‫‪lim1‬‬ ‫∞‪2 ≥ +‬‬ ‫ﻓﺎن‬
‫‪1‬‬
‫‪x→−‬‬
‫‪2‬‬ ‫‪x+‬‬
‫‪x >−‬‬
‫‪1‬‬ ‫‪2‬‬
‫‪2‬‬

‫‪1‬‬ ‫اذن ‪ :‬اﻟﺪاﻟﺔ ‬


‫‪.−‬‬ ‫‪ F‬ﻏﻴﺮ ق ش ﻋﻠﻰ اﻟﻴﻤﻴﻦ ﻓﻲ‬
‫‪2‬‬

‫ﺑﻌﺜﻪ ‪ :‬ﻳﺎﺳﺮ ﻏﺮﻳﺰ‬


‫اﻟﺜﺎﻧﻴﺔ ﺑﻜﺎﻟﻮرﻳﺎ ﻋﻠﻮم رﻳﺎﺿﻴﺔ‬ ‫ﺗﺼﺤﻴﺢ اﻻﻣﺘﺤﺎن اﻟﻮﻃﻨﻲ‬
‫اﻟﺮﻳﺎﺿﻴﺎت‬ ‫اﻟﻤﻮﺣﺪ ﻟﻠﺒﻜﺎﻟﻮرﻳﺎ‬ ‫اﻷﺳﺘﺎذ ‪ :‬اﻟﺤﻴﺎن‬
‫اﻟﺪورة اﻟﻌﺎدﻳﺔ ‪2007‬‬
‫اﻟﺘﻤﺮﻳﻦ اﻷول ‪:‬‬
‫⎫ ‪⎧ 1‬‬
‫(‬ ‫)‬
‫‪. ∀ a,b ∈ E 2‬‬ ‫‪ .I‬ﻟﻴﻜﻦ ⎬ ⎨ ‪ . E = \ −‬ﻟﺪﻳﻨﺎ ‪: a ⊥ b = a + b − ab 2 :‬‬
‫⎭‪⎩ 2‬‬
‫‪ .1‬أ‪ -‬ﻟﻴﻜﻦ ‪ . (a,b ) ∈ E 2‬ﻟﺪﻳﻨﺎ ‪:‬‬
‫‪1‬‬
‫‪2‬‬
‫‪−‬‬
‫‪1‬‬
‫(‬
‫‪2‬‬
‫()‬
‫= ‪a 2 −1 b 2 −1‬‬ ‫)‬ ‫‪1‬‬
‫‪2‬‬
‫‪−‬‬
‫‪1‬‬
‫‪2‬‬
‫(‬
‫‪2ab + 1 − a 2 − b 2 = a + b − ab 2 = a ⊥ b‬‬ ‫)‬
‫ب‪ -‬ﻟﻴﻜﻦ ‪ . (a,b ) ∈ E 2‬ﻟﺪﻳﻨﺎ ‪⇔ a 2 − 1 ≠ 0 :‬‬
‫‪1‬‬
‫≠ ‪ a ∈ E ⇔ a‬و ‪ . b ∈ E ⇔ b 2 − 1 ≠ 0‬إذن ‪:‬‬
‫‪2‬‬
‫≠ ‪a ⊥b‬‬ ‫‪1‬‬
‫‪2‬‬
‫‪. a ⊥ b −‬أي ‪:‬‬
‫‪1‬‬
‫‪2‬‬
‫‪=−‬‬
‫‪1‬‬
‫‪2‬‬
‫(‬ ‫()‬ ‫)‬
‫‪ (a 2 −1)(b 2 − 1) ≠ 0‬وﻣﻨﻩ ﻓﺈن‪a 2 − 1 b 2 −1 ≠ 0 :‬‬

‫وﻣﻨﻩ ﻓﺈن ‪ . a ⊥ b ∈ E :‬وﺑﺎﻟﺘﺎﻟﻲ ﻓﺈن ‪. ∀ (a,b ) ∈ E 2 : a ⊥ b ∈ E :‬‬


‫إذن ⊥ ﻗﺎﻧﻮن ﺗﺮآﻴﺐ داﺧﻠﻲ ﻓﻲ ‪. E‬‬
‫‪ .2‬ﻟﺪﻳﻨﺎ ‪ ⊥ :‬ﻗﺎﻧﻮن ﺗﺮآﻴﺐ داﺧﻠﻲ ﻓﻲ ‪. E‬‬
‫وﺑﻤﺎ أن اﻟﺠﻤﻊ واﻟﻀﺮب ﻗﺎﻧﻮﻧﻴﻦ ﺗﺒﺎدﻟﻴﻴﻦ وﺗﺠﻤﻴﻌﻴﻴﻦ ﻓﻲ \ ‪ ،‬ﻓﺈن ⊥ ﻗﺎﻧﻮن ﺗﺒﺎدﻟﻲ وﺗﺠﻤﻴﻌﻲ ﻓﻲ ‪. E‬‬
‫ﻟﺪﻳﻨﺎ ‪ . ∀a ∈ E : a ⊥ 0 = a æ 0 ⊥ a = a :‬إذن ‪ 0‬هﻮ اﻟﻌﻨﺼﺮ اﻟﻤﺤﺎﻳﺪ ﺑﺎﻟﻨﺴﺒﺔ ل ⊥ ﻓﻲ ‪. E‬‬
‫(‬ ‫)‬
‫ﻟﻴﻜﻦ ‪ . a,b ∈ E 2‬ﻟﺪﻳﻨﺎ ‪:‬‬

‫⇔ ‪a ⊥b = 0‬‬
‫‪1‬‬
‫‪2‬‬
‫‪−‬‬
‫‪1‬‬
‫‪2‬‬
‫(‬
‫‪a 2 −1 b 2 −1 = 0‬‬ ‫()‬ ‫)‬
‫⇔‬ ‫‪(a‬‬ ‫()‬
‫‪2 −1 b 2 −1 = 1‬‬ ‫)‬
‫‪1‬‬
‫⇔‬ ‫= ‪b 2 −1‬‬
‫‪a 2 −1‬‬
‫‪1‬‬
‫⇔‬ ‫=‪b 2‬‬ ‫‪+1‬‬
‫‪a 2 −1‬‬
‫‪a 2‬‬
‫⇔‬ ‫=‪b 2‬‬
‫‪a 2 −1‬‬
‫‪a‬‬
‫⇔ ‪a ⊥b = 0‬‬ ‫=‪b‬‬
‫‪a 2 −1‬‬
‫‪1‬‬ ‫‪a‬‬ ‫‪1‬‬
‫= ‪ b‬وهﺬا ﻏﻴﺮ ﻣﻤﻜﻦ‪ ،‬ﻓﺈن ‪b ∈ E :‬‬ ‫⇔‬ ‫=‬ ‫وﺑﻤﺎ أن ‪⇔ a 2 = a 2 − 1 ⇔ 0 = −1 :‬‬
‫‪2‬‬ ‫‪a 2 −1‬‬ ‫‪2‬‬
‫‪a‬‬
‫= ‪ b‬ﻓﻲ ‪ E‬ﺑﺎﻟﻨﺴﺒﺔ ﻟﻠﻘﺎﻧﻮن ⊥ ‪.‬‬ ‫وﺑﺎﻟﺘﺎﻟﻲ ﻓﺈن ﻟﻜﻞ ‪ a ∈ E‬ﻣﻤﺎﺛﻞ وﺣﻴﺪ‬
‫‪a 2 −1‬‬
‫زﻣﺮة ﺗﺒﺎدﻟﻴﺔ ‪.‬‬ ‫وﺑﺎﻟﺘﺎﻟﻲ ﻓﺈن ‪( E , ⊥ ) :‬‬
‫⎞‪⎛1 0‬‬
‫⎜= ‪I‬‬ ‫ﺣﻠﻘﺔ واﺣﺪﻳﺔ وﺣﺪﺗﻬﺎ ⎟‬ ‫‪ .II‬ﻧﻌﻠﻢ أن ‪( M2 ( \ ) , +,×) :‬‬
‫⎠‪⎝0 1‬‬
‫ﻓﻀﺎء ﻣﺘﺠﻬﻲ ﺣﻘﻴﻘﻲ ‪.‬‬ ‫وأن ‪( M2 ( \ ) , +,.) :‬‬
‫‪1 ⎛⎜ 2 − a‬‬ ‫‪a‬‬ ‫⎞‬
‫ﻣﺠﻤﻮﻋﺔ اﻟﻤﺼﻔﻮﻓﺎت ﻣﻦ ) \ ( ‪ M2‬اﻟﺘﻲ ﺗﻜﺘﺐ ﻋﻠﻰ ﺷﻜﻞ ⎟‬
‫= ) ‪ M (a‬ﺣﻴﺚ ‪. a ∈ E‬‬ ‫‪F‬‬
‫‪2 ⎜⎝ a‬‬ ‫⎠⎟ ‪2 − a‬‬

‫‪18‬‬ ‫ﺗﺼﺤﻴﺢ ﻣﻮﺿﻮع اﻟﺪورة اﻟﻌﺎدﻳﺔ ‪2007‬‬ ‫اﻷﺳﺘﺎذ ‪ :‬ﻣﺤﻤﺪ اﻟﺤﻴﺎن اﻟﺜﺎﻧﻴﺔ ﺑﻜﺎﻟﻮرﻳﺎ ﻋﻠﻮم رﻳﺎﺿﻴﺔ ‪- 1 -‬‬
‫⎞ ‪⎛ −1 1‬‬
‫⎜= ‪. A‬‬ ‫ﻧﻀﻊ ‪⎟ :‬‬
‫⎠‪⎝ 1 −1‬‬
‫‪⎛ −1 1 ⎞ ⎛ −1 1 ⎞ ⎛ 2‬‬ ‫⎞‪2‬‬ ‫⎞ ‪⎛ −1 1‬‬
‫‪A 2 = −2A‬‬ ‫⎜= ‪. A2‬‬ ‫⎜×⎟‬ ‫⎜=⎟‬ ‫⎜ ‪⎟ = −2‬‬ ‫‪ .1‬أ‪ -‬ﻟﺪﻳﻨﺎ ‪⎟ = −2A :‬‬
‫⎠ ‪⎝ 1 −1⎠ ⎝ 1 −1⎠ ⎝ −2 −2‬‬ ‫⎠‪⎝ 1 −1‬‬
‫‪a‬‬ ‫‪a‬‬ ‫⎞ ‪⎛ 1 0 ⎞ a ⎛ −1 1‬‬ ‫‪1 ⎛⎜ 2 − a‬‬ ‫⎞ ‪a‬‬
‫‪M (a ) = I +‬‬ ‫‪A .I+‬‬ ‫⎜= ‪A‬‬ ‫‪⎟+‬‬ ‫⎜‬ ‫=⎟‬ ‫وﻟﺪﻳﻨﺎ ‪⎟ = M (a ) :‬‬
‫‪2‬‬ ‫‪2‬‬ ‫⎠‪⎝0 1‬‬ ‫⎠ ‪2 ⎝ 1 −1‬‬ ‫‪2 ⎜⎝ a‬‬ ‫⎠⎟ ‪2 − a‬‬
‫⎛‬ ‫⎛ ⎞ ‪a‬‬ ‫⎞ ‪b‬‬
‫) (‬
‫‪M (a ) × M b = ⎜ I +‬‬ ‫‪A ⎟×⎜ I +‬‬
‫‪ab‬‬
‫‪A ⎟=I + A2 +‬‬
‫‪a‬‬
‫‪A+‬‬
‫‪b‬‬
‫ب‪ -‬ﻟﻴﻜﻦ ‪ . a,b ∈ E 2‬ﻟﺪﻳﻨﺎ ‪A :‬‬ ‫(‬ ‫)‬
‫⎝‬ ‫⎝ ⎠ ‪2‬‬ ‫⎠ ‪2‬‬ ‫‪2‬‬ ‫‪2‬‬ ‫‪2‬‬
‫⎛‬ ‫⎛ ⎞ ‪a‬‬ ‫⎞ ‪b‬‬ ‫‪a + b − ab 2‬‬
‫) (‬
‫‪M (a ) × M b = ⎜ I +‬‬ ‫‪A ⎟×⎜ I +‬‬ ‫‪A ⎟ = I − abA +‬‬
‫‪a‬‬
‫‪A+‬‬
‫‪b‬‬
‫‪A =I +‬‬ ‫‪A‬‬
‫⎝‬ ‫⎝ ⎠ ‪2‬‬ ‫⎠ ‪2‬‬ ‫‪2‬‬ ‫‪2‬‬ ‫‪2‬‬
‫‪a ⊥b‬‬
‫‪M (a ) × M (b ) = I +‬‬ ‫) ‪A = M (a ⊥ b‬‬
‫‪2‬‬
‫) ﺣﺴﺐ اﻟﺴﺆال ‪.1.I‬ب (‬ ‫‪a ⊥b ∈E‬‬ ‫) (‬ ‫(‬
‫‪ M (a ) × M b = M a ⊥ b‬و‬ ‫)‬ ‫إذن ‪:‬‬

‫‪.‬‬ ‫ﺟﺰء ﻣﺴﺘﻘﺮ ﻣﻦ )×‪( M2 ( \ ) ,‬‬ ‫‪F‬‬ ‫‪ . M (a ) × M (b ) = M (a ⊥ b ) ∈ F‬إذن‬ ‫وﻣﻨﻩ ﻓﺈن ‪:‬‬
‫‪ .2‬ﻧﻌﺘﺒﺮ اﻟﺘﻄﺒﻴﻖ ‪:‬‬
‫‪ϕ :‬‬ ‫)⊥ ‪( E ,‬‬ ‫→‬ ‫)× ‪( F ,‬‬
‫‪a‬‬ ‫) ‪6 ϕ (a ) = M (a‬‬
‫‪.‬‬ ‫أ‪ -‬ﻟﻴﻜﻦ ‪ . (a,b ) ∈ E 2‬ﻟﺪﻳﻨﺎ ‪ϕ (a ⊥ b ) = M (a ⊥ b ) = M (a ) × M (b ) = ϕ (a ) ⊥ ϕ (b ) :‬‬
‫إذن ‪ ϕ :‬ﺗﺸﺎآﻞ ﻣﻦ ) ⊥ ‪ ( E ,‬ﻧﺤﻮ )× ‪. ( F ,‬‬
‫‪. ∃a ∈ E‬‬ ‫‪ . ∃a ∈ E‬إذن ‪/ B = ϕ (a ) :‬‬
‫ﻟﻴﻜﻦ ‪ ، B ∈ F‬إذن ‪/ B = M (a ) :‬‬
‫وﻋﻠﻴﻩ ﻓﺈن ‪ ϕ‬ﺷﻤﻮﻟﻲ ﻣﻦ ‪ E‬ﻧﺤﻮ ‪. F‬‬

‫وﻟﺪﻳﻨﺎ ‪ϕ (a ) = ϕ (b ) ⇒ M (a ) = M (b ) ⇒ I + A = I + A ⇒ A = A ⇒ a = b :‬‬
‫‪a‬‬ ‫‪b‬‬ ‫‪a‬‬ ‫‪b‬‬
‫‪2‬‬ ‫‪2‬‬ ‫‪2‬‬ ‫‪2‬‬
‫إذن ‪ ϕ :‬ﺗﺒﺎﻳﻨﻲ ﻣﻦ ‪ E‬ﻧﺤﻮ ‪. F‬‬
‫وﺑﺎﻟﺘﺎﻟﻲ ﻓﺈن ‪ ϕ‬ﺗﺸﺎآﻞ ﺗﻘﺎﺑﻠﻲ ﻣﻦ ) ⊥ ‪ ( E ,‬ﻧﺤﻮ )×‪. ( F ,‬‬
‫زﻣﺮة ﺗﺒﺎدﻟﻴﺔ ‪ ،‬ﻓﺈن )× ‪ ( F ,‬زﻣﺮة ﺗﺒﺎدﻟﻴﺔ‪.‬‬ ‫)× ‪ ( F ,‬و ) ⊥ ‪( E ,‬‬ ‫ﺗﺸﺎآﻞ ﺗﻘﺎﺑﻠﻲ ﻣﻦ ) ⊥ ‪ ( E ,‬ﻧﺤﻮ‬ ‫ب‪ -‬ﺑﻤﺎ أن ‪ϕ‬‬
‫اﻟﺘﻤﺮﻳﻦ ‪: 2‬‬
‫‪.‬‬ ‫ﻟﻴﻜﻦ } ‪a ∈ ^ − {−i , i‬‬
‫‪ .‬ﻟﻴﻜﻦ ‪ . u = a + i :‬ﻟﺪﻳﻨﺎ ‪:‬‬ ‫) ‪(E‬‬ ‫(‬ ‫)‬
‫‪ .1 .I‬أ‪ -‬ﻧﻌﺘﺒﺮ ﻓﻲ ^ اﻟﻤﻌﺎدﻟﺔ ‪: z 2 − (1 + a ) (1 + i ) z + 1 + a 2 i = 0 :‬‬

‫(‬
‫= ‪u 2 − (1+ a ) (1+ i )u + 1+ a 2 i‬‬ ‫)‬ ‫‪(a + i ) − (1+ a ) (1+ i )(a + i ) + (1+ a 2 ) i‬‬
‫‪2‬‬

‫=‬ ‫‪a 2 + 2ai −1− (1+ a ) (a + i + ai −1) + i + a 2i‬‬


‫‪= a 2 + 2ai −1− a − i − ai +1− a 2 − ai − a 2i +1+ i + a 2i‬‬
‫(‬
‫= ‪u 2 − (1+ a ) (1+ i )u + 1+ a 2 i‬‬ ‫)‬ ‫‪0‬‬

‫إذن ‪ u = a + i :‬ﺣﻞ ﻟﻠﻤﻌﺎدﻟﺔ ) ‪. ( E‬‬

‫‪18‬‬ ‫ﺗﺼﺤﻴﺢ ﻣﻮﺿﻮع اﻟﺪورة اﻟﻌﺎدﻳﺔ ‪2007‬‬ ‫اﻷﺳﺘﺎذ ‪ :‬ﻣﺤﻤﺪ اﻟﺤﻴﺎن اﻟﺜﺎﻧﻴﺔ ﺑﻜﺎﻟﻮرﻳﺎ ﻋﻠﻮم رﻳﺎﺿﻴﺔ ‪- 2 -‬‬
: ‫ ﻟﺪﻳﻨﺎ‬. ( E ) ‫ اﻟﺤﻞ اﻵﺧﺮ ﻟﻠﻤﻌﺎدﻟﺔ‬v ‫ ﻟﻴﻜﻦ‬-‫ب‬

u +v = − = (1 + a ) (1 + i ) ⇒ v = (1 + a ) (1 + i ) − u
b
a
⇒ v = (1 + a ) (1 + i ) − (a + i )
⇒ v = 1 + i + a + ai − a − i
⇒ v = 1 + ai
1 2
: ‫ وﻣﻨﻩ ﻓﺈن‬، aa = a = 1 : ‫ إذن‬. a = 1 : ‫ ﻧﻔﺘﺮض أن‬.2
. a=
a
1
u ⎛u ⎞ u a + i a − i a − i 1 − ai i (1 − ai ) a + i u
. ∈ \ : ‫ ﻓﺈن‬، ⎜ ⎟ = = = = = = = = : ‫ ﺑﻤﺎ أن‬-‫أ‬
v ⎝ v ⎠ v 1 + ai 1 − ai 1 − i a − i i (a − i ) 1 + ai v
a
. a ⎡(a − a ) + 2i ⎤ = a
⎣( ) ⎦
u 2 = a ⎡⎢ a − a + 2i ⎤⎥ − aa + 2ai = a 2 − 1 + 2ai = (a + i ) = u 2 : ‫ ﻟﺪﻳﻨﺎ‬-‫ب‬
2 2
⎣⎢ ⎦⎥

arg (u ) ≡ arg ⎛⎜ a ⎡⎢(a − a ) + 2i ⎤⎥ ⎞⎟ ⎡⎣ 2π ⎤⎦


2
( )
: ‫ و ﻟﺪﻳﻨﺎ‬arg u 2 ≡ 2arg (u ) ⎡⎣ 2π ⎤⎦ : ‫ ﻟﺪﻳﻨﺎ‬-‫ﺟـ‬
⎝ ⎣ ⎦⎠
≡ arg (a ) + arg ( (a − a ) + 2i ) ⎡⎣ 2π ⎤⎦
. (a − a ) + 2i = 2i ℑm (a ) + 2i = 2 ( ℑm (a ) +1) i : ‫وﻟﺪﻳﻨﺎ‬

. ℑm (a ) ≤ a ⇒ ℑm (a ) ≤ 1 ⇒ −1 ≤ ℑm (a ) ≤ 1 ⇒ 0 ≤ ℑm (a ) +1 ≤ 2 : ‫وﻟﺪﻳﻨﺎ‬

. ( )
arg (a − a ) + 2i ≡ π ⎡⎣ 2π ⎤⎦
2
: ‫ إذن‬. (a − a ) + 2i = ⎡⎢⎣2 ( ℑm (a ) +1) , π2 ⎤⎥⎦ : ‫وﻣﻨﻩ ﻓﺈن‬
. 2arg (u ) ≡ arg (a ) + π ⎡⎣2π ⎤⎦ : ‫ أي‬. arg (u 2 ) ≡ arg (a ) + π ⎡⎣ 2π ⎤⎦ : ‫وﻋﻠﻴﻩ ﻓﺈن‬
2 2
. arg (u ) ≡ 1 arg (a ) + π ⎡⎣π ⎤⎦ : ‫وﺑﺎﻟﺘﺎﻟﻲ ﻓﺈن‬
2 4
: 1 ‫ ﻃﺮﻳﻘﺔ‬.3

u + iv = 2i = 2 ‫وﻣﻨﻩ ﻓﺈن‬. u + iv = a + i + i (1+ ai ) = 2i ‫و‬، u + v = u + iv ≥ u + iv :‫ﻟﺪﻳﻨﺎ‬

. u +v ≥ 2 : ‫وﺑﺎﻟﺘﺎﻟﻲ ﻓﺈن‬

: 2 ‫ﻃﺮﻳﻘﺔ‬

: ‫وﻣﻨﻩ ﻓﺈن‬ aa = 1 : ‫ إذن‬. a = 1 : ‫ﻟﺪﻳﻨﺎ‬

u + v = a + i + 1+ ai = a + i + aa + ai = a + i + a a + i = a + i + a + i ≥ a + i + a + i
. a + i + a + i = a + a + 2i = 2ℜe (a ) + 2i = 2 (ℜe (a ) + i ) : ‫وﻟﺪﻳﻨﺎ‬

a + i + a + i = 2 ℜe ( a ) + i = 2 ( ℜ e ( a ) ) + 1
2
. : ‫إذن‬

: ‫وﺑﻤﺎ أن‬

18 2007 ‫ﺗﺼﺤﻴﺢ ﻣﻮﺿﻮع اﻟﺪورة اﻟﻌﺎدﻳﺔ‬ - 3 - ‫ ﻣﺤﻤﺪ اﻟﺤﻴﺎن اﻟﺜﺎﻧﻴﺔ ﺑﻜﺎﻟﻮرﻳﺎ ﻋﻠﻮم رﻳﺎﺿﻴﺔ‬: ‫اﻷﺳﺘﺎذ‬
‫⇒ ‪ℜe ( a ) ≤ a‬‬ ‫‪ℜe ( a ) ≤ 1‬‬
‫⇒‬ ‫‪−1 ≤ ℜ e ( a ) ≤ 1‬‬

‫(‬ ‫)‬
‫‪2‬‬
‫⇒‬ ‫‪0 ≤ ℜe ( a ) ≤ 1‬‬

‫(‬ ‫)‬
‫‪2‬‬
‫‪⇒ 1 ≤ 1 + ℜe ( a ) ≤ 2‬‬

‫(‬ ‫)‬
‫‪2‬‬
‫) ‪⇒ 1 ≤ 1 + ℜe ( a‬‬

‫‪ℜe ( a ) ≤ a‬‬ ‫) ) ‪⇒ 2 ≤ 2 1 + ( ℜe ( a‬‬


‫‪2‬‬

‫⇒‬ ‫‪2 ≤ a +i +a +i‬‬


‫‪ ،‬وﻣﻨﻩ ﻧﺴﺘﻨﺘﺞ أن ‪:‬‬ ‫‪a +i +a +i ≥ 2‬‬ ‫و‬ ‫‪u + v ≥ a +i +a +i‬‬ ‫إذن ‪:‬‬

‫‪u +v ≥ 2‬‬
‫‪JJG JJG‬‬
‫(‬ ‫)‬
‫ﻣﻨﺴﻮب إﻟﻰ ﻣﻌﻠﻢ ﻣﺘﻌﺎﻣﺪ ﻣﻤﻨﻈﻢ ‪ . O ,e1,e 2‬ﻟﻴﻜﻦ ⎣⎡ ∞‪. m ∈ ⎤⎦ 2, +‬‬ ‫‪ .II‬اﻟﻤﺴﺘﻮى اﻟﻌﻘﺪي ‪P‬‬
‫ﺑﺤﻴﺚ ‪. u + v = m :‬‬ ‫) ‪ ( E m‬ﻣﺠﻤﻮﻋﺔ اﻟﻨﻘﻁ ) ‪ M (a‬ﻣﻦ اﻟﻤﺴﺘﻮى اﻟﻌﻘﺪي ‪P‬‬ ‫ﻧﻌﺘﺒﺮ‬
‫‪.‬‬ ‫‪u + v = a +i + a −i‬‬ ‫‪ .1‬ﻧﻌﻠﻢ أن ‪:‬‬
‫‪.‬‬ ‫‪ i‬و ‪−i‬‬ ‫اﻟﻠﺘﺎن ﻟﺤﻘﺎهﻤﺎ ﻋﻠﻰ اﻟﺘﻮاﻟﻲ‬ ‫) ‪ F ′ ( −i‬ﻧﻘﻄﺘﻴﻦ ﻣﻦ اﻟﻤﺴﺘﻮى اﻟﻌﻘﺪي ‪P‬‬ ‫‪ F i‬و‬ ‫ﻟﺘﻜﻦ ) (‬
‫‪.‬‬ ‫‪u + v = m ⇔ a + i + a − i = m ⇔ MF + MF ′ = m‬‬ ‫ﻟﺪﻳﻨﺎ ‪:‬‬

‫‪ ،‬ﻓﺈن ‪:‬‬ ‫‪ FF ′ = z F ′ − z F = −i − i = −2i = 2‬و ‪m = u + v ≥ 2‬‬ ‫وﺑﻤﺎ أن اﻟﻤﺴﺎﻓﺔ اﻟﺒﺆرﻳﺔ‬

‫‪ . m‬إذن ‪ ( E m ) :‬إهﻠﻴﻠﺞ ﻣﺮآﺰﻩ ﻣﻨﺘﺼﻒ اﻟﻘﻄﻌﺔ ⎦⎤‪ ⎡⎣ FF ′‬أي ‪ O‬أﺹﻞ اﻟﻤﻌﻠﻢ ‪.‬‬ ‫‪= 2b ≥ 2c‬‬

‫‪.‬‬ ‫‪( x , y ) ∈\2‬‬ ‫‪ .2‬ﻧﻀﻊ ‪ a = x + iy :‬ﺣﻴﺚ‬


‫أ‪ -‬ﻟﺪﻳﻨﺎ ‪:‬‬
‫⇔ ) ‪M (a ) ∈ ( E m‬‬ ‫‪u +v =m‬‬
‫‪⇔ MF + MF ′ = m‬‬
‫ﻧﻌﻠﻢ أن ‪ F ( 0,1) :‬و )‪ F ′ ( 0, −1‬و ) ‪، M ( x , y‬‬
‫‪JJJJJJG‬‬ ‫‪JJJJJJG‬‬
‫إذن ‪ MF ( −x ,1 − y ) :‬و ) ‪. MF ′ ( −x , −1 − y‬‬

‫) ‪MF ′2 = x 2 + (1+ y‬‬ ‫) ‪æ MF 2 = x 2 + (1− y‬‬


‫‪2‬‬ ‫‪2‬‬
‫‪.‬‬ ‫وﻣﻨﻩ ﻓﺈن ‪:‬‬

‫‪،‬‬ ‫‪( MF − MF ′)( MF + MF ′) = −4 y‬‬ ‫‪ ،‬وﻣﻨﻩ‬ ‫‪MF 2 − MF ′2 = −4 y‬‬ ‫إذن ‪:‬‬


‫‪،‬‬ ‫‪MF + MF ′ = m‬‬ ‫وﺑﻤﺎ أن ‪:‬‬
‫‪.‬‬ ‫‪m ( MF − MF ′) = −4 y‬‬ ‫ﻓﺈن ‪:‬‬

‫‪. MF − MF ′ = −‬‬
‫‪4y‬‬ ‫أي ‪:‬‬
‫‪m‬‬

‫‪18‬‬ ‫ﺗﺼﺤﻴﺢ ﻣﻮﺿﻮع اﻟﺪورة اﻟﻌﺎدﻳﺔ ‪2007‬‬ ‫اﻷﺳﺘﺎذ ‪ :‬ﻣﺤﻤﺪ اﻟﺤﻴﺎن اﻟﺜﺎﻧﻴﺔ ﺑﻜﺎﻟﻮرﻳﺎ ﻋﻠﻮم رﻳﺎﺿﻴﺔ ‪- 4 -‬‬
‫‪MF + MF ′‬‬ ‫=‬ ‫‪m‬‬
‫)‪(+‬‬
‫‪4‬‬
‫‪MF − MF ′‬‬ ‫=‬ ‫‪−‬‬ ‫‪y‬‬ ‫وﻋﻠﻴﻩ ﻓﺈﻧﻨﺎ ﻧﺠﺪ ‪:‬‬
‫‪m‬‬
‫‪−−−−−−−−− −−−−−−−−− −−−−−−−−−‬‬

‫‪2MF‬‬ ‫=‬ ‫‪m− 4 y‬‬


‫‪m‬‬
‫‪2‬‬
‫⎛‬ ‫⎞‬
‫‪MF 2 = ⎜⎜ m − 2 y‬‬ ‫⎟‬
‫⎟‬
‫=‬
‫‪m2‬‬ ‫‪4‬‬
‫‪− 2y + 2 y 2‬‬ ‫‪ .‬وﻣﻨﻩ ﻓﺈن ‪:‬‬ ‫= ‪MF‬‬
‫‪m 2‬‬
‫إذن ‪− y :‬‬
‫‪⎝ 2‬‬
‫‪m‬‬ ‫⎠‬
‫‪4‬‬ ‫‪m‬‬ ‫‪2 m‬‬
‫‪m 2 − 2y + 4‬‬ ‫) ‪y 2 = x 2 + (1− y‬‬ ‫) ‪MF 2 = x 2 + (1− y‬‬
‫‪2‬‬ ‫‪2‬‬
‫‪.‬‬ ‫‪ ،‬ﻓﺈن ‪:‬‬ ‫و ﺑﻤﺎ أن‬
‫‪4‬‬ ‫‪2‬‬
‫‪m‬‬
‫‪.‬‬
‫‪m 2 + 4 y 2 = x 2 +1+ y 2‬‬ ‫ﻳﻜﺎﻓﺊ‬
‫‪m 2 − 2 y + 4 y 2 = x 2 +1 − 2 y + y 2‬‬ ‫أي ‪:‬‬
‫‪4 m2‬‬ ‫‪4‬‬ ‫‪m2‬‬
‫⎛‬ ‫⎞‬ ‫‪2‬‬
‫‪.‬‬ ‫‪x 2 + ⎜⎜1− 42 ⎟⎟ y 2 = m −1‬‬ ‫وﺑﺎﻟﺘﺎﻟﻲ ﻓﺈن ﻣﻌﺎدﻟﺔ دﻳﻜﺎرﺗﻴﺔ ﻟﻺهﻠﻴﻠﺞ ) ‪ ( E m‬هﻲ ‪:‬‬
‫⎝‬ ‫‪m‬‬ ‫⎠‬ ‫‪4‬‬

‫ﺑﻄﺮﻳﻘﺔ أﺧﺮى ‪:‬‬

‫‪ ( E‬ﻓﻲ اﻟﻤﻌﻠﻢ ) ‪(O , i , j‬‬


‫‪JJG JJG‬‬
‫ﺗﻜﺘﺐ ﻋﻠﻰ ﺷﻜﻞ ‪:‬‬ ‫‪m‬‬ ‫ﻧﻌﻠﻢ أن ﻣﻌﺎدﻟﺔ دﻳﻜﺎرﺗﻴﺔ ﻟﻺهﻠﻴﻠﺞ )‬
‫‪FF ′ 2‬‬ ‫‪m‬‬ ‫‪a2‬‬ ‫‪x2 y2‬‬
‫‪.‬‬ ‫=‪c‬‬ ‫= ‪ 2b = m ⇒ b‬و ‪= = 1‬‬ ‫أي ‪ x 2 + 2 y 2 = a 2‬و‬ ‫‪+‬‬ ‫‪=1‬‬
‫‪2‬‬ ‫‪2‬‬ ‫‪2‬‬ ‫‪b‬‬ ‫‪a2 b 2‬‬
‫‪2‬‬
‫⎞ ‪2 ⎛m‬‬ ‫‪m2‬‬
‫= ‪. c = b − a ⇒ a = b − c = ⎜ ⎟ −12‬‬
‫‪2‬‬ ‫‪2‬‬ ‫‪2‬‬ ‫‪2‬‬ ‫‪2‬‬
‫إذن ‪−1 :‬‬
‫⎠‪⎝ 2‬‬ ‫‪4‬‬
‫⎛‬ ‫⎞‬ ‫‪2‬‬
‫‪.‬‬ ‫‪x 2 + ⎜⎜1− 42 ⎟⎟ y 2 = m −1‬‬ ‫وﺑﺎﻟﺘﺎﻟﻲ ﻓﺈن دﻳﻜﺎرﺗﻴﺔ ﻟﻺهﻠﻴﻠﺞ ) ‪ ( E m‬هﻲ ‪:‬‬
‫⎝‬ ‫‪m‬‬ ‫⎠‬ ‫‪4‬‬

‫‪2‬‬ ‫‪2‬‬
‫) (‬
‫‪. E4‬‬ ‫‪:‬‬
‫‪x‬‬
‫‪2‬‬
‫‪ . ( E 4 ) : x 2 + 3 y 2 = 3‬إذن ‪+ y 2 = 1 :‬‬
‫‪2‬‬ ‫‪4‬‬
‫ب‪ -‬ﻟﺪﻳﻨﺎ ‪:‬‬
‫‪3‬‬
‫‪b =2‬‬
‫‪2‬‬
‫‪ .‬وﻣﻨﻩ ﻓﺈن ‪. c = b 2 − a 2 = 22 − 3 = 1 :‬‬ ‫و‬ ‫إذن ‪a = 3 :‬‬
‫إهﻠﻴﻠﺞ ‪:‬‬ ‫وﻣﻨﻩ ﻓﺈن ) ‪( E 4‬‬
‫ﻣﺮآﺰﻩ ‪. O‬‬ ‫‪-‬‬
‫‪ A‬و ‪ A ′ − 3,0‬و ) ‪ B ( 0, 2‬و ) ‪. B ′ ( 0, −2‬‬ ‫(‬ ‫( )‬ ‫)‬
‫‪ -‬رؤوﺳﻩ ‪3,0‬‬
‫ﺑﺆرﺗﻴﻩ )‪ F ( 0,1‬و )‪. F ′ ( 0, −1‬‬ ‫‪-‬‬
‫‪c 1‬‬
‫‪.‬‬ ‫=‪e‬‬ ‫=‬ ‫ﺗﺒﺎﻋﺪﻩ اﻟﻤﺮآﺰي‬ ‫‪-‬‬
‫‪b 2‬‬

‫‪18‬‬ ‫ﺗﺼﺤﻴﺢ ﻣﻮﺿﻮع اﻟﺪورة اﻟﻌﺎدﻳﺔ ‪2007‬‬ ‫اﻷﺳﺘﺎذ ‪ :‬ﻣﺤﻤﺪ اﻟﺤﻴﺎن اﻟﺜﺎﻧﻴﺔ ﺑﻜﺎﻟﻮرﻳﺎ ﻋﻠﻮم رﻳﺎﺿﻴﺔ ‪- 5 -‬‬
‫‪ .3‬ﻃﺮﻳﻘﺔ ‪: 1‬‬
‫⎛‬ ‫⎞‬ ‫‪2‬‬
‫‪8‬‬
‫⎜‬ ‫‪4‬‬ ‫⎟‬ ‫‪9 2 9‬‬ ‫⎛‬ ‫⎞‬
‫‪. x 2 + ⎜1 − 2‬‬ ‫⇔ ‪⎟ y 2 = 7 −1‬‬ ‫‪x2+‬‬ ‫= ‪y‬‬ ‫‪: ⎜E‬‬ ‫ﻣﻌﺎدﻟﺔ اﻹهﻠﻴﻠﺞ ⎟‬
‫⎜‬ ‫‪8‬‬ ‫⎟‬ ‫‪4‬‬ ‫‪16‬‬ ‫‪7‬‬ ‫⎜‬ ‫‪8‬‬ ‫⎟‬
‫⎜‬ ‫⎟‬ ‫⎝‬ ‫‪7‬‬ ‫⎠‬
‫⎝‬ ‫‪7‬‬ ‫⎠‬
‫‪x −xA‬‬ ‫ﻣﻌﺎدﻟﺔ اﻟﻤﺴﺘﻘﻴﻢ ‪ AB‬هﻲ ‪y − y A ⇔ x − 3 = y − 0 ⇔ y = − 2 3 x + 2 :‬‬
‫‪.‬‬ ‫=‬
‫‪xB −xA y B − y A‬‬ ‫‪− 3‬‬ ‫‪2‬‬ ‫‪3‬‬ ‫) (‬
‫‪⎧ 2 9 2 9‬‬
‫‪⎪x +‬‬ ‫= ‪y‬‬ ‫⎛‬ ‫⎞‬
‫⎨ ‪. (*) :‬‬ ‫ﻟﻨﺤﺪ د ﺗﻘﺎﻃﻊ اﻟﻤﺴﺘﻘﻴﻢ ) ‪ ( AB‬واﻹهﻠﻴﻠﺞ ⎟ ‪ ⎜ E 8‬ﺑﺤﻞ اﻟﻨﻈﻤﺔ ‪:‬‬
‫⎪‬ ‫‪16‬‬ ‫‪7‬‬
‫‪⎪y = − 2 3 x + 2‬‬ ‫⎟‪⎜ 7‬‬
‫⎝‬ ‫⎠‬
‫⎩⎪‬ ‫‪3‬‬
‫‪⎧7 2 3‬‬
‫‪⎪ x -‬‬ ‫‪3x‬‬ ‫‪+ 9 = 9 ⎧⎪49x 2 − 42 3x + 27 = 0 ⎧⎪(7x -3 3)2 = 0‬‬
‫‪⎪4‬‬ ‫‪2‬‬ ‫‪4‬‬ ‫⎪⇔‪7‬‬ ‫⎨⎪ ⇔‬
‫)(‬
‫⎨⇔ *‬ ‫⎨‬ ‫‪2‬‬ ‫‪3‬‬ ‫‪2 3x +2‬‬
‫⎪‬ ‫‪2 3‬‬ ‫‪⎪y = −‬‬ ‫‪x +2‬‬ ‫‪⎪y = −‬‬
‫‪⎪y = − 3 x‬‬ ‫‪+2‬‬ ‫⎩⎪‬ ‫‪3‬‬ ‫⎪⎩‬ ‫‪3‬‬
‫⎩‬
‫⎧‬
‫‪⎪x‬‬ ‫‪= 373‬‬
‫⎨⎪ ⇔ *‬
‫)(‬
‫‪⎪y‬‬
‫⎪‬ ‫‪=8‬‬
‫⎩‬ ‫‪7‬‬
‫‪18‬‬ ‫ﺗﺼﺤﻴﺢ ﻣﻮﺿﻮع اﻟﺪورة اﻟﻌﺎدﻳﺔ ‪2007‬‬ ‫اﻷﺳﺘﺎذ ‪ :‬ﻣﺤﻤﺪ اﻟﺤﻴﺎن اﻟﺜﺎﻧﻴﺔ ﺑﻜﺎﻟﻮرﻳﺎ ﻋﻠﻮم رﻳﺎﺿﻴﺔ ‪- 6 -‬‬
‫⎛‬ ‫⎞‬ ‫⎛‬ ‫⎞‬
‫‪.‬‬ ‫⎟ ‪Ω⎜ 3 3 , 8‬‬
‫⎟‪⎜ 7 7‬‬
‫‪ ⎜ E‬ﻳﺘﻘﺎﻃﻌﺎن وﻓﻖ اﻟﻨﻘﻄﺔ‬
‫⎜‬ ‫‪8‬‬
‫واﻹهﻠﻴﻠﺞ ⎟‬
‫⎟‬
‫وﻣﻨﻩ ﻓﺈن اﻟﻤﺴﺘﻘﻴﻢ ) ‪( AB‬‬
‫⎝‬ ‫⎠‬ ‫⎝‬ ‫‪7‬‬ ‫⎠‬
‫⎛‬ ‫⎞‬
‫هﻲ ‪:‬‬ ‫‪ ⎜⎜ E 8‬ﻓﻲ اﻟﻨﻘﻄﺔ ‪Ω‬‬ ‫‪9‬‬ ‫‪9‬‬
‫ﻧﻌﻠﻢ أن ﻣﻌﺎدﻟﺔ اﻟﻤﻤﺎس ﻟﻺهﻠﻴﻠﺞ = ‪⎟ : x 2 + y 2‬‬
‫⎟‬ ‫‪16‬‬ ‫‪7‬‬
‫⎝‬ ‫‪7‬‬ ‫⎠‬

‫‪x x 0 + 9 yy 0 = 9 ⇔ x 3 3 + 9 y 8 = 9‬‬
‫‪16‬‬ ‫‪7‬‬ ‫‪7 16 7 7‬‬
‫‪⇔ 3 3x + 9 y = 9‬‬
‫‪7‬‬ ‫‪14‬‬ ‫‪7‬‬
‫‪⇔ 2x + 3y = 2 3‬‬
‫⎛‬ ‫⎞‬ ‫⎛‬ ‫⎞‬
‫‪.‬‬ ‫⎟ ‪Ω⎜ 3 3 , 8‬‬
‫⎟‪⎜ 7 7‬‬
‫‪ ⎜ E‬ﻓﻲ اﻟﻨﻘﻄﺔ‬
‫⎜‬ ‫‪8‬‬
‫ﻣﻤﺎس ﻟﻺهﻠﻴﻠﺞ ⎟‬
‫⎟‬
‫وﺑﺎﻟﺘﺎﻟﻲ ﻓﺈن اﻟﻤﺴﺘﻘﻴﻢ ) ‪( AB‬‬
‫⎝‬ ‫⎠‬ ‫⎝‬ ‫‪7‬‬ ‫⎠‬
‫ﻃﺮﻳﻘﺔ ‪: 2‬‬
‫⎛‬ ‫⎞‬ ‫‪8‬‬ ‫‪2‬‬
‫⎜‬ ‫‪4‬‬ ‫⎟‬ ‫‪9 2 9‬‬ ‫‪16 16 2‬‬ ‫⎛‬ ‫⎞‬
‫‪x 2 + ⎜1 − 2‬‬ ‫‪⎟ y = 7 −1 ⇔ x 2 +‬‬
‫‪2‬‬
‫‪y = ⇔ y =±‬‬ ‫‪− x‬‬ ‫ﻣﻌﺎدﻟﺔ اﻹهﻠﻴﻠﺞ ⎟ ‪: ⎜ E 8‬‬
‫⎜‬ ‫‪8‬‬ ‫⎟‬ ‫‪4‬‬ ‫‪16‬‬ ‫‪7‬‬ ‫‪7 9‬‬ ‫⎟‪⎜ 7‬‬
‫⎜‬ ‫⎟‬ ‫⎝‬ ‫⎠‬
‫⎝‬ ‫‪7‬‬ ‫⎠‬
‫⎤‪⎡ 3 7 3 7‬‬ ‫‪16 16 2‬‬
‫‪. ∀x ∈ ⎢ −‬‬ ‫‪,‬‬ ‫= ) ‪⎥ : g (x‬‬ ‫ﻧﻌﺘﺒﺮ اﻟﺪاﻟﺔ اﻟﻌﺪدﻳﺔ ‪ g‬اﻟﻤﻌﺮﻓﺔ ﺑﻤﺎ ﻳﻠﻲ ‪− x :‬‬
‫⎢⎣‬ ‫‪7‬‬ ‫⎥⎦ ‪7‬‬ ‫‪7 9‬‬
‫‪x − 3 y −0‬‬ ‫ﻣﻌﺎدﻟﺔ ‪ AB‬هﻲ ‪2 3 x + 2 :‬‬
‫‪.‬‬ ‫=‬ ‫‪⇔ y =−‬‬ ‫) (‬
‫‪− 3‬‬ ‫‪2‬‬ ‫‪3‬‬
‫‪2 3‬‬ ‫⎡‪⎤ 3 7 3 7‬‬ ‫⎛‬ ‫⎞‬
‫‪.‬‬ ‫‪g ′(x ) = −‬‬ ‫‪ x ∈ ⎥ −‬ﺑﺤﻴﺚ ‪:‬‬ ‫‪,‬‬ ‫‪ ، ⎜ E‬إذا وﺟﺪ ﻋﺪد ﺣﻘﻴﻘﻲ ⎢‬ ‫) ‪ ( AB‬ﻣﻤﺎس ﻟﻺهﻠﻴﻠﺞ ⎟‬
‫‪3‬‬ ‫‪⎦ 7‬‬ ‫⎣ ‪7‬‬ ‫⎜‬ ‫‪8‬‬ ‫⎟‬
‫⎝‬ ‫‪7‬‬ ‫⎠‬
‫‪2 3‬‬
‫‪.‬‬ ‫‪−g ′( x ) = −‬‬ ‫أو‬
‫‪3‬‬
‫‪⎛3 3⎞ 8‬‬ ‫‪2 3‬‬
‫‪g ′(x ) = −‬‬
‫‪3 3‬‬ ‫‪16‬‬ ‫‪x‬‬
‫⎜ ‪.y =g‬‬ ‫وﻣﻨﻩ = ⎟‬ ‫= ‪⇔x‬‬ ‫‪ g ′(x ) = −‬و‬ ‫ﻟﺪﻳﻨﺎ ‪:‬‬
‫‪⎜ 7 ⎟ 7‬‬ ‫‪3‬‬ ‫‪7‬‬ ‫‪9 16 19 2‬‬
‫⎝‬ ‫⎠‬ ‫‪− x‬‬
‫‪7‬‬ ‫‪9‬‬
‫⎛ ⎞‪⎛3 3 8‬‬ ‫⎞‬
‫⎜‪Ω‬‬ ‫‪, ⎟∈⎜ E‬‬ ‫ﻟﺪﻳﻨﺎ ‪⎟ :‬‬
‫⎜ ⎟‪⎜ 7 7‬‬ ‫‪8‬‬ ‫⎟‬
‫⎝‬ ‫⎝ ⎠‬ ‫‪7‬‬ ‫⎠‬
‫⎛‬ ‫⎞‬
‫‪ ⎜ E‬ﻓﻲ اﻟﻨﻘﻄﺔ ‪ Ω‬ﺗﺤﺤﺪد ﺑﻤﺎ ﻳﻠﻲ ‪:‬‬ ‫وﻟﺪﻳﻨﺎ ﻣﻌﺎدﻟﺔ اﻟﻤﻤﺎس ﻟﻺهﻠﻴﻠﺞ ⎟‬
‫⎜‬ ‫‪8‬‬ ‫⎟‬
‫⎝‬ ‫‪7‬‬ ‫⎠‬
‫‪3 3 9 8 9‬‬ ‫‪9‬‬ ‫‪2 3‬‬
‫‪x‬‬ ‫‪+ y = ⇔ 3 3x + y = 9 ⇔ y = −‬‬ ‫‪x +2‬‬
‫‪7 16 7 7‬‬ ‫‪2‬‬ ‫‪3‬‬
‫وهﻲ اﻟﻤﻌﺎدﻟﺔ اﻟﻤﺨﺘﺼﺮة ﻟﻠﻤﺴﺘﻘﻴﻢ ) ‪. ( AB‬‬
‫⎛‬ ‫⎞‬
‫‪. ⎜E‬‬ ‫وﺑﺎﻟﺘﺎﻟﻲ ﻓﺈن ) ‪ ( AB‬ﻣﻤﺎس ﻟﻺهﻠﻴﻠﺞ ⎟‬
‫⎜‬ ‫‪8‬‬ ‫⎟‬
‫⎝‬ ‫‪7‬‬ ‫⎠‬

‫‪18‬‬ ‫ﺗﺼﺤﻴﺢ ﻣﻮﺿﻮع اﻟﺪورة اﻟﻌﺎدﻳﺔ ‪2007‬‬ ‫اﻷﺳﺘﺎذ ‪ :‬ﻣﺤﻤﺪ اﻟﺤﻴﺎن اﻟﺜﺎﻧﻴﺔ ﺑﻜﺎﻟﻮرﻳﺎ ﻋﻠﻮم رﻳﺎﺿﻴﺔ ‪- 7 -‬‬
‫⎞‪⎛3 3‬‬ ‫‪2 3‬‬
‫‪ − g ′ ( x ) = −‬وﻣﻨﻩ ‪⎟ = −‬‬
‫‪8‬‬ ‫‪3 3‬‬
‫⎜ ‪. y = −g‬‬ ‫‪⇔x =−‬‬ ‫وﻟﺪﻳﻨﺎ ‪:‬‬
‫⎟ ‪⎜ 7‬‬ ‫‪7‬‬ ‫‪3‬‬ ‫‪7‬‬
‫⎝‬ ‫⎠‬
‫⎛‬ ‫⎞‬ ‫⎛ ⎞‪⎛ 3 3 8‬‬ ‫⎞‬
‫‪ Ω′ ⎜ −‬و ﻟﺪﻳﻨﺎ ﻣﻌﺎدﻟﺔ اﻟﻤﻤﺎس ﻟﻺهﻠﻴﻠﺞ ⎟ ‪ ⎜ E 8‬ﻓﻲ اﻟﻨﻘﻄﺔ ‪ Ω‬هﻲ ‪:‬‬ ‫‪, − ⎟∈⎜ E 8‬‬ ‫إذن ‪⎟ :‬‬
‫⎟‪⎜ 7‬‬ ‫‪⎜ 7‬‬ ‫‪7 ⎟⎠ ⎜⎝ 7‬‬ ‫⎟‬
‫⎝‬ ‫⎠‬ ‫⎝‬ ‫⎠‬
‫‪3 3 9 8 9‬‬ ‫‪9‬‬ ‫‪2 3‬‬
‫‪ . −x‬هﺬﻩ اﻟﻤﻌﺎدﻟﺔ ﻟﻴﺴﺖ اﻟﻤﻌﺎدﻟﺔ‬ ‫‪− y = ⇔ −3 3x − y = 9 ⇔ y = −‬‬ ‫‪x −2‬‬
‫‪7 16 7 7‬‬ ‫‪2‬‬ ‫‪3‬‬
‫اﻟﻤﺨﺘﺼﺮة ﻟﻠﻤﺴﺘﻘﻴﻢ ) ‪ ، ( AB‬إﻧﻤﺎ هﻲ اﻟﻤﻌﺎدﻟﺔ اﻟﻤﺨﺘﺼﺮة ﻟﻠﻤﺴﺘﻘﻴﻢ اﻟﻤﻮازي ل ) ‪ ( AB‬واﻟﻤﺎر ﻣﻦ ‪. Ω′‬‬

‫اﻟﺘﻤﺮﻳﻦ اﻟﺜﺎﻟﺚ ‪:‬‬


‫‪.‬‬ ‫) ‪(E‬‬ ‫‪ .1‬ﻧﻌﺘﺒﺮ ﻓﻲ ‪ ]2‬اﻟﻤﻌﺎدﻟﺔ‪: 195x − 232 y = 1 :‬‬
‫ﻟﺪﻳﻨﺎ ‪:‬‬ ‫أ‪ -‬ﻃﺮﻳﻘﺔ ‪: 1‬‬

‫إذن ‪ 195 = 3 × 5 ×13 :‬و ‪. 232 = 23 × 29‬‬

‫‪.‬‬ ‫‪232 ∧195 = 1‬‬ ‫وﻣﻨﻩ ﻓﺈن ‪:‬‬

‫‪18‬‬ ‫ﺗﺼﺤﻴﺢ ﻣﻮﺿﻮع اﻟﺪورة اﻟﻌﺎدﻳﺔ ‪2007‬‬ ‫اﻷﺳﺘﺎذ ‪ :‬ﻣﺤﻤﺪ اﻟﺤﻴﺎن اﻟﺜﺎﻧﻴﺔ ﺑﻜﺎﻟﻮرﻳﺎ ﻋﻠﻮم رﻳﺎﺿﻴﺔ ‪- 8 -‬‬
‫ﺣﺴﺐ ﺗﻘﻨﻴﺔ اﻟﻘﺴﻤﺔ اﻷﻗﻠﻴﺪﻳﺔ اﻟﻤﺘﺘﺎﺑﻌﺔ ﻷﻗﻠﻴﺪس ‪ ،‬ﻟﺪﻳﻨﺎ ‪:‬‬ ‫ﻃﺮﻳﻘﺔ ‪: 2‬‬

‫‪ .‬ﺑﺎﻹﻇﺎﻓﺔ إﻟﻰ ﻣﻌﺎﻣﻠﻲ ‪.Bezout‬‬ ‫وﻣﻨﻩ ﻧﺴﺘﻨﺘﺞ أن ‪232 ∧195 = 1‬‬


‫ب‪ -‬ﻟﺪﻳﻨﺎ ‪:‬‬

‫) ‪232 / 195 ( x + 69‬‬


‫‪Gauss‬‬
‫‪∃h ∈ \ / x + 69 = 232h‬‬ ‫‪ .‬وﻣﻨﻩ ﻓﺈن ‪:‬‬ ‫⇒‬ ‫‪232 / x + 69‬‬ ‫إذن ‪:‬‬

‫‪ ،‬ﻓﻨﺠﺪ ‪:‬‬‫أي ‪ . ∃h ∈ \ / x = −69 + 232h :‬ﻧﻌﻮض هﺬا اﻟﺘﻌﺒﻴﺮ ﻓﻲ اﻟﻌﻻﻗﺔ )*(‬


‫‪. 195 × 232h = 232 ( y + 58) ⇔ 195h = y + 58 ⇔ y = −58 +195h‬‬

‫وﻣﻨﻩ ﻓﺈن ‪ ( x , y ) = ( −69 + 232h , −58 + 195h ) :‬ﺣﻴﺚ ] ∈ ‪ . h‬أي ‪:‬‬

‫) )‪. ( x , y ) = ( −69 + 232 + 232 ( h −1) , −58 + 195 + 195 ( h − 1‬‬

‫ﻳﻜﺎﻓﺊ ‪ ( x , y ) = (163 + 232 ( h − 1) ,137 + 195 ( h −1) ) :‬وﺑﻮﺿﻊ ‪ ، k = h −1‬ﻧﺠﺪ ‪ k ∈] :‬و‬

‫) ‪ ( x , y ) = (163 + 232k ,137 + 195k‬ﺣﻴﺚ ] ∈ ‪. k‬‬

‫وﺑﻤﺎ أن اﻷزواج ) ‪ ، (163 + 232k ,137 + 195k‬ﺣﻴﺚ ] ∈ ‪ ، k‬ﺗﺤﻘﻖ اﻟﻤﻌﺎدﻟﺔ ) ‪ ، ( E‬ﻓﺈن ﻣﺠﻤﻮﻋﺔ‬

‫}] ∈ ‪S = {(163 + 232k ,137 +195k ) / k‬‬ ‫ﺣﻠﻮل اﻟﻤﻌﺎدﻟﺔ ) ‪ ( E‬هﻲ ‪:‬‬

‫‪ 0 ≤ d‬و ⎦⎤ ‪.195d ≡ 1 ⎡⎣ 232‬‬ ‫ﺟـ‪ -‬ﻟﻴﻜﻦ ‪ d‬ﻋﺪدا ﺹﺤﻴﺤﺎ ﻃﺒﻴﻌﻴﺎ ﺑﺤﻴﺚ ‪≤ 232‬‬
‫‪ .‬ﺣﺴﺐ اﻟﺴﺆال ‪.1‬ب ‪ ،‬ﻟﺪﻳﻨﺎ ‪:‬‬ ‫ﻟﺪﻳﻨﺎ ‪∃m ∈ ] / 195d − 232m = 1 :‬‬
‫‪. ∃k ∈ ] /‬‬ ‫‪d = 163 + 232k‬‬ ‫‪ .‬إذن ‪:‬‬ ‫‪∃k ∈ ] /‬‬ ‫) ‪(d , m ) = (163 + 232k ,137 +195k‬‬

‫‪18‬‬ ‫ﺗﺼﺤﻴﺢ ﻣﻮﺿﻮع اﻟﺪورة اﻟﻌﺎدﻳﺔ ‪2007‬‬ ‫اﻷﺳﺘﺎذ ‪ :‬ﻣﺤﻤﺪ اﻟﺤﻴﺎن اﻟﺜﺎﻧﻴﺔ ﺑﻜﺎﻟﻮرﻳﺎ ﻋﻠﻮم رﻳﺎﺿﻴﺔ ‪- 9 -‬‬
‫‪0 ≤ d ≤ 232 ⇔ 0 ≤ 163 + 232k ≤ 232‬‬
‫⇔‬ ‫‪−163 ≤ 232k ≤ 69‬‬
‫‪163‬‬ ‫‪69‬‬
‫⇔‬ ‫‪−‬‬ ‫≤ ‪≤k‬‬
‫‪232‬‬ ‫‪232‬‬
‫‪163‬‬ ‫‪69‬‬
‫‪ ، −‬ﻓﺈن ‪ . k = 0 :‬إذن ‪. d = 163 :‬‬ ‫و ‪≈ −0,70‬‬ ‫وﺑﻤﺎ أن ‪ k ∈] :‬و ‪≈ 0,29‬‬
‫‪232‬‬ ‫‪232‬‬
‫‪N = pq + r ; 0 ≤ r < p‬‬ ‫‪ .2‬ﻟﺪﻳﻨﺎ ‪. N = 233 :‬‬
‫‪p‬‬ ‫‪q‬‬ ‫‪r‬‬ ‫‪p‬‬ ‫‪2‬‬

‫‪2 116 1‬‬ ‫‪4‬‬


‫‪3 77 2‬‬ ‫‪9‬‬
‫‪5 46 3‬‬ ‫‪25‬‬
‫‪7 33 2‬‬ ‫‪49‬‬
‫‪11 21 2‬‬ ‫‪121‬‬
‫‪13 17 12‬‬ ‫‪169‬‬
‫‪17 13 12 289‬‬ ‫‪Stop‬‬
‫إذن ‪ N = 233‬ﻋﺪد أوﻟﻲ ‪.‬‬
‫ﻧﺘﻮﻗﻒ ﻓﻲ ﺣﺎﻟﺔ ‪ ) q < p‬أو ‪.( p > N‬‬
‫‪2‬‬

‫‪.3‬ﻟﺘﻜﻦ ` ∩ ⎦⎤‪ . A = ⎡⎣0,232‬ﻟﻴﻜﻦ ‪ f : A → A‬ﺗﻄﺒﻴﻘﺎ ﻳﺮﺑﻁ آﻞ ﻋﻨﺼﺮ ‪ a ∈ A‬ﺑﺎﻟﻌﻨﺼﺮ ) ‪ f (a‬ﺣﻴﺚ‬

‫‪.‬‬ ‫‪233‬‬ ‫ﻋﻠﻰ‬ ‫‪a195‬‬ ‫هﻮ ﺑﺎﻗﻲ اﻟﻘﺴﻤﺔ اﻷﻗﻠﻴﺪﻳﺔ ﻟﻠﻌﺪد‬ ‫) ‪f (a‬‬
‫‪. ∀a ∈ A‬‬ ‫⎦⎤‪\ {0} : a 232 ≡ 1 ⎡⎣233‬‬ ‫ﻧﻘﺒﻞ أن ‪:‬‬

‫ﻣﺒﺮهﻨﺔ ﻓﻴﺮﻣﺎ ‪ :‬إذا آﺎن ‪ n‬ﻋﺪدا ﺹﺤﻴﺤﺎ ﻃﺒﻴﻌﻴﺎ أوﻟﻴﺎ ‪ ،‬ﻓﺈن ‪∀a ∈ ` \ {0} : a n −1 ≡ 1 ⎡⎣ n ⎤⎦ :‬‬
‫‪.‬‬ ‫‪233‬‬ ‫هﻮ ﺑﺎﻗﻲ اﻟﻘﺴﻤﺔ اﻷﻗﻠﻴﺪﻳﺔ ﻟﻠﻌﺪد ‪ a195‬ﻋﻠﻰ‬ ‫) ‪f (a‬‬ ‫‪ .‬ﻟﺪﻳﻨﺎ‬ ‫‪f (a ) = b‬‬ ‫ﺑﺤﻴﺚ‬ ‫أ‪ -‬ﻟﻴﻜﻦ ‪(a,b ) ∈ A 2‬‬
‫‪.‬‬ ‫‪0 ≤ b < 233‬‬ ‫و‬ ‫⎦⎤‪a195 ≡ b ⎡⎣ 233‬‬ ‫‪.‬وﻣﻨﻩ ﻓﺈن ‪:‬‬ ‫‪0 ≤ f (a ) < 233‬‬ ‫و‬ ‫⎦⎤‪a195 ≡ f (a ) ⎡⎣ 233‬‬ ‫إذن ‪:‬‬

‫وﻟﺪﻳﻨﺎ ‪ . 195 ×163 ≡ 1 ⎡⎣ 232⎤⎦ :‬إذن ‪ 195 ×163 = 1 + 232k :‬ﺣﻴﺚ‬ ‫⎦⎤‪a195×163 ≡ b 163 ⎡⎣233‬‬ ‫وﻣﻨﻩ ﻓﺈن ‪:‬‬

‫‪. a 232k‬‬ ‫⎦⎤‪≡ 1 ⎡⎣ 233‬‬ ‫‪ ،‬إذن ‪:‬‬ ‫⎦⎤‪a 232 ≡ 1 ⎡⎣233‬‬ ‫‪ .‬ﻧﻌﻠﻢ أن ‪:‬‬ ‫⎦⎤‪a1+232k ≡ b 163 ⎡⎣233‬‬ ‫] ∈ ‪ . k‬إذن ‪:‬‬

‫و ‪. 0 ≤ a ≤ 232 < 233‬‬ ‫⎦⎤‪a ≡ b 163 ⎡⎣ 233‬‬ ‫‪ .‬إذن ‪:‬‬ ‫⎦⎤‪a1+232k ≡ 1⎡⎣ 233‬‬ ‫وﻋﻠﻴﻩ ﻓﺈن ‪:‬‬

‫‪.‬‬ ‫وﺑﺎﻟﺘﺎﻟﻲ ﻧﺴﺘﻨﺘﺞ أن ‪ a‬هﻮ ﺑﺎﻗﻲ اﻟﻘﺴﻤﺔ اﻷﻗﻠﻴﺪﻳﺔ ل ‪ b 163‬ﻋﻠﻰ ‪233‬‬

‫هﻮ اﻟﺘﻄﺒﻴﻖ اﻟﺬي ﻳﺮﺑﻁ آﻞ ﻋﻨﺼﺮ‬ ‫ﺟـ‪ -‬ﺣﺴﺐ ‪.3‬أ ‪ ،‬ﻟﺪﻳﻨﺎ ‪ f‬ﺗﻘﺎﺑﻞ ﻣﻦ ‪ A‬ﻧﺤﻮ ‪ A‬وﻟﺪﻳﻨﺎ ‪f −1 : A → A‬‬
‫‪ b ∈ A‬ﺑﺎﻟﻌﻨﺼﺮ ‪ f b‬ﺣﻴﺚ ‪ f b‬هﻮ ﺑﺎﻗﻲ اﻟﻘﺴﻤﺔ اﻷﻗﻠﻴﺪﻳﺔ ل ‪ b 163‬ﻋﻠﻰ ‪. 233‬‬ ‫) (‬ ‫) (‬
‫اﻟﺘﻤﺮﻳﻦ اﻟﺮاﺑﻊ ‪:‬‬
‫ﺑﻤﺎ ﻳﻠﻲ ‪:‬‬ ‫\‬ ‫اﻟﻤﻌﺮﻓﺔ ﻋﻠﻰ‬ ‫‪ .I‬ﻧﻌﺘﺒﺮ اﻟﺪاﻟﺔ اﻟﻌﺪدﻳﺔ ‪g‬‬
‫→ \ ‪g :‬‬ ‫\‬
‫‪x 6 g ( x ) = 1 + ( x −1)e x‬‬
‫‪. g (x‬‬ ‫‪) = 1+ ( x −1)e x‬‬ ‫‪= 1 − e x + xe x‬‬ ‫‪ ،‬ﻟﺪﻳﻨﺎ ‪:‬‬ ‫‪ .1‬ﻟﻴﻜﻦ \ ∈ ‪x‬‬
‫‪ . g ′ ( x ) = (1 + ( x −1)e x ) = e x + ( x −1)e x = xe x‬إﺷﺎرة ) ‪ g ′ ( x‬هﻲ إﺷﺎرة ‪x‬‬
‫‪′‬‬
‫‪.‬‬ ‫إذن ‪:‬‬

‫‪lim‬‬
‫∞‪x →−‬‬
‫‪g ( x ) = xlim‬‬
‫∞‪→−‬‬
‫‪1+ x e x − e x = 1‬‬ ‫‪ lim‬و‬
‫∞‪x →+‬‬
‫‪g ( x ) = xlim‬‬
‫∞‪→+‬‬
‫∞‪1 + ( x −1)e x = +‬‬
‫‪18‬‬ ‫ﺗﺼﺤﻴﺢ ﻣﻮﺿﻮع اﻟﺪورة اﻟﻌﺎدﻳﺔ ‪2007‬‬ ‫اﻷﺳﺘﺎذ ‪ :‬ﻣﺤﻤﺪ اﻟﺤﻴﺎن اﻟﺜﺎﻧﻴﺔ ﺑﻜﺎﻟﻮرﻳﺎ ﻋﻠﻮم رﻳﺎﺿﻴﺔ ‪- 10 -‬‬
‫‪.‬‬ ‫‪ 0‬ﻗﻴﻤﺔ دﻧﻴﺎ ﻣﻄﻠﻘﺔ ﻟﻠﺪاﻟﺔ ‪ g‬ﻋﻠﻰ \ ‪ .‬إذن ‪∀x ∈ \ : g ( x ) ≥ 0 :‬‬
‫‪ . g‬وﻟﺪﻳﻨﺎ ‪:‬‬ ‫ﻟﺪﻳﻨﺎ ‪( 0) = 0‬‬ ‫‪.2‬‬
‫‪.‬‬ ‫‪x < 0 ⇒ g ( x ) > g ( 0) ⇒ g ( x ) > 0‬‬ ‫ﺗﻨﺎﻗﺼﻴﺔ ﻗﻄﻌﺎ ﻋﻠﻰ ⎣⎡ ‪ . ⎤⎦ −∞,0‬إذن ‪:‬‬ ‫‪g‬‬
‫‪.‬‬ ‫‪x > 0 ⇒ g ( x ) > g ( 0) ⇒ g ( x‬‬ ‫ﺗﺰاﻳﺪﻳﺔ ﻗﻄﻌﺎ ﻋﻠﻰ ⎣⎡ ∞‪ . ⎤⎦ 0, +‬إذن ‪) > 0 :‬‬ ‫‪g‬‬
‫‪.‬‬ ‫‪∀x ∈ \* : g ( x ) > 0‬‬ ‫وﺑﺎﻟﺘﺎﻟﻲ ﻓﺈن ‪:‬‬

‫ﺑﻤﺎ ﻳﻠﻲ ‪:‬‬ ‫‪ .II‬ﻟﺘﻜﻦ ‪ f‬اﻟﺪاﻟﺔ اﻟﻌﺪدﻳﺔ اﻟﻤﻌﺮﻓﺔ ﻋﻠﻰ \‬


‫⎧‬
‫‪⎪f‬‬ ‫‪( x ) = e xx−1‬‬ ‫‪; x ≠0‬‬
‫⎨‬
‫⎪‬ ‫‪f ( 0) = 1‬‬
‫⎩‬
‫‪1‬‬ ‫‪x‬‬ ‫‪x‬‬ ‫‪x‬‬ ‫‪1‬‬
‫‪. lim‬‬ ‫‪ ، lim f ( x ) = lim x‬ﻷن ‪ lim x = 0 :‬و ‪= 0‬‬ ‫=‬ ‫‪lim‬‬ ‫‪= 0 .1‬‬
‫‪x →+∞ e‬‬ ‫‪x‬‬ ‫‪x →+∞ e‬‬ ‫∞‪x →+‬‬ ‫‪x →+∞ e − 1 x →+∞ e‬‬ ‫‪x‬‬ ‫‪1‬‬
‫‪1− x‬‬
‫‪e‬‬
‫‪x‬‬
‫‪x‬‬ ‫‪xe‬‬
‫‪ ، lim f ( x ) + x = lim x‬ﻷن ‪ lim xe x = 0 :‬و ‪. lim e x = 0‬‬
‫∞‪x →−‬‬ ‫∞‪x →−‬‬ ‫∞‪x →−‬‬ ‫‪x →−∞ e − 1‬‬
‫‪+ x = xlim‬‬ ‫‪= 0 =0‬‬
‫‪→−∞ e x − 1 0 − 1‬‬
‫‪e x −1‬‬
‫‪= 1 = f ( 0 ) .2‬‬
‫‪x‬‬
‫‪ ، xlim‬ﻷن ‪= 1 :‬‬ ‫‪f ( x ) = xlim‬‬
‫‪1‬‬
‫‪. lim‬‬
‫‪x →0 x‬‬ ‫‪→0‬‬
‫‪= lim‬‬
‫‪→0 e x − 1 x →0 e x − 1‬‬

‫‪x‬‬
‫إذن ‪ f‬ﻣﺘﺼﻠﺔ ﻓﻲ ‪. 0‬‬
‫‪ .3‬أ‪ -‬ﻟﻴﻜﻦ *\ ∈ ‪ ، x‬ﻟﺪﻳﻨﺎ ‪:‬‬

‫‪f ′(x ) = ⎜ x‬‬ ‫=‬


‫‪x‬‬
‫(‬ ‫‪x‬‬
‫)‬
‫‪⎛ x ⎞′ x ′ e − 1 − x e − 1‬‬
‫=‬
‫(‬
‫‪e x −1 − xe x‬‬ ‫‪)′‬‬
‫=‬
‫‪−1 − ( x −1)e x‬‬
‫‪=−‬‬
‫) ‪g (x‬‬
‫⎟‬
‫⎠ ‪⎝ e −1‬‬ ‫(‬ ‫)‬ ‫(‬ ‫)‬ ‫)‪(e x −1‬‬ ‫)‪(e x −1‬‬
‫‪2‬‬ ‫‪2‬‬ ‫‪2‬‬ ‫‪2‬‬
‫‪e x −1‬‬ ‫‪e x −1‬‬
‫‪.‬‬ ‫ب‪ -‬إﺷﺎرة ) ‪ f ′ ( x‬ﻋﻠﻰ *\ هﻲ إﺷﺎرة ) ‪ . − g ( x‬وﻣﻨﻩ ﻧﺴﺘﻨﺘﺞ ﺟﺪول ﺗﻐﻴﺮات اﻟﺪاﻟﺔ ‪ f‬ﻋﻠﻰ \‬
‫‪x‬‬
‫‪.‬‬ ‫‪lim e x = 0‬‬ ‫‪ ، lim f ( x ) = lim‬ﻷن ‪:‬‬ ‫∞‪= +‬‬
‫∞‪x →−‬‬ ‫∞‪x →−‬‬ ‫‪x →−∞ e − 1‬‬
‫‪x‬‬

‫‪18‬‬ ‫ﺗﺼﺤﻴﺢ ﻣﻮﺿﻮع اﻟﺪورة اﻟﻌﺎدﻳﺔ ‪2007‬‬ ‫اﻷﺳﺘﺎذ ‪ :‬ﻣﺤﻤﺪ اﻟﺤﻴﺎن اﻟﺜﺎﻧﻴﺔ ﺑﻜﺎﻟﻮرﻳﺎ ﻋﻠﻮم رﻳﺎﺿﻴﺔ ‪- 11 -‬‬
J ( x ) = ∫ te −t dt : ‫ ﻟﺪﻳﻨﺎ‬، x ∈ \ ‫ ﻟﻴﻜﻦ‬.4
x
.
0
u (t ) = −e −t
u ′ (t ) = e −t
: ‫ إذن‬، : ‫ ﻧﻀﻊ‬-‫أ‬
v ′ (t ) = 1 v (t ) = t
: ‫ ﻟﺪﻳﻨﺎ‬، ‫ ﺣﺴﺐ ﺗﻘﻨﻴﺔ اﻟﻤﻜﺎﻣﻠﺔ ﺑﺎﻷﺟﺰاء‬. \ ‫ ﻣﺘﺼﻠﺘﺎن وﻗﺎﺑﻠﺘﺎن ﻟﻻﺷﺘﻘﺎق ﻋﻠﻰ‬v ‫ و‬u : ‫ﻟﺪﻳﻨﺎ‬
x
J( ) ∫x = te −t dt = 0 u ′ (t )v (t ) dt = ⎡⎣u (t )v (t )⎤⎦ 0 − 0 u ′ (t )v (t ) dt
x x x

0 ∫ ∫
J ( x ) = ⎡⎣−te −t ⎤⎦ 0 − ∫0 −e −t dt = −xe − x − 0 − ⎡⎣e −t ⎤⎦ 0 = −xe − x − e − x + 1
x x x

J ( x ) = e −x ⎛⎜e x −1− x ⎞⎟ : ‫وﺑﺎﻟﺘﺎﻟﻲ ﻓﺈن‬


⎝ ⎠
J ( x ) = ∫ te −t dt
x
. : ‫ ﻟﺪﻳﻨﺎ‬، x ∈\ ‫ ﻟﻴﻜﻦ‬-‫ب‬
0
. 0 ≤ t ≤ x ⇒ −x ≤ −t ≤ 0 ⇒ e −x ≤ e −t ≤ 1 ⇒ te −x ≤ te ≤ t : ‫ ﻓﺈن‬، x ∈ \ + ‫إذا آﺎن‬ −t
x x
⎡t 2 ⎤ ⎡t 2 ⎤ x 2 −x 2
te −x dt ≤ ∫0 te −t dt ≤ ∫0 t dt ⇒ e ( ) ⎢2⎥ e ≤ J (x )≤ x
x x x
∫0
−x
⎢ ⎥ ≤ J x ≤ ⇒ : ‫إذن‬
⎢⎣ 2 ⎥⎦ 0 ⎢⎣ ⎥⎦ 0 2 2
x +x x −x
x2 − 2 x2 −
. e ≤ J (x ) ≤ e 2 : ‫وﻣﻨﻩ ﻓﺈن‬
2 2
. t ≤ 0 ‫ﻷن‬، x ≤ t ≤ 0 ⇒ 0 ≤ −t ≤ −x ⇒ 1 ≤ e −t ≤ e − x ⇒ te − x ≤ te −t ≤ t : ‫ ﻓﺈن‬، x ∈ \ − ‫إذا آﺎن‬
0 0
⎡t 2 ⎤ ⎡t 2 ⎤ x 2 −x 2
te −x dt ≤ ∫x te −t dt ≤ ∫x t dt ⇒ e ⎢ ⎥ ≤ −J ( x ) ≤ ⎢ ⎥ ⇒ − e ≤ −J ( x ) ≤ − x2
0 0 0
∫x
−x
:‫إذن‬
⎣⎢ 2 ⎦⎥ x ⎣⎢ 2 ⎥⎦ x 2
x +x x −x
x2 − x2 − x2 x2
. e 2 ≤ J (x ) ≤ e 2 : ‫ وﻣﻨﻩ ﻓﺈن‬، ≤ J ( x ) ≤ e −x : ‫وﻋﻠﻴﻩ ﻓﺈن‬
2 2 2 2
. x = 0 ‫هﺬﻩ اﻟﻌﻻﻗﺔ ﺗﻈﻞ ﺹﺤﻴﺤﺔ ﻣﻦ أﺟﻞ‬
x +x x −x
x2 − x2 −
∀x ∈ \ : e 2 ≤ J (x ) ≤ e 2 : ‫وﺑﺎﻟﺘﺎﻟﻲ ﻓﺈن‬
2 2
x +x x −x
.
x2 −
2
e 2 x2 −
≤ J ( x ) ≤ e 2 ‫ و‬J ( x ) = e −x e x −1− x : ‫ ﻟﺪﻳﻨﺎ‬، x ∈ \* ‫ ﻟﻴﻜﻦ‬-‫ﺟـ‬
2 ( )
x +x x −x x +x x −x
1 x − e x −1 − x 1 x − 2 x2 − 2 x2 − 2
e e 2 ≤ ≤ e e : ‫ وﻣﻨﻩ‬، e ≤ e (e −1 − x ) ≤ e
−x x
: ‫إذن‬
2 x2 2 2 2
x −x x +x
≤ e −12− x ≤ 1 e
1e x
. ∀x ∈ \* : 2 2 : ‫وﺑﺎﻟﺘﺎﻟﻲ ﻓﺈن‬
2 x 2
18 2007 ‫ﺗﺼﺤﻴﺢ ﻣﻮﺿﻮع اﻟﺪورة اﻟﻌﺎدﻳﺔ‬ - 12 - ‫ ﻣﺤﻤﺪ اﻟﺤﻴﺎن اﻟﺜﺎﻧﻴﺔ ﺑﻜﺎﻟﻮرﻳﺎ ﻋﻠﻮم رﻳﺎﺿﻴﺔ‬: ‫اﻷﺳﺘﺎذ‬
‫‪x +x‬‬ ‫‪x −x‬‬ ‫‪x −x‬‬ ‫‪x +x‬‬
‫‪≤ e −12− x ≤ 1 e‬‬
‫‪x‬‬
‫‪lim e‬‬
‫‪1‬‬ ‫‪2‬‬ ‫‪1‬‬
‫‪ xlim‬و =‬
‫‪1‬‬
‫‪e‬‬ ‫‪2‬‬ ‫‪1‬‬ ‫‪1‬‬
‫‪ ∀x ∈ \* : e‬و =‬ ‫‪2‬‬ ‫د‪ -‬ﻟﺪﻳﻨﺎ ‪2 :‬‬
‫‪x →0 2‬‬ ‫‪2‬‬ ‫→‬ ‫‪02‬‬ ‫‪2‬‬ ‫‪2‬‬ ‫‪x‬‬ ‫‪2‬‬
‫‪e x −1 − x 1‬‬
‫‪lim‬‬ ‫=‬ ‫ﺣﺴﺐ ﺧﺎﺹﻴﺎت اﻟﺘﺮﺗﻴﺐ واﻟﻨﻬﺎﻳﺎت ‪ ،‬ﻟﺪﻳﻨﺎ ‪:‬‬
‫‪x →0‬‬ ‫‪x2‬‬ ‫‪2‬‬
‫‪x‬‬
‫)‪f ( x ) − f ( 0‬‬ ‫‪−1‬‬
‫وﻣﻨﻩ ﻓﺈن ‪e − 1 = lim x − e + 1 = lim − e −1 − x 1 = − 1 :‬‬
‫‪x‬‬ ‫‪x‬‬ ‫‪x‬‬
‫‪lim‬‬ ‫=‬ ‫‪lim‬‬
‫‪x →0‬‬ ‫‪x −0‬‬ ‫‪x →0‬‬ ‫‪x‬‬ ‫‪( ) x →0‬‬
‫‪x →0 x e x − 1‬‬ ‫‪x2‬‬ ‫‪e x −1‬‬ ‫‪2‬‬
‫‪x‬‬
‫‪f ′ ( 0) = −‬‬
‫‪1‬‬
‫‪.‬‬ ‫ﻗﺎﺑﻠﺔ ﻟﻻﺷﺘﻘﺎق ﻓﻲ ‪ 0‬وﻟﺪﻳﻨﺎ ‪:‬‬ ‫‪f‬‬ ‫إذن‬
‫‪2‬‬
‫‪ ،‬ﻟﺪﻳﻨﺎ ‪:‬‬ ‫‪ .5‬أ‪ -‬ﻟﻴﻜﻦ *\ ∈ ‪x‬‬
‫⎛‬
‫⎜⎜ = ) ‪f ′′ ( x‬‬
‫‪−‬‬ ‫‪g‬‬ ‫=⎟) (‬
‫‪x‬‬
‫=‬
‫(‬
‫‪⎞′ − g ′ ( x ) (e x − 1)2 + g ( x ) (e x − 1)2 ′ −xe x e x − 1 2 + 2e x g x‬‬
‫(‬ ‫)‬ ‫(‬ ‫)‬ ‫)‪) (e x −1‬‬
‫⎟‪2‬‬
‫(‬ ‫)‬ ‫)‪(e x − 1‬‬ ‫‪4‬‬
‫(‬ ‫)‬
‫‪4‬‬
‫⎟ ‪⎜ e x −1‬‬ ‫‪e x −1‬‬
‫⎝‬ ‫⎠‬
‫)‪−xe x (e x − 1) + 2e x g ( x ) (e x − 1‬‬
‫‪2‬‬

‫= ) ‪f ′′ ( x‬‬ ‫=‬
‫‪ex‬‬
‫)) ‪( −x (e −1) + 2g ( x‬‬
‫‪x‬‬

‫‪(e‬‬ ‫)‪−1‬‬ ‫‪(e‬‬ ‫)‪− 1‬‬


‫‪x‬‬ ‫‪4‬‬ ‫‪x‬‬ ‫‪3‬‬

‫= ) ‪f ′′ ( x‬‬
‫‪ex‬‬
‫‪( −xe‬‬ ‫‪x‬‬
‫‪+ x + 2 + 2 ( x − 1) e x‬‬ ‫)‬
‫‪(e‬‬ ‫)‪−1‬‬
‫‪x‬‬ ‫‪3‬‬

‫= ) ‪f ′′ ( x‬‬ ‫‪ex‬‬
‫) ‪(e x ( x − 2) + 2 + x‬‬
‫)‪(e x −1‬‬
‫‪3‬‬

‫‪ ،‬ﻟﺪﻳﻨﺎ ‪:‬‬ ‫ب‪ -‬ﻟﻴﻜﻦ \ ∈ ‪ ، x‬ﻧﻀﻊ ‪h (x ) = e x ( x − 2 ) + 2 + x :‬‬

‫‪h ′(x ) = (e x ( x − 2) + 2 + x ) = e x + e x ( x − 2) + 1 = e x ( x −1) + 1 = g ( x ) ≥ 0‬‬


‫‪′‬‬

‫‪ ،‬ﻓﺈن ‪:‬‬ ‫إذن ‪ h‬ﺗﺰاﻳﺪﻳﺔ ﻋﻠﻰ \ ‪ .‬وﺑﻤﺎ أن ‪h ( 0) = 0‬‬


‫)‪x ≤ 0 ⇒ h ( x ) ≤ h ( 0‬‬ ‫)‪x ≥ 0 ⇒ h ( x ) ≥ h ( 0‬‬
‫و‬
‫⇒‬ ‫‪h (x ) ≤ 0‬‬ ‫⇒‬ ‫‪h (x ) ≥ 0‬‬
‫‪ex‬‬ ‫) ‪h (x‬‬
‫‪.‬‬ ‫= ) ‪∀x ∈ \* : f ′′ ( x‬‬ ‫×‬ ‫ﺟـ‪ -‬ﻟﺪﻳﻨﺎ ‪:‬‬
‫‪e x −1‬‬
‫)‪(e x −1‬‬
‫‪2‬‬

‫) ‪h (x‬‬
‫‪.‬‬ ‫هﻲ إﺷﺎرة‬ ‫إﺷﺎرة ) ‪ f ′′ ( x‬ﻋﻠﻰ *\‬
‫‪e x −1‬‬
‫‪ .‬ﻟﺪﻳﻨﺎ ‪:‬‬ ‫ﻟﻴﻜﻦ *\ ∈ ‪x‬‬

‫‪18‬‬ ‫ﺗﺼﺤﻴﺢ ﻣﻮﺿﻮع اﻟﺪورة اﻟﻌﺎدﻳﺔ ‪2007‬‬ ‫اﻷﺳﺘﺎذ ‪ :‬ﻣﺤﻤﺪ اﻟﺤﻴﺎن اﻟﺜﺎﻧﻴﺔ ﺑﻜﺎﻟﻮرﻳﺎ ﻋﻠﻮم رﻳﺎﺿﻴﺔ ‪- 13 -‬‬
‫⇒ ‪x <0‬‬ ‫‪e x <1 æ h (x ) < 0‬‬ ‫⇒ ‪x >0‬‬ ‫‪e x >1 æ h (x ) > 0‬‬
‫‪⇒ e x −1 < 0 æ h ( x ) < 0‬‬ ‫‪⇒ e x −1 > 0 æ h ( x ) > 0‬‬
‫) ‪h (x‬‬ ‫و‬ ‫) ‪h (x‬‬
‫⇒‬ ‫‪>0‬‬ ‫⇒‬ ‫‪>0‬‬
‫‪e x −1‬‬ ‫‪e x −1‬‬
‫⇒‬ ‫‪f ′′ ( x ) > 0‬‬ ‫⇒‬ ‫‪f ′′ ( x ) > 0‬‬
‫‪.‬‬ ‫‪∀x ∈ \* : f ′′ ( x ) > 0‬‬ ‫ﻓﻲ آﻠﺘﺎ اﻟﺤﺎﻟﺘﻴﻦ ‪ ،‬ﻟﺪﻳﻨﺎ ‪:‬‬

‫د‪ -‬إﻧﺸﺎء اﻟﻤﻨﺤﻨﻰ ) ‪ ( C‬ﻓﻲ اﻟﻤﺴﺘﻮى ‪ P‬اﻟﻤﻨﺴﻮب إﻟﻰ ﻣﻌﻠﻢ ﻣﺘﻌﺎﻣﺪ ﻣﻤﻨﻈﻢ ) ‪(O , i , j‬‬
‫‪JJG JJG‬‬
‫‪:‬‬ ‫‪f‬‬

‫‪.‬‬ ‫‪y =0‬‬ ‫ﻣﻌﺎدﻟﺘﻩ‬ ‫) ‪ ( Cf‬ﻳﻘﺒﻞ ﻣﻘﺎرﺑﺎ أﻓﻘﻴﺎ ﺑﺠﻮار ∞‪+‬‬ ‫ﻟﺪﻳﻨﺎ ‪ lim f ( x ) = 0‬إذن‬
‫∞‪x →+‬‬

‫‪.‬‬ ‫‪y = −x‬‬ ‫ﻣﻌﺎدﻟﺘﻩ‬ ‫) ‪ ( Cf‬ﻳﻘﺒﻞ ﻣﻘﺎرﺑﺎ ﻣﺎﺋﻻ ﺑﺠﻮار ∞‪−‬‬ ‫‪ lim f ( x‬إذن‬
‫∞‪x →−‬‬
‫‪)+x‬‬ ‫وﻟﺪﻳﻨﺎ ‪= 0‬‬

‫‪18‬‬ ‫ﺗﺼﺤﻴﺢ ﻣﻮﺿﻮع اﻟﺪورة اﻟﻌﺎدﻳﺔ ‪2007‬‬ ‫اﻷﺳﺘﺎذ ‪ :‬ﻣﺤﻤﺪ اﻟﺤﻴﺎن اﻟﺜﺎﻧﻴﺔ ﺑﻜﺎﻟﻮرﻳﺎ ﻋﻠﻮم رﻳﺎﺿﻴﺔ ‪- 14 -‬‬
‫اﻟﻤﺘﺘﺎﻟﻴﺔ اﻟﻌﺪدﻳﺔ اﻟﻤﻌﺮﻓﺔ ﺑﻤﺎ ﻳﻠﻲ ‪:‬‬ ‫‪ .III‬ﻟﺘﻜﻦ `∈‪(u n )n‬‬
‫⎪⎧‬ ‫‪u0 =1‬‬
‫⎨‬
‫) ‪⎪⎩u n +1 = f (u n‬‬ ‫`∈ ‪; n‬‬
‫‪ .‬ﻟﺪﻳﻨﺎ ‪:‬‬ ‫‪ .1‬ﻟﻴﻜﻦ *\ ∈ ‪x‬‬
‫‪x‬‬
‫⇔ ‪f (x ) = x‬‬ ‫‪=x‬‬
‫‪e −1‬‬ ‫‪x‬‬

‫‪x‬‬
‫⇔‬ ‫‪−x =0‬‬
‫‪e −1‬‬
‫‪x‬‬

‫‪⎛ 1‬‬ ‫⎞‬


‫⇔‬ ‫‪x ⎜⎜ x‬‬ ‫‪−1⎟⎟ = 0‬‬
‫⎠ ‪⎝ e −1‬‬
‫‪1‬‬
‫⇔‬ ‫‪x = 0 æ_ x‬‬ ‫‪=1‬‬
‫‪e −1‬‬
‫⇔‬ ‫‪e x −1 = 1‬‬
‫‪⇔ ex = 2‬‬
‫‪f (x ) = x‬‬ ‫‪⇔ x = ln 2‬‬
‫‪.‬‬ ‫وﻟﺪﻳﻨﺎ ‪ . f ( 0 ) = 1 ≠ 0‬إذن ‪ x = ln 2‬هﻮ اﻟﺤﻞ اﻟﻮﺣﻴﺪ ﻟﻠﻤﻌﺎدﻟﺔ ‪ f ( x ) = x‬ﻓﻲ \‬
‫‪−1− ( x −1)e x‬‬ ‫) ‪g (x‬‬
‫‪.‬‬ ‫= ) ‪f ′( x‬‬ ‫‪=−‬‬ ‫‪ ، x ∈ \+‬ﻟﺪﻳﻨﺎ ‪≤ 0 :‬‬ ‫‪ .2‬أ‪ -‬ﻟﻴﻜﻦ‬
‫(‬ ‫)‪−1‬‬ ‫(‬ ‫)‪−1‬‬
‫‪2‬‬ ‫‪2‬‬
‫‪ex‬‬ ‫‪ex‬‬
‫ﺗﺰاﻳﺪﻳﺔ ﻋﻠﻰ ‪. \ +‬‬ ‫وﻟﺪﻳﻨﺎ ‪ . ∀x ∈ \* : f ′′ ( x ) > 0 :‬إذن ‪f ′‬‬
‫‪x ≥ 0 ⇒ f ′ ( x ) ≥ f ′ ( 0) ⇒ f ′ ( x ) ≥ − 1‬‬
‫‪.‬‬ ‫إذن ‪:‬‬
‫‪2‬‬
‫وﻣﻨﻩ ﻓﺈن ‪ . ∀x ∈ \ + : − ≤ f ′ ( x ) ≤ 0 :‬إذن ‪: − ≤ f ′ ( x ) ≤ :‬‬
‫‪. ∀x‬‬ ‫‪∈ \+‬‬
‫‪1‬‬ ‫‪1‬‬ ‫‪1‬‬
‫‪2‬‬ ‫‪2‬‬ ‫‪2‬‬
‫‪. ∀x ∈ \ + : f ′ ( x ) ≤ −‬‬
‫‪1‬‬ ‫وﺑﺎﻟﺘﺎﻟﻲ ﻓﺈن ‪:‬‬
‫‪2‬‬
‫ﻣﺘﺼﻠﺔ ﻋﻠﻰ اﻟﻤﺠﺎل ⎣⎡ ∞‪ . ⎤⎦ 0, +‬ﺣﺴﺐ ﻣﺒﺮهﻨﺔ اﻟﺘﺰاﻳﺪات‬ ‫‪f′‬‬ ‫و‬ ‫‪\+‬‬ ‫ب‪ -‬ﻟﺪﻳﻨﺎ ‪ f‬ﻗﺎﺑﻠﺔ ﻟﻻﺷﺘﻘﺎق ﻋﻠﻰ‬

‫ﻣﺤﺼﻮر ﺑﻴﻦ‬ ‫) ‪ ∀x ∈ \ + ; ∃c ∈ \ + / f ( x ) − f ( ln 2 ) = f ′ (c ) ( x − ln 2‬ﺣﻴﺚ ‪c‬‬ ‫اﻟﻤﻨﺘﻬﻴﺔ‪ ،‬ﻟﺪﻳﻨﺎ ‪:‬‬

‫≤ ) ‪ f ′ (c‬و ‪، f ( ln 2 ) = ln 2‬‬ ‫‪ . f ( x ) − f ( ln 2 ) = f ′ (c ) x − ln 2‬وﺑﻤﺎ أن ‪:‬‬


‫‪1‬‬
‫‪ .‬إذن ‪:‬‬ ‫‪ ln 2‬و ‪x‬‬
‫‪2‬‬
‫‪. ∀x‬‬ ‫≤ ‪∈ \ + : f ( x ) − ln 2‬‬
‫‪1 x − ln 2‬‬ ‫ﻓﺈن ‪:‬‬
‫‪2‬‬
‫‪.f‬‬ ‫‪(u ) − ln 2 ≤ 12 u‬‬
‫‪n‬‬ ‫‪n‬‬ ‫‪− ln 2‬‬ ‫‪ .‬إذن ‪:‬‬ ‫ﻟﻴﻜﻦ ` ∈ ‪ ، n‬ﻟﺪﻳﻨﺎ ‪ ، u n ∈\+‬ﻷن ‪f ( \ + ) = ⎤⎦ 0,1⎤⎦ ⊂ \ +‬‬

‫‪∀n ∈ ` : u n +1 − ln 2 ≤ 1 u n − ln 2‬‬ ‫وﺑﺎﻟﺘﺎﻟﻲ ﻓﺈن ‪:‬‬


‫‪2‬‬
‫‪n‬‬
‫⎞‪⎛1‬‬
‫‪.‬‬ ‫)*(‬ ‫‪∀n ∈ ` : u n − ln 2‬‬ ‫⎟⎟ ⎜⎜ ≤‬ ‫‪u 0 − ln 2‬‬ ‫ب‪ -‬ﻧﺒﻴﻦ ﺑﺎﻟﺘﺮﺟﻊ أن ‪:‬‬
‫⎠‪⎝ 2‬‬
‫‪18‬‬ ‫ﺗﺼﺤﻴﺢ ﻣﻮﺿﻮع اﻟﺪورة اﻟﻌﺎدﻳﺔ ‪2007‬‬ ‫اﻷﺳﺘﺎذ ‪ :‬ﻣﺤﻤﺪ اﻟﺤﻴﺎن اﻟﺜﺎﻧﻴﺔ ﺑﻜﺎﻟﻮرﻳﺎ ﻋﻠﻮم رﻳﺎﺿﻴﺔ ‪- 15 -‬‬
‫اﻟﻌﻻﻗﺔ )*( ﺹﺤﻴﺤﺔ ﻣﻦ أﺟﻞ ‪. n = 0‬‬
‫‪.‬‬ ‫ﻟﻴﻜﻦ ` ∈ ‪ ، n‬ﻧﻔﺘﺮض أن اﻟﻌﻻﻗﺔ )*( ﺹﺤﻴﺤﺔ ﻣﻦ أﺟﻞ ‪ n‬وﻧﺒﻴﻦ أﻧﻬﺎ ﺹﺤﻴﺤﺔ ﻣﻦ أﺟﻞ ‪n + 1‬‬
‫‪n‬‬
‫⎞‪⎛1‬‬ ‫‪1‬‬
‫‪.‬‬ ‫‪u n − ln 2‬‬ ‫⎟⎟ ⎜⎜ ≤‬ ‫ﻟﺪﻳﻨﺎ ‪ u n + 2 − ln 2 ≤ u n +1 − ln 2 :‬و ‪u 0 − ln 2‬‬
‫⎠‪⎝2‬‬ ‫‪2‬‬

‫‪n +1‬‬ ‫‪n‬‬


‫⎞‪⎛1‬‬ ‫‪⎛1⎞ 1‬‬
‫‪. u n +1 − ln 2‬‬ ‫⎟⎟ ⎜⎜ ≤‬ ‫‪ . u n +1 − ln 2‬أي ‪u 0 − ln 2 :‬‬ ‫إذن ‪≤ ⎜⎜ ⎟⎟ × × u 0 − ln 2 :‬‬
‫⎠‪⎝ 2‬‬ ‫‪⎝ 2⎠ 2‬‬
‫‪n‬‬
‫⎞ ⎛‬
‫‪.‬‬ ‫‪∀n ∈ ` : u n − ln 2‬‬ ‫⎟⎟ ‪≤ ⎜⎜ 1‬‬ ‫‪u 0 − ln 2‬‬ ‫وﺑﺎﻟﺘﺎﻟﻲ ﻓﺈن ‪:‬‬
‫⎠‪⎝ 2‬‬
‫‪n‬‬ ‫‪n‬‬
‫‪1‬‬ ‫⎞‪⎛1‬‬ ‫⎞‪⎛1‬‬
‫ﺟـ‪ -‬ﻟﺪﻳﻨﺎ ‪ ∀n ∈ ` : u n − ln 2 ≤ ⎜ ⎟ 1− ln 2 :‬و ‪. −1 < < 1 ⇒ lim ⎜⎜ ⎟⎟ = 0‬‬
‫‪2‬‬ ‫‪n →+∞ 2‬‬
‫⎠ ⎝‬ ‫⎟‪⎜ 2‬‬
‫⎠ ⎝‬
‫‪.‬‬ ‫‪lim u‬‬
‫‪n →+∞ n‬‬
‫‪= ln 2‬‬ ‫ﻣﺘﺘﺎﻟﻴﺔ ﻣﺘﻘﺎرﺑﺔ ﻧﻬﺎﻳﺘﻬﺎ ‪:‬‬ ‫ﺣﺴﺐ ﻣﺼﺎدﻳﻖ اﻟﺘﻘﺎرب ‪ ،‬ﻟﺪﻳﻨﺎ `∈‪(u n )n‬‬
‫ﺑﻤﺎ ﻳﻠﻲ ‪:‬‬ ‫‪ .IV‬ﻧﻌﺘﺒﺮ ‪ F‬اﻟﺪاﻟﺔ اﻟﻌﺪدﻳﺔ اﻟﻤﻌﺮﻓﺔ ﻋﻠﻰ \‬
‫⎧‬ ‫‪t‬‬
‫‪⎪⎪F ( x‬‬ ‫‪) = ∫x‬‬
‫‪2x‬‬
‫‪dt ; x ≠ 0‬‬
‫⎨‬ ‫‪e −1‬‬ ‫‪t‬‬

‫⎪‬
‫⎩⎪‬
‫‪F ( 0) = 0‬‬
‫‪ ،‬إذن ‪:‬‬ ‫‪ .1‬أ‪ -‬ﻟﻴﻜﻦ *\∈ ‪ . x‬ﻧﻌﻠﻢ أن ‪ f‬ﺗﻨﺎﻗﺼﻴﺔ ﻗﻄﻌﺎ ﻋﻠﻰ \‬

‫‪ ،‬ﻓﺈن ‪:‬‬ ‫‪x >0‬‬ ‫إذا آﺎن‬


‫⇒ ‪x ≤ t ≤ 2x‬‬ ‫) ‪f ( 2x ) ≤ f (t ) ≤ f ( x‬‬
‫‪f ( 2x )dt ≤ ∫ f (t )dt ≤ ∫ f ( x )dt‬‬
‫‪2x‬‬ ‫‪2x‬‬ ‫‪2x‬‬
‫⇒‬ ‫‪∫x‬‬ ‫‪x‬‬ ‫‪x‬‬
‫⇒‬ ‫) ‪x f ( 2x ) ≤ F ( x ) ≤ x f ( x‬‬
‫‪2x 2‬‬ ‫‪x2‬‬
‫⇒‬
‫‪e 2x −1‬‬
‫≤‬ ‫‪F‬‬ ‫‪( ) e x −1‬‬
‫‪x‬‬ ‫≤‬
‫‪ ،‬ﻓﺈن ‪:‬‬ ‫‪x <0‬‬ ‫إذا آﺎن‬
‫⇒ ‪2x ≤ t ≤ x‬‬ ‫) ‪f ( x ) ≤ f (t ) ≤ f ( 2x‬‬
‫‪f ( x )dt ≤ ∫ f (t )dt ≤ ∫ f ( 2x )dt‬‬
‫‪x‬‬ ‫‪x‬‬ ‫‪x‬‬
‫⇒‬ ‫‪∫2x‬‬ ‫‪2x‬‬ ‫‪2x‬‬
‫⇒‬ ‫) ‪−xf ( x ) ≤ −F ( x ) ≤ −xf ( 2x‬‬
‫‪2x 2 ≤ F x ≤ x 2‬‬
‫⇒‬
‫‪e 2 x −1‬‬
‫‪( ) e x −1‬‬

‫‪2x 2‬‬ ‫‪x2‬‬


‫‪.‬‬ ‫\ ∈ ‪∀x‬‬ ‫*‬
‫‪: 2x‬‬ ‫‪≤ F (x ) ≤ x‬‬ ‫وﺑﺎﻟﺘﺎﻟﻲ ﻓﺈن ‪:‬‬
‫‪e −1‬‬ ‫‪e −1‬‬

‫‪18‬‬ ‫ﺗﺼﺤﻴﺢ ﻣﻮﺿﻮع اﻟﺪورة اﻟﻌﺎدﻳﺔ ‪2007‬‬ ‫اﻷﺳﺘﺎذ ‪ :‬ﻣﺤﻤﺪ اﻟﺤﻴﺎن اﻟﺜﺎﻧﻴﺔ ﺑﻜﺎﻟﻮرﻳﺎ ﻋﻠﻮم رﻳﺎﺿﻴﺔ ‪- 16 -‬‬
‫‪2‬‬ ‫‪2‬‬
‫‪.‬‬ ‫‪∀x ∈ \* : 22xx ≤ F ( x ) ≤ xx‬‬ ‫ب‪ -‬ﻟﺪﻳﻨﺎ ‪:‬‬
‫‪e‬‬ ‫‪−1‬‬ ‫‪e −1‬‬
‫‪x2‬‬ ‫‪x‬‬ ‫‪0‬‬ ‫‪2x 2‬‬ ‫‪2x‬‬ ‫‪0‬‬
‫‪lim‬‬ ‫‪= lim x‬‬ ‫‪= =0‬‬ ‫و‬ ‫‪lim‬‬ ‫‪= lim x‬‬ ‫=‬ ‫‪=0‬‬ ‫وﺑﻤﺎ أن ‪:‬‬
‫‪x →0 e −1 x →0 e − 1 1‬‬
‫‪x‬‬ ‫‪x →0 e‬‬ ‫‪2‬‬ ‫‪x‬‬
‫‪−1 x →0 e − 1 (e x + 1) 1× 2‬‬
‫‪x‬‬ ‫‪x‬‬
‫‪.‬‬ ‫‪0‬‬ ‫ﻣﺘﺼﻠﺔ ﻓﻲ‬ ‫‪F‬‬ ‫‪ ،‬وﺑﺎﻟﺘﺎﻟﻲ ﻓﺈن‬ ‫) ‪lim F ( x ) = 0 = F ( 0‬‬ ‫ﻓﺈن ‪:‬‬
‫‪x →0‬‬
‫‪2‬‬ ‫‪2‬‬
‫‪.‬‬ ‫∀‬‫‪x‬‬ ‫∈‬ ‫\‬‫*‬
‫‪:‬‬ ‫‪2‬‬‫‪x‬‬ ‫≤‬ ‫‪F‬‬ ‫‪x‬‬ ‫≤‬
‫‪( ) x‬‬‫‪x‬‬
‫‪e −1‬‬
‫ﺟـ‪ -‬ﻟﺪﻳﻨﺎ ‪:‬‬
‫‪−1‬‬
‫‪2x‬‬ ‫‪e‬‬
‫‪.‬‬
‫‪2x ≤ F ( x ) − F ( 0) ≤ x‬‬
‫‪∀x ∈ \* : 2x‬‬ ‫إذن ‪:‬‬
‫‪e −1‬‬ ‫‪x −0‬‬ ‫‪e x −1‬‬
‫‪x‬‬ ‫‪1‬‬ ‫‪1‬‬ ‫‪2x‬‬ ‫‪2‬‬ ‫‪2‬‬
‫‪lim‬‬ ‫‪= lim x‬‬ ‫‪= =1‬‬ ‫و‬ ‫‪lim‬‬ ‫‪= lim x‬‬ ‫=‬ ‫وﺑﻤﺎ أن ‪= 1 :‬‬
‫‪x →0 e x‬‬ ‫‪−1‬‬ ‫→‬ ‫‪e −1 1‬‬ ‫‪x →0 e 2 x‬‬ ‫‪−1‬‬ ‫→‬ ‫‪e −1 x‬‬ ‫‪1× 2‬‬
‫)‪(e + 1‬‬
‫‪x‬‬ ‫‪0‬‬ ‫‪x‬‬ ‫‪0‬‬
‫‪x‬‬ ‫‪x‬‬
‫)‪F ( x ) − F ( 0‬‬
‫‪.‬‬ ‫‪F ′ (0) = 1‬‬ ‫و‬ ‫ﻓﻲ ‪0‬‬ ‫ﻗﺎﺑﻠﺔ ﻟﻻﺷﺘﻘﺎق‬ ‫‪F‬‬ ‫‪ ،‬وﻣﻨﻩ ﻓﺈن‬ ‫‪lim‬‬ ‫ﻓﺈن ‪= 1 :‬‬
‫‪x →0‬‬ ‫‪x −0‬‬

‫داﻟﺔ ﻣﺘﺼﻠﺔ ﻋﻠﻰ اﻟﻤﺠﺎل ⎦⎤ ‪ ⎡⎣ x ,2x‬أو ⎦⎤ ‪ ⎡⎣ 2x , x‬ﺣﺴﺐ‬ ‫‪f : t 6 tt‬‬ ‫‪ ،‬ﻟﺪﻳﻨﺎ‬ ‫*\ ∈ ‪x‬‬ ‫‪ .2‬أ‪ -‬ﻟﻴﻜﻦ‬
‫‪e −1‬‬
‫‪. ∀x‬‬ ‫إﺷﺎرة ‪، x‬ﻓﻬﻲ ﺗﻘﺒﻞ داﻟﺔ أﺹﻠﻴﺔ ‪ ϕ‬ﻋﻠﻰ اﻟﻤﺠﺎل ⎦⎤ ‪ ⎡⎣ x ,2x‬أو ⎦⎤ ‪ ، ⎡⎣ 2x , x‬و ) ‪∈ \* : ϕ ′ ( x ) = f ( x‬‬
‫) ‪⎡ϕ (t ) ⎤ = ϕ ( 2x ) − ϕ ( x‬‬
‫‪t‬‬ ‫‪2x‬‬
‫∫ = ) ‪. F (x‬‬
‫‪2x‬‬
‫‪dt‬‬ ‫=‬ ‫إذن ‪:‬‬
‫‪x e t −1‬‬ ‫⎣‬ ‫‪⎦x‬‬
‫ﻟﺪﻳﻨﺎ ‪ ϕ‬و ‪ x 6 2x‬ﻗﺎﺑﻠﺘﻴﻦ ﻟﻻﺷﺘﻘﺎق ﻋﻠﻰ *\ و *\ ∈ ‪. ∀x ∈ \* : 2x‬‬
‫إذن ‪ F :‬ﻗﺎﺑﻠﺔ ﻟﻻﺷﺘﻘﺎق ﻋﻠﻰ *\ و ﻟﻜﻞ *\ ∈ ‪ ، x‬ﻟﺪﻳﻨﺎ ‪:‬‬
‫× ‪F ′ ( x ) = (ϕ ( 2x ) − ϕ ( x ) ) = ( 2x )′ ϕ ′ ( 2x ) − ϕ ′ ( x ) = 2‬‬
‫‪′‬‬ ‫‪ex‬‬
‫) ‪f (x ) − f (x‬‬
‫‪2x‬‬ ‫‪4‬‬
‫‪−‬‬ ‫‪= x‬‬
‫‪e −1 e −1 e + 1‬‬
‫‪2x‬‬ ‫‪x‬‬

‫‪3 −e x‬‬
‫⎜⎛ = ) ‪F ′ ( x‬‬ ‫⎞‬
‫(‬ ‫)‬ ‫) ‪f (x‬‬
‫‪4‬‬
‫‪−‬‬ ‫‪1‬‬‫⎟‬ ‫‪f‬‬ ‫‪x‬‬ ‫=‬
‫⎠ ‪⎝ e +1‬‬
‫‪x‬‬
‫‪e x +1‬‬
‫‪∀x ∈ \* : F ′ ( x ) = 3 −‬‬
‫‪x‬‬
‫) ( ‪e +1‬‬
‫‪.‬‬
‫‪e f x‬‬ ‫وﺑﺎﻟﺘﺎﻟﻲ ﻓﺈن ‪:‬‬
‫‪x‬‬

‫ب‪ -‬ﻧﻌﻠﻢ أن ‪ f‬ﻣﺘﺼﻠﺔ وﺗﻨﺎﻗﺼﻴﺔ ﻗﻄﻌﺎ ﻋﻠﻰ اﻟﻤﺠﺎل ⎣⎡ ∞‪. ⎤⎦ −∞, +‬‬

‫‪∀x‬‬ ‫‪ . f ⎤⎦ −∞, +∞ ⎡⎣ = ⎤⎥ xlim‬وﻣﻨﻩ ﻓﺈن ‪∈ \ : f ( x ) > 0 :‬‬


‫∞‪⎦ →+‬‬‫(‬ ‫‪f ( x ) , xlim‬‬
‫∞‪→−‬‬ ‫)‬
‫إذن ‪f ( x ) ⎡⎢ = ⎤⎦ 0, +∞ ⎡⎣ :‬‬
‫⎣‬
‫‪3 −e x‬‬
‫‪ ، ∀x ∈ \* : F ′ ( x ) = x‬ﻓﺈن إﺷﺎرة ) ‪ F ′ ( x‬ﻋﻠﻰ *\ هﻲ إﺷﺎرة ) ‪. ( 3 − e x‬‬ ‫وﺑﻤﺎ أن ) ‪f ( x‬‬
‫‪e +1‬‬
‫‪3 − e x = 0 ⇔ e x = 3 ⇔ x = ln 3‬‬
‫إذن ‪ F :‬ﺗﻨﺎﻗﺼﻴﺔ ﻗﻄﻌﺎ ﻋﻠﻰ اﻟﻤﺠﺎل ⎣⎡ ∞‪ ⎡⎣ln 3, +‬وﺗﺰاﻳﺪﻳﺔ ﻗﻄﻌﺎ ﻋﻠﻰ آﻞ ﻣﻦ اﻟﻤﺠﺎﻟﻴﻦ ⎦⎤‪ ⎤⎦ 0,ln 3‬و‬
‫⎣⎡ ‪. ⎤⎦ −∞,0‬‬
‫اﻟﻨﻬﺎﻳﺔ‪.‬‬

‫‪18‬‬ ‫ﺗﺼﺤﻴﺢ ﻣﻮﺿﻮع اﻟﺪورة اﻟﻌﺎدﻳﺔ ‪2007‬‬ ‫اﻷﺳﺘﺎذ ‪ :‬ﻣﺤﻤﺪ اﻟﺤﻴﺎن اﻟﺜﺎﻧﻴﺔ ﺑﻜﺎﻟﻮرﻳﺎ ﻋﻠﻮم رﻳﺎﺿﻴﺔ ‪- 17 -‬‬
‫ﺑﻤﺎ أن ‪2x 2 ≤ F x ≤ x 2 :‬‬
‫‪lim e x‬‬
‫∞‪x →−‬‬
‫‪ ،‬ﻷن ‪= 0‬‬
‫‪x2‬‬
‫‪lim‬‬
‫‪e x −1‬‬
‫∞‪x →−‬‬
‫=‬ ‫∞‪−‬‬ ‫و‬ ‫∀‬‫‪x‬‬ ‫∈‬ ‫\‬ ‫*‬
‫‪:‬‬
‫‪e 2 x −1‬‬
‫‪( ) e x −1‬‬
‫‪2x 2‬‬ ‫‪1 t2‬‬ ‫‪1t2 1‬‬
‫‪، lim 2x‬‬ ‫‪= lim‬‬ ‫=‬ ‫‪lim‬‬ ‫‪ xlim‬وﺑﻤﺎ أن ‪= 0 :‬‬ ‫ﻓﺈن ‪F ( x ) = −∞ :‬‬
‫‪x →+∞ e‬‬ ‫‪−1 t →+∞ 2 e t −1 t →+∞ 2 e t 1 − 1‬‬ ‫∞‪→−‬‬
‫‪et‬‬
‫‪2‬‬
‫‪2‬‬ ‫⎛‬ ‫⎞‬
‫⎛‬ ‫⎞‬ ‫‪2‬‬
‫‪⎜ 2‬‬‫⎟‬
‫‪1‬‬ ‫‪t2‬‬ ‫⎟ ‪⎜ t‬‬ ‫⎞ ‪⎛ 2u‬‬
‫وذﻟﻚ ﺑﻮﺿﻊ ‪، u = t‬‬ ‫‪lim‬‬ ‫‪t = t lim‬‬ ‫‪= lim ⎜ u ⎟ = lim ⎜ u‬‬ ‫ﺣﻴﺚ ‪ ، t = 2x‬ﻷن ‪⎟ = 0 :‬‬
‫‪2‬‬ ‫∞‪t →+‬‬ ‫‪e‬‬ ‫∞‪→+‬‬ ‫⎜‬
‫⎟⎟ ‪⎜ 2t‬‬
‫‪1‬‬ ‫⎠ ‪u →+∞ ⎝ e‬‬ ‫‪u →+∞ e‬‬
‫⎜‬ ‫⎟‬
‫⎠ ‪⎝e‬‬ ‫⎜‬ ‫⎟‬
‫‪⎝u‬‬ ‫⎠‬

‫‪.‬‬ ‫‪lim F ( x ) = 0‬‬ ‫‪ ، lim x‬ﻓﺈن ‪:‬‬


‫‪x‬‬ ‫‪2‬‬
‫و أن ‪x 2 1 = 0 :‬‬
‫‪= lim x‬‬
‫∞‪x →+‬‬ ‫‪x →+∞ e −1 x →+∞ e‬‬ ‫‪1‬‬
‫‪1−‬‬
‫‪ex‬‬
‫آﻤﺎ ﻳﻠﻲ ‪:‬‬ ‫وﻣﻨﻩ ﻧﺴﺘﻨﺘﺞ ﺟﺪول ﺗﻐﻴﺮات اﻟﺪاﻟﺔ ‪ F‬ﻋﻠﻰ \‬

‫∫ = )‪F ( ln 3‬‬
‫‪2ln3‬‬ ‫‪t‬‬ ‫‪3‬‬
‫‪dt = - ( ln(3) ) - di log(9) + di log(3) ≈ 0,4385061927‬‬
‫‪2‬‬

‫‪ln3‬‬ ‫‪e −1‬‬


‫‪t‬‬ ‫‪2‬‬
‫‪t 6 ln t‬‬ ‫‪ln t‬‬
‫‪x‬‬
‫ﻋﻠﻰ واﻟﺘﻲ ﺗﻨﻌﺪم ﻓﻲ ‪1‬‬
‫‪1- t‬‬
‫= ) ‪ di log(x‬هﻲ اﻟﺪاﻟﺔ اﻷﺹﻠﻴﺔ ﻟﻠﺪاﻟﺔ‬ ‫‪∫1 1- t‬‬ ‫ﺣﻴﺚ ‪dt :‬‬
‫) هﺬﻩ اﻟﺪاﻟﺔ ﺧﺎرج اﻟﻤﻘﺮر (‬
‫ﻣﻌﺎدﻟﺘﻩ ‪. y = 0‬‬ ‫) ‪ ( CF‬ﻳﻘﺒﻞ ﻣﻘﺎرﺑﺎ أﻓﻘﻴﺎ ﺑﺠﻮار ∞‪+‬‬
‫‪2x‬‬
‫‪∀x ∈ \* :‬‬ ‫∞‪ lim F x = −‬و ﻟﺪﻳﻨﺎ ‪2x ≤ F ( x ) ≤ x‬‬
‫‪lim‬‬ ‫∞‪= +‬‬ ‫و‬
‫‪e x −1‬‬ ‫) (‬ ‫ﻟﺪﻳﻨﺎ‬
‫∞‪x →−‬‬ ‫‪e −1‬‬
‫‪2x‬‬
‫‪e 2x‬‬‫‪−1‬‬ ‫‪x‬‬ ‫∞‪x →−‬‬

‫) ‪F (x‬‬
‫اﺗﺠﺎهﻩ ﻣﺤﻮر اﻷراﺗﻴﺐ‪.‬‬ ‫) ‪ ( CF‬ﻳﻘﺒﻞ ﻓﺮﻋﺎ ﺷﻠﺠﻤﻴﺎ ﺑﺠﻮار ∞‪−‬‬ ‫‪ lim‬وﻣﻨﻩ ﻓﺈن‬
‫‪x →−∞ x‬‬
‫إذن ‪= +∞ :‬‬
‫إﻧﺸﺎء اﻟﻤﻨﺤﻨﻰ ) ‪: ( CF‬‬

‫‪18‬‬ ‫ﺗﺼﺤﻴﺢ ﻣﻮﺿﻮع اﻟﺪورة اﻟﻌﺎدﻳﺔ ‪2007‬‬ ‫اﻷﺳﺘﺎذ ‪ :‬ﻣﺤﻤﺪ اﻟﺤﻴﺎن اﻟﺜﺎﻧﻴﺔ ﺑﻜﺎﻟﻮرﻳﺎ ﻋﻠﻮم رﻳﺎﺿﻴﺔ ‪- 18 -‬‬
http://arabmaths.ift.fr
http://arabmaths.ift.fr
http://arabmaths.ift.fr
‫اﻟﺜﺎﻧﻴﺔ ﻋﻠﻮم رﻳﺎﺿﻴﺔ‬ ‫ﺗﺼﺤﻴﺢ اﻻﻣﺘﺤﺎن اﻟﻮﻃﻨﻲ اﻟﻤﻮﺡﺪ‬ ‫ﺛﺎﻧﻮﻳﺔ ﻣﺤﻤﺪ اﻟﺨﺎﻣﺲ ‪ -‬اﻟﻘﻨﻴﻄﺮة ‪-‬‬
‫أوب‬ ‫ﻟﺪورة ﻳﻮﻟﻴﻮز ‪2007‬‬ ‫اﻷﺳﺘﺎذ ﻣﺤﻤﺪ ﻏﺮﻳﺰ‬
‫اﻻﺳﺘﺪراآﻴﺔ‬

‫اﻟﺘﻤﺮﻳﻦ اﻻول‬
‫‪ x ≡ a‬‬ ‫] ‪[p‬‬
‫)‪(S‬‬ ‫‪‬‬ ‫‪-1‬أ‪-‬‬
‫‪ x = b‬‬ ‫] ‪[q‬‬
‫∈ ) ‪∃ ( u 0 , v0‬‬ ‫‪2‬‬
‫‪: pu 0 + qv0 = 1‬‬ ‫إذن ﺡﺴﺐ ‪Bezout‬‬ ‫ﻟﺪﻳﻨﺎ‬
‫‪p ∧q =1‬‬
‫‪x 0 = bpu + aqv0‬‬ ‫‪0‬‬
‫ب ‪ -‬ﻟﻴﻜﻦ‬
‫ﻓﺎن ] ‪qv0 ≡ 1[ p‬‬ ‫و ﺑﻤﺎ أن ‪pu 0 + qv 0 = 1‬‬ ‫ﻟﺪﻳﻨﺎ ] ‪x 0 ≡ aqv0 [ p‬‬
‫وﻣﻨﻪ ] ‪(1) x 0 ≡ a [ p‬‬ ‫اذن ]‪aqv0 ≡ a [ p‬‬
‫اذن ] ‪x 0 ≡ bp [ q‬‬ ‫ﻟﺪﻳﻨﺎ ‪x 0 = bpu + aqv0‬‬
‫‪0‬‬

‫اذن ] ‪bpu 0 ≡ b [ q‬‬ ‫ﻓﺎن ] ‪pu 0 ≡ 1[ q‬‬ ‫و ﺑﻤﺎ أن ‪pu 0 + qv 0 = 1‬‬


‫و ﻣﻨﻪ ] ‪(2) x 0 ≡ b [q‬‬
‫ﻣﻦ )‪ (1‬و )‪ (2‬ﻧﺴﺘﻨﺘﺞ أن ‪ x 0‬ﺡﻞ ﻟﻠﻨﻈﻤﺔ )‪(S‬‬
‫‪ - 2‬ﻟﻴﻜﻦ ‪ x‬ﺡﻼ ﻟﻠﻨﻈﻤﺔ )‪(S‬‬
‫] ‪ x 0 ≡ a [ p‬‬ ‫] ‪ x ≡ a [ p‬‬
‫‪‬‬ ‫و‬ ‫‪‬‬ ‫ﻟﺪﻳﻨﺎ‬
‫] ‪ x 0 = b [ q‬‬ ‫] ‪ x = b [ q‬‬
‫‪ x − x 0 = mp‬‬
‫]‪x − x 0 ≡ 0 [ p‬‬
‫‪2‬‬
‫∈ )‪∃(m, n‬‬ ‫‪:‬‬ ‫أي‬ ‫و‬ ‫]‪x − x0 ≡ 0 [q‬‬ ‫اذن‬
‫‪ x − x 0 = nq‬‬
‫‪qm‬‬ ‫ﻓﺎﻧﻪ ﺡﺴﺐ ﻣﺒﺮهﻨﺔ ‪Gauss‬‬ ‫‪p∧q =1‬‬ ‫و ﺑﻤﺎ أن‬ ‫‪q x − x 0 ⇒ q mp‬‬
‫‪(∃k1 ∈ ) m = k1q‬‬ ‫اذن‬
‫اذن اﻟﻌﺪد ‪ pq‬ﻳﻘﺴﻢ اﻟﻌﺪد ‪x − x 0‬‬ ‫ﻓﺎن ‪x − x 0 = k1pq‬‬ ‫‪x − x 0 = mp‬‬ ‫وﺑﻤﺎ أن‬
‫‪ - 3‬ﻟﻴﻜﻦ ‪ x‬ﻋﺪدا ﺹﺤﻴﺤﺎ ﻧﺴﺒﻴﺎ ﺑﺤﻴﺚ ‪ pq‬ﻳﻘﺴﻢ اﻟﻌﺪد ‪x − x 0‬‬
‫‪(∃k ∈ ) : x − x0 = kpq‬‬ ‫اذن‬ ‫ﻟﺪﻳﻨﺎ ] ‪x − x 0 ≡ 0 [ pq‬‬
‫] ‪ x ≡ x 0 [ p‬‬
‫‪‬‬ ‫أي‬ ‫اذن ‪ p x − x 0‬و ‪q x − x 0‬‬ ‫اذن ‪x − x0 = (kp)q = (kq)p‬‬
‫] ‪ x ≡ x 0 [ q‬‬
‫] ‪ x ≡ a [ p‬‬ ‫] ‪ x 0 ≡ a [ p‬‬
‫اذن ‪ x‬ﺡﻞ ﻟﻠﻨﻈﻤﺔ )‪(S‬‬ ‫‪‬‬ ‫اذن‬ ‫‪‬‬ ‫وﺑﻤﺎ أن ‪ x 0‬ﺡﻞ ﻟﻠﻨﻈﻤﺔ )‪ (S‬ﻓﺎن‬
‫] ‪ x ≡ b [ q‬‬ ‫] ‪ x 0 ≡ b [ q‬‬
‫‪ - 4‬ﻟﺪﻳﻨﺎ ‪ x 0 = bpu + aqv0‬ﺡﻞ ﻟﻠﻨﻈﻤﺔ )‪(S‬‬
‫‪0‬‬

‫اذن ﺡﻠﻮل اﻟﻨﻈﻤﺔ )‪ (S‬هﻲ اﻷﻋﺪاد اﻟﻨﺴﺒﻴﺔ ‪ x‬ﺑﺤﻴﺚ ] ‪x ≡ x 0 [ pq‬‬


‫]‪ x ≡ 1[8‬‬
‫‪‬‬ ‫‪ - 5‬ﻟﺪﻳﻨﺎ‬
‫]‪ x ≡ 3 [13‬‬
‫)‪ ∃(u 0 , v0 ) = (5, −3‬ﺑﺤﻴﺚ ‪8(5)+13(-3)=1‬‬ ‫اذن‬ ‫اﻟﻌﺪدﻳﻦ ‪ 8‬و ‪ 13‬أوﻟﻴﺎن ﻓﻴﻤﺎ ﺑﻴﻨﻬﻤﺎ‬
‫‪x 0 = (3 × 8 × 5) + (1× 13 × (−3)) = 120 − 39‬‬ ‫أي‬ ‫ﻟﺪﻳﻨﺎ ‪x 0 = bpu 0 + aqv0‬‬
‫اذن ‪ x 0 = 81‬ﺡﻞ ﻟﻠﻨﻈﻤﺔ )‪(S‬‬
‫و ﺑﺎﻟﺘﺎﻟﻲ ﻣﺠﻤﻮﻋﺔ ﺡﻠﻮل اﻟﻨﻈﻤﺔ )‪ (S‬هﻲ اﻻﻋﺪاد اﻟﻨﺴﺒﻴﺔ ‪ x‬ﺑﺤﻴﺚ ]‪x ≡ 81[104‬‬

‫‪-1-‬‬
‫اﻟﺘﻤﺮﻳﻦ اﻟﺜﺎﻧﻲ‬
‫ﻟﻴﻜﻦ ‪ n‬ﻋﺪدا ﺹﺤﻴﺤﺎ ﻃﺒﻴﻌﻴﺎ ﻓﺮدﻳﺎ أآﺒﺮ ﻣﻦ أو ﻳﺴﺎوي ‪ . 3‬ﻟﺪﻳﻨﺎ ‪ n‬ﺹﻨﺪوﻗﺎ ﻣﺮﻗﻤﺎ ﻣﻦ ‪ 1‬إﻟﻰ ‪n‬‬
‫اﻟﺼﻨﺪوق رﻗﻢ ‪ (1 ≤ k ≤ n) k‬ﻳﺤﺘﻮي ﻋﻠﻰ آﺮة ‪ k‬ﺑﻴﻀﺎء و ‪ n-k‬آﺮة ﺳﻮداء‬
‫ﻟﻜﻞ ‪ k‬ﻣﻦ }‪{1, 2.............n‬‬ ‫‪ - 1‬ﻟﻴﻜﻦ ‪ B‬اﻟﺤﺪث اﻟﻜﺮة اﻟﻤﺴﺤﻮﺑﺔ ﺑﻴﻀﺎء‬
‫ﻧﻀﻊ ‪ Ek‬اﻟﺤﺪث اﺥﺘﻴﺎر اﻟﺼﻨﺪوق رﻗﻢ ‪k‬‬
‫∅ = ‪Ek ∩ Ep‬‬ ‫)‪(k ≠ p‬‬ ‫و‬ ‫‪Ω = E1 ∪ E 2 .............. ∪ E n‬‬ ‫ﻟﺪﻳﻨﺎ‬
‫)) ‪p(B) = p(B ∩ Ω) = p(B ∩ (E1 ∪ E 2 ∪ .................... ∪ E n‬‬
‫)) ‪p(B) = p((B ∩ E1 ) ∪ (B ∩ E 2 ) ∪ ..................... ∪ (B ∩ E n‬‬
‫) ‪p(B) = p(B ∩ E1 ) + (B ∩ E 2 ) + ....................... + (B ∩ E n‬‬
‫) ‪p(B ∩ E k‬‬
‫)‪p(B ∩ E k ) = p(E k ) × pEk (B‬‬ ‫اذن‬ ‫= )‪p E k (B‬‬
‫) ‪p(E k‬‬
‫ﻟﺪﻳﻨﺎ‬
‫)‪p(B) = p(E1 ) × p E1 (B) + p(E 2 ) × p E2 (B) + .....................p(En ) × pEn (B‬‬ ‫اذن‬
‫‪1‬‬
‫= ) ‪p(E1 ) = p(E 2 ) = .........................p(E n‬‬
‫‪n‬‬
‫ﻟﺪﻳﻨﺎ‬
‫‪k‬‬
‫) ﻻن اﻟﺼﻨﺪوق رﻗﻢ ‪ k‬ﻳﺤﺘﻮي ﻋﻠﻰ ‪ k‬آﺮة ﺑﻴﻀﺎء (‬ ‫= ) ‪p E k (B‬‬
‫‪n‬‬
‫و‬
‫‪11 2‬‬ ‫)‪n  1 n(n + 1‬‬
‫= )‪p(B‬‬ ‫‪‬‬ ‫× = ‪+ + .............. + ‬‬
‫‪nn n‬‬ ‫‪n n‬‬ ‫‪2n‬‬
‫اذن‬
‫)‪(n + 1‬‬
‫= )‪p(B‬‬
‫‪2n‬‬
‫و ﺑﺎﻟﺘﺎﻟﻲ‬
‫‪ - 2‬ﻟﻴﻜﻦ ‪ I‬اﻟﺤﺪث ‪ :‬اﻟﺼﻨﺪوق ﻳﺤﻤﻞ رﻗﻤﺎ ﻓﺮدﻳﺎ‬
‫‪(2k + 1) + 1‬‬
‫‪2‬‬
‫‪= k +1‬‬ ‫ﻟﺪﻳﻨﺎ ‪ n‬ﻓﺮدي ‪ . n=2k+1‬ﻋﺪد اﻟﺼﻨﺎدﻳﻖ اﻟﺘﻲ ﺗﺤﻤﻞ رﻗﻤﺎ ﻓﺮدﻳﺎ هﻮ‬
‫‪n −1‬‬
‫‪+1‬‬
‫‪n −1‬‬ ‫‪C1k +1‬‬ ‫‪k +1‬‬
‫‪p(I) = 2‬‬ ‫اذن‬ ‫=‪k‬‬ ‫ﻓﺈن‬ ‫‪n = 2k + 1‬‬ ‫وﺑﻤﺎ أن‬ ‫= )‪p(I‬‬ ‫=‬ ‫اذن‬
‫‪n‬‬ ‫‪2‬‬ ‫‪C1n‬‬ ‫‪n‬‬

‫)‪(n + 1‬‬
‫= )‪p(I‬‬
‫‪2n‬‬
‫و ﺑﺎﻟﺘﺎﻟﻲ‬
‫)‪p(B ∩ I‬‬
‫= )‪p I (B‬‬
‫)‪p(I‬‬
‫‪ - 3‬اﺡﺘﻤﺎل اﻟﺤﺼﻮل ﻋﻠﻰ آﺮة ﺑﻴﻀﺎء ﻋﻠﻤﺎ أن اﻟﺴﺤﺐ ﺗﻢ ﻣﻦ ﺹﻨﺪوق رﻗﻤﻪ ﻓﺮدي هﻮ ‪:‬‬
‫‪I = E1 ∪ E3 ∪ E5 ∪..................En‬‬ ‫ﻟﺪﻳﻨﺎ‬
‫)) ‪p(B ∩ I) = p(B ∩ (E1 ∪ E 3 ∪ E 5 ∪ ...............E n‬‬ ‫اذن‬
‫)) ‪= p((B ∩ E1 ) ∪ (B ∩ E3 ) ∪ ....................... ∪ (B ∩ E n‬‬
‫) ‪= p(B ∩ E1 ) + (B ∩ E3 ) + ....................... + (B ∩ E n‬‬
‫)‪p(B ∩ I) = p(E1 ) × p E1 (B) + p(E3 ) × p E3 (B) + ........................... + p(E n ) × p E n (B‬‬ ‫اذن‬
‫‪1 1 1 3‬‬ ‫‪1 n‬‬
‫‪p(B ∩ I) =  ×  +  ×  + ................................ +  × ‬‬
‫‪n n n n‬‬ ‫‪n n‬‬
‫‪1‬‬
‫) ‪p(B ∩ I) = 2 (1 + 3 + 5 + ................. + n‬‬
‫‪n‬‬
‫‪1+ 3+ 5 +........................ + n = 1+ 3+ 5+.................................... + 2k +1‬‬ ‫ﻳﻤﻜﻦ أن ﻧﺒﻴﻦ ﺑﺎﻟﺘﺮﺝﻊ أن ‪:‬‬
‫‪2‬‬
‫‪ n +1 ‬‬
‫‪= ( k + 1) = ‬‬
‫‪2‬‬
‫‪‬‬ ‫)ﻻن )‪( ( n = 2k + 1‬‬
‫‪ 2 ‬‬
‫‪n +1‬‬ ‫‪( n + 1)2‬‬ ‫‪1  n +1 ‬‬
‫‪2‬‬
‫= )‪p I (B‬‬ ‫و ﺑﺎﻟﺘﺎﻟﻲ‬ ‫= )‪p I (B‬‬ ‫×‬
‫‪2n‬‬
‫اذن‬ ‫= )‪p(B ∩ I‬‬ ‫‪‬‬ ‫‪‬‬ ‫اذن‬
‫‪2n‬‬ ‫‪4n 2‬‬ ‫‪n +1‬‬ ‫‪n2  2 ‬‬

‫‪-2-‬‬
‫اﻟﺘﻤﺮﻳﻦ اﻟﺜﺎﻟﺚ‬
{ 2
(H) = M(z) ∈ (P) / z 2 + z − z = 1
2
} ‫ﻧﻌﺘﺒﺮ اﻟﻤﺠﻤﻮﻋﺔ‬
z = x + iy ‫ ﻟﻴﻜﻦ‬- ‫ أ‬- 1
2 2
M ∈ (H) ⇔ z 2 + z − z = 1
⇔ x 2 − y 2 + 2ixy + x 2 − y 2 − 2ixy − x 2 − y 2 = 1
⇔ x 2 − 3y 2 = 1 (1)
(H) ‫( هﻲ ﻣﻌﺎدﻟﺔ دﻳﻜﺎرﺗﻴﺔ ﻟﻠﻤﺠﻤﻮﻋﺔ‬1)
x2 y2
O(0, 0) ‫( هﺬﻟﻮل ﻣﺮآﺰﻩ‬H) ‫ﻓﺎن‬ − =1 : ‫( هﻲ‬H) ‫ ﺑﻤﺎ أن ﻣﻌﺎدﻟﺔ اﻟﻤﺠﻤﻮﻋﺔ‬- ‫ب‬
12  1 
2

 
 3
3 3
y=−
3
x ‫و‬ y=
3
x ‫وﻣﻘﺎرﺑﻴﻪ‬ A '(−1, 0) ‫ و‬A(1,0) ‫رأﺳﻴﻪ‬ ‫و‬

z ' = x '+ iy ' ‫و‬ z = x + iy ‫ ﺑﺤﻴﺚ‬. (H) ‫ﻧﻘﻄﺘﺎن ﻣﻦ‬ M '(z) ‫ و‬M ( z ) ‫ ﻟﺘﻜﻦ‬- 2
ϕ(z, z ') = zz '+ zz '− zz ' ‫ﻧﻀﻊ‬
zz' = (xx '+ yy') + i(xy'− x ' y) ‫و‬ zz ' = (xx '+ yy ') + i(x ' y − xy ') ‫ﻟﺪﻳﻨﺎ‬
ϕ(z, z ') = (xx '+ 3yy ') + i(xy '+ x ' y) ‫اذن‬ zz' = (xx'− yy') − i(xy'+ x' y) ‫و‬
(xx '+ 3yy ')2 − 3(xy '+ x ' y)2 = x 2 x '2 + 9y 2 y '2 − 3x 2 y '2 − 3x '2 y 2 ‫ﻟﺪﻳﻨﺎ‬
2 2
x 2 − 3y2 = 1 ‫ و‬x ' − 3y ' = 1 ‫ﻓﺎن‬ (H)‫ ﻧﻘﻄﺘﺎن ﻣﻦ‬M '(z ') ‫ و‬M(z) ‫ﺑﻤﺎ أن‬
x 2 x '2 + 9y2 y'2 − 3x 2 y'2 − 3x '2 y2 = 1 ‫أي‬ ( x 2 − 3y2 )( x '2 − 3y '2 ) = 1 ‫اذن‬
M(ϕ(z, z ')) ∈ (H) ‫و ﺑﺎﻟﺘﺎﻟﻲ‬
2
ϕ(z, z) = z × z + z × z − z × z = z 2 + z − z
2
‫ و ﻟﺪﻳﻨﺎ‬ϕ(z,1) = z ×1 + z ×1 − z ×1 = z ‫ ﻟﺪﻳﻨﺎ‬- ‫ب‬
ϕ(z, z) = 1 ‫و ﻣﻨﻪ‬ 2 2
z 2 + z − z = 1 ‫ﻓﺎن‬ M(z) ∈ (H) ‫وﺑﻤﺎ أن‬
M(z) * M(z ') = M(ϕ(z, z ')) (H) ‫ ﻣﻦ‬M(z') ‫ و‬M(z) ‫( ﺑﻘﺎﻧﻮن اﻟﺘﺮآﻴﺐ اﻟﺪاﺥﻠﻲ * ﺡﻴﺚ ﻟﻜﻞ‬H) ‫ ﻧﺰود‬- 3
M(z") ‫ و‬M(z') ‫ و‬M(z) ‫ﻟﻜﻞ‬
M(z) * (M(z ') * M(z")) = M(z) * M(ϕ(z ', z '')) ‫ﻟﺪﻳﻨﺎ‬
= M(z)*M(z'z'' + z'z"− z'z") = M(ϕ(z,z'z" + z'z "− z'z"))
= M(z(z'z"+ z'z" − z'z") + z(z'z" + z'z"− z'z") − zz'z "− zz'z" + zz'z")
= M(zz 'z "+ zz 'z" − zz 'z"− zz 'z '' + zz 'z") (1 )
-3-
‫)"‪(M(z) * M(z ')) * M(z") = M(zz ' + zz '− zz ') * M(z‬‬
‫))"‪= M(ϕ(zz ' + zz '− zz ', z‬‬
‫)"‪= M(zz 'z" + zz"z" − zz 'z '' + zz ' z"+ zz 'z "− zz ' z"− zz ' z" − zz 'z '' + zz ' z‬‬
‫)'' ‪= M(zz 'z "+ zz 'z" − zz 'z"− zz 'z" + zz 'z‬‬ ‫)‪(2‬‬
‫ﻣﻦ )‪ (1‬و )‪ (2‬ﻧﺴﺘﻨﺘﺞ أن )'' ‪M(z) *(M(z ') * M(z '')) = (M(z) * M(z ')) * M(z‬‬
‫اذن اﻟﻘﺎﻧﻮن * ﺗﺠﻤﻴﻌﻲ‬
‫)' ‪M(z) * M(z ') = M(ϕ(z, z ')) = M(zz ' + zz '− zz‬‬ ‫ﻟﺪﻳﻨﺎ‬
‫)' ‪M(z ') * M(z) = M(ϕ(z ', z)) = M(z 'z + z 'z − zz‬‬ ‫و‬
‫)‪ M(z) * M(z ') = M(z ') * M(z‬ﻟﻜﻞ )‪ M(z‬و )' ‪ M(z‬ﻣﻦ)‪(H‬‬ ‫اذن‬
‫اذن اﻟﻘﺎﻧﻮن * ﺗﺒﺎدﻟﻲ‬
‫أي )‪ M(z) * M(1) = M(z‬ﻟﻜﻞ )‪ M(z‬ﻣﻦ)‪(H‬‬ ‫)‪M(ϕ(z,1)) = M(z‬‬ ‫ﻟﺪﻳﻨﺎ‬
‫اذن )‪ M(1‬هﻮ اﻟﻌﻨﺼﺮ اﻟﻤﺤﺎﻳﺪ ﻟﻠﻘﺎﻧﻮن *‬
‫اذن )‪M(z) * M(z) = M(1‬‬ ‫)‪M(ϕ(z, z)) = M(1‬‬ ‫ﻟﺪﻳﻨﺎ‬
‫اذن آﻞ ﻋﻨﺼﺮ )‪ M(z‬ﻣﻦ )‪ (H‬ﻳﻘﺒﻞ ﻣﻤﺎﺛﻼ ﻓﻲ )‪ (H‬هﻮ )‪ M(z‬و ﺑﺎﻟﺘﺎﻟﻲ )*‪ ((H),‬زﻣﺮة ﺗﺒﺎدﻟﻴﺔ‬

‫اﻟﺘﻤﺮﻳﻦ اﻟﺮاﺑﻊ‬
‫‪‬‬ ‫‪a + b‬‬ ‫‪−b ‬‬ ‫‪2‬‬
‫‪‬‬
‫‪F = M(a, b) = ‬‬ ‫∈ )‪ /(a, b‬‬ ‫‪‬‬
‫‪‬‬ ‫‪ 5b a − 3b ‬‬ ‫‪‬‬
‫ﻧﻀﻊ )‪ I = M(1, 0‬و )‪ J = M(0,1‬و‬
‫)‪O = M(0, 0‬‬
‫‪ - 1‬أ ‪ F ⊂ M 2 ( ) -‬و ∅ ≠ ‪ F‬ﻻن ‪O ∈ F‬‬
‫‪M(a, b) − M(c, d) = M(a − c, b − d) ∈ F‬‬ ‫ﻟﻜﻞ )‪ M(a, b‬و )‪ M(c, d‬ﻣﻦ ‪F‬‬
‫اذن )‪ (F, +‬زﻣﺮة ﺗﺒﺎدﻟﻴﺔ‬ ‫اذن ) ‪ (F, +‬زﻣﺮة ﺝﺰﺋﻴﺔ ﻣﻦ اﻟﺰﻣﺮة اﻟﺘﺒﺎدﻟﻴﺔ )‪(M 2 (R), +‬‬
‫‪∀M(a, b) ∈ F‬‬ ‫) ‪(∀λ ∈ R‬‬ ‫‪λ.M = λ.(aI + bJ) = (λa)I + (λb)J‬‬
‫اذن ‪ F‬ﺝﺰء ﻣﺴﺘﻘﺮ ﺑﺎﻟﻨﺴﺒﺔ ﻟﻠﻘﺎﻧﻮن اﻟﺨﺎرﺝﻲ ‪.‬‬ ‫‪λ.M ∈ F‬‬ ‫اذن‬
‫و )‪ (M 2 (R), +,.‬ﻓﻀﺎء ﻣﺘﺠﻬﻲ ﺡﻘﻴﻘﻲ ﻓﺎن اﻟﺨﺎﺹﻴﺎت اﻷرﺑﻊ ﺗﺒﻘﻰ ﻣﺘﺤﻘﻘﺔ‬ ‫) (‪F ⊂ M2‬‬ ‫ﺑﻤﺎ أن‬
‫‪∀(M, M ') ∈ F2‬‬ ‫)‪(∀α ∈ R‬‬ ‫' ‪α.(M + M ') = α.M + α.M‬‬
‫‪∀M ∈ F‬‬ ‫‪∀(α, β) ∈ R‬‬ ‫‪2‬‬
‫‪( α + β ) .M = α.M + βM‬‬
‫)‪( αβ ) .M = α.(β.M‬‬
‫‪∀M ∈ F‬‬ ‫‪1.M = M‬‬
‫ﻓﻀﺎء ﻣﺘﺠﻬﻲ ﺡﻘﻴﻘﻲ‬ ‫)‪(F, +,.‬‬‫اذن‬
‫ﻣﻮﻟﺪة ﻟﻠﻔﻀﺎء اﻟﻤﺘﺠﻬﻲ اﻟﺤﻘﻴﻘﻲ ‪F‬‬ ‫)‪(I, J‬‬ ‫اذن اﻷﺳﺮة‬ ‫)‪(∀M ∈ F‬‬ ‫‪M(a, b) = aI + bJ‬‬ ‫ب ‪ -‬ﻟﺪﻳﻨﺎ‬
‫‪0 0‬‬ ‫‪α +β‬‬ ‫‪−β   0 0 ‬‬
‫∈ )‪∀(α, β‬‬ ‫‪2‬‬ ‫‪αI + β J = ‬‬ ‫‪‬‬ ‫‪⇒‬‬ ‫‪=‬‬ ‫‪‬‬
‫‪0 0‬‬ ‫‪‬‬ ‫‪5‬‬‫‪β‬‬ ‫‪α‬‬ ‫‪− 3β   0 0 ‬‬
‫‪α = 0‬‬
‫‪⇒‬‬
‫‪β = 0‬‬
‫أﺳﺎس ﻟﻠﻔﻀﺎء اﻟﻤﺘﺠﻬﻲ ‪F‬‬ ‫وﻣﻨﻪ اﻷﺳﺮة‬
‫)‪(I, J‬‬ ‫اذن اﻷﺳﺮة )‪ (I, J‬ﺡﺮة‬
‫اذن ‪dim F = 2‬‬ ‫ﻋﺪد ﻋﻨﺎﺹﺮ اﻷﺳﺎس )‪ (I, J‬هﻮ ‪2‬‬
‫‪ - 2‬ﻟﻴﻜﻦ ‪ α‬ﻋﺪدا ﻋﻘﺪﻳﺎ ﻻ ﻳﻨﺘﻤﻲ إﻟﻰ‬
‫*‬
‫× ∈ ) ‪(x1 , y1‬‬ ‫ﺡﻴﺚ‬ ‫‪α = x1 + iy1‬‬ ‫ﻧﻀﻊ‬

‫‪-4-‬‬
2
z = x + iy ‫ﺑﺤﻴﺚ‬ ‫( ﻣﻦ‬x, y) ‫ﻳﻮﺝﺪ زوج وﺡﻴﺪ‬ ‫ ﻣﻦ‬z ‫ﻟﻜﻞ‬
‫؟‬ z = β + γα ‫ ﺑﺤﻴﺚ‬2 ‫( ﻣﻦ‬β, γ ) ‫هﻞ ﻳﻮﺝﺪ زوج وﺡﻴﺪ‬
x + iy = β + γ (x1 + iy1 ) ⇔ x + iy = β + γx1 + iγy1
 y
β + γx1 = x γ = y
 1
⇔  ⇔
 γy1 = y β + x1 y = x
 y1
 x1
β = x − y y
 1
⇔ 
γ = y
 y1
2
z = β.1 + γ.α ‫ﺑﺤﻴﺚ‬ ‫( ﻣﻦ‬β, γ) ‫ﻳﻮﺝﺪ زوج وﺡﻴﺪ‬ ‫ ﻣﻦ‬z ‫ﻟﻜﻞ‬
( , +,.) ‫( أﺳﺎس ﻟﻠﻔﻀﺎء اﻟﻤﺘﺠﻬﻲ‬1, α) : ‫و ﺑﺎﻟﺘﺎﻟﻲ‬
ψ: →F -3
(a, b) ∈ 2 z = a + α b ‫ﺡﻴﺚ‬ ‫ ﻣﻦ‬z ‫ﻟﻜﻞ ﻋﻨﺼﺮ‬ z → ψ (z) = M(a, b)
 −4 2   1 −1 
J2 =   ‫اذن‬ J=  ‫ ﻟﺪﻳﻨﺎ‬- ‫أ‬
 −10 4   5 −3 
 2 −1 
J 2 = −2(I + J) ‫اذن‬ I+J =  ‫و ﻟﺪﻳﻨﺎ‬
 5 −2 
ψ (α ) = M(0,1) = J ‫اذن‬ ψ (α) = ψ (0 + α1) ‫ﻟﺪﻳﻨﺎ‬
‫ ﻋﻨﺼﺮان ﻣﻦ‬z ' = a '+ αb ' ‫ و‬z = a + αb ‫ ﻟﻴﻜﻦ‬- ‫ب‬
ψ (z.z ') = ψ ((a + α b)(a '+ α b ')) ‫ﻟﺪﻳﻨﺎ‬
= ψ(aa '+ (ab '+ a 'b)α + α 2 bb ')
ψ (z) × ψ (z ') = M(a, b) × M(a ', b ') ‫وﻟﺪﻳﻨﺎ‬
= (aI + bJ) × (a ' I + b ' J)
= aa 'I + (ab '+ a 'b)J + bb 'J 2 = aa 'I + (ab '+ a 'b)J + bb '(−2I − 2J)
= (aa '− 2bb ')I + (ab '+ a ' b − 2bb ')J
= M(aa '− 2bb ', ab '+ a ' b − 2bb ')
= ψ((aa '− 2bb ') + α(ab '+ a 'b − 2bb '))
aa '+ (ab '+ a 'b)α + α bb ' = (aa '− 2bb ') + α(ab '+ a 'b − 2bb ') :
2
‫ ﺗﺸﺎآﻼ ﺗﻘﺎﺑﻠﻴﺎ ﻳﺠﺐ أن ﻳﻜﻮن‬ψ ‫ﻟﻜﻲ ﻳﻜﻮن‬
α 2 + 2α + 2 = 0 ‫أي‬ α 2 bb ' = −2bb '− 2αbb ' ‫اذن‬
α = −1 − i ‫ أو‬α = −1 + i ‫اذن‬ ∆ ' = 1 − 2 = −1 ‫ﻟﺪﻳﻨﺎ‬
α = − 1 + i ‫ ﻧﺄﺥﺬ‬- 4
ψ (α 2007 ) = ψ (α.α................α) ‫و‬ ψ (α) = J ‫ﻟﺪﻳﻨﺎ‬
2007‫ﺮة‬ ‫ﻣ‬

ψ(α 2007 ) = ψ (α).ψ(α)...................ψ(α) ( (F, ×) ‫ﻧﺤﻮ‬ ( , ×) ‫ﻣﻦ‬ ‫ﺗﺸﺎآﻞ‬ ‫اذن )ﻻن‬


ψ
ψ (α 2007 ) = J 2007 ‫اذن‬
‫ ﻟﺪﻳﻨﺎ‬Moivre ‫ﺡﺴﺐ ﺹﻴﻐﺔ‬
3π 3π 3π 3π
α2007 = (−1 + i)2007 = ( 2(cos + i sin ))2007 = ( 2)2007 (cos .2007 + i sin .2007)
4 4 4 4
π π 2 2 −21004 21004
α 2007 = ( 2 ) 2007 (− cos
4
− i sin ) = ( 2 ) 2007 (−
4 2
−i
2
)=
2
−i
2
‫اذن‬
-5-
α 2007 = −21003 − i21003 = −21003 (1 + i) = −21003 (−1 + i + 2) = −21004 − 21003 (−1 + i) ‫اذن‬
α 2007 = −21004 + α (−21003 )

ψ (a + αb) = M(a, b) ‫أي‬ ψ (z) = M(a, b) ‫ﻧﻌﻠﻢ أن‬


ψ (α 2007 ) = J 2007 ⇔ ψ (−21004 + α(−21003 )) = J 2007
⇔ M(−21004 , −21003 ) = J 2007

⇔ J 2007 = −21004 I − 21003 J

‫اﻟﺘﻤﺮﻳﻦ اﻟﺨﺎﻡﺲ‬
g(x) = 1 + x − e− x -1 (I
‫ ﺗﺰاﻳﺪﻳﺔ ﻗﻄﻌﺎ ﻋﻠﻰ‬g ‫اذن‬ (∀x ∈ ) g '(x) = 1 + x − e− x 0 -‫أ‬
−x
lim g(x) = lim 1 + x − e -‫ب‬
x →−∞ x →−∞
1
= lim 1 + x(1 − )
x →−∞ xe x
= (−∞)(−(−∞ )) = −∞
lim g(x) = lim 1 + x − e− x
x →+∞ x →+∞

= +∞

x −∞ 0 +∞
g’(x) +
+∞
g(x) 0
−∞

‫ﻧﺤﻮ‬ ‫ ﺗﻘﺎﺑﻞ ﻣﻦ‬g ‫اذن‬ lim g(x) = +∞


x →+∞
‫و‬ lim g(x) = −∞
x →−∞
‫و‬ ‫ ﻣﺘﺼﻠﺔ و ﺗﺰاﻳﺪﻳﺔ ﻗﻄﻌﺎ ﻋﻠﻰ‬g - ‫ج‬
g(x) = 0 ‫هﻮ اﻟﺤﻞ اﻟﻮﺡﻴﺪ ﻟﻠﻤﻌﺎدﻟﺔ‬ x0 = 0 ‫ﻓﺎن‬ g(0) = 0 ‫و ﺑﻤﺎ أن‬
1
f (x) = −x
‫ ﻟﺪﻳﻨﺎ‬- 2
1+ x − e
1 1
lim f (x) = = 0 ‫ و‬lim f (x) = =0 - ‫أ‬
x →+∞ +∞ x →−∞ −∞
1 1 1
lim f (x) = lim = −∞ ‫و‬ lim f (x) = lim = lim x = +∞
x →0 x →0 ex − 1 x →0 x →0 1 + x − e− x x →0 e − 1
x≺0 x≺0 +x x 0 x 0 x 0 +x
x ex
−(1 + x − e − x ) ' −g '(x)
f '(x) = −x 2
= (∀x ∈ ) ‫ ﻟﺪﻳﻨﺎ‬- ‫ب‬
(1 + x − e ) g 2 (x)
f '(x) ≺ 0 ‫ﻓﺎن‬ g '(x) 0 ‫ ﺑﻤﺎ أن‬- ‫ج‬
x −∞ 0 +∞

f’(x) - -
0 +∞
f(x)
−∞ 0

-6-
‫د‪-‬‬

‫‪ - 3‬أ ‪ -‬ﻟﻴﻜﻦ ‪ n‬ﻣﻦ‬


‫*‬

‫∞‪lim h(x) = +∞ − n = +‬‬


‫‪x →0 +‬‬
‫و‬ ‫‪lim h(x) = lim f (x) − n = − n‬‬
‫∞‪x →+‬‬ ‫∞‪x →+‬‬
‫ﻟﺪﻳﻨﺎ‬ ‫‪h(x) = f (x) − n‬‬ ‫ﻟﺘﻜﻦ‬
‫‪*+‬‬
‫) ‪ . (∀x ∈ *+‬اذن ‪ h‬ﺗﻨﺎﻗﺼﻴﺔ ﻗﻄﻌﺎ ﻋﻠﻰ‬ ‫‪h '(x) = f '(x) ≺ 0‬‬

‫ﻓﺎن ‪ h‬ﺗﻘﺎﺑﻞ ﻣﻦ [∞‪ ]0, +‬ﻧﺤﻮ [∞‪]−n, +‬‬ ‫‪*+‬‬


‫ﺑﻤﺎ أن ﻣﺘﺼﻠﺔ و ﺗﻨﺎﻗﺼﻴﺔ ﻗﻄﻌﺎ ﻋﻠﻰ‬
‫اذن ﻳﻮﺝﺪ ﻋﺪد ﺡﻘﻴﻘﻲ وﺡﻴﺪ ‪ x n‬ﻣﻦ [∞‪ ]0, +‬ﺑﺤﻴﺚ ‪h(x n ) = 0‬‬

‫اذن اﻟﻤﻌﺎدﻟﺔ ‪ f (x) = n‬ﺗﻘﺒﻞ ﺡﻼ وﺡﻴﺪا ‪ x n‬ﻓﻲ اﻟﻤﺠﺎل [∞‪]0, +‬‬


‫ب ‪ -‬ﻟﺪﻳﻨﺎ ‪ f (x n ) = n‬و ‪ f (x n +1 ) = n + 1‬اذن ) ‪f (x n +1 ) f (x n‬‬

‫وﺑﻤﺎ أن ‪ f‬ﺗﻨﺎﻗﺼﻴﺔ ﻗﻄﻌﺎ ﻋﻠﻰ [∞‪ ]0, +‬ﻓﺎن ‪ x n +1 ≺ x n‬اذن اﻟﻤﺘﺘﺎﻟﻴﺔ )‪ (xn‬ﺗﻨﺎﻗﺼﻴﺔ‬
‫ﻟﺪﻳﻨﺎ [∞‪ x n ∈ ]0, +‬اذن )‪ (xn‬ﻣﺼﻐﻮرة ﺑﺎﻟﻌﺪد ‪0‬‬
‫و ﺑﻤﺎ أن )‪ (xn‬ﺗﻨﺎﻗﺼﻴﺔ و ﻣﺼﻐﻮرة ﺑﺎﻟﻌﺪد ‪ 0‬ﻓﺎن )‪ (xn‬ﻣﺘﻘﺎرﺑﺔ‬
‫ج ‪ -‬ﻟﺘﻜﻦ ‪= lim x n‬‬
‫∞→ ‪n‬‬
‫‪1‬‬
‫⇔ ‪f (x n ) = n‬‬ ‫‪=n‬‬
‫‪1 + x n − e− x n‬‬

‫‪1‬‬
‫= ‪⇔ 1 + x n − e− x n‬‬
‫‪n‬‬
‫‪1‬‬
‫‪lim (1 + x n − e− x n ) = lim‬‬ ‫‪⇔ 1 + − e− = 0‬‬
‫∞‪n →+‬‬ ‫∞‪n →+‬‬ ‫‪n‬‬
‫‪⇔ 1 + = e−‬‬
‫ﻧﻔﺘﺮض أن‬‫‪0‬‬
‫‪−‬‬
‫ﻏﻴﺮ ﻣﻤﻜﻦ‬ ‫‪e− ≺ 1‬‬ ‫و‬ ‫‪1+‬‬ ‫‪1‬‬ ‫اذن‬ ‫‪e‬‬ ‫أي ‪≺ 1‬‬ ‫اذن ‪ 1 + 1‬و ‪− ≺ 0‬‬
‫‪lim x n = 0‬‬ ‫و ﺑﺎﻟﺘﺎﻟﻲ‬ ‫اذن ‪= 0‬‬
‫∞‪n →+‬‬
‫‪1‬‬
‫⇔ ‪f (x) = 1‬‬ ‫‪=1‬‬ ‫‪ - 1 ( II‬أ ‪-‬‬
‫‪1 + x − e− x‬‬
‫‪⇔ 1 + x − e− x = 1‬‬
‫‪⇔ e−x = x‬‬
‫‪α = x1‬‬ ‫‪0‬‬ ‫ﺗﻘﺒﻞ ﺡﻼ وﺡﻴﺪا‬ ‫‪f (x) = 1‬‬ ‫ب ‪ -‬ﺡﺴﺐ س ‪ - 3‬أ ‪ -‬اﻟﻤﻌﺎدﻟﺔ‬

‫‪-7-‬‬
‫‪1‬‬
‫‪e‬‬
‫‪≤ α ≤1‬‬ ‫ﻟﻨﺒﻴﻦ أن‬
‫‪1 ‬‬ ‫‪1 ‬‬
‫‪ e ,1‬‬ ‫اذن ‪ u‬ﺗﻨﺎﻗﺼﻴﺔ ﻗﻄﻌﺎ‬ ‫)‪u '(x) = f '(x‬‬ ‫‪0‬‬ ‫و‬ ‫‪ e ,1‬‬ ‫ﻟﺪﻳﻨﺎ ‪ u‬ﻣﺘﺼﻠﺔ ﻋﻠﻰ‬ ‫‪u(x) = f (x) − 1‬‬ ‫ﻟﺘﻜﻦ‬
‫‪‬‬ ‫‪‬‬ ‫‪‬‬ ‫‪‬‬
‫‪1‬‬ ‫‪1‬‬
‫‪e‬‬
‫‪≺ α ≺1‬‬ ‫ﺗﻘﺒﻞ ﺡﻼ وﺡﻴﺪا‬ ‫‪e− x = x‬‬ ‫اذن ﺡﺴﺐ ﻣﺒﺮهﻨﺔ اﻟﻘﻴﻢ اﻟﻮﺳﻄﻴﺔ اﻟﻤﻌﺎدﻟﺔ‬ ‫‪u(1) × u( ) ≺ 0‬‬
‫‪e‬‬
‫ﻟﺪﻳﻨﺎ‬
‫‪y n +1 = e − yn‬‬ ‫‪ - 2‬أ ‪ -‬ﻟﺪﻳﻨﺎ ‪ y1 = 1‬و‬
‫‪1‬‬
‫‪e‬‬
‫‪≤ y1 ≤ 1‬‬ ‫اذن‬ ‫‪y1 = 1‬‬ ‫ﻣﻦ أﺝﻞ ‪ n = 1‬ﻟﺪﻳﻨﺎ‬
‫‪1‬‬ ‫‪1‬‬
‫‪e‬‬
‫‪≤ y n +1 ≤ 1‬‬ ‫و ﻧﺒﻴﻦ أن‬ ‫‪e‬‬
‫ﻧﻔﺘﺮض أن ‪≤ y n ≤ 1‬‬

‫‪1‬‬
‫‪−1 ≤ − y n ≤ 0‬‬ ‫أي‬ ‫‪−1 ≤ − y n ≤ −‬‬
‫‪e‬‬
‫ﻟﺪﻳﻨﺎ ‪ 1e ≤ yn ≤ 1‬اذن‬
‫‪1‬‬
‫‪e‬‬
‫‪≤ y n +1 ≤ 1‬‬ ‫أي‬ ‫‪e−1 ≤ e − yn ≤ 1‬‬ ‫اذن‬
‫) ∈ ‪(∀n‬‬ ‫*‬ ‫‪1‬‬
‫‪e‬‬
‫‪≤ yn ≤ 1‬‬ ‫و ﺑﺎﻟﺘﺎﻟﻲ‬
‫‪yn‬‬ ‫‪ f‬ﻣﺘﺼﻠﺔ ﻋﻠﻰ اﻟﻤﺠﺎل اﻟﻤﻐﻠﻮق اﻟﺬي ﻃﺮﻓﺎﻩ ‪ α‬و‬ ‫ب ‪ -‬ﻟﺘﻜﻦ‬ ‫‪f (x) = e− x‬‬
‫و ﻗﺎﺑﻠﺔ ﻟﻼﺵﺘﻘﺎق ﻋﻠﻰ اﻟﻤﺠﺎل اﻟﻤﻔﺘﻮح اﻟﺬي ﻃﺮﻓﺎﻩ ‪ α‬و ‪y n‬‬
‫‪yn‬‬ ‫و‬ ‫‪α‬‬ ‫اذن ﺡﺴﺐ ﻣﺒﺮهﻨﺔ اﻟﺘﺰاﻳﺪات اﻟﻤﻨﺘﻬﻴﺔ ﻳﻮﺝﺪ ﻋﺪد ﺡﻘﻴﻘﻲ ‪ c‬ﻣﺤﺼﻮر ﺑﻴﻦ‬
‫اذن ‪y n +1 − α = f '(c) y n − α‬‬ ‫ﺑﺤﻴﺚ ‪f (y n ) − f (α) = f '(c) yn − α‬‬
‫أي ‪f '(c) = e− c‬‬ ‫اذن ‪f '(c) = −e− c‬‬ ‫ﻟﺪﻳﻨﺎ ‪f '(x) = −e− x‬‬

‫اذن ‪− y n ≺ −c ≺ −α‬‬ ‫‪α ≺ c ≺ yn‬‬ ‫ﻟﺪﻳﻨﺎ‬


‫‪1‬‬ ‫‪1‬‬
‫‪−1 ≺ −α ≺ −‬‬
‫‪e‬‬
‫ﻓﺎن‬ ‫‪e‬‬
‫‪≺ α ≺1‬‬ ‫و ﺑﻤﺎ أن‬
‫‪1‬‬
‫‪−‬‬
‫‪0 ≺ e − c ≺ e−α ≺ e‬‬ ‫‪e‬‬‫اذن‬
‫‪1‬‬
‫‪−‬‬
‫∈ ‪(∀n‬‬ ‫*‬
‫)‬ ‫≤ ‪y n +1 − α‬‬ ‫‪e e‬‬ ‫‪yn − α‬‬ ‫و ﺑﺎﻟﺘﺎﻟﻲ‬
‫‪1‬‬
‫‪−‬‬
‫≤ ‪y2 − α‬‬ ‫‪e e‬‬ ‫‪y1 − α‬‬ ‫ج ‪ -‬ﻟﺪﻳﻨﺎ‬
‫‪1‬‬
‫‪−‬‬
‫‪y3 − α ≤ e‬‬ ‫‪e‬‬ ‫‪y2 − α‬‬

‫‪........‬‬
‫‪1‬‬
‫‪−‬‬
‫‪yn − α ≤ e‬‬ ‫‪e‬‬ ‫‪y n −1 − α‬‬
‫‪n −1‬‬
‫‪‬‬ ‫‪‬‬ ‫‪−‬‬
‫‪1‬‬
‫‪1‬‬
‫‪y n − α ≤  1‬‬ ‫‪‬‬
‫‪‬‬ ‫‪y1 − α‬‬ ‫و ﻣﻨﻪ‬ ‫‪y n − α ≤ (e‬‬ ‫‪e ) n −1‬‬ ‫‪y1 − α‬‬ ‫اذن‬
‫‪ e‬‬ ‫‪‬‬
‫‪e‬‬ ‫‪‬‬
‫‪n −1‬‬
‫‪‬‬ ‫‪‬‬
‫‪1‬‬
‫‪lim y n = α‬‬ ‫ﻣﺘﻘﺎرﺑﺔ و‬ ‫) ‪(y n‬‬ ‫ﻓﺎن‬ ‫‪lim ‬‬ ‫‪‬‬ ‫‪=0‬‬ ‫ﺑﻤﺎ أن‬
‫∞‪n →+‬‬ ‫‪n →+∞  1‬‬ ‫‪‬‬
‫‪ e‬‬ ‫‪‬‬
‫‪e‬‬ ‫‪‬‬
‫‪‬‬ ‫‪2x‬‬

‫‪‬‬ ‫∫‬
‫‪ F(x) = f (t)dt, x‬‬ ‫‪0‬‬
‫‪‬‬ ‫‪x‬‬
‫ﺑﻤﺎ ﻳﻠﻲ ‪:‬‬ ‫‪+‬‬
‫‪ ( III‬ﻟﺘﻜﻦ ‪ F‬اﻟﺪاﻟﺔ اﻟﻤﻌﺮﻓﺔ ﻋﻠﻰ‬
‫‪‬‬ ‫‪1‬‬
‫‪ F(0) = ln 2‬‬
‫‪‬‬ ‫‪2‬‬
‫‪t ≺ 1 + t − e− t‬‬ ‫أي‬ ‫‪t + e− t ≺ 1 + t‬‬ ‫اذن‬ ‫‪e− t ≺ 1‬‬ ‫أي‬ ‫اذن ‪− t ≺ 0‬‬ ‫ﻟﺪﻳﻨﺎ ‪t 0‬‬ ‫‪-1‬أ‪-‬‬
‫‪1‬‬ ‫‪1‬‬ ‫‪1‬‬
‫)‪(1‬‬ ‫≺ )‪f (t‬‬ ‫أي‬ ‫‪−t‬‬
‫≺‬ ‫اذن‬
‫‪t‬‬ ‫‪1+ t − e‬‬ ‫‪t‬‬
‫‪-8-‬‬
‫‪1‬‬ ‫‪1‬‬ ‫‪1‬‬
‫)‪(2‬‬ ‫‪1+ t‬‬
‫)‪≺ f (t‬‬ ‫أي‬ ‫≺‬
‫‪1 + t 1 + t − e− t‬‬
‫إذن‬ ‫‪1 + t − e− t ≺ 1 + t‬‬ ‫اذن‬ ‫‪−e − t ≺ 0‬‬ ‫ﻟﺪﻳﻨﺎ‬
‫‪1‬‬ ‫‪1‬‬
‫‪(∀t‬‬ ‫)‪0‬‬
‫‪1+ t‬‬
‫≤ )‪≤ f (t‬‬
‫‪t‬‬
‫ﻣﻦ )‪ (1‬و )‪ (2‬ﻧﺴﺘﻨﺘﺞ أن‬
‫‪1‬‬ ‫‪1‬‬
‫‪1+ t‬‬
‫≤ )‪≤ f (t‬‬
‫‪t‬‬
‫ب ‪ -‬ﻟﺪﻳﻨﺎ‬
‫‪2x‬‬ ‫‪2x‬‬ ‫‪x‬‬
‫‪1‬‬ ‫‪1‬‬
‫‪(x‬‬ ‫)‪0‬‬ ‫∫‬
‫‪x‬‬
‫‪1+ t‬‬
‫≤ ‪dt‬‬ ‫∫‬
‫‪x‬‬
‫≤ ‪f (t)dt‬‬ ‫اذن ‪∫ t dt‬‬
‫‪x‬‬

‫)‪ln(1 + 2x) − ln(1 + x) ≤ F(x) ≤ ln(2x) − ln(x‬‬ ‫اذن‬


‫‪1 + 2x‬‬ ‫‪2x‬‬
‫(‪ln‬‬
‫‪1+ x‬‬
‫)‪) ≤ F(x) ≤ ln( ) = ln(2‬‬
‫‪x‬‬
‫اذن‬
‫‪1 + 2x‬‬
‫‪lim F(x) = ln 2‬‬
‫∞‪x →+‬‬
‫ﻓﺎن‬ ‫(‪lim ln‬‬
‫∞‪x →+‬‬ ‫‪1+ x‬‬
‫ﺑﻤﺎ أن ‪) = ln 2‬‬ ‫و‬
‫‪t2‬‬
‫)‪(∀t ≥ 0‬‬ ‫‪1 − t ≤ e− t ≤ 1 − t +‬‬
‫‪2‬‬
‫‪ - 2‬أ ‪ -‬ﻟﻨﺒﻴﻦ أن‬
‫ﻟﺘﻜﻦ ‪ u(t) = 1 − t‬و‬
‫‪v(t) = e− t‬‬
‫‪v(0) = 1‬‬ ‫‪ u‬و ‪ v‬داﻟﺘﺎن ﻗﺎﺑﻠﺘﺎن ﻟﻼﺵﺘﻘﺎق ﻋﻠﻰ [∞‪ [0, +‬و ‪ u(0) = 1‬و‬
‫[∞‪∀t ∈ [ 0, +‬‬ ‫‪ v '(t) = −e− t‬و ‪u '(t) = −1‬‬

‫)‪u '(t) ≤ v '(t‬‬ ‫أي‬ ‫اي ‪−1 ≤ −e− t‬‬ ‫أي ‪e − t ≤ 1‬‬ ‫اذن ‪− t ≤ 0‬‬ ‫ﻟﺪﻳﻨﺎ ‪t ≥ 0‬‬
‫اذن )‪ u(t) ≤ v(t‬ﻟﻜﻞ )‪(1) (t ≥ 0‬‬
‫‪t2‬‬
‫‪u(0) = 1‬‬ ‫و‬ ‫‪w(0) = 1‬‬ ‫ﻟﺪﻳﻨﺎ‬ ‫‪w(t) = 1 − t +‬‬
‫‪2‬‬
‫ﻟﺘﻜﻦ‬
‫[∞‪∀t ∈ [ 0, +‬‬ ‫‪w '(t) = −1 + t‬‬
‫‪− e − t ≤ −1 + t‬‬ ‫أي‬ ‫‪1 − t ≤ e− t‬‬ ‫ﻓﺎن‬ ‫)‪u(t) ≤ v(t‬‬
‫ﺑﻤﺎ أن‬
‫اذن )‪v '(t) ≤ w '(t‬‬
‫و ﻣﻨﻪ )‪ v(t) ≤ w(t‬ﻟﻜﻞ ) ‪(2) ( t ≥ 0‬‬
‫ﻣﻦ ) ‪ ( 1‬و ) ‪ (2‬ﻧﺴﺘﻨﺘﺞ أن )‪u(t) ≤ v(t) ≤ w(t‬‬
‫‪t2‬‬
‫)‪(∀t ≥ 0‬‬ ‫‪1 − t ≤ e− t ≤ 1 − t +‬‬
‫‪2‬‬
‫و ﺑﺎﻟﺘﺎﻟﻲ‬
‫‪t≥0‬‬ ‫ﻟﻜﻞ‬ ‫‪1 − t ≤ e− t‬‬ ‫ب ‪ -‬ﻟﺪﻳﻨﺎ‬
‫‪1‬‬ ‫‪1‬‬
‫≤‬
‫‪2t 1 + t − e− t‬‬
‫اي‬ ‫‪1 + t − e− t ≤ 2t‬‬ ‫أي‬ ‫اذن ‪1 − e− t ≤ t‬‬

‫‪1‬‬
‫)‪(1‬‬ ‫‪2t‬‬
‫)‪≤ f (t‬‬ ‫اذن‬
‫‪1 1‬‬ ‫‪1‬‬ ‫‪1 1‬‬ ‫‪1‬‬ ‫‪1‬‬
‫‪( +‬‬
‫‪2 t 4−t‬‬
‫‪) − f (t) = ( +‬‬ ‫‪)−‬‬
‫‪2 t 4 − t 1 + t − e− t‬‬
‫ﻟﺪﻳﻨﺎ‬
‫‪2 − 2t + t 2 − 2e − t‬‬
‫=‬
‫) ‪t(4 − t)(1 + t − e − t‬‬
‫‪t2‬‬
‫‪2 − 2t + t 2 − 2e− t ≥ 0‬‬ ‫ﻓﺎن‬ ‫‪e− t ≤ 1 − t +‬‬
‫‪2‬‬
‫ﺑﻤﺎ أن‬
‫‪1 + t − e− t‬‬ ‫‪0‬‬ ‫ﻓﺎن‬ ‫‪(t‬‬ ‫)‪0‬‬ ‫ﻟﻜﻞ‬ ‫)‪f (t‬‬ ‫‪0‬‬ ‫و ﺑﻤﺎ أن‬
‫‪0≺ t≺ 4‬‬ ‫ﻻن‬ ‫)‪t(4 − t‬‬ ‫و ﻟﺪﻳﻨﺎ ‪0‬‬
‫‪1 1‬‬ ‫‪1‬‬
‫)‪(2‬‬ ‫≤ )‪f (t‬‬ ‫‪( +‬‬
‫‪2 t 4−t‬‬
‫)‬ ‫اذن‬
‫‪1‬‬ ‫‪1 1‬‬ ‫‪1‬‬
‫ﻟﻜﻞ ‪ t‬ﻣﻦ [‪]0, 4‬‬ ‫‪2t‬‬
‫‪≤ f (t) ≤ ( +‬‬
‫‪2 t 4−t‬‬
‫)‬ ‫ﻣﻦ )‪ (1‬و )‪ (2‬ﻧﺴﺘﻨﺘﺞ أن‬

‫‪-9-‬‬
‫‪1‬‬ ‫‪1 1‬‬ ‫‪1‬‬
‫‪2t‬‬
‫‪≤ f (t) ≤ ( +‬‬
‫‪2 t 4−t‬‬
‫)‬ ‫ج ‪ -‬ﻟﺪﻳﻨﺎ‬
‫‪2x‬‬ ‫‪2x‬‬ ‫‪2x‬‬
‫‪1‬‬ ‫‪1‬‬ ‫‪1‬‬ ‫‪1 1‬‬ ‫‪1‬‬
‫‪2‬‬
‫[‬ ‫‪ln t ]x ≤ F(x) ≤ [ lnt − ln(4 − t) ]x‬‬
‫‪2x‬‬
‫‪2‬‬
‫‪2x‬‬
‫أي‬ ‫∫‬ ‫‪2t‬‬
‫≤ ‪dt‬‬ ‫اذن ‪∫ f (t)dt ≤ ∫ 2 ( t + 4 − t )dt‬‬
‫‪x‬‬ ‫‪x‬‬ ‫‪x‬‬
‫‪ln 2‬‬ ‫‪ln 2 1 4 − 2x‬‬
‫≤ )‪≤ F(x‬‬ ‫(‪− ln‬‬ ‫أي )‬
‫‪2‬‬ ‫‪2‬‬ ‫‪2‬‬ ‫‪4−x‬‬
‫‪ln 2‬‬ ‫‪ 4 − 2x ‬‬
‫= )‪lim F(x‬‬
‫‪x →0‬‬ ‫‪2‬‬
‫)‪= F(0‬‬ ‫ﻓﺎن‬ ‫‪lim ln ‬‬
‫‪x →0  4 − x ‬‬
‫و ﺑﻤﺎ أن ‪ = ln1 = 0‬‬

‫‪x0 = 0‬‬ ‫اذن ‪ F‬ﻣﺘﺼﻠﺔ ﻓﻲ‬


‫‪2x‬‬
‫= )‪F(x‬‬ ‫‪ - 3‬أ ‪ -‬ﻟﺪﻳﻨﺎ ‪∫ f (t)dt‬‬
‫‪x‬‬

‫اذن ‪ f‬ﺗﻘﺒﻞ داﻟﺔ أﺹﻠﻴﺔ ‪ ϕ‬ﻋﻠﻰ ] ‪ [ x, 2x‬ﺑﺤﻴﺚ ‪ ϕ‬ﻗﺎﺑﻠﺔ ﻟﻼﺵﺘﻘﺎق ﻋﻠﻰ‬ ‫‪ f‬ﻣﺘﺼﻠﺔ ﻋﻠﻰ ] ‪[ x, 2x‬‬
‫‪(x‬‬ ‫)‪0‬‬ ‫] ‪[ x, 2x‬‬
‫و‬ ‫)‪ϕ '(x) = f (x‬‬
‫ﻟﺪﻳﻨﺎ )‪F(x) = [ ϕ(t) ]x = ϕ(2x) − ϕ(x‬‬
‫‪2x‬‬

‫اﻟﺪاﻟﺔ )‪ x → ϕ(x‬ﻗﺎﺑﻠﺔ ﻟﻼﺵﺘﻘﺎق ﻋﻠﻰ ] ‪[ x, 2x‬‬


‫اﻟﺪاﻟﺔ )‪ x → ϕ(2x‬ﻗﺎﺑﻠﺔ ﻟﻼﺵﺘﻘﺎق ﻋﻠﻰ ] ‪ [ x, 2x‬ﻷﻧﻬﺎ ﻣﺮآﺐ داﻟﺘﻴﻦ ﻗﺎﺑﻠﺘﻴﻦ ﻟﻼﺵﺘﻘﺎق‬
‫اذن ‪ F‬ﻗﺎﺑﻠﺔ ﻟﻼﺵﺘﻘﺎق ﻋﻠﻰ ] ‪ [ x, 2x‬ﺡﻴﺚ )‪(x 0‬‬
‫‪*+‬‬
‫اذن ‪ F‬ﻗﺎﺑﻠﺔ ﻟﻼﺵﺘﻘﺎق ﻋﻠﻰ‬
‫) ∈ ‪(∀x‬‬ ‫‪*+‬‬
‫)‪F '(x) = ϕ '(2x).2 − ϕ '(x) = 2f (2x) − f (x‬‬
‫‪2‬‬ ‫‪1‬‬
‫= )‪F '(x‬‬ ‫‪−‬‬
‫‪1 + 2x − e −2x‬‬ ‫‪1 + x − e− x‬‬
‫‪2‬‬ ‫‪1‬‬
‫‪1−‬‬ ‫‪+ 2x‬‬ ‫‪2x‬‬ ‫‪x‬‬
‫=‬ ‫‪e‬‬ ‫‪x‬‬
‫‪e = e − 2e + 1‬‬
‫)‪g(x).g(2x‬‬ ‫)‪e 2x .g(x).g(2x‬‬
‫‪(e x − 1) 2‬‬
‫= )‪F '(x‬‬
‫)‪e 2x .g(x).g(2x‬‬
‫‪*+‬‬
‫ﻟﻜﻞ ‪ x‬ﻣﻦ‬ ‫)‪g(2x‬‬ ‫‪0‬‬ ‫و‬ ‫)‪g(x‬‬ ‫‪0‬‬ ‫ب ‪ -‬ﻟﺪﻳﻨﺎ ‪ (ex − 1)2 ≥ 0‬و‬
‫‪*+‬‬
‫اذن ‪ F‬ﺗﺰاﻳﺪﻳﺔ ﻗﻄﻌﺎ ﻋﻠﻰ‬

‫‪- 10 -‬‬
SUJET DE BACCALAURÉAT (MAROC , Juin 2006)
EPREUVE DE MATHEMATIQUES, FILIERE SCIENCES MATH
La durée de l’épreuve est de 4 heures, coefficient 10 et l’usage des calculatrices NON programmables
est autorisé.

Exercice 1 (3,5 Points)


⎛ 1 0⎞
On note G l’ensemble des matrices de M 2 ( \ ) s’écrivant sous la forme M ( a , b ) = ⎜ ⎟ , avec
⎝a b⎠
(a , b)∈\ × \* .
( )
On rappelle que M 2 ( \ ) , + , × est un anneau unitaire.

Partie I
(
1. Montrer que G est une partie stable de M 2 ( \ ) , × . )
2. Montrer que ( G , × ) est un groupe. Ce groupe est-il commutatif ?
*
3. Soit H l’ensemble des matrices M ( a , b ) de G telles que ( a , b)∈\ × \ + .

Montrer que H est un sous groupe de ( G , × ) .

⎛ 1 0⎞
4. Soit A un élément de G tel que A = ⎜ ⎟ , a∈\ .
⎝a 1⎠
1 2 n +1
On pose A = A , A = A × A et A = A n × A pour tout entier naturel non nul n .
n
Donner l’expression de A en fonction de a et de n .

Partie II
*
Pour tout ( a , b) et ( x , y ) dans \ × \ , on définit la loi de composition interne T par :

(a , b) T ( x , y ) = (a + bx , by )

Soit ϕ * *
l’application définie de G vers \ × \ par : ∀ ( a , b)∈\ × \ , on a ( )
ϕ M ( a , b ) = ( a , b) .

1. Montrer que ϕ ( *
)
est un morphisme bijectif de ( G , × ) vers \ × \ , T .

(
*
2. En déduire la structure algébrique de \ × \ , T . )

1
3. Pour tout réel a et pour tout entier naturel n ≥ 2 , déterminer le symétrique de
( )
(a ,1) T (a ,1) T ............(a ,1) T dans \ × \* , T .

n fois

EXERCICE 2 (2,5 Points)


* * 2 2 2
On considère dans ` × ` l’équation x ( x + y ) = y ( x − y ) , on note (E) cette équation.

1. Soit le couple ( x , y ) une solution de (E).


On pose d = PGCD ( x , y ) , x = a d et y = b d .
2 2 2
a. Vérifier que d b ( a − b) = ( a + b) a .
b. En déduire que b = 1 .
c. Montrer que a ≠ 1 et que ( a − 1) divise ( a + 1) .
d. En déduire que a = 2 ou a = 3 .
* *
2. Résoudre dans ` × ` l’équation (E).

EXERCICE 3 (5 Points)
2
Pour tout nombre complexe z , on pose P ( z ) = z − (2 + 6 i ) z .

Partie I
JJG JJG
( )
Dans le plan complexe est rapporté à un repère orthonormé O , e1 , e 2 , on considère l’ensemble des
points M d’affixe z tels que P ( z ) est imaginaire pur. On note ( H ) cet ensemble.

2 2
1. Montrer que x − y − 2 x + 6 y = 0 est une équation cartésienne de ( H ) .
2. Montrer que ( H ) est une hyperbole puis déterminer son centre, ses sommets ainsi que deux
JJG JJG
(
équations de ses asymptotes dans le repère O , e1 , e 2 . )
3. Vérifier que le point O (Origine du repère) est un point de ( H ) puis donner une équation
JJG JJG
(
cartésienne de la tangente à ( H ) au point O dans le repère O , e1 , e 2 . )
JJG JJG
(
4. Tracer ( H ) dans le repère O , e1 , e 2 . )
Partie II
1. Résoudre dans ^ l’équation P ( z ) = 4 − 6 i .
⎛1⎞ ⎛ 1 ⎞
2. On pose u = 1 + 5 i , v = 1 + i , w = 239 − i , α = arctan ⎜ ⎟ et β = arctan ⎜ ⎟.
5 ⎝ ⎠ ⎝ 239 ⎠

2
4
a. Vérifier que u × v = 4 w .
b. Exprimer un argument de u en fonction de α et un argument de w en fonction de β.
⎛1⎞ ⎛ 1 ⎞ π
c. En déduire que 4arctan ⎜ ⎟ − arctan ⎜ ⎟= .
⎝5⎠ ⎝ 239 ⎠ 4

EXERCICE 4 (9 Points)

Partie I
Dans cette partie, n est un entier naturel supérieur ou égal à 3
*
On considère la fonction g n définie sur \ + par g n ( x ) = n x + 2 ln( x ) .

1. Dresser le tableau de variations de g n .


*
2. Montrer que pour tout x ∈ \ + , on a x > ln( x) .
3.
*
a. Montrer que l’équation g n ( x ) = 0 admet dans \ + une unique solution notée α n , puis
1 1
montrer que <αn < .
n n
b. En déduire que lim α n = 0 .
n→+∞

Partie II
I. Soit f [
la fonction définie sur 0 , + ∞ [ par f ( x) = 3 x e− x .
G G
On note C f sa représentation graphique dans un repère orthonormé O , i , j . ( )
G G
On prend i = j = 3 cm.

1. Etudier la dérivabilité de f à droite de zéro puis donner une interprétation géométrique de ce


résultat.
2. Calculer lim f ( x ) puis donner une interprétation géométrique de ce résultat.
x →+ ∞
3.
⎛ 1 − 3x ⎞
a. Montrer que pour tout réel x appartenant à ]0 , +∞ [ , f ' ( x) = ⎜ ⎟ f ( x) .
⎝ 3x ⎠
b. Dresser le tableau de variations de f .
⎛1⎞
4. Tracer C f . On prendra f ⎜ ⎟ ≈ 0,5 .
⎝3⎠

II. On pose I = ⎡⎢ , 1⎤⎥ .


1
⎣3 ⎦

3
1.
a. Montrer que f ( I ) ⊂ I .
' 2
b. A l’aide de la question 3.a de la Partie II, montrer que ∀x∈ I , f ( x ) ≤ .
3
c. Montrer que ⎡ x = α 3 ⇔ ( x > 0 et f ( x ) = x) ⎤ , où α3 est la solution de l’équation
⎣ ⎦
g 3 ( x) = 0 (Cf. Partie I).

( )n ≥ 0 la suite définie par u 0 = 13 et pour entier naturel n ,


2. Soit u n u n +1 = f (u n ) .

a. Montrer que pour entier naturel n , u n ∈ I .


2
b. Montrer que pour entier naturel n , u n +1 − α 3 ≤ u n −α 3 .
3
n +1
⎛2⎞
c. En déduire que pour entier naturel n , u n − α 3 ≤ ⎜ ⎟ .
⎝3⎠
d. Montrer que la suite u n
n≥0
( )
est convergente et donner sa limite.

Partie III
8x
[
Soit F la fonction définie sur 0 , + ∞ [ par F ( x) =
∫ x
f (t ) dt .

1.
a. Montrer que F est dérivable sur 0 , + ∞ . [ [
'
b. Donner l’expression de F ( x ) pour tout x appartenant à 0 , + ∞ [ [ et en déduire le sens
de variations de F .
2.

[ [
a. Montrer que pour tout x appartenant à 0 , + ∞ , on a 0 ≤ F ( x ) ≤ 2 f ( x) 1 − e ( −7x
).
b. En déduire la valeur de lim F ( x) .
x→+∞
c. Dresser le tableau de variations de F .

FIN DU SUJET

4
‫‪1 0‬‬
‫‪ I = ‬ﺗﺤﻘﻖ ‪. I = M(0,1) :‬ﺇﺫﻥ ‪ I‬ﻫﻮ ﺍﻟﻌﻨﺼﺮ‬ ‫‪ (2‬ﺍﻟﻤﺼﻔﻮﻓﺔ ‪‬‬
‫‪0 1‬‬ ‫א‬ ‫א‬ ‫א‬
‫ﺍﻟﻤﺤﺎﻳﺪ ﻝ )×‪. (G,‬‬ ‫‪:‬א‬
‫ﻟﻴﻜﻦ ‪ (a,b) ∈ \ 2‬ﺑﺤﻴﺚ ‪ . b ≠ 0‬ﻟﺪﻳﻨﺎ ‪det (M(a,b) ) = b ≠ 0 :‬‬ ‫מא‬ ‫‪:‬א‬
‫ﺇﺫﻥ ‪ M(a,b) :‬ﺗﻘﺒﻞ ﻣﻘﻠﻮﺑﺎ ﻓﻲ )×‪ (M 2 ( \ ),‬ﻭﻫﻮ ‪:‬‬
‫‪2006‬‬ ‫‪:‬‬ ‫א‬ ‫א‬
‫‪−1‬‬ ‫‪1  b 0‬‬ ‫إﳒﺎز ‪ :‬ﻣﺤﻤﺪ أﻳﺖ اﳊﺴﲔ‬
‫= ) )‪(M(a,b‬‬ ‫‪‬‬ ‫‪‬‬
‫‪b −a 1‬‬ ‫أﺳﺘﺎذ ﺑﺜﺎﻧﻮﻳﺔ ﻣﻮﻻي رﺷﻴﺪ ‪ :‬ﻓﺎس‬
‫‪−1‬‬
‫) )‪(M(a,b‬‬ ‫‪= M‬‬ ‫‪‬‬
‫‪− a , 1 ‬‬
‫ﻧﻼﺣﻆ ﺃﻥ ‪∈ G :‬‬
‫‪ b b ‬‬

‫× ﺗﺠﻤﻴﻌﻲ ﻓﻲ ) \ ( ‪ M 2‬ﺇﺫﻥ ﻓﻲ ‪ G‬ﺑﺎﻷﺣﺮﻯ‪.‬‬ ‫‪@ Þëþa@åí‹ànÛa‬‬


‫ﻣﻦ ﻛﻞ ﻣﺎ ﺳﺒﻖ ﻧﺴﺘﻨﺘﺞ ﺃﻥ ‪ (G,×) :‬ﺯﻣﺮﺓ‪.‬‬ ‫‪:‬‬ ‫א‬ ‫א‬
‫‪:‬‬ ‫‪3‬‬
‫ﺣﺴﺐ ﻣﺎ ﺳﺒﻖ ﺃﻋﻼﻩ ﻟﺪﻳﻨﺎ ﻟﻜﻞ \ ∈ )‪ (a,a ',b‬ﺑﺤﻴﺚ ‪b ≠ 0‬‬ ‫‪ (1‬ﻟﺪﻳﻨﺎ ﻟﻜﻞ ‪ (a,a ',b,b') ∈ \ 4‬ﺑﺤﻴﺚ ‪ b ≠ 0‬ﻭ ‪: b' ≠ 0‬‬
‫)‪ M(a,b) × M(a ',b) = M(a ',b) × M(a,b‬ﺇﺫﺍ ﻭﻓﻘﻂ ﺇﺫﺍ ‪:‬‬ ‫‪1 0   1 0 ‬‬
‫‪M(a,b) × M(a ',b') = ‬‬ ‫‪×‬‬ ‫‪‬‬
‫‪a b a ' b'‬‬
‫‪ a + ba ' = a '+ ba‬ﺇﺫﺍ ﻭﻓﻘﻂ ﺇﺫﺍ ‪:‬‬
‫‪ 1‬‬ ‫‪0 ‬‬
‫‪ (a − a ')(1− b) = 0‬ﺃﻱ ‪ b = 1 :‬ﺃﻭ ' ‪. a = a‬‬ ‫‪= ‬‬ ‫)'‪ = M(a+ba ',bb‬‬
‫‪a + ba ' bb'‬‬
‫ﻳﻜﻔﻲ ﺇﺫﻥ ﺃﻥ ﻧﺨﺘﺎﺭ ‪ (a,a ',b) ∈ \ 3‬ﺑﺤﻴﺚ ‪:‬‬
‫‪ b ≠ 1‬ﻭ ' ‪ a ≠ a‬ﻟﻜﻲ ﺗﻜﻮﻥ ‪:‬‬
‫ﻓﺈﻥ ‪ M(a+ba ',bb') ∈ G :‬ﻭﻣﻨﻪ ‪ G :‬ﻣﺴﺘﻘﺮﺓ ﻗﻲ‬ ‫ﺑﻤﺎ ﺃﻥ ‪bb' ≠ 0 :‬‬
‫)‪M(a,b) × M(a ',b) ≠ M(a ',b) × M(a,b‬‬
‫)×‪(M 2 ( \ ),‬‬
‫ﻣﺜﻼ ‪ (a,a ',b) = (0,1,−1) :‬ﻧﺠﺪ ‪:‬‬
‫)‪M(0,−1) × M(1,−1) = M(−1,−1‬‬
‫‪1‬‬
 1 0 1 0  1 0 M(1,−1) × M(0,−1) = M(1,−1) ‫ﻭ‬
A n+1 = A n × A =  ×  =  
na 1 a 1 (n +1)a 1 .‫( ﻟﻴﺴﺖ ﺗﺒﺎﺩﻟﻴﺔ‬G,×) ‫ﺇﺫﻥ ﺍﻟﺰﻣﺮﺓ‬

 1 0 . M(0,1) ∈ H : ‫ ﻷﻥ‬H ≠ ∅ ‫ ﻭ‬H ⊂ G (3


( ∀n ∈ ` * ) A n =   : ‫ﻭﻣﻨﻪ‬
na 1
: ‫( ﻟﺪﻳﻨﺎ‬b,b') ∈ \ 2 ‫ﻟﻜﻞ‬
(b > 0 ‫ ﻭ‬b' > 0 ) ⇒ bb' > 0
: ‫א‬ ‫א‬ .(G,×) ‫ ﻣﺴﺘﻘﺮﺓ ﻗﻲ‬H ‫ﺇﺫﻥ‬
: ‫ ﻣﻌﺮﻑ ﻷﻥ‬ϕ (1 1
b> 0 ⇒ > 0 : b ‫ﻭ ﻟﺪﻳﻨﺎ ﻟﻜﻞ ﻋﺪﺩ ﺣﻘﻴﻘﻲ‬
( ∀(a,a ',b,b') ∈ \ 2 × \ *2 ) M(a,b) = M(a ',b') ⇒ (a,b) = (a ',b') b
.‫ ﺳﺎﺑﻘﺎ ﻟﻪ‬M(a,b) ‫( ﻳﻘﺒﻞ‬a,b) : ‫ ﺷﻤﻮﻟﻲ ﻷﻥ‬ϕ : ‫ﺇﺫﻥ‬
: ‫ ﺗﺒﺎﻳﻨﻲ ﻷﻥ‬ϕ M(a,b) ∈ H ⇒ (M(a,b) )−1 = M  ∈H
− a , 1 
 b b 
ϕ(M(a,b) ) = ϕ(M(a ',b') ) ⇒ (a,b) = (a 'b')
. (G,×) ‫ ﺯﻣﺮﺓ ﺟﺰﺋﻴﺔ ﻟﻠﺰﻣﺮﺓ‬H ‫ ﻭﻣﻨﻪ‬.‫ ﻣﺴﺘﻘﺮﺓ ﺑﺎﻟﻤﻘﻠﻮﺏ‬H ‫ﺇﺫﻥ‬
⇒ M(a,b) = M(a ',b')
. ‫ ﺗﻘﺎﺑﻞ‬ϕ ‫ﺇﺫﻥ‬
1 0 1 0  1 0
2
(a,a ',b,b') ∈ \ × \ *2
‫ ﻟﻜﻞ‬: ‫ ﺗﺸﺎﻛﻞ ﻷﻥ‬ϕ A 2 =  ×  =   : ‫( ﻟﺪﻳﻨﺎ‬4
a 1 a 1 2a 1
ϕ (M(a,b) × M(a ',b') ) = ϕ (M(a+ba ',bb') )  1 0
P(n) : A n =   : ‫ﻟﻨﺒﻴﻦ ﺑﺎﻟﺘﺮﺟﻊ ﺃﻥ‬
= (a + ba ',bb') = (a,b)T(a ',b') na 1
= (ϕ (M(a,b) )) T (ϕ (M(a ',b') )) . ‫ ﺻﺤﻴﺤﺔ‬P(1) ‫ﻟﺪﻳﻨﺎ‬
: ‫ﺇﺫﻥ‬. P(n) ‫ﻧﻔﺘﺮﺽ ﺃﻥ‬

2
‫‪a 2d 2 (ad + bd) = b2d 2 (ad − bd)2‬‬ ‫‪ ( \ × \ * ,T ) (2‬ﺯﻣﺮﺓ ﻏﻴﺮ ﺗﺒﺎﺩﻟﻴﺔ ﻷﻧﻬﺎ ﺻﻮﺭﺓ ﺯﻣﺮﺓ ﻏﻴﺮ ﺗﺒﺎﺩﻟﻴﺔ‬
‫ﻭﺑﻤﺎ ﺃﻥ ‪ d ≠ 0 :‬ﻓﺈﻥ ‪:‬‬ ‫ﺑﺘﺸﺎﻛﻞ ﺗﻘﺎﺑﻠﻲ‪.‬‬
‫ﻣﻼﺣﻈﺔ ‪ :‬ﺍﻟﻌﻨﺼﺮ ﺍﻟﻤﺤﺎﻳﺪ ﻟﻬﺬﻩ ﺍﻟﺰﻣﺮﺓ ﻫﻮ ‪ϕ (M(0,1) ) = (0,1) :‬‬
‫‪a 2 (a + b) = b2d(a − b)2‬‬
‫ﻭﻣﻤﺎﺛﻞ )‪ (a,b‬ﻫﻮ ‪:‬‬
‫‪3‬‬
‫(‪.‬‬ ‫ﺍﺧﺘﺰﻟﻨﺎ ﺏ ‪d‬‬ ‫)‬ ‫‪‬‬ ‫‪  a 1 ‬‬
‫(‬
‫) )‪(a,b)' = ϕ (M(a,b‬‬
‫‪−1‬‬
‫)‬ ‫‪‬‬
‫‪= ϕ M a 1   = − , ‬‬
‫‪ −b, b    b b ‬‬
‫ﺏ( ﻟﺪﻳﻨﺎ ﺣﺴﺐ ﺃ( ‪ b|(a + b)a 2 :‬ﻭ ﺑﻤﺎ ﺃﻥ ‪ d = x ∧ y :‬ﻓﺈﻥ ‪:‬‬
‫‪(3‬‬
‫‪2‬‬
‫‪ a ∧ b = 1‬ﻭﻣﻨﻪ ﺃﻳﻀﺎ ‪ a ∧ b = 1 :‬ﻭﻣﻨﻪ ﺣﺴﺐ ﻣﺒﺮﻫﻨﺔ ﻛﻮﺹ ‪:‬‬
‫)‪ . b|(a + b‬ﺇﺫﻥ ‪b|(a + b) − b = a‬‬
‫')) )‪((a,1)T...T(a,1))' = (ϕ (M(a,1) ) T...Tϕ (M(a,1‬‬
‫‪b =1‬‬
‫((‬ ‫))‬
‫'‬
‫ﺃﻱ ‪ a ∧ b = b :‬ﻭﻣﻨﻪ ‪:‬‬
‫(‬ ‫)‬
‫'‬
‫) )‪= ϕ ((M(a,1) )×...×(M(a,1) )) = ϕ (M(a,1‬‬
‫‪n‬‬

‫ﺝ( ﺗﺼﺒﺢ ﺍﻟﻤﺘﺴﺎﻭﻳﺔ ﺃﻋﻼﻩ ﻓﻲ ﺃ( ‪:‬‬


‫(‬
‫) )‪= (ϕ (M(na,1) )) = ϕ (M(na,1‬‬ ‫)‬
‫'‬ ‫‪−1‬‬

‫‪d(a −1)2 = (a +1)a 2‬‬


‫ﻭﻣﻨﻪ ‪ a = 0) :‬ﺃﻭ ‪a = 1 ⇒ (a = −1‬‬ ‫(‬ ‫)‬
‫)‪= ϕ M(−na,1) = (−na,1‬‬
‫ﻫﺬﺍ ﻳﻌﻨﻲ ﺑﺎﻟﺨﺼﻮﺹ ﺃﻥ ﺍﻟﻌﺒﺎﺭﺓ ‪ a = 1‬ﺧﺎﻃﺌﺔ‪.‬‬
‫)‪((a,1)T...T(a,1))' = (−na,1‬‬
‫ﻣﻠﺨﺺ ‪:‬‬
‫‪a ≠1‬‬
‫ﺇﺫﻥ ‪:‬‬
‫ﻟﺪﻳﻨﺎ ﺇﺫﻥ ‪(a −1)|(a +1)a 2 :‬‬ ‫‪@ Z@ïãbrÛa@åí‹ànÛa‬‬

‫‪ (1‬ﺃ( ﺑﺎﻟﺘﻌﻮﻳﺾ ﺍﻟﻤﺒﺎﺷﺮ ﻧﺠﺪ ‪:‬‬


‫‪3‬‬
‫ﺇﺫﺍ ﻛﺎﻥ ‪ a = 3‬ﻓﺈﻥ ‪. d = 9 :‬‬ ‫ﻭﻧﻌﻠﻢ ﺃﻥ ‪ ) (a −1) ∧ a = 1 :‬ﺣﺴﺐ ﻣﺘﻄﺎﺑﻘﺔ ﺑﻮﺯﻭ ﻭﺍﻟﻌﻼﻗﺔ ‪:‬‬
‫ﻭﻣﻨﻪ ‪(x,y) ∈ {(24,2),(27,9)} :‬‬ ‫‪ ( a − (a −1) = 1‬ﺇﺫﻥ ﻛﻤﺎ ﺳﺒﻖ ‪ (a −1) ∧ a 2 = 1 :‬ﻭﻣﻨﻪ ﺣﺴﺐ‬
‫ﺑﺈﻧﺠﺎﺯ ﺍﻟﺤﺴﺎﺏ ﺗﺤﻘﻘﻨﺎ ﻣﻦ ﻛﻮﻥ ﺍﻟﺰﻭﺟﻴﻦ ﺃﻋﻼﻩ ﺣﻠﻴﻦ ﻟﻠﻤﻌﺎﺩﻟﺔ ﻭﻣﻨﻪ‬ ‫ﻣﺒﺮﻫﻨﺔ ﻛﻮﺹ ‪:‬‬
‫ﻣﺠﻤﻮﻋﺔ ﺍﻟﺤﻠﻮﻝ ‪:‬‬
‫)‪(a −1)|(a +1‬‬
‫})‪S = {(24,2),(27,9‬‬
‫ﺩ( ﻟﺪﻳﻨﺎ ‪:‬‬
‫‪‬‬
‫‪‬‬
‫‪‬‬‫)‪(a −1)|(a +1‬‬
‫‪‬‬ ‫))‪⇒ (a −1)|((a +1) − (a −1‬‬
‫‪‬‬
‫‪‬‬
‫‪‬‬‫‪(a‬‬ ‫‪−‬‬‫‪1)|(a‬‬ ‫‪−‬‬ ‫)‪1‬‬
‫‪@ Z@sÛbrÛa@åí‹ànÛa‬‬
‫ﻭﻣﻨﻪ ‪.(a −1)|2 :‬‬
‫ﻭﺑﻤﺎ ﺃﻥ ‪ a −1 > 0‬ﻓﺈﻥ ‪a −1 ∈ {1,2} :‬‬
‫‪:‬‬ ‫א‬ ‫א‬
‫‪ (1‬ﻧﻀﻊ ‪ z = x + yi :‬ﺑﺤﻴﺚ ‪.(x,y) ∈ \ 2 :‬ﻟﺪﻳﻨﺎ ‪:‬‬ ‫‪ a = 2‬ﺃﻭ ‪a = 3‬‬ ‫ﺃﻱ ‪:‬‬
‫‪M(z) ∈ (H) ⇔ Re(P(z)) = 0‬‬
‫‪⇔ Re(x 2 − y 2 + 2ixy − (2 + 6i)(x + yi)) = 0‬‬ ‫‪ (2‬ﻟﺪﻳﻨﺎ ﺣﺴﺐ ﻣﺎ ﺳﺒﻖ ‪ :‬ﺇﺫﺍ ﻛﺎﻥ )‪ (x,y‬ﺣﻼ ﻟﻠﻤﻌﺎﺩﻟﺔ )‪ (E‬ﻓﺈﻧﻪ ﻳﻮﺟﺪ‬
‫‪⇔ Re((x 2 − y 2 − 2x + 6y) + i(2xy − 6x − 2y)) = 0‬‬
‫* ` ∈ ‪ d‬ﺑﺤﻴﺚ ‪:‬‬
‫‪⇔ x 2 − y 2 − 2x + 6y = 0‬‬
‫‪‬‬
‫‪‬‬ ‫‪x = da‬‬
‫‪‬‬
‫‪‬‬
‫‪x 2 − y 2 − 2x + 6y = 0‬‬ ‫‪‬‬
‫‪y = d‬‬
‫ﻣﻌﺎﺩﻟﺔ ﺩﻳﻜﺎﺭﺗﻴﺔ ﻝ )‪. (H‬‬ ‫ﻭﻣﻨﻪ ‪:‬‬ ‫‪‬‬
‫‪‬‬
‫‪‬‬‫}‪a ∈ {2,3‬‬
‫‪‬‬
‫‪‬‬ ‫‪2‬‬ ‫‪2‬‬
‫‪d(a −1) = (a +1)a‬‬
‫‪‬‬
‫ﺇﺫﺍ ﻛﺎﻥ ‪ a = 2‬ﻓﺈﻥ ‪d = 12 :‬‬

‫‪4‬‬
‫‪ (2‬ﻟﺪﻳﻨﺎ ‪:‬‬
‫‪ (D) :y = x + 2‬ﻭ ‪(D'): y − 3 = −x + 4‬‬ ‫ﺃﻱ ‪:‬‬
‫‪x 2 − y 2 − 2x + 6y = 0 ⇔ (x −1)2 − (y − 3)2 = −8‬‬
‫‪X2‬‬ ‫‪Y2‬‬
‫ﻣﻠﺤﻮﻇﺔ ‪ :‬ﻟﻢ ﺗﻄﻠﺐ ﺍﻟﻌﻨﺎﺻﺮ ﺍﻟﻤﻤﻴﺰﺓ ﻟﻜﻦ ﻧﻌﻄﻴﻬﺎ ﺇﺗﻤﺎﻣﺎ ﻟﻠﻔﺎﺋﺪﺓ ‪:‬‬ ‫⇔‬ ‫‪−‬‬ ‫‪= −1‬‬
‫)‪( 8) ( 8‬‬
‫‪2‬‬ ‫‪2‬‬

‫ﻟﺪﻳﻨﺎ ‪ a = b = 8 :‬ﻭﻣﻨﻪ ‪c = a 2 + b2 = 4 :‬‬


‫‪‬‬
‫‪‬‬
‫‪‬‬ ‫‪X = x −1‬‬
‫‪c‬‬ ‫‪4‬‬ ‫‪‬‬ ‫ﺑﺤﻴﺚ ‪:‬‬
‫= =‪e‬‬ ‫ﻭﻣﻨﻪ ﺍﻟﺘﺒﺎﻋﺪ ﺍﻟﻤﺮﻛﺰﻱ ‪= 2 :‬‬ ‫‪‬‬
‫‪‬‬ ‫‪Y‬‬ ‫=‬ ‫‪y‬‬ ‫‪−‬‬ ‫‪3‬‬
‫‪a‬‬ ‫‪8‬‬ ‫‪‬‬
‫‪a2‬‬ ‫ﻭﻣﻨﻪ )‪ (H‬ﻫﺬﻟﻮﻝ ﻣﺮﻛﺰﻩ ﻫﻮ ‪ Ω(1,3) :‬ﻭ ‪:‬‬
‫ﺍﻟﺒﺆﺭﺓ ‪ . F(0,4) :‬ﺍﻟﺪﻟﻴﻞ ﺍﻟﻤﺮﺗﺒﻂ ﺑﻬﺎ ‪(∆): Y = = 2 :‬‬
‫‪c‬‬ ‫‪X2‬‬ ‫‪Y2‬‬
‫‪−‬‬ ‫‪= −1‬‬
‫)‪( 8) ( 8‬‬
‫‪2‬‬ ‫‪2‬‬ ‫‪2‬‬
‫‪a‬‬
‫ﺍﻟﺒﺆﺭﺓ ‪ . F(0, −4) :‬ﺍﻟﺪﻟﻴﻞ ﺍﻟﻤﺮﺗﺒﻂ ﺑﻬﺎ ‪(∆): Y = − = −2 :‬‬
‫‪c‬‬ ‫‪JJG JJG‬‬
‫‪JJG JJG‬‬
‫(‬
‫ﻓﻲ ﺍﻟﻤﻌﻠﻢ ‪. Ω,e1,e2‬‬ ‫)‬ ‫(‬ ‫)‬
‫ﻣﻌﺎﺩﻟﺔ ﻟﻪ ﻓﻲ ﺍﻟﻤﻌﻠﻢ ‪. Ω,e1,e2 :‬‬

‫(‬ ‫)‬
‫ﺍﻟﺮﺃﺳﺎﻥ ‪ :‬ﻟﺪﻳﻨﺎ ‪ a = b = 8 :‬ﻭﻣﻨﻪ ﺍﻟﺮﺃﺳﺎﻥ ‪ A 0, 8 :‬ﻭ‬
‫‪JJG JJG‬‬
‫‪ (4‬ﺍﻹﻧﺸﺎﺀ ‪ :‬ﺍﻧﻈﺮ ﺍﻟﺸﻜﻞ ﺃﺳﻔﻠﻪ‪.‬‬ ‫(‬ ‫)‬ ‫(‬
‫‪ A ' 0,− 8‬ﻓﻲ ﺍﻟﻤﻌﻠﻢ ‪ Ω,e1,e2‬ﺃﻱ ‪ A 1,3 + 8 :‬ﻭ‬‫)‬ ‫(‬ ‫)‬
‫‪JJG JJG‬‬
‫‪:‬‬ ‫א‬ ‫א‬ ‫(‬ ‫)‬
‫‪ A ' 3,3 − 8‬ﻓﻲ ﺍﻟﻤﻌﻠﻢ ‪. Ω,e1,e2‬‬ ‫(‬ ‫)‬
‫‪ (1‬ﺍﻟﻤﻌﺎﺩﻟﺔ ‪ P(z) = 4 − 6i :‬ﺗﻜﺎﻓﺊ ‪:‬‬ ‫‪JJG JJG‬‬
‫‪2‬‬
‫(‬ ‫)‬
‫ﺍﻟﻤﻘﺎﺭﺑﺎﻥ ‪ :‬ﻣﻌﺎﺩﻟﺘﺎ ﺍﻟﻤﻘﺎﺭﺑﻴﻦ ﻓﻲ ﺍﻟﻤﻌﻠﻢ ‪: Ω,e1,e2‬‬
‫‪z − (2 + 6i)z − 4 + 6i = 0‬‬ ‫و ‪(D'): Y = −X‬‬ ‫‪(D): Y = X‬‬
‫ﻣﻤﻴﺰﻫﺎ ﺍﻟﻤﺨﺘﺼﺮ ﻫﻮ ‪. ∆ ' = (1 + 3i) + 4 − 6i = −4 = (2i)2 :‬‬
‫‪2‬‬ ‫‪JJG JJG‬‬
‫(‬ ‫)‬
‫ﻭﻣﻨﻪ ﻣﻌﺎﺩﻟﺘﻴﻬﻤﺎ ﻓﻲ ﺍﻟﻤﻌﻠﻢ ‪: O,e1,e 2 :‬‬
‫ﻭﻣﻨﻪ ﺟﺬﺭﺍﻫﺎ ‪ z1 = 1 + 3i − 2i = 1 + i :‬ﻭ‬
‫ﻭ ‪(D'): y − 3 = 1− x‬‬ ‫‪(D): y − 3 = x −1‬‬
‫‪z2 = 1 + 3i + 2i = 1 + 5i‬‬

‫‪5‬‬
‫‪1‬‬
‫‪ tan(arg(w)) = −‬ﻭ‬ ‫ﺑﺎﻟﻤﺜﻞ ‪:‬‬
‫‪239‬‬
‫‪1‬‬
‫‪ sin(arg(w)) = −‬ﺇﺫﻥ ‪:‬‬ ‫‪<0‬‬
‫‪169 2‬‬
‫‪ 1 ‬‬
‫‪arg(w) ≡ −Arc tan ‬‬
‫] [ ‪ 239 ‬‬
‫‪2π‬‬

‫‪π‬‬
‫≡ )‪arg(w) ≡ −β [2π] arg(u‬‬ ‫‪−α‬‬ ‫ﻣﻠﺨﺺ ‪[2π] :‬‬
‫‪2‬‬
‫ﺝ( ﻣﻦ ﺍﻟﻌﻼﻗﺔ )‪ (1‬ﺃﻋﻼﻩ ﻧﺴﺘﻨﺘﺞ ﺃﻥ ‪:‬‬
‫]‪arg(u 4 v) ≡ arg(4w) [2π‬‬
‫}‪S = {1 + i,1 + 5i‬‬
‫‪π‬‬ ‫ﻭﻣﻨﻪ ﻣﺠﻤﻮﻋﺔ ﺣﻠﻮﻟﻬﺎ ‪:‬‬
‫ﻭﻣﻨﻪ ‪ . 2π − 4α + ≡ −β [2π] :‬ﺇﺫﻥ ‪:‬‬
‫‪4‬‬
‫‪π‬‬ ‫‪ (2‬ﺃ( ﻟﺪﻳﻨﺎ ﺑﺎﺳﺘﻌﻤﺎﻝ ﺻﻴﻐﺔ ﺣﺪﺍﻧﻴﺔ ﻧﻴﻮﺗﻦ ‪u 4 = 476 − 480i :‬‬
‫≡‪4α − β‬‬ ‫)‪[2π] (2‬‬
‫‪4‬‬
‫ﻭﻣﻨﻪ ‪ . u 4 v = 956 − 4i = 4(239 − i) :‬ﺇﺫﻥ ‪:‬‬
‫ﻣﻦ ﺟﻬﺔ ﺃﺧﺮﻯ ‪:‬‬
‫‪1‬‬ ‫‪π‬‬ ‫‪u 4 v = 4w‬‬ ‫)‪(1‬‬
‫‪0 < < 1 ⇒ 0 < α < ⇒ 0 < 4α < π‬‬
‫‪5‬‬ ‫‪4‬‬
‫‪1‬‬ ‫‪π‬‬ ‫‪5‬‬
‫<‪0‬‬ ‫ﻭ < ‪< 1 <⇒ 0 < β‬‬ ‫= ))‪sin(arg(u‬‬ ‫ﺏ( ﻟﺪﻳﻨﺎ ‪ tan(arg(u)) = 5 :‬ﻭ ‪> 0‬‬
‫‪239‬‬ ‫‪4‬‬ ‫‪26‬‬
‫‪π‬‬ ‫‪π‬‬
‫ﻭﻣﻨﻪ ‪− < 4α − β < π :‬‬ ‫ﻭﻣﻨﻪ ‪arg(u) ≡ Arc tan (5) ≡ −α [2π] :‬‬
‫‪4‬‬ ‫‪2‬‬

‫‪6‬‬
‫‪1‬‬ ‫‪1‬‬ ‫‪x‬‬ ‫‪π‬‬
‫= )‪( ∀x > 0) u '(x‬‬
‫‪2 x‬‬ ‫‪x‬‬
‫‪−‬‬
‫=‬
‫‪2x x‬‬
‫‪x −2‬‬ ‫(‬ ‫)‬ ‫ﻭﻣﻦ )‪ (2‬ﺃﻋﻼﻩ ﻧﺴﺘﻨﺘﺞ ﺃﻥ ‪(∃k ∈ ]) 4α − β = + k2π :‬‬
‫‪4‬‬
‫ﻭ ﻣﻨﻪ ‪ u '(x) > 0 ⇔ x > 4 :‬ﻭ ‪u '(x) = 0 ⇔ x = 4‬‬ ‫ﻭﻣﻦ ﺍﻟﻤﺘﻔﺎﻭﺗﺔ ﺍﻟﻤﺰﺩﻭﺟﺔ ﺃﻋﻼﻩ ﻧﺤﺼﻞ ﻋﻠﻰ ‪:‬‬

‫ﻭﻫﺬﺍ ﻳﻌﻨﻲ ﺑﺎﻟﺨﺼﻮﺹ ﺃﻥ )‪ u(4‬ﻗﻴﻤﺔ ﺩﻧﻴﺎ ﻣﻄﻠﻘﺔ ﻟﻠﺪﺍﻟﺔ ‪. u‬‬ ‫‪1‬‬ ‫‪3‬‬ ‫‪π π‬‬
‫‪ − < + k2π < π‬ﺃﻱ ‪ − < k < :‬ﻭﻣﻨﻪ ‪k = 0 :‬‬
‫‪2‬‬ ‫‪4‬‬ ‫‪4 4‬‬
‫ﺇﺫﻥ ‪( ∀x > 0) u(x) ≥ u(4) = 2 − ln4 = 2(1− ln2) > 0 :‬‬ ‫‪π‬‬
‫ﺇﺫﻥ ‪ 4α − β= :‬ﻭﻣﻨﻪ ‪:‬‬
‫ﻷﻥ ‪ 1− ln(2) = ln(e) − ln(2) :‬ﻭ ‪ e > 2‬ﻭ ‪ ln‬ﺗﺰﺍﻳﺪﻳﺔ ﻗﻄﻌﺎ‪.‬‬ ‫‪4‬‬
‫‪1‬‬ ‫‪ 1  π‬‬
‫) ‪(∀x ∈ \ *+‬‬ ‫‪x − ln x > 0‬‬ ‫‪4Arc tan   − Arc tan ‬‬ ‫=‪‬‬
‫ﺇﺫﻥ ‪:‬‬ ‫‪ 5 ‬‬ ‫‪ 239  4‬‬

‫‪ (3‬ﺃ( ﺍﻟﺪﺍﻟﺔ ‪ gn‬ﻣﺘﺼﻠﺔ ﻭﺗﺰﺍﻳﺪﻳﺔ ﻗﻄﻌﺎ ﻋﻠﻰ ﺍﻟﻤﺠﺎﻝ [∞‪I =]0, +‬‬
‫‪: Éia‹Ûa@åí‹ànÛa‬‬
‫ﺇﺫﻥ ‪ gn‬ﺗﻘﺎﺑﻞ ﻣﻦ ‪ I‬ﻧﺤﻮ )‪. g n (I‬ﻟﺪﻳﻨﺎ ﺣﺴﺐ ﺍﻟﺴﺆﺍﻝ ‪: (1‬‬
‫‪:‬‬ ‫א‬ ‫א‬
‫\ = )‪gn (I‬‬
‫‪2‬‬
‫ﺑﺎﻟﺨﺼﻮﺹ ‪ 0‬ﻳﻘﺒﻞ ﺳﺎﺑﻘﺎ ﻭﺣﻴﺪﺍ ‪ α n‬ﺏ ‪ gn‬ﻓﻲ ‪. I‬‬ ‫‪ ( ∀x > 0) g n '(x) = n +‬ﻭﻣﻨﻪ ﺍﻟﺪﺍﻟﺔ ‪g n‬‬‫‪ (1‬ﻟﺪﻳﻨﺎ ‪> 0 :‬‬
‫‪x‬‬
‫‪ 1 ‬‬ ‫ﺗﺰﺍﻳﺪﻳﺔ ﻗﻄﻌﺎ ﻋﻠﻰ [∞‪ ]0,+‬ﻭﻣﻨﻪ ﺟﺪﻭﻝ ﺗﻐﻴﺮﺍﺗﻬﺎ‬
‫‪ gn ‬ﺣﺴﺐ ﺍﻟﺴﺆﺍﻝ ‪(2‬‬ ‫ﻟﺪﻳﻨﺎ ‪= n − ln(n) > 0 :‬‬
‫‪ n ‬‬
‫‪1‬‬
‫ﻭ ‪ gn   = 1− 2ln(n) < 0 :‬ﻷﻥ ‪ . n ≥ 3 :‬ﻫﺬﺍ ﻳﻌﻨﻲ ﺃﻥ ‪:‬‬
‫‪ n ‬‬
‫‪ 1‬‬ ‫‪ 1‬‬
‫‪gn   < gn (α n ) < gn  ‬‬
‫‪ n‬‬ ‫‪ n ‬‬

‫‪ (2‬ﻟﻨﺪﺭﺱ ﺗﻐﻴﺮﺍﺕ ﺍﻟﺪﺍﻟﺔ ‪ . x 6 u(x) = x − ln x :‬ﻟﺪﻳﻨﺎ ‪:‬‬


‫‪7‬‬
‫‪1‬‬ ‫‪1‬‬
‫‪1‬‬ ‫‪‬‬ ‫< ‪< αn‬‬
‫‪2‬‬ ‫‪1‬‬
‫‪1‬‬ ‫‪‬‬ ‫‪1‬‬
‫‪1− 3x‬‬ ‫ﻭﺑﻤﺎ ﺃﻥ ‪ g n‬ﺗﺰﺍﻳﺪﻳﺔ ﻗﻄﻌﺎ ﻓﺈﻥ ‪:‬‬
‫= ‪− x  e−x =  x−1 −1 x e−x‬‬
‫‪−‬‬
‫‪f '(x) =  x‬‬ ‫‪3‬‬ ‫‪3‬‬ ‫‪3‬‬
‫)‪f(x‬‬ ‫‪n‬‬ ‫‪n‬‬
‫‪ 3‬‬ ‫‪‬‬ ‫‪ 3‬‬ ‫‪‬‬ ‫‪3x‬‬
‫‪lim α n = 0‬‬ ‫‪ 1‬‬ ‫‪ 1‬‬
‫‪1− 3x‬‬ ‫∞‪n→+‬‬ ‫ﺏ( ﻟﺪﻳﻨﺎ ‪ lim   = lim   = 0 :‬ﻭﻣﻨﻪ ‪:‬‬
‫= )‪( ∀x > 0) f '(x‬‬ ‫)‪f(x‬‬ ‫‪n→+∞  n ‬‬ ‫‪n→+∞ ‬‬‫‪ n ‬‬
‫‪3x‬‬ ‫ﺇﺫﻥ ‪:‬‬
‫‪:‬‬ ‫א‬ ‫א‬
‫ﺏ( ﺣﺴﺐ ﻣﺎ ﺳﺒﻖ ﻓﺈﻥ ﺇﺷﺎﺭﺓ ' ‪ f‬ﻫﻲ ﺇﺷﺎﺭﺓ ‪ x 6 −3x +1 :‬ﻭﻣﻨﻪ‬
‫) ‪ (1 (I‬ﻟﺪﻳﻨﺎ ‪= lim+ 3 2 e−x = +∞ :‬‬
‫)‪f(x‬‬ ‫‪1‬‬
‫ﺟﺪﻭﻝ ﺍﻟﺘﻐﻴﺮﺍﺕ ﻟﻠﺪﺍﻟﺔ ‪: f‬‬ ‫‪lim+‬‬
‫‪x→0‬‬ ‫‪x‬‬ ‫‪x→0‬‬ ‫‪x‬‬
‫ﻭﻣﻨﻪ ﺍﻟﺪﺍﻟﺔ ‪ f‬ﻻ ﺗﻘﺒﻞ ﺍﻹﺷﺘﻘﺎﻕ ﻓﻲ ‪ 0‬ﻋﻠﻰ ﺍﻟﻴﻤﻴﻦ ﻭ ﻣﻨﺤﻨﺎﻫﺎ ﻳﻘﺒﻞ‬

‫ﻧﺼﻒ ﻣﻤﺎﺱ ﻣﻮﺍﺯﻱ ﻟﻤﺤﻮﺭ ﺍﻷﺭﺍﺗﻴﺐ ﻓﻲ ﺍﻟﻨﻘﻄﺔ ‪ O‬ﺃﺻﻞ ﺍﻟﻤﻌﻠﻢ ‪.‬‬

‫‪x‬‬ ‫‪1‬‬
‫‪lim f(x) = lim‬‬ ‫‪=0‬‬
‫ﻷﻥ ‪:‬‬ ‫∞‪x→+‬‬ ‫‪x→+∞ ex‬‬ ‫‪3‬‬
‫‪x‬‬ ‫‪2‬‬ ‫‪(2‬‬

‫‪x‬‬ ‫‪1‬‬
‫‪ (4‬ﻣﻨﺤﻨﻰ ﺍﻟﺪﺍﻟﺔ ‪: f‬‬ ‫‪lim‬‬ ‫‪x‬‬
‫‪= lim‬‬ ‫‪=0‬‬
‫‪x→+∞ e‬‬ ‫‪x→+∞ 3 2‬‬
‫‪x‬‬

‫)‪ (C‬ﻳﻘﺒﻞ ﻣﺤﻮﺭ ﺍﻷﻓﺎﺻﻴﻞ ﻣﻘﺎﺭﺑﺎ ﻟﻪ ﺑﺠﻮﺍﺭ ∞‪. +‬‬ ‫ﻭﻣﻨﻪ‬

‫‪ (3‬ﺃ( ﻟﺪﻳﻨﺎ ﻟﻜﻞ ‪: x > 0‬‬

‫‪8‬‬
1 
2 I =  ,1 (II )
( ∀x ∈ I ) |f '(x)|≤  3 
3 : ‫ﻣﻠﺨﺺ‬
‫ ﻣﺘﺼﻠﺔ ﻭﺗﻨﺎﻗﺼﻴﺔ ﻗﻄﻌﺎ‬f ‫( ﺃ( ﺣﺴﺐ ﺍﻟﺪﺭﺍﺳﺔ ﺍﻟﻤﻨﺠﺰﺓ ﺃﻋﻼﻩ ﻓﺈﻥ‬1
  1 
f(I) = f(1),f   : ‫ ﻭﻣﻨﻪ‬I ‫ﻋﻠﻰ‬

‫ﺝ( ﻟﺪﻳﻨﺎ‬   3 

1 1 1
: ‫ ﺇﺫﻥ‬f(1) = > ‫ ﻭ‬f   < 0,5 < 1 : ‫ﻟﺪﻳﻨﺎ‬
x > 0 e 3  3 
 x > 0 

 x > 0 
 ⇔  1 ⇔1 1 1
< f(1) < f   < 1

 f(x) = x 
 −x  ln(x) − x = ln(x)  3 
x e = x  3
3
3


x > 0 x > 0
 f(I) ⊂ I
⇔ ⇔ ⇔ x = α3 : ‫ﻭﻣﻨﻪ‬

ln(x) − 3x = 3ln(x)  g
 3
 (x) = 0
1− 3x
v(x) = ‫∀ ( ﺑﺤﻴﺚ‬x ∈ I ) f '(x) = v(x)f(x) : ‫ﺏ( ﻟﺪﻳﻨﺎ‬
3x
1
. ‫ ﻭﺍﺿﺢ‬: u 0 = ∈ I : ‫ ( ﺃ( ﺑﺎﻟﺘﺮﺟﻊ‬2 : ‫ ﺍﻟﻤﺤﺪﺩﺓ‬: ‫ )ﻷﻥ‬I ‫ﻭ ﻫﻲ ﺩﺍﻟﺔ ﻣﺘﺨﺎﻃﺔ ﺗﻨﺎﻗﺼﻴﺔ ﻗﻄﻌﺎ ﻋﻠﻰ‬
3
  1   2  −3 1
( ∀n ∈ ` ) u n ∈ I ⇒ u n+1 = f(u n ) ∈ f(I) ⊂ I ⇒ u n+1 ∈ I v(I) =  v(1), v   = − ,0 : ‫( ﻭﻣﻨﻪ‬ = −3 < 0
  3   3  3 0

( ∀n ∈ ` ) u n ∈ I 2
: ‫ﻭﻣﻨﻪ‬ : ‫∀ ( ﻭﺑﻤﺎ ﺃﻥ‬x ∈ I ) |v(x)|≤ : ‫ﺑﺎﻟﺨﺼﻮﺹ‬
3
(‫( ﺃ‬3 ‫ ﻭﺣﺴﺐ ﺍﻟﺴﺆﺍﻝ‬. f(α 3 ) = α 3 : ‫( ﺝ( ﻓﺈﻥ‬1 ‫ﺏ( ﺣﺴﺐ ﺍﻟﺴﺆﺍﻝ‬
( ∀x ∈ I ) |f(x)|≤ 1
1 1 2
. α 3 ∈ I : ‫ﻭﻣﻨﻪ‬ < α3 < : ‫ﻓﺈﻥ‬ ( ∀x ∈ I ) |f '(x)|=|v(x)||f(x)|≤ : ‫ﻓﺈﻥ‬
3 3 3
|u n+1 −α 3 |=|f(u n ) − f(α 3 )| : n ∈ ` ‫ﻟﻜﻞ‬

9
‫ﺣﺴﺐ ﻧﺘﻴﺠﺔ ﺍﻟﺴﺆﺍﻝ ﺏ( ﺃﻋﻼﻩ ‪:‬‬ ‫ﺑﻤﺎ ﺃﻥ ‪ f‬ﻗﺎﺑﻠﺔ ﻟﻺﺷﺘﻘﺎﻕ ﻋﻠﻰ ‪ I‬ﻭ ﺑﻤﺎ ﺃﻥ ﻛﻼ ﻣﻦ ‪ u n‬ﻭ ‪ α 3‬ﻋﻨﺼﺮﺍﻥ‬
‫‪n +1‬‬ ‫‪n +2‬‬
‫‪2‬‬ ‫‪2 2‬‬ ‫‪2‬‬ ‫ﻣﻦ ‪ I‬ﻓﺈﻧﻪ ﺣﺴﺐ ﻣﺒﺮﻫﻨﺔ ﺍﻟﺘﺰﺍﻳﺪﺍﺕ ﺍﻟﻤﻨﺘﻬﻴﺔ ﻳﻮ ﺟﺪ ﻋﻨﺼﺮ ‪c n‬‬
‫‪|u n+1 −α 3 |≤ |u n −α 3 |≤  ‬‬ ‫‪=  ‬‬
‫‪3‬‬ ‫‪3  3 ‬‬ ‫‪ 3 ‬‬ ‫ﻣﺎ ﺑﻴﻦ ‪ u n‬ﻭ ‪ α 3‬ﺑﺤﻴﺚ ‪:‬‬

‫‪2‬‬
‫‪n +1‬‬ ‫) ‪f(u n ) − f(α 3 ) = f '(c n )(u n −α n‬‬
‫‪( ∀n ∈ ` ) |u n −α 3 |≤  ‬‬
‫‪3‬‬ ‫ﻭﻣﻨﻪ ‪:‬‬ ‫ﻭﻣﻨﻪ ‪:‬‬
‫‪2‬‬
‫‪2‬‬ ‫‪ 2 ‬‬
‫‪n +1‬‬ ‫| ‪|u n+1 −α 3 |=|f '(c n )||u n −α n |≤ |u n −α n‬‬
‫ﻭﺣﺴﺐ‬ ‫ﺗﺆﻭﻝ ﺇﻟﻰ ‪ 0‬ﻷﻥ ‪< 1 :‬‬ ‫‪ ‬‬ ‫ﺩ( ﺍﻟﻤﺘﺘﺎﻟﻴﺔ ﺍﻟﻬﻨﺪﺳﻴﺔ‬ ‫‪3‬‬
‫‪3‬‬ ‫‪ 3 ‬‬ ‫ﻷﻥ ‪c n ∈ I :‬‬
‫ﺍﻟﻤﺘﻔﺎﻭﺗﺔ ﺃﻋﻼﻩ ﻓﺈﻥ‪:‬‬
‫‪2‬‬
‫| ‪( ∀n ∈ `)|u n+1 −α 3 |≤ |u n −α n‬‬
‫ﺍﻟﻤﺘﺘﺎﻟﻴﺔ ‪ (u n )n≥0‬ﻣﺘﻘﺎﺭﺑﺔ ﻭ ‪lim u n = α 3‬‬ ‫‪3‬‬ ‫ﺇﺫﻥ ‪:‬‬
‫∞‪n→+‬‬

‫ﺝ( ﺑﺎﻟﺘﺮﺟﻊ ‪:‬‬


‫‪8x‬‬ ‫ﻷﺟﻞ ‪ n = 0 :‬ﻳﻜﻔﻲ ﺃﻥ ﻧﺒﻴﻦ ﺃﻥ ‪:‬‬
‫)[∞‪( ∀x ∈ [0, +‬‬ ‫∫ = )‪F(x‬‬ ‫‪f(t)dt‬‬ ‫) ‪(III‬‬
‫‪x‬‬ ‫‪1 2‬‬
‫‪ (1‬ﺃ( ﺍﻟﺪﺍﻟﺔ ‪ f‬ﻣﺘﺼﻠﺔ ﻋﻠﻰ [∞‪ [0, +‬ﺇﺫﻥ ﺗﻘﺒﻞ ﺩﺍﻟﺔ ﺃﺻﻠﻴﺔ ‪ G‬ﻋﻠﻰ‬ ‫‪α3 −‬‬ ‫≤‬
‫‪3 3‬‬
‫[∞‪ [0, +‬ﻭﻣﻨﻪ ‪:‬‬ ‫‪1‬‬ ‫‪1‬‬
‫ﻭﻣﻨﻪ ‪:‬‬ ‫< ‪< α3‬‬ ‫ﻧﻌﻠﻢ ﺃﻥ ‪:‬‬
‫‪3‬‬ ‫‪3‬‬
‫)‪( ∀x ∈ [0, +∞[) F(x) = G(8x) − G(x‬‬
‫ﺇﺫﻥ ‪ F‬ﻛﻔﺮﻕ ﻣﺮﻛﺒﺎﺕ ﺩﻭﺍﻝ ﻗﺎﺑﻠﺔ ﻟﻺﺷﺘﻘﺎﻕ ﺗﻘﺒﻞ ﺍﻹﺷﺘﻘﺎﻕ ﻋﻠﻰ‬ ‫‪1‬‬ ‫‪1‬‬ ‫‪1‬‬ ‫‪3 −1 2‬‬
‫≤ ‪ α 3 −‬ﻷﻥ ‪3 < 3 :‬‬ ‫= ‪−‬‬ ‫<‬
‫‪3‬‬ ‫‪3 3‬‬ ‫‪3‬‬ ‫‪3‬‬
‫[∞‪.[0, +‬‬ ‫‪n +1‬‬
‫‪ 2 ‬‬
‫ﺏ( ﻟﺪﻳﻨﺎ ﻟﻜﻞ ‪ x‬ﻣﻦ [∞‪: [0, +‬‬ ‫ﻧﻔﺘﺮﺽ ﺃﻥ ‪ |u n −α 3 |≤   :‬ﻷﺟﻞ ` ∈ ‪. n‬‬
‫‪ 3 ‬‬
‫)‪F '(x) = 8G'(8x) − G'(x) = 8f(8x) − f(x‬‬
‫‪10‬‬
‫ﻭﺑﻤﺎ ﺃﻥ ‪ f ≥ 0‬ﻋﻠﻰ ] ‪ [ x,8x‬ﻓﺈﻥ ‪ F(x) ≥ 0 :‬ﻭﻣﻨﻪ ‪:‬‬ ‫ﺃﻱ ‪F '(x) = 16 3 x e−8x − 3 xe−x = 3 xe−x (16e−7x −1) :‬‬
‫‪+‬‬
‫‪( ∀x ∈ \ ) 0 ≤ F(x) ≤ 2f(x)(1− e‬‬ ‫‪−7x‬‬
‫)‬ ‫)‪( ∀x ∈ [0, +∞[) F '(x) = 3 x e−x (16e−7x −1‬‬
‫ﻭﻣﻨﻪ ‪:‬‬
‫‪−7x‬‬
‫‪lim (1− e‬‬ ‫ﻭ ﻟﺪﻳﻨﺎ ‪) = 1 :‬‬ ‫ﺏ( ﻧﻌﻠﻢ ﺃﻥ ‪lim f(x) = 0 :‬‬ ‫)‪4ln(2‬‬
‫∞‪x→+‬‬ ‫∞‪x→+‬‬
‫‪( ∀x ∈ [0, +∞[) F '(x) = 0 ⇔ x = 0‬‬ ‫= ‪ x‬ﺃﻭ‬
‫ﺇﺫﻥ ‪ lim (2f(x)(1− e−7x )) = 0 :‬ﻭﺣﺴﺐ ﺍﻟﻤﺘﻔﺎﻭﺗﺔ ﺍﻟﻤﺰﺩﻭﺟﺔ‬ ‫‪7‬‬
‫∞‪x→+‬‬
‫)‪4ln(2‬‬
‫‪lim F(x) = 0‬‬
‫< ‪( ∀x ∈ [0, +∞[) F '(x) > 0 ⇔ 16e−7x > 1 ⇔ x‬‬
‫‪7‬‬
‫∞‪x→+‬‬ ‫ﺃﻋﻼﻩ ﻓﺈﻥ ‪:‬‬ ‫ﻭﻣﻨﻪ ﺗﻐﻴﺮﺍﺕ ‪: F‬‬
‫ﺝ( ﺟﺪﻭﻝ ﺍﻟﺘﻐﻴﺮﺍﺕ ﻟﻠﺪﺍﻟﺔ ‪: F‬‬ ‫‪ 4ln 2 ‬‬
‫‪0,‬‬ ‫* ‪ F‬ﺗﺰﺍﻳﺪﻳﺔ ﻗﻄﻌﺎ ﻋﻠﻰ ‪ :‬‬
‫‪‬‬ ‫‪7 ‬‬
‫‪ 4ln 2‬‬ ‫‪‬‬
‫‪‬‬ ‫* ‪ F‬ﺗﻨﺎﻗﺼﻴﺔ ﻗﻄﻌﺎ ﻋﻠﻰ ‪, +∞  :‬‬
‫‪ 7‬‬ ‫‪‬‬
‫‪ 4ln 2 ‬‬
‫‪ F ‬ﻗﻴﻤﺔ ﻗﺼﻮﻯ ﻣﻄﻠﻘﺔ ﻟﻠﺪﺍﻟﺔ ‪. F‬‬
‫‪ 7 ‬‬
‫*‬

‫‪ ( 2‬ﺃ( ﻟﻴﻜﻦ ‪. x ∈ \ + :‬ﻟﺪﻳﻨﺎ ‪ x ≤ 8x :‬ﻭﻣﻨﻪ ‪:‬‬


‫)]‪( ∀t ∈ [x,8x‬‬ ‫‪3‬‬
‫‪t ≤ 3 8x = 2 3 x‬‬
‫‪8x‬‬ ‫‪8x‬‬
‫∫ = )‪F(x‬‬ ‫‪3‬‬
‫∫ ‪t e−t dt ≤ 2 3 x‬‬ ‫ﺇﺫﻥ ‪e−t dt :‬‬
‫‪x‬‬ ‫‪x‬‬
‫‪−t 8x‬‬
‫) ‪= 2 3 x −e  = 2 3 x (e−x − e−8x‬‬
‫‪x‬‬

‫) ‪= 2 3 x e−x (1− e−7x ) = 2f(x)(1− e−7x‬‬

‫‪11‬‬
‫اﳌﻤﻠﻜﺔ اﳌﻐﺮﺑﻴﺔ‬
‫وزارة اﻟﱰﺑﻴﺔ اﻟﻮﻃﻨﻴﺔ واﻟﺘﻌﻠﻴﻢ اﻟﻌﺎﱄ و ﺗﻜﻮﻳﻦ اﻷﻃﺮ و اﻟﺒﺤﺚ اﻟﻌﻠﻤﻲ‬
‫ﻗﻄﺎع اﻟﱰﺑﻴﺔ اﻟﻮﻃﻨﻴﺔ‬

‫اﻻﻣﺘﺤﺎن اﻟﻮﻃﲏ اﳌﻮﺣﺪ ﻟﻠﺒﻜﺎﻟﻮرﻳﺎ‬


‫اﻟﺪورة اﻟﻌﺎدﻳﺔ ‪2005‬‬
‫اﻟﺸﻌﺒﺔ‪ :‬اﻟﻌﻠﻮم‬
‫اﳌﻌﺎﻣﻞ‪10 :‬‬ ‫اﳌﺪة‪4:‬ﺳﺎﻋﺎت‬ ‫اﳌﺎدة‪:‬اﻟﺮﻳﺎﺿﻴﺎت‬
‫اﻟﺮﻳﺎﺿﻴﺔ)أ و ب(‬

‫اﻟﺘﻤﺮﻳﻦ اﻷول ‪:‬‬

‫‪:‬‬ ‫ﻧﻌﺘﱪ ﰲ ‪ \ 2‬ﻗﺎﻧﻮن اﻟﱰآﻴﺐ اﻟﺪاﺧﻠﻲ * اﳌﻌﺮف ﲟﺎ ﻳﻠﻲ‬


‫‪ ax + by ay + bx ‬‬
‫* ) ‪( a, b‬‬ ‫‪x, y =(‬‬ ‫‪),‬‬ ‫‪2‬‬
‫ﻣﻦ \ ‪ :‬‬ ‫) ‪( x, y‬‬ ‫و‬ ‫ﻟﻜﻞ ) ‪( a, b‬‬
‫‪ 2‬‬ ‫‪2 ‬‬

‫‪‬‬ ‫‪1‬‬ ‫‪1‬‬ ‫‪*‬‬


‫‪E =  m + , m‬‬ ‫ﻟﺘﻜﻦ اﺠﻤﻟﻤﻮﻋﺔ ∈‪ \ −/ m \  :‬‬
‫‪2‬‬
‫∈‬
‫‪‬‬ ‫‪m‬‬ ‫‪m‬‬ ‫‪‬‬
‫ﻗﺎﻧﻮن داﺧﻠﻲ ﰲ اﺠﻤﻟﻤﻮﻋﺔ ‪E‬‬ ‫‪ (1‬ﺑﲔ أن *‬
‫‪‬‬ ‫‪1‬‬ ‫‪1‬‬
‫)‪( ∀m ∈ \ ) ϕ (m‬‬
‫*‬
‫=‪ m‬‬
‫‪‬‬ ‫‪m‬‬
‫‪, m+‬‬ ‫اﳌﻌﺮف \ ﻣﻦ ﳓﻮ ‪ E‬ﲟﺎ ﻳﻠﻲ ‪ :−‬‬
‫‪m‬‬
‫*‬
‫‪ (2‬ﻟﻴﻜﻦ اﻟﺘﻄﺒﻴﻖ ‪ϕ‬‬

‫ﳓﻮ )*‪( E ,‬‬ ‫ﻣﻦ )×‪( \ ,‬‬


‫*‬
‫أ( ﺑﲔ أن ‪ ϕ‬ﺗﺸﺎآﻞ ﺗﻘﺎﺑﻠﻲ‬

‫‪‬‬ ‫‪1‬‬ ‫‪1‬‬


‫زﻣﺮة ﺗﺒﺎدﻟﻴﺔ ﳏﺪدا ﻋﻨﺼﺮهﺎ اﶈﺎﻳﺪوﳑﺎﺛﻞ آﻞ ﻋﻨﺼﺮ ‪ m + , m − ‬‬ ‫أن )*‪( E ,‬‬ ‫ب( اﺳﺘﻨﺘﺞ‬
‫‪‬‬ ‫‪m‬‬ ‫‪m‬‬
‫ﺣﻴﺚ ﻋﺪد ‪ m‬ﺣﻘﻴﻘﻲ ﻏﲑ ﻣﻨﻌﺪم ‪.‬‬

‫‪F = {( x, y ) ∈ \ 2 / x‬‬ ‫‪2‬‬ ‫و‬ ‫‪y 2 ≥= x 2 − 4‬‬ ‫}‬ ‫‪ (3‬ﻧﻌﺘﱪ اﺠﻤﻟﻤﻮﻋﺔ‬

‫‪‬‬ ‫‪1‬‬ ‫‪1‬‬ ‫‪‬‬


‫∈‪F =  m + , m −  \ 2 / m 0‬‬
‫أ>( ﺑﲔ أن ‪ :‬‬
‫‪‬‬ ‫‪m‬‬ ‫‪m‬‬ ‫‪‬‬
‫)*‪( E ,‬‬ ‫زﻣﺮة ﺟﺰﺋﻴﺔ ﻣﻦ‬ ‫أن )*‪( F ,‬‬ ‫ب( ﺑﲔ‬

‫اﻟﺘﻤﺮﻳﻦ اﻟﺜﺎﻧﻲ‪:‬‬

‫اﳉﺰء اﻷول ‪:‬‬


‫‪ p‬ﻋﺪد ﺻﺤﻴﺢ ﻃﺒﻴﻌﻲ أوﱄ أآﱪ أو ﻳﺴﺎوي ﻣﻦ ‪5‬‬

‫‪ (1‬ﺑﲔ ان ‪p 2 ≡ 1 [3] :‬‬


‫ﲝﻴﺚ‬ ‫‪ ،‬ﺑﲔ أﻧﻪ ﻳﻮﺟﺪ ﻋﺪد ﺻﺤﻴﺢ ﻃﺒﻴﻌﻲ ‪q‬‬ ‫‪ (2‬أ( ﺑﺎﺳﺘﻌﻤﺎل زوﺟﻴﺔ اﻟﻌﺪد ‪p‬‬

‫‪http://mathkas.ici.ma‬‬ ‫‪1‬‬
‫)‪p 2 − 1 = 4q ( q + 1‬‬

‫]‪p 2 ≡ 1 [8‬‬ ‫ب( اﺳﺘﻨﺘﺞ أن‪:‬‬

‫]‪p 2 ≡ 1 [24‬‬ ‫‪ (3‬ﺑﲔ أن ‪:‬‬

‫اﳉﺰء اﻟﺜﺎﻧﻲ ‪:‬‬


‫ﻟﻴﻜﻦ ‪ a‬ﻋﺪدا ﺻﺤﻴﺤﺎ ﻃﺒﻴﻌﻴﺎ أوﻟﻴﺎ ﻣﻊ اﻟﻌﺪد‪. 24‬‬
‫‪ (1‬ﺑﲔ أن ‪a 2 ≡ 1 [24] :‬‬

‫و‬ ‫ﻣﻦ }‪{1,..., 23‬‬ ‫ﺣﻴﺚ ‪ ak ∧ 24 = 1 :‬ﻟﻜﻞ ‪k‬‬ ‫‪ (2‬هﻞ ﺗﻮﺟﺪ أﻋﺪاد ﺻﺤﻴﺤﺔ ﻃﺒﻴﻌﻴﺔ ‪a23 ، ... ، a1‬‬

‫و ‪( 24‬‬ ‫‪ a12 + ... + a23‬؟ ) ‪ ak ∧ 24‬هﻮ اﻟﻘﺎﺳﻢ اﳌﺸﱰك اﻷآﱪ ﻟﻠﻌﺪدﻳﻦ ‪ak‬‬
‫‪2‬‬
‫=‬
‫‪23997‬‬

‫اﻟﺘﻤﺮﻳﻦ اﻟﺜﺎﻟﺚ‬

‫اﳉﺰء اﻷول ‪:‬‬


‫ﲟﺎ ﻳﻠﻲ ‪:‬‬ ‫[∞‪[0, +‬‬ ‫اﳌﻌﺮﻓﺔ ﻋﻠﻰ اﺠﻤﻟﺎل‬ ‫ﻧﻌﺘﱪ اﻟﺪاﻟﺔ اﻟﻌﺪدﻳﺔ ‪f‬‬

‫‪‬‬ ‫‪−‬‬
‫‪2‬‬
‫‪ f ( x) = ( x + 2 ) e x ; x > 0‬‬
‫‪‬‬
‫‪ f (0) = 0‬‬
‫‪G G‬‬
‫‪( 2cm‬‬ ‫)اﻟﻮﺣﺪة‬ ‫) ‪( O, i , j‬‬ ‫ﻟﻴﻜﻦ ‪ C f‬ﻣﻨﺤﻨﺎهﺎ ﰲ ﻣﻌﻠﻢ ﻣﺘﻌﺎﻣﺪ ﳑﻨﻈﻢ‬

‫‪ (1‬أ( ﺑﲔ أن ‪ f‬ﻣﺘﺼﻠﺔ ﻋﻠﻰ اﻟﻴﻤﲔ ﰲ ‪0‬‬


‫ب( ﺑﲔ أن ‪ f‬ﻗﺎﺑﻠﺔ ﻟﻺﺷﺘﻘﺎق ﻋﻠﻰ اﻟﻴﻤﲔ ﰲ ‪0‬‬

‫[∞‪[0, +‬‬ ‫‪ f‬ﺗﺰاﻳﺪﻳﺔ ﻗﻄﻌﺎ ﻋﻠﻰ‬ ‫ج( ﺑﲔ أن‬

‫)‪lim f ( x‬‬ ‫‪ (2‬أ( اﺣﺴﺐ‬


‫∞‪x →+‬‬

‫‪t2‬‬
‫) ‪( ∀t ≥ 0‬‬ ‫‪0 ≤ e−t t 1‬‬ ‫ب( ‪−‬ﺑﲔ أن ‪:+‬‬ ‫≤‬
‫‪2‬‬
‫‪4‬‬ ‫‪4‬‬ ‫‪2‬‬
‫) ‪( ∀x > 0‬‬ ‫‪−‬‬ ‫≤‪f ( x) x‬‬ ‫‪ −‬ج( ﺑﲔ≤ ان ‪−:‬‬
‫‪x‬‬ ‫‪x2‬‬ ‫‪x‬‬
‫ﻳﻨﺒﻐﻲ ﲢﺪﻳﺪ ﻣﻌﺎدﻟﺔ ﻟﻪ‪.‬‬ ‫ﻣﺎﺋﻼ ) ∆ (‬ ‫د( اﺳﺘﻨﺘﺞ أن اﳌﻨﺤﲎ ‪ C f‬ﻳﻘﺒﻞ ﻣﻘﺎرﺑﺎ‬

‫اﳌﺴﺘﻘﻴﻢ ) ∆ (‬ ‫‪ (3‬أﻧﺸﺊ اﳌﻨﺤﲎ ‪ C f‬و‬


‫اﳉﺰء اﻟﺜﺎﻧﻲ‬
‫‪ n‬ﻋﺪد ﺻﺤﻴﺢ ﻃﺒﻴﻌﻲ ﻏﲑ ﻣﻨﻌﺪم ‪.‬‬
‫ﲟﺎ ﻳﻠﻲ ‪:‬‬ ‫اﺠﻤﻟﺎل [∞‪[0, +‬‬ ‫ﻧﻌﺘﱪ اﻟﺪاﻟﺔ اﻟﻌﺪدﻳﺔ ‪ f n‬اﳌﻌﺮﻓﺔ ﻋﻠﻰ‬

‫‪http://mathkas.ici.ma‬‬ ‫‪2‬‬
‫‪‬‬ ‫‪‬‬ ‫‪2  − 2x‬‬
‫‪ f n ( x) =  x +  e‬‬ ‫‪; x>0‬‬
‫‪‬‬ ‫‪‬‬ ‫‪n‬‬
‫‪ f (0) = 0‬‬
‫‪ n‬‬
‫‪ (1‬ﺑﲔ أن ‪ f n‬ﻗﺎﺑﻠﺔ ﻟﻺﺷﺘﻘﺎق ﻋﻠﻰ اﻟﻴﻤﲔ ﰲ ‪.0‬‬

‫[∞‪[0, +‬‬ ‫‪ (2‬ادرس ﺗﻐﲑات اﻟﺪاﻟﺔ ‪ f n‬ﻋﻠﻰ اﺠﻤﻟﺎل‬

‫‪2‬‬
‫[∞ ‪]0. +‬‬ ‫‪ an‬ﰲ اﺠﻤﻟﺎل‬ ‫ﺗﻘﺒﻞ ﺣﻼ وﺣﻴﺪا‬ ‫= )‪f n ( x‬‬ ‫‪ (3‬ا( ﺑﲔ أن ‪،‬ﻟﻜﻞ ‪ n‬ﻣﻦ *` ‪ ،‬اﳌﻌﺎدﻟﺔ‬
‫‪n‬‬
‫‪2‬‬ ‫‪2‬‬
‫‪( ∀x > 0 ) ( ∀n‬‬ ‫∈ ‪`* ) f‬‬
‫) ‪n +1 ( x‬‬ ‫)‪f n (−x‬‬ ‫‪−‬ب( ﺑﲔ أن> ‪:‬‬
‫‪n +1‬‬ ‫‪n‬‬
‫ج( اﺳﺘﻨﺘﺞ أن اﳌﺘﺘﺎﻟﻴﺔ ) ‪ (an‬ﺗﻨﺎﻗﺼﻴﺔ ﰒ ﺑﲔ أن ) ‪ (an‬ﻣﺘﻘﺎرﺑﺔ ‪.‬‬

‫‪lim a n = a :‬‬ ‫ﻧﻀﻊ‬


‫∞‪n→+‬‬

‫‪2‬‬

‫) ` ∈ ‪( ∀n‬‬ ‫*‬
‫‪nan = 2e‬‬ ‫‪an‬‬
‫‪2‬‬ ‫‪−‬‬
‫د( ﺑﲔ أن ‪:‬‬

‫‪. a=0‬‬ ‫ه( ﺑﲔ أن‬


‫اﳉﺰء اﻟﺜﺎﻟﺚ‬
‫ﲟﺎ ﻳﻠﻲ ‪:‬‬ ‫[∞‪[0, +‬‬ ‫ﻧﻌﺘﱪ اﻟﺪاﻟﺔ اﻟﻌﺪدﻳﺔ ‪ F‬اﳌﻌﺮﻓﺔ ﻋﻠﻰ اﺠﻤﻟﺎل‬
‫‪2x‬‬
‫∫ = )‪F ( x‬‬ ‫‪f (t ) dt‬‬
‫‪x‬‬

‫) ‪ f‬هﻲ اﻟﺪاﻟﺔ اﳌﻌﺮﻓﺔ ﰲ اﳉﺰء اﻷول(‬

‫) ‪( ∀x > 0‬‬ ‫)‪xf ( x‬‬ ‫)‪F≤( x‬‬ ‫)‪≤ x‬‬


‫‪xf (2‬‬ ‫‪(1‬أ( ﺑﲔ أن ‪:‬‬

‫)‪lim F ( x‬‬ ‫ب( اﺣﺴﺐ‬


‫∞‪x →+‬‬

‫[∞‪[0, +‬‬ ‫‪ (2‬أ( ﺑﲔ أن ‪ F‬ﻗﺎﺑﻠﺔ ﻟﻺﺷﺘﻘﺎق ﻋﻠﻰ اﺠﻤﻟﺎل‬

‫‪‬‬ ‫‪− ‬‬


‫‪2‬‬
‫‪ 1x ‬‬ ‫‪1‬‬
‫‪‬‬
‫‪‬‬ ‫‪F‬‬ ‫('‬ ‫‪x‬‬ ‫)‬ ‫=‬ ‫‪e‬‬ ‫‪x‬‬
‫( ‪‬‬ ‫‪x‬‬ ‫‪+‬‬ ‫‪2‬‬ ‫‪)  e − 1‬‬ ‫(‬ ‫‪3‬‬ ‫‪x‬‬ ‫‪2‬‬ ‫‪) x  +; x‬‬
‫‪e‬‬ ‫‪0‬‬ ‫‪+‬‬ ‫>‬
‫‪‬‬ ‫‪‬‬ ‫‪‬‬ ‫‪‬‬ ‫‪‬‬ ‫ب( ﺑﲔ أن ‪:‬‬
‫‪‬‬
‫‪ Fd ' (0) = 0‬‬
‫ﻋﻠﻰ اﻟﻴﻤﲔ(‬ ‫) )‪ Fd '(0‬هﻮ اﻟﻌﺪد اﳌﺸﺘﻖ ﻟﻠﺪاﻟﺔ ‪ F‬ﰲ‪0‬‬

‫‪ (3‬أﻋﻂ ﺟﺪول اﻟﺘﻐﲑات اﻟﺪاﻟﺔ ‪F‬‬

‫اﻟﺘﻤﺮﻳﻦ اﻟﺮاﺑﻊ‬
‫‪iz − 1‬‬
‫= )‪f ( z‬‬ ‫ﻟﻜﻞ ﻋﺪد ﻋﻘﺪي ‪ z‬ﳐﺎﻟﻒ ﻟﻠﻌﺪد ‪ ، −1‬ﻧﺼﻊ ‪:‬‬
‫)‪( z + 1‬‬
‫‪2‬‬

‫‪ (1‬أ( ﺣﺪد اﻟﻌﺪد اﳊﻘﻴﻘﻲ ‪ y‬ﲝﻴﺚ ‪f (iy ) = iy :‬‬

‫)‪(E‬‬ ‫‪f ( z) = z‬‬ ‫اﳌﻌﺎدﻟﺔ ‪:‬‬ ‫ب(ﺣﻞ ﰲ ^‬

‫‪3‬‬ ‫‪http://mathkas.ici.ma‬‬
‫‪ Re( z0 ) = 0‬و ) ‪Re( z1 ) > Re( z2‬‬ ‫ﺣﻴﺚ‬ ‫)‪(E‬‬ ‫ﳊﻠﻮل اﳌﻌﺎدﻟﺔ‬ ‫و ‪z2‬‬ ‫ﻧﺮﻣﺰ ب ‪ z0‬و ‪z1‬‬
‫‪7π‬‬ ‫‪11π‬‬
‫‪i‬‬ ‫‪i‬‬
‫‪z2 + 1 = e‬‬ ‫‪6‬‬
‫و‬ ‫‪z1 + 1 = e‬‬ ‫‪6‬‬
‫‪(2‬أ( ﲢﻘﻖ أن ‪:‬‬

‫و ‪z2‬‬ ‫ب( اﺳﺘﻨﺘﺞ اﻟﻜﺘﺎﺑﺔ اﳌﺜﻠﺜﻴﺔ ﻟﻜﻞ ﻣﻦ اﻟﻌﺪدﻳﻦ ‪z1‬‬

‫ﺣﻴﺚ ‪. 0 ≤ α < π‬‬ ‫‪z = ei α‬‬ ‫‪ (3‬ﰲ هﺬا اﻟﺴﺆال ﻧﻔﱰض أن‬

‫أ( ﺑﲔ أن ‪f ( z ) = izf ( z ) :‬‬

‫ب( ﺣﺪد ‪ α‬إذا ﻋﻠﻤﺖ أن ‪f ( z ) + f ( z ) = 0 :‬‬


‫‪iϕ‬‬
‫\ × ‪( r ,ϕ ) ∈ \*+‬‬ ‫‪ f ( z ) = re‬ﺣﻴﺚ ‪:‬‬ ‫ﻋﻠﻰ اﻟﺸﻜﻞ‬ ‫ج( اآﺘﺐ ) ‪f ( z‬‬

‫‪| z |= 1‬‬
‫‪‬‬
‫‪‬‬ ‫‪1‬‬ ‫‪ (4‬ﺣﺪد ‪ z‬إذا ﻋﻠﻤﺖ أن‬
‫‪Re( z ) = 2‬‬

‫اﻧﺘﻬﻰ‬

‫‪htt‬‬
‫‪p:/‬‬
‫‪/m‬‬
‫‪ath‬‬
‫‪kas‬‬
‫‪.ici‬‬
‫‪.m‬‬
‫‪a‬‬

‫‪4‬‬
‫اﻷﺳﺘﺎذ ‪ :‬اﻟﺤﻴﺎن‬ ‫ﺗﺼﺤﻴﺢ اﻹﻣﺘﺤﺎن اﻟﻮﻃﻨﻲ اﻟﻤﻮﺣﺪ‬ ‫اﻟﺜﺎﻧﻴﺔ ﺑﻜﺎﻟﻮرﻳﺎ ﻋﻠﻮم رﻳﺎﺿﻴﺔ‬
‫اﻟﺪورة اﻟﻌﺎدﻳﺔ ‪2005 / 06 / 10‬‬
‫⎞ ‪ax + by ay + bx‬‬
‫; ‪∀ ( a, b ) ∈ \ 2 ; ∀ ( x , y ) ∈ \ 2‬‬ ‫⎜⎛ = ) ‪( a, b ) * ( x , y‬‬ ‫‪,‬‬ ‫⎟‬ ‫اﻟﺘﻤﺮﻳﻦ ‪: 1‬‬
‫‪⎝ 2‬‬ ‫⎠ ‪2‬‬
‫⎛⎧‬ ‫‪1‬‬ ‫‪1‬‬ ‫⎞‬ ‫⎫*‬
‫‪E = ⎨⎜ m + , m −‬‬ ‫⎬ \∈ ‪⎟∈\ / m‬‬
‫‪2‬‬

‫⎝⎩‬ ‫‪m‬‬ ‫‪m‬‬ ‫⎠‬ ‫⎭‬


‫⎛‬ ‫‪1‬‬ ‫⎞‪1‬‬ ‫⎛‬ ‫‪1‬‬ ‫⎞‪1‬‬
‫‪ (1‬ﻟﻴﻜﻦ ⎟ ‪ X (m ) = ⎜ m + , m −‬و ⎟ ‪ X (n ) = ⎜ n + , n −‬ﻋﻨﺼﺮﻳﻦ ﻣﻦ اﻟﻤﺠﻤﻮﻋﺔ ‪ E‬؛ إذن *\ ∈ ‪ m‬و *\ ∈ ‪. n‬‬
‫⎝‬ ‫‪n‬‬ ‫⎠‪n‬‬ ‫⎝‬ ‫‪m‬‬ ‫⎠‪m‬‬
‫⎛‬ ‫‪1‬‬ ‫⎛ ⎞‪1‬‬ ‫‪1‬‬ ‫⎞‪1‬‬
‫⎟ ‪X (m ) * X (n ) = ⎜ m + , m − ⎟ * ⎜ n + , n −‬‬ ‫ﻟﺪﻳﻨﺎ ‪:‬‬
‫⎝‬ ‫‪m‬‬ ‫⎝ ⎠‪m‬‬ ‫‪n‬‬ ‫⎠‪n‬‬
‫⎛⎛‬ ‫⎛⎞ ‪1‬‬ ‫⎛ ⎞‪1‬‬ ‫⎛⎞ ‪1‬‬ ‫⎛ ⎞‪1‬‬ ‫⎛⎞ ‪1‬‬ ‫⎛ ⎞‪1‬‬ ‫⎛⎞ ‪1‬‬ ‫⎞⎞‪1‬‬
‫⎟ ⎟ ‪⎜ ⎜ m + m ⎟ ⎜ n + n ⎟ + ⎜ m − m ⎟ ⎜ n − n ⎟ ⎜ m + m ⎟⎜ n − n ⎟ + ⎜ m − m ⎟ ⎜ n + n‬‬
‫⎝⎜=‬ ‫⎝⎠‬ ‫⎝ ⎠‬ ‫⎝⎠‬ ‫⎝‪⎠,‬‬ ‫⎝⎠‬ ‫⎝ ⎠‬ ‫⎝⎠‬ ‫⎟⎠‬
‫⎜‬ ‫‪2‬‬ ‫‪2‬‬ ‫⎟‬
‫⎜‬ ‫⎟‬
‫⎝‬ ‫⎠‬
‫⎛‬ ‫‪m n‬‬ ‫‪1‬‬ ‫‪m n‬‬ ‫‪1‬‬ ‫‪m n‬‬ ‫‪1‬‬ ‫‪m n‬‬ ‫⎞ ‪1‬‬
‫⎟ ‪⎜ mn + n + m + mn + mn − n − m + mn mn − n + m − mn + mn + n − m − mn‬‬
‫⎜=‬ ‫‪,‬‬ ‫⎟‬
‫⎜‬ ‫‪2‬‬ ‫‪2‬‬ ‫⎟‬
‫⎝‬ ‫⎠‬
‫⎛‬ ‫‪1‬‬ ‫⎞ ‪1‬‬
‫‪= ⎜ mn +‬‬ ‫‪, mn −‬‬ ‫⎟‬
‫⎝‬ ‫‪mn‬‬ ‫⎠ ‪mn‬‬
‫) ‪X (m ) * X (n ) = X (mn‬‬
‫وﺑﻤﺎ أن *\ ∈ ‪ m‬و *\ ∈ ‪ , n‬ﻓﺈن *\ ∈ ‪ . mn‬وﻋﻠﻴﻪ ﻓﺈن ‪. X (m ) * X (n ) ∈ E :‬‬
‫وﺑﺎﻟﺘﺎﻟﻲ ﻓﺈن ‪ . ∀ ( X ,Y ) ∈ E 2 ; X *Y ∈ E :‬إذن * ﻗﺎﻧﻮن داﺧﻠﻲ ﻓﻲ اﻟﻤﺠﻤﻮﻋﺔ ‪. E‬‬
‫→ *\ ‪ϕ :‬‬ ‫‪E‬‬
‫‪.‬‬ ‫⎛‬ ‫‪1‬‬ ‫‪ (2‬ﻟﺪﻳﻨﺎ ‪1 ⎞ :‬‬
‫‪m‬‬ ‫⎟ ‪6 ϕ (m ) = ⎜ m + , m −‬‬
‫⎝‬ ‫‪m‬‬ ‫⎠‪m‬‬
‫أ(‬
‫⎛‬ ‫‪1‬‬ ‫⎛ ⎞ ‪1‬‬ ‫‪1‬‬ ‫‪1‬‬ ‫⎛ ⎞‬ ‫‪1‬‬ ‫⎞‪1‬‬
‫‪ . i‬ﻟﻴﻜﻦ \ ∈ ) ‪ , ( m , n‬ﻟﺪﻳﻨﺎ ‪⎟ * ⎜ n + , n − ⎟ = ϕ (m ) *ϕ (n ) :‬‬
‫‪*2‬‬
‫‪ϕ (mn ) = ⎜ mn +‬‬ ‫‪, mn −‬‬ ‫‪⎟ = ⎜m + ,m −‬‬
‫⎝‬ ‫‪mn‬‬ ‫⎝ ⎠ ‪mn‬‬ ‫‪m‬‬ ‫‪m‬‬ ‫⎝ ⎠‬ ‫‪n‬‬ ‫⎠‪n‬‬
‫) ‪∀ ( m , n ) ∈ \* : ϕ (mn ) = ϕ (m ) * ϕ (n‬‬ ‫)×‪ ( \* ,‬ﻧﺤﻮ )*‪( E ,‬‬
‫‪2‬‬
‫‪.‬‬ ‫إذن ‪ ϕ‬ﺗﺸﺎآﻞ ﻣﻦ‬
‫⎛‬ ‫‪1‬‬ ‫⎞‪1‬‬
‫‪ . ii‬ﻟﻴﻜﻦ ‪ , X ∈ E‬إذن‪ ∃!m ∈ \* / X = ⎜ m + , m − ⎟ :‬؛ وﻣﻨﻪ ) ‪ ∃!m ∈ \* / X = ϕ (m‬؛ وﻋﻠﻴﻪ ﻓﺈن ‪ ϕ‬ﺗﻘﺎﺑﻞ ﻣﻦ *\ ﻧﺤﻮ ‪E‬‬
‫⎝‬ ‫‪m‬‬ ‫⎠‪m‬‬
‫وﺑﺎﻟﺘﺎﻟﻲ ﻓﺈن ‪ ϕ‬ﺗﺸﺎآﻞ ﺗﻘﺎﺑﻠﻲ ﻣﻦ ) × ‪ ( \ ,‬ﻧﺤﻮ )*‪. ( E ,‬‬
‫*‬

‫ب( ﺑﻤﺎ أن ‪ ϕ‬ﺗﺸﺎآﻞ ﺗﻘﺎﺑﻠﻲ ﻣﻦ ) × ‪ ( \* ,‬ﻧﺤﻮ )*‪ ( E ,‬وأن ) × ‪ ( \* ,‬زﻣﺮة ﺗﺒﺎدﻟﻴﺔ ‪ ,‬ﻓﺈن )*‪ ( E ,‬زﻣﺮة ﺗﺒﺎدﻟﻴﺔ ‪.‬‬
‫ﻟﺪﻳﻨﺎ ‪ 1‬هﻮ اﻟﻌﻨﺼﺮ اﻟﻤﺤﺎﻳﺪ ﺑﺎﻟﻨﺴﺒﺔ ﻟﻠﻘﺎﻧﻮن × ﻓﻲ ) × ‪ , ( \* ,‬وﻣﻨﻪ ﻓﺈن اﻟﻌﻨﺼﺮ اﻟﻤﺤﺎﻳﺪ ﺑﺎﻟﻨﺴﺒﺔ ﻟﻠﻘﺎﻧﻮن * ﻓﻲ )*‪ ( E ,‬هﻮ ‪. ϕ (1) = (2, 0) :‬‬
‫⎛‬ ‫‪1‬‬ ‫⎞‪1‬‬
‫ﻟﻴﻜﻦ ⎟ ‪ ϕ (m ) = ⎜ m + , m −‬ﻋﻨﺼﺮا ﻣﻦ ‪ . E‬ﻣﻤﺎﺛﻞ ) ‪ ϕ (m‬ﺑﺎﻟﻨﺴﺒﺔ ﻟﻠﻘﺎﻧﻮن * ﻓﻲ )*‪ ( E ,‬هﻮ ‪:‬‬
‫⎝‬ ‫‪m‬‬ ‫⎠‪m‬‬
‫⎛‬ ‫⎞‬
‫‪1‬‬ ‫⎛‬ ‫‪1‬‬ ‫⎞‬ ‫⎜‬ ‫‪1‬‬ ‫‪1‬‬ ‫‪1‬‬ ‫‪1‬‬ ‫‪⎟ ⎛1‬‬ ‫‪1‬‬ ‫⎞‬
‫⎟ ‪( m ≠ 0 ⇒ ≠ 0 ) . (ϕ (m ) )′ = ϕ (m −1 ) = ϕ ⎜ ⎟ = ⎜ + , − ⎟ = ⎜ + m , − m‬‬
‫‪m‬‬ ‫‪⎝m ⎠ ⎜m 1 m 1 ⎟ ⎝m‬‬ ‫‪m‬‬ ‫⎠‬
‫⎝‬ ‫‪m‬‬ ‫⎠‪m‬‬
‫}‪ y 2 = x 2 − 4‬و ‪F = {(x , y ) ∈ \ 2 / x ≥ 2‬‬ ‫‪ (3‬ﻧﻌﺘﺒﺮ اﻟﻤﺠﻤﻮﻋﺔ ‪:‬‬

‫‪-1-‬‬
‫⎛⎧‬ ‫‪1‬‬ ‫⎞‪1‬‬ ‫⎫‬
‫‪ .‬ﻟﻨﺒﻴﻦ أن ‪ F = G :‬؟‬ ‫أ ( ﻧﻀﻊ ‪G = ⎨⎜ m + , m − ⎟ ∈ \ 2 / m > 0 ⎬ :‬‬
‫⎝⎩‬ ‫‪m‬‬ ‫⎠‪m‬‬ ‫⎭‬
‫⎛‬ ‫‪1‬‬ ‫⎞‪1‬‬
‫‪ : G ⊂ F‬ﻟﻴﻜﻦ ‪ , X = ⎜ m + m − ⎟ ∈G‬إذن ‪ . m > 0‬ﻟﺪﻳﻨﺎ ‪:‬‬
‫⎝‬ ‫‪m‬‬ ‫⎠‪m‬‬
‫‪2‬‬ ‫‪2‬‬
‫⎛‬ ‫⎞‪1‬‬ ‫‪1‬‬ ‫⎛ ‪1‬‬ ‫⎞‪1‬‬
‫⎟ ‪. ⎜m + ⎟ −4 = m2 + 2+ 2 −4 = m2 −2+ 2 = ⎜m −‬‬
‫⎝‬ ‫⎠‪m‬‬ ‫‪m‬‬ ‫‪m‬‬ ‫⎝‬ ‫⎠‪m‬‬
‫‪1‬‬ ‫‪1‬‬ ‫‪1‬‬ ‫‪2‬‬
‫‪m+ ≥2 m× ⇒ m+ ≥2‬‬ ‫‪ . ∀(a , b ) ∈ \ + : a + b ≥ 2 ab‬ﺗﻄﺒﻴﻖ ‪:‬‬ ‫ﺗﺬآﻴﺮ ‪:‬‬
‫‪m‬‬ ‫‪m‬‬ ‫‪m‬‬
‫وﺑﻨﺎءا ﻋﻠﻰ ﻣﺎ ﺳﺒﻖ ‪ ,‬ﻧﺠﺪ ‪ . X ∈ F :‬وﻣﻨﻪ ﻧﺴﺘﻨﺘﺞ أن ‪. G ⊂ F‬‬
‫⎛‬ ‫‪1‬‬ ‫⎞‪1‬‬
‫‪ : F ⊂ G‬ﻟﻴﻜﻦ ‪ , (x , y ) ∈ F‬إذن ‪ x ≥ 2 :‬و ‪ . y 2 = x 2 − 4‬ﻧﺒﺤﺚ ﻋﻦ ‪ m > 0‬ﺑﺤﻴﺚ ‪ (x , y ) = ⎜ m + , m − ⎟ :‬؟‬
‫⎝‬ ‫‪m‬‬ ‫⎠‪m‬‬
‫⎧‬ ‫‪1‬‬ ‫⎧‬ ‫‪x +y‬‬
‫‪m+ =x‬‬ ‫‪⎧ 2m = x + y‬‬ ‫⎪‬ ‫=‪m‬‬
‫⎛‬ ‫‪1‬‬ ‫⎞‪1‬‬ ‫⎪‬
‫⎪‬ ‫‪m‬‬ ‫⎪‬ ‫⎪‬ ‫‪2‬‬
‫⎨ ⇔ ⎟ ‪(x , y ) = ⎜ m + , m −‬‬ ‫‪⇔⎨2‬‬ ‫⎨⇔‬
‫⎝‬ ‫‪m‬‬ ‫⎠‪m‬‬ ‫‪⎪m − 1 = y‬‬ ‫‪⎪⎩ m = x − y‬‬ ‫‪⎪m = 2‬‬
‫⎪⎩‬ ‫‪m‬‬ ‫⎩⎪‬ ‫‪x −y‬‬
‫‪x +y‬‬ ‫‪2‬‬
‫⇒ ‪y 2 = x 2 − 4 ⇒ x 2 − y 2 = 4 ⇒ (x + y )(x − y ) = 4‬‬ ‫=‬ ‫وﻧﻌﻠﻢ أن ‪ ) :‬ﻷن ‪( x ≠ y‬‬
‫‪2‬‬ ‫‪x −y‬‬
‫⎛‬ ‫‪1‬‬ ‫⎞‪1‬‬ ‫‪x +y‬‬
‫= ‪ , m‬ﻟﺪﻳﻨﺎ ‪ . (x , y ) = ⎜ m + , m − ⎟ :‬ﻟﻨﺒﻴﻦ أن ‪ m > 0 :‬؟‬ ‫ﻣﻦ أﺟﻞ‬
‫⎝‬ ‫‪m‬‬ ‫⎠‪m‬‬ ‫‪2‬‬
‫ﻟﺪﻳﻨﺎ ‪ , (x + y )(x − y ) = 4 > 0‬إذن ‪ x + y‬و ‪ x − y‬ﻟﻬﻤﺎ ﻧﻔﺲ اﻹﺷﺎرة ‪ .‬وﻟﺪﻳﻨﺎ ‪ . x ≥ 2 > 0‬إذن ‪:‬‬
‫‪x +y‬‬
‫=‪.m‬‬ ‫‪ y ≥ 0 ⇒ x + y ≥ x > 0‬و ‪ . y ≤ 0 ⇒ − y ≥ 0 ⇒ x − y ≥ x > 0 ⇒ x + y > 0‬وﻣﻨﻪ ﻓﺈن ‪> 0 :‬‬
‫‪2‬‬
‫إذن ‪ . (x , y ) ∈G‬وﻣﻨﻪ ﻧﺠﺪ ‪. F ⊂ G :‬‬
‫وﺑﺎﻟﺘﺎﻟﻲ ﻓﺈن ‪. F = G :‬‬
‫⎛⎧‬ ‫‪1‬‬ ‫⎞‪1‬‬ ‫⎫‬
‫ب( ﻟﺪﻳﻨﺎ ‪. F = ⎨⎜ m + , m − ⎟ ∈ \ 2 / m > 0 ⎬ ⊂ E . i :‬‬
‫⎝⎩‬ ‫‪m‬‬ ‫⎠‪m‬‬ ‫⎭‬
‫‪ F ≠ ∅ . ii‬ﻷن ﻣﻦ أﺟﻞ ‪ , m = 1‬ﻟﺪﻳﻨﺎ ‪. (2, 0) ∈ F :‬‬
‫⎛‬ ‫‪1‬‬ ‫⎞‪1‬‬ ‫⎛‬ ‫‪1‬‬ ‫⎞‪1‬‬
‫‪ . iii‬ﻟﻴﻜﻦ ⎟ ‪ X = ⎜ m + , m −‬و ⎟ ‪ Y = ⎜ n + , n −‬ﻋﻨﺼﺮﻳﻦ ﻣﻦ اﻟﻤﺠﻤﻮﻋﺔ ‪. F‬إذن ‪:‬‬
‫⎝‬ ‫‪n‬‬ ‫⎠‪n‬‬ ‫⎝‬ ‫‪m‬‬ ‫⎠‪m‬‬
‫‪⎧m > 0 m‬‬ ‫⎞‪⎛1‬‬ ‫⎞ ‪⎛m ⎞ ⎛m n m n‬‬
‫⎨‪.‬‬ ‫⎟ ‪ X *Y ′ = ϕ (m ) *ϕ (n )′ = ϕ (m ) *ϕ ⎜ ⎟ = ϕ ⎜ ⎟ = ⎜ + , −‬و ‪⇒ > 0‬‬
‫‪⎩n > 0‬‬ ‫‪n‬‬ ‫⎠ ‪⎝n‬‬ ‫⎠‪⎝n ⎠ ⎝n m n m‬‬
‫‪∀ ( X ,Y ) ∈ F 2 : X *Y ′ ∈ F‬‬ ‫وﻣﻨﻪ ﻓﺈن ‪. X *Y ′ ∈ F :‬‬
‫‪.‬‬ ‫)*‪( E ,‬‬ ‫وﺑﺎﻟﺘﺎﻟﻲ ﻓﺈن ‪ ( F ,*) :‬زﻣﺮة ﺟﺰﺋﻴﺔ ﻣﻦ‬
‫اﻟﺘﻤﺮﻳﻦ ‪: 2‬‬
‫‪ p I‬ﻋﺪد ﺻﺤﻴﺢ ﻃﺒﻴﻌﻲ أوﻟﻲ و ‪. p ≥ 5‬‬
‫‪ (1‬ﻟﺪﻳﻨﺎ ‪ p‬أوﻟﻲ و ‪ p ≥ 5‬و ‪ 3‬أوﻟﻲ إذن ‪ 3 ) 3 ∧ p = 1‬و ‪ p‬أوﻟﻴﺎن ﻓﻴﻤﺎ ﺑﻴﻨﻬﻤﺎ ( وﻣﻨﻪ ‪ 3 ∧ p 2 = 1‬وﻋﻠﻴﻪ ﻓﺈن ‪. p ≡ 0 [3] :‬‬
‫]‪ . p ≡ 2 [3‬ﻧﺪرس هﺎﺗﻴﻦ اﻟﺤﺎﻟﺘﻴﻦ ‪:‬‬ ‫أو‬ ‫]‪p ≡ 1[3‬‬ ‫وﻣﻨﻪ ﻧﺴﺘﻨﺘﺞ أن ‪:‬‬
‫‪ . ii‬إذا آﺎن ]‪ , p ≡ 2 [3‬ﻓﺈن ‪ p 2 ≡ 4 [3] :‬إذن ]‪. p 2 ≡ 1[3‬‬ ‫‪ . i‬إذا آﺎن ]‪ , p ≡ 1[3‬ﻓﺈن ‪. p 2 ≡ 1[3] :‬‬
‫‪.‬‬ ‫]‪p 2 ≡ 1[3‬‬ ‫وﺑﺎﻟﺘﺎﻟﻲ ﻓﺈن ‪:‬‬
‫‪ ∃q ∈ ] /‬وﻣﻨﻪ‬ ‫‪ (2‬أ( ﻟﺪﻳﻨﺎ ‪ p‬ﻋﺪد ﺻﺤﻴﺢ ﻃﺒﻴﻌﻲ أوﻟﻲ و ‪ , p ≥ 5‬إذن ‪ p‬ﻋﺪد ﻓﺮدي وﻣﻨﻪ ‪p = 2q + 1‬‬
‫‪ . p 2 = (2q + 1) 2 = 4q 2 + 4q + 1 = 4q (q + 1) + 1‬إذن ‪. p 2 − 1 = 4q (q + 1) :‬‬
‫ب( ﻟﺪﻳﻨﺎ ‪ q‬و ‪ q + 1‬ﻋﺪدان ﺻﺤﻴﺤﺎن ﻃﺒﻴﻌﻴﺎن ﻣﺘﺘﺎﺑﻌﺎن ‪ ,‬إذن )‪ q (q + 1‬ﻋﺪد ﺻﺤﻴﺢ ﻃﺒﻴﻌﻲ زوﺟﻲ ‪.‬وﻣﻨﻪ‬

‫‪-2-‬‬
‫‪.‬‬ ‫]‪p 2 ≡ 1[8‬‬ ‫وﻣﻨﻪ ﻧﺠﺪ ‪:‬‬ ‫‪ . ∃k ∈ ` / q (q + 1) = 2k‬إذن ‪. p 2 − 1 = 8k :‬‬
‫‪⎧a / c‬‬
‫⎪‬
‫‪⎨b / c‬‬ ‫‪ (3‬ﺧﺎﺻﻴﺔ ‪ a :‬و ‪ b‬و ‪ c‬أﻋﺪاد ﻧﺴﺒﻴﺔ ‪ .‬ﻟﺪﻳﻨﺎ ‪⇒ ab / c :‬‬
‫‪⎪a ∧ b = 1‬‬
‫⎩‬
‫‪Bezout‬‬
‫ﺑﺮهﺎن ‪ :‬ﻟﺪﻳﻨﺎ ‪ a / c ⇒ ∃m ∈ ] / c = am‬و ‪ b / c ⇒ ∃n ∈ ] / c = bn‬و ‪. a ∧ b = 1 ⇒ ∃(u ,v ) ∈ ] 2 / au + bv = 1‬‬
‫إذن ‪ c = cau + cbv = bnau + ambv = ab (nu + mv ) :‬وﻣﻨﻪ ﻧﺴﺘﻨﺘﺞ أن ‪. ab / c :‬‬
‫ﻣﻼﺣﻈﺔ ‪ :‬اﻟﺨﺎﺻﻴﺔ ﻟﻴﺴﺖ ﺻﺤﻴﺤﺔ إذا آﺎن ‪ . a ∧ b ≠ 1‬ﻣﺜﺎل ﻣﻀﺎد ‪ 2 :‬ﻳﻘﺴﻢ ‪ 12‬و ‪ 4‬ﻳﻘﺴﻢ ‪ 12‬ﻟﻜﻦ ‪ 8 = 2 × 4‬ﻻ ﻳﻘﺴﻢ ‪ 12‬؛ ‪. 2 ∧ 4 = 2‬‬
‫ﺗﻄﺒﻴﻖ ‪ :‬ﻟﺪﻳﻨﺎ ]‪ p 2 ≡ 1[3‬و ]‪ p 2 ≡ 1[8‬إذن )‪ 3 / ( p 2 − 1‬و )‪ 8 / ( p 2 − 1‬و ‪ 3 ∧ 8 = 1‬إذن )‪ 24 = 3 × 8 / ( p 2 − 1‬أي ]‪p 2 ≡ 1[ 24‬‬
‫‪ II‬ﻟﻴﻜﻦ ‪ a‬ﻋﺪدا ﺻﺤﻴﺤﺎ ﻃﺒﻴﻌﻴﺎ ﺣﻴﺚ ‪. a ∧ 24 = 1 :‬‬
‫‪ (1‬أ( ﻟﺪﻳﻨﺎ ‪ , a ∧ 24 = 1‬إذن ‪ a‬ﻋﺪد ﺻﺤﻴﺢ ﻃﺒﻴﻌﻲ ﻓﺮدي) ﻟﻮ آﺎن ‪ a‬زوﺟﻴﺎ ﻟﻜﺎن ‪ 2‬ﻗﺎﺳﻤﺎ ﻣﺸﺘﺮآﺎ ل ‪ a‬و ‪ 24‬وهﺬا ﻳﺘﻨﺎﻗﺾ ﻣﻊ ‪( a ∧ 24 = 1‬‬
‫وﻣﻨﻪ ﻧﺴﺘﻨﺘﺞ أن ‪ ) a 2 ≡ 1[8] :‬اﺗﺒﻊ ﻧﻔﺲ ﺧﻄﻮات اﻟﺴﺆاﻟﻴﻦ ‪ -2- I‬أ و‪ -2- I‬ب ( ‪.‬‬
‫وﻟﺪﻳﻨﺎ ‪ , a ∧ 24 = 1‬إذن ]‪ ) a ≡ 0 [3‬ﻟﻮ آﺎن ]‪ a ≡ 0 [3‬ﻟﻜﺎن ‪ 3‬ﻗﺎﺳﻤﺎ ﻣﺸﺘﺮآﺎ ل ‪ a‬و‪ 24‬وهﺬا ﻳﺘﻨﺎﻗﺾ ﻣﻊ ‪. ( a ∧ 24 = 1‬‬
‫إذن ]‪ a ≡ 1[3‬أو ]‪ . a ≡ 2 [3‬إذن ]‪ a ≡ 1[3] ⇒ a 2 ≡ 1[3‬و ]‪. a ≡ 2 [3] ⇒ a 2 ≡ 4 [3] ⇒ a 2 ≡ 1[3‬‬
‫]‪. a 2 ≡ 1[ 24‬‬ ‫ﻟﺪﻳﻨﺎ ]‪ a 2 ≡ 1[8‬و ]‪ a 2 ≡ 1[3‬و ‪ 3 ∧ 8 = 1‬؛ إذن ‪:‬‬
‫‪∀k ∈ {1,..., 23} : ak ∧ 24 = 1‬‬ ‫ب( ﻧﻔﺘﺮض أﻧﻪ ﺗﻮﺟﺪ أﻋﺪاد ﺻﺤﻴﺤﺔ ﻃﺒﻴﻌﻴﺔ ‪ a1‬و ‪ a2‬و‪ ...‬و ‪ a23‬ﺣﻴﺚ‪:‬‬
‫‪. a12 + a22 + ... + a23‬‬
‫‪2‬‬
‫و ‪= 23997‬‬
‫‪a12 + a22 + ... + a23‬‬
‫‪2‬‬
‫ﻟﺪﻳﻨﺎ ‪ . ∀k ∈ {1,..., 23} : ak ∧ 24 = 1‬إذن ]‪ ∀k ∈ {1,..., 23} : ak2 ≡ 1[ 24‬وﻣﻨﻪ ﻓﺈن ‪≡ 23 [ 24] :‬‬
‫وﻟﺪﻳﻨﺎ ]‪ . 23997 ≡ 21[ 24‬إذن ‪ 23 ≡ 21[ 24] :‬أي ]‪ 2 ≡ 0 [ 24‬وهﺬا ﺗﻨﺎﻗﺾ ! ‪.‬‬
‫‪∀k ∈ {1,..., 23} : ak ∧ 24 = 1‬‬ ‫ﺧﻼﺻﺔ ‪ :‬ﻻ ﺗﻮﺟﺪ أﻋﺪاد ﺻﺤﻴﺤﺔ ﻃﺒﻴﻌﻴﺔ ‪ a1‬و ‪ a2‬و‪ ...‬و ‪ a23‬ﺣﻴﺚ‪:‬‬
‫‪. a12 + a22 + ... + a23‬‬
‫‪2‬‬
‫و ‪= 23997‬‬
‫اﻟﺘﻤﺮﻳﻦ ‪: 3‬‬
‫⎧‬ ‫‪−‬‬
‫‪2‬‬
‫‪⎪f ( x ) = ( x + 2 ) e x‬‬ ‫‪; x >0‬‬
‫⎨ ‪.‬‬ ‫‪ I‬ﻧﻌﺘﺒﺮ ‪ f‬اﻟﺪاﻟﺔ اﻟﻌﺪدﻳﺔ اﻟﻤﻌﺮﻓﺔ ﻋﻠﻰ اﻟﻤﺠﺎل [∞‪ [ 0, +‬ﺑﻤﺎ ﻳﻠﻲ ‪:‬‬
‫‪⎪⎩f (0) = 0‬‬
‫‪G JG‬‬
‫(‬ ‫)‬
‫وﻟﻴﻜﻦ ) ‪ (C f‬ﻣﻨﺤﻨﺎهﺎ ﻓﻲ ﻣﻌﻠﻢ ﻣﺘﻌﺎﻣﺪ ﻣﻤﻨﻈﻢ ‪ ) . O , i , j‬اﻟﻮﺣﺪة = ‪. ( 2cm‬‬
‫‪2‬‬
‫‪−‬‬ ‫‪⎛ 2‬‬ ‫⎞‬ ‫‪2‬‬
‫‪lim+ f (x ) = lim+ ( x + 2 ) e‬‬ ‫‪x‬‬
‫‪ (1‬أ( ﻧﻀﻊ ‪ X = −‬إذن ∞‪ x → 0+ ⇒ X → −‬وﻣﻨﻪ ‪= lim ⎜ − + 2 ⎟ e X = 0 = f (0) :‬‬
‫‪x →0‬‬ ‫‪x →0‬‬ ‫∞‪X →−‬‬
‫‪⎝ X‬‬ ‫⎠‬ ‫‪x‬‬
‫ﻷن ‪ . lim e = 0‬وﺑﺎﻟﺘﺎﻟﻲ ﻓﺈن ‪ f‬ﻣﺘﺼﻠﺔ ﻋﻠﻰ اﻟﻴﻤﻴﻦ ﻓﻲ اﻟﻨﻘﻄﺔ ‪. 0‬‬
‫‪X‬‬
‫∞‪X →−‬‬

‫‪2‬‬
‫‪ X = −‬إذن ∞‪ x → 0+ ⇒ X → −‬وﻣﻨﻪ ‪:‬‬ ‫ب( ﻧﻀﻊ‬
‫‪x‬‬
‫)‪f (x ) − f (0‬‬ ‫‪x + 2 − x2‬‬
‫‪lim e‬‬ ‫‪X‬‬
‫‪ lim X e‬و ‪= 0‬‬ ‫‪X‬‬
‫‪ lim+‬ﻷن‪= 0 :‬‬ ‫‪= lim+‬‬ ‫‪e = lim (1 − X ) e X = lim e X − X e X = 0‬‬
‫∞‪X →−‬‬ ‫∞‪X →−‬‬ ‫‪x →0‬‬ ‫‪x −0‬‬ ‫‪x‬‬ ‫→‬ ‫‪0‬‬ ‫‪x‬‬ ‫∞‪X →−‬‬ ‫∞‪X →−‬‬

‫إذن ‪ f‬ﻗﺎﺑﻠﺔ ﻟﻺﺷﺘﻘﺎق ﻋﻠﻰ اﻟﻴﻤﻴﻦ ﻓﻲ ‪ 0‬وﻟﺪﻳﻨﺎ ‪. f d' ( 0 ) = 0‬‬


‫ج( ﻟﻴﻜﻦ [∞‪ . x ∈ ]0, +‬ﻟﺪﻳﻨﺎ‪:‬‬

‫‪⎞′‬‬ ‫⎛ ‪⎛ 2 ⎞′ − x‬‬ ‫‪⎞ − x ( x + 1) + 3 − x‬‬


‫‪2‬‬
‫⎛‬ ‫‪−‬‬
‫‪2‬‬
‫‪−‬‬
‫‪2‬‬ ‫‪2‬‬
‫‪2‬‬ ‫‪2‬‬ ‫‪2‬‬
‫‪f ( x ) = ⎜ (x + 2)e‬‬
‫‪′‬‬ ‫‪x‬‬ ‫‪′‬‬
‫= ‪⎟ = (x + 2) e + (x + 2) ⎜ − ⎟ e = ⎜ 1 + 2 (x + 2) ⎟ e‬‬
‫‪x‬‬
‫‪e >0‬‬
‫⎝‬ ‫⎠‬ ‫⎠ ‪⎝ x‬‬ ‫‪⎝ x‬‬ ‫⎠‬ ‫‪x2‬‬
‫إذن ‪ f‬ﺗﺰاﻳﺪﻳﺔ ﻗﻄﻌﺎ ﻋﻠﻰ اﻟﻤﺠﺎل [∞‪. [ 0, +‬‬
‫‪2‬‬
‫‪−‬‬ ‫‪eX‬‬ ‫‪2‬‬
‫‪lim f (x ) = lim (x + 2)e‬‬ ‫‪x‬‬
‫‪= lim− ( −2 + 2X‬‬ ‫)‬ ‫‪ (2‬أ( ﻧﻀﻊ ‪ . X = −‬إذن ‪ x → +∞ ⇒ X → 0−‬وﻣﻨﻪ‪= +∞ :‬‬
‫∞‪x →+‬‬ ‫∞‪x →+‬‬ ‫‪X →0‬‬ ‫‪X‬‬ ‫‪x‬‬

‫‪-3-‬‬
‫‪t2‬‬
‫= ) ‪ v (t‬ﻟﻜﻞ [∞‪. t ∈ [ 0, +‬‬ ‫ب( ﻧﻀﻊ ‪ u (t ) = e −t + t − 1 :‬و‬
‫‪2‬‬
‫ﻟﺪﻳﻨﺎ ‪ u‬و ‪ v‬ﻗﺎﺑﻠﺘﻴﻦ ﻟﻺﺷﺘﻘﺎق ﻣﺮﺗﻴﻦ ﻋﻠﻰ اﻟﻤﺠﺎل [∞‪ [ 0, +‬و ‪ u ′‬و ‪ v ′‬ﻣﺘﺼﻠﺘﻴﻦ ﻋﻠﻰ اﻟﻤﺠﺎل [∞‪ [ 0, +‬وﻟﺪﻳﻨﺎ ‪:‬‬
‫‪ u ′(t ) = −e −t + 1‬و ‪ u ′′(t ) = e −t‬و ‪ v ′(t ) = t‬و ‪ v ′′(t ) = 1‬ﻟﻜﻞ [∞‪. t ∈ [ 0, +‬‬
‫ﺑﻤﺎ أن ﻟﻜﻞ [∞‪ t ≥ 0 ⇒ −t ≤ 0 ⇒ 0 ≤ e −t ≤ 1 ⇒ 0 ≤ u ′′(t ) ≤ v ′′(t ) : t ∈ [ 0, +‬؛‬
‫‪x‬‬ ‫‪x‬‬
‫) ‪ ∀x ≥ 0 : 0 ≤ ∫ u ′′(t )dt ≤ ∫ v ′′(t )dt ⇒ 0 ≤ u ′(x ) ≤ v ′(x‬ﻷن ‪. u ′(0) = v ′(0) = 0‬‬ ‫ﻓﺈن ‪:‬‬
‫‪0‬‬ ‫‪0‬‬
‫‪t‬‬ ‫‪t‬‬
‫) ‪ ∀t ≥ 0 : 0 ≤ ∫ u ′(x )dx ≤ ∫ v ′(x )dx ⇒ 0 ≤ u (t ) ≤ v (t‬ﻷن ‪. u (0) = v (0) = 0‬‬ ‫وﻣﻨﻪ ﻓﺈن ‪:‬‬
‫‪0‬‬ ‫‪0‬‬

‫‪t2‬‬
‫≤ ‪. ∀t ∈ [ 0, +∞[ : 0 ≤ e + t − 1‬‬
‫‪−t‬‬
‫وﺑﺎﻟﺘﺎﻟﻲ ﻓﺈن ‪:‬‬
‫‪2‬‬
‫‪2‬‬ ‫‪2‬‬ ‫‪2‬‬
‫‪−‬‬ ‫‪2‬‬ ‫⎞ ‪1⎛ 2‬‬ ‫‪−‬‬ ‫‪2‬‬ ‫‪2‬‬ ‫‪2‬‬
‫‪0 ≤e‬‬ ‫‪x‬‬
‫‪+ −1 ≤ ⎜ ⎟ ⇒ 0 ≤ e x + −1 ≤ 2‬‬ ‫وﺑﺘﻄﺒﻴﻖ اﻟﺴﺆال اﻟﺴﺎﺑﻖ ‪ ,‬ﻧﺠﺪ ‪:‬‬ ‫ج( ﻟﻴﻜﻦ ‪ . x > 0‬ﻟﺪﻳﻨﺎ ‪> 0‬‬
‫‪x‬‬ ‫⎠ ‪2⎝x‬‬ ‫‪x‬‬ ‫‪x‬‬ ‫‪x‬‬
‫‪2‬‬
‫‪2‬‬ ‫‪−‬‬ ‫‪2 2‬‬
‫‪⇒ 1− ≤ e ≤ 1− + 2‬‬
‫‪x‬‬
‫‪x‬‬ ‫‪x x‬‬
‫‪2‬‬
‫⎞‪⎛ 2‬‬ ‫‪−‬‬ ‫⎞ ‪⎛ 2 2‬‬
‫) ‪⇒ ⎜1 − ⎟ ( x + 2 ) ≤ ( x + 2 ) e x ≤ ⎜1 − + 2 ⎟ ( x + 2‬‬
‫⎠ ‪⎝ x‬‬ ‫⎠ ‪⎝ x x‬‬
‫‪4‬‬ ‫‪4 2 4‬‬
‫‪⇒ x + 2 − 2 − ≤ f (x ) ≤ x + 2 − 2 − + + 2‬‬
‫‪x‬‬ ‫‪x x x‬‬
‫‪4‬‬ ‫‪4 2‬‬
‫‪⇒ x − ≤ f (x ) ≤ x + 2 −‬‬
‫‪x‬‬ ‫‪x‬‬ ‫‪x‬‬
‫‪4‬‬ ‫‪4 2‬‬
‫‪⇒ − ≤ f (x ) − x ≤ 2 −‬‬
‫‪x‬‬ ‫‪x‬‬ ‫‪x‬‬
‫‪4 2‬‬ ‫‪4‬‬ ‫‪4‬‬ ‫‪4 2‬‬
‫د( ﻟﺪﻳﻨﺎ ‪ lim f (x ) = +∞ :‬وﻟﺪﻳﻨﺎ ‪ ∀x > 0 : − ≤ f (x ) − x ≤ 2 −‬و ‪ lim − = 0‬و ‪. lim 2 − = 0‬‬
‫‪x →+∞ x‬‬ ‫‪x‬‬ ‫‪x‬‬ ‫∞‪→+‬‬ ‫‪x‬‬ ‫‪x‬‬ ‫‪x‬‬ ‫‪x‬‬ ‫∞‪x →+‬‬

‫إذن ‪ . lim f (x ) − x = 0 :‬وﻣﻨﻪ ﻧﺴﺘﻨﺘﺞ أن اﻟﻤﻨﺤﻨﻰ ) ‪ (C f‬ﻳﻘﺒﻞ ﻣﻘﺎرﺑﺎ ﻣﺎﺋﻼ ) ∆ ( ﺑﺠﻮار ∞‪ +‬ﻣﻌﺎدﻟﺘﻪ ‪. y = x‬‬
‫∞‪x →+‬‬

‫‪ (3‬إﻧﺸﺎء اﻟﻤﻨﺤﻨﻰ ) ‪: (C f‬‬

‫‪(∆) : y‬‬ ‫‪=x‬‬


‫) ‪(C f‬‬

‫‪JG‬‬
‫‪j‬‬

‫‪G‬‬
‫‪O i‬‬

‫‪-4-‬‬
‫⎧‬ ‫⎛‬ ‫‪2 ⎞ − x2‬‬
‫‪⎪ n‬‬
‫‪f‬‬ ‫(‬ ‫‪x‬‬ ‫)‬ ‫=‬ ‫⎜‬ ‫‪x‬‬ ‫‪+‬‬ ‫‪⎟e‬‬ ‫‪; x >0‬‬
‫⎨‬ ‫⎝‬ ‫⎠‪n‬‬ ‫ﻟﻴﻜﻦ *` ∈ ‪ n‬و ‪ f n‬اﻟﺪاﻟﺔ اﻟﻌﺪدﻳﺔ اﻟﻤﻌﺮﻓﺔ ﻋﻠﻰ اﻟﻤﺠﺎل [∞‪ [ 0, +‬ﺑﻤﺎ ﻳﻠﻲ ‪:‬‬ ‫‪II‬‬
‫‪⎪f (0) = 0‬‬
‫‪⎩ n‬‬
‫‪2‬‬
‫‪ . X = −‬إذن ‪ x → 0+ ⇒ X → −∞ :‬وﻣﻨﻪ ﻓﺈن ‪:‬‬ ‫‪ (1‬ﻧﻀﻊ‬
‫‪x‬‬
‫)‪f (x ) − f n (0‬‬ ‫⎛‬ ‫‪2‬‬ ‫‪⎞ −x‬‬
‫‪2‬‬
‫‪⎛ X ⎞ X‬‬ ‫‪1‬‬
‫‪. lim X e‬‬ ‫‪X‬‬
‫‪ lim+ n‬ﻷن ‪= 0‬‬ ‫‪= lim+ ⎜1 +‬‬ ‫‪⎟ e = Xlim‬‬ ‫‪⎜1 − ⎟ e = Xlim‬‬ ‫‪e X − XeX = 0‬‬
‫∞‪X →−‬‬ ‫‪x →0‬‬ ‫‪x −0‬‬ ‫⎝ ‪x →0‬‬ ‫‪nx‬‬ ‫⎠‬ ‫∞‪→−‬‬
‫⎝‬ ‫⎠ ‪n‬‬ ‫∞‪→−‬‬ ‫‪n‬‬
‫إذن ‪ f n‬ﻗﺎﺑﻠﺔ ﻟﻺﺷﺘﻘﺎق ﻋﻠﻰ اﻟﻴﻤﻴﻦ ﻓﻲ ‪ 0‬وﻟﺪﻳﻨﺎ ‪. ( f n′ )d (0) = 0 :‬‬

‫⎛⎛‬ ‫‪2 ⎞ −2‬‬ ‫⎛ ‪⎞′‬‬ ‫⎛ ‪2 ⎞′ − x2‬‬ ‫⎛ ⎛ ‪2 ⎞⎛ 2 ⎞′ − x2‬‬ ‫‪2 ⎞ 2 ⎞ − x2‬‬


‫‪f n′(x ) = ⎜ ⎜ x + ⎟ e x‬‬ ‫=‬
‫⎜ ⎟‬ ‫‪+‬‬ ‫⎟‬ ‫‪+‬‬ ‫⎜‬ ‫‪+‬‬ ‫⎜⎟‬ ‫‪−‬‬ ‫⎟‬ ‫=‬ ‫‪1‬‬
‫⎜ ⎜‬‫‪+‬‬ ‫‪+‬‬ ‫⎟‬ ‫‪ (2‬ﻟﻴﻜﻦ ‪ . x > 0‬ﻟﺪﻳﻨﺎ‪e > 0 :‬‬
‫⎠⎟ ‪n ⎠ x 2‬‬
‫‪x‬‬ ‫‪e‬‬ ‫‪x‬‬ ‫‪e‬‬ ‫‪x‬‬
‫⎝⎝‬ ‫⎠‪n‬‬ ‫⎝ ⎠‬ ‫⎠‪n‬‬ ‫⎝‬ ‫⎠ ‪n ⎠⎝ x‬‬ ‫⎝ ⎝‬
‫إذن ‪ f n‬داﻟﺔ ﺗﺰاﻳﺪﻳﺔ ﻗﻄﻌﺎ ﻋﻠﻰ اﻟﻤﺠﺎل [∞‪. [ 0, +‬‬
‫‪ (3‬أ( ﻟﺪﻳﻨﺎ ‪ f n‬ﻣﺘﺼﻠﺔ وﺗﺰاﻳﺪﻳﺔ ﻗﻄﻌﺎ ﻋﻠﻰ اﻟﻤﺠﺎل [∞‪ ]0, +‬؛ إذن ‪ f n‬ﺗﻘﺎﺑﻞ ﻣﻦ اﻟﻤﺠﺎل [∞‪ ]0, +‬ﻧﺤﻮ اﻟﻤﺠﺎل [∞‪. f n ( ]0, +∞[ ) = ]0, +‬‬
‫‪2‬‬ ‫‪2‬‬
‫= ) ‪. ∃!an ∈ ]0, +∞[ / f n ( an‬‬ ‫وﺑﻤﺎ أن [∞‪ ∈ ]0, +‬؛ ﻓﺈن‬
‫‪n‬‬ ‫‪n‬‬
‫ب( ﻟﻴﻜﻦ ‪ x > 0‬و *` ∈ ‪ . n‬ﻟﺪﻳﻨﺎ ‪:‬‬
‫⎛‬ ‫⎛ ⎞ ‪2‬‬ ‫⎛⎛ ⎞ ‪2‬‬ ‫‪2 ⎞ − x2‬‬ ‫⎛⎛ ⎞ ‪2‬‬ ‫‪2 ⎞ − x2‬‬ ‫‪2⎞ ⎛ 2‬‬ ‫⎞ ‪2 ⎞⎛ −2‬‬
‫‪⎜ n +1‬‬
‫‪f‬‬ ‫(‬ ‫‪x‬‬ ‫)‬ ‫‪−‬‬ ‫‪−‬‬
‫‪⎟ ⎜ n‬‬
‫‪f‬‬ ‫(‬ ‫‪x‬‬ ‫)‬ ‫‪−‬‬ ‫=‬
‫⎜⎜ ⎟‬ ‫‪x‬‬ ‫‪+‬‬ ‫⎟‬ ‫‪e‬‬ ‫‪−‬‬ ‫‪−‬‬
‫⎜⎜ ⎟‬ ‫‪x‬‬ ‫‪+‬‬ ‫‪⎟e‬‬ ‫⎜=⎟ ‪−‬‬ ‫⎟‪− ⎟ ⎜ e x − 1‬‬
‫⎝‬ ‫⎝ ⎠‪n +1‬‬ ‫⎝⎝ ⎠ ‪n‬‬ ‫⎠‪n +1‬‬ ‫⎝⎝ ⎠‪n +1‬‬ ‫⎠‪n‬‬ ‫⎝ ⎠ ‪n ⎠ ⎝ n +1 n‬‬ ‫⎠‬
‫⎛‬ ‫⎛ ⎞ ‪2‬‬ ‫⎞‪2‬‬ ‫⎞ ‪2 ⎛ − x2‬‬
‫‪⎜ n +1‬‬
‫‪f‬‬ ‫(‬ ‫‪x‬‬ ‫)‬ ‫‪−‬‬ ‫‪−‬‬
‫‪⎟ ⎜ n‬‬
‫‪f‬‬ ‫(‬ ‫‪x‬‬ ‫)‬ ‫‪−‬‬ ‫⎟‬ ‫=‬ ‫‪−‬‬ ‫⎟‪⎜ e − 1‬‬ ‫إذن ‪:‬‬
‫⎝‬ ‫⎝ ⎠‪n +1‬‬ ‫⎠‪n‬‬ ‫⎝ )‪n (n + 1‬‬ ‫⎠‬
‫‪2‬‬ ‫‪−‬‬
‫‪2‬‬
‫⎞ ‪2 ⎛ − x2‬‬
‫‪. x >0⇒−‬‬ ‫‪< 0 ⇒ e x <1⇒ −‬‬ ‫وﻣﻨﻪ ﻓﺈن ‪⎜ e − 1⎟ > 0 :‬‬
‫‪x‬‬ ‫⎝ )‪n (n + 1‬‬ ‫⎠‬
‫‪2‬‬ ‫‪2‬‬
‫‪. ∀x > 0; ∀n ∈ `* ; f n +1 (x ) −‬‬ ‫‪> f n (x ) −‬‬ ‫وﺑﺎﻟﺘﺎﻟﻲ ﻓﺈن ‪:‬‬
‫‪n +1‬‬ ‫‪n‬‬
‫‪2‬‬ ‫‪2‬‬ ‫‪2‬‬
‫‪f n +1 (an +1 ) −‬‬ ‫)ﻷن ‪> f n (an +1 ) − ⇒ 0 > f n (an +1 ) − ( f n +1 (an +1 ) = 0‬‬ ‫ج( ﻟﺪﻳﻨﺎ ‪:‬‬
‫‪n +1‬‬ ‫‪n‬‬ ‫‪n‬‬
‫‪2‬‬
‫< ) ‪⇒ f n (an +1‬‬
‫‪n‬‬
‫⎞‪⎛2‬‬
‫⎟ ⎜ ‪⇒ an +1 < f n−1‬‬ ‫) ﻷن ‪ f n‬ﺗﻘﺎﺑﻞ ﻣﻦ [∞‪ [ 0, +‬ﻧﺤﻮ [∞‪( [ 0, +‬‬
‫⎠ ‪⎝n‬‬
‫‪2‬‬
‫‪⇒ an +1 < an‬‬ ‫) ﻷن = ) ‪( f n (an‬‬
‫‪n‬‬
‫وﺑﺎﻟﺘﺎﻟﻲ ﻓﺈن *`∈‪ ( an )n‬ﻣﺘﺘﺎﻟﻴﺔ ﺗﻨﺎﻗﺼﻴﺔ ؛ وﺑﻤﺎ أﻧﻬﺎ ﻣﺼﻐﻮرة ﺑﺎﻟﻌﺪد ‪ ( ∀n ∈ ` : an > 0 ) 0‬؛ ﻓﺈﻧﻬﺎ ﻣﺘﻘﺎرﺑﺔ‪ .‬ﻧﻀﻊ ‪lim an = a‬‬
‫*‬
‫∞‪n →+‬‬
‫‪2‬‬
‫‪2‬‬ ‫⎛‬ ‫‪2⎞ −‬‬ ‫‪2‬‬ ‫‪2 2 2‬‬ ‫‪2‬‬ ‫‪2‬‬
‫‪. f n (an ) = ⇒ ⎜ an + ⎟ e an = ⇒ an + = e n ⇒ nan + 2 = 2e n ⇒ nan = 2e n − 2‬‬ ‫د( ﻟﺪﻳﻨﺎ ‪:‬‬
‫‪n‬‬ ‫⎝‬ ‫⎠‪n‬‬ ‫‪n‬‬ ‫‪n n‬‬
‫هـ( ﻟﺪﻳﻨﺎ ‪ . ∀n ∈ `* : an > 0‬إذن ‪ . a = lim an ≥ 0‬ﻧﻔﺘﺮض أن ‪ a ≠ 0‬؛ إذن ‪. a > 0 :‬‬
‫∞‪n →+‬‬
‫‪2‬‬ ‫‪2‬‬ ‫‪2‬‬

‫‪ . lim 2e‬وهﺬا ﺗﻨﺎﻗﺾ ‪ .‬وﺑﺎﻟﺘﺎﻟﻲ ﻓﺈن ‪:‬‬ ‫‪an‬‬


‫‪ ∀n ∈ ` : nan = 2e‬و ∞‪ lim nan = +‬ﻟﻜﻦ ‪− 2 = 2e − 2‬‬
‫‪a‬‬ ‫*‬ ‫‪an‬‬
‫وﻟﺪﻳﻨﺎ ‪− 2 :‬‬
‫∞‪n →+‬‬ ‫∞‪n →+‬‬

‫‪lim an = 0‬‬
‫∞‪n →+‬‬

‫‪. ∀x ∈ [ 0, +∞[ : F (x ) = ∫ f (t )dt‬‬ ‫‪ III‬ﻧﻌﺘﺒﺮ اﻟﺪاﻟﺔ اﻟﻌﺪدﻳﺔ ‪ F‬اﻟﻤﻌﺮﻓﺔ ﻋﻠﻰ اﻟﻤﺠﺎل [∞‪ [ 0, +‬ﺑﻤﺎﻳﻠﻲ ‪:‬‬
‫‪2x‬‬

‫‪x‬‬

‫‪ (1‬أ( ﻟﻴﻜﻦ ‪ . x > 0‬ﻟﺪﻳﻨﺎ ‪ f‬ﺗﺰاﻳﺪﻳﺔ ﻗﻄﻌﺎ ﻋﻠﻰ اﻟﻤﺠﺎل [∞‪ [ 0, +‬و ‪ . x ≤ 2x‬إذن ‪:‬‬

‫‪-5-‬‬
‫) ‪ . ∀t ∈ [ x , 2x ] : x ≤ t ≤ 2x ⇒ f (x ) ≤ f (t ) ≤ f (2x‬إذن ‪. f (x ) ∫ dt ≤ ∫ f (t )dt ≤ f (2x ) ∫ dt :‬‬
‫‪2x‬‬ ‫‪2x‬‬ ‫‪2x‬‬

‫‪x‬‬ ‫‪x‬‬ ‫‪x‬‬

‫ﻟﻜﻞ ‪. x > 0‬‬ ‫) ‪xf (x ) ≤ F (x ) ≤ xf (2x‬‬ ‫وﻣﻨﻪ ﻓﺈن ‪:‬‬


‫ب( ﻟﺪﻳﻨﺎ ) ‪ ∀x > 0 : F (x ) ≥ xf (x‬وﻟﺪﻳﻨﺎ ∞‪ . lim f (x ) = +∞ ⇒ lim xf (x ) = +‬وﻣﻨﻪ ﻧﺴﺘﻨﺘﺞ أن ‪lim F (x ) = +∞ :‬‬
‫∞‪x →+‬‬ ‫∞‪x →+‬‬ ‫∞‪x →+‬‬

‫‪ (2‬أ( ﻟﺪﻳﻨﺎ ‪ f‬ﻣﺘﺼﻠﺔ ﻋﻠﻰ اﻟﻤﺠﺎل [∞‪ . [ 0, +‬ﻟﺘﻜﻦ ‪ ψ‬داﻟﺔ أﺻﻠﻴﺔ ﻟﻠﺪاﻟﺔ ‪ f‬ﻋﻠﻰ اﻟﻤﺠﺎل [∞‪ . [ 0, +‬إذن ‪:‬‬
‫) ‪. ∀x ∈ [ 0, +∞[ : F (x ) = ∫ f (t )dt =ψ (2x ) −ψ (x‬‬
‫‪2x‬‬

‫‪x‬‬

‫ﺑﻤﺎ أن ‪ x 6 2x‬ﻗﺎﺑﻠﺔ ﻟﻺﺷﺘﻘﺎق ﻋﻠﻰ اﻟﻤﺠﺎل [∞‪ ]0, +‬وﺗﺤﻮل اﻟﻤﺠﺎل [∞‪ ]0, +‬ﻧﺤﻮ اﻟﻤﺠﺎل [∞‪ ]0, +‬وﺑﻤﺎ أن ‪ ψ‬ﻗﺎﺑﻠﺔ ﻟﻺﺷﺘﻘﺎق ﻋﻠﻰ‬
‫؛ ﻓﺈن ) ‪ x 6 ψ (2x‬ﻗﺎﺑﻠﺔ ﻟﻺﺷﺘﻘﺎق ﻋﻠﻰ اﻟﻤﺠﺎل [∞‪ . ]0, +‬وﺑﺎﻟﺘﺎﻟﻲ ﻓﺈن ‪ F‬ﻗﺎﺑﻠﺔ ﻟﻺﺷﺘﻘﺎق ﻋﻠﻰ اﻟﻤﺠﺎل [∞‪. ]0, +‬‬ ‫اﻟﻤﺠﺎل [∞‪]0, +‬‬
‫⎡‬ ‫) ‪F (x‬‬ ‫⎤‬
‫≤ ) ‪ [∀x > 0 : xf (x ) ≤ F (x ) ≤ xf (2x ) ] ⇒ ⎢∀x > 0 : f (x‬و ‪ lim+ f (x ) = 0‬و‬ ‫وﻟﺪﻳﻨﺎ ‪≤ f (2x ) ⎥ :‬‬
‫‪x →0‬‬ ‫⎣‬ ‫‪x‬‬ ‫⎦‬
‫)‪F (x ) − F (0‬‬ ‫) ‪F (x‬‬
‫‪ lim+‬إذن ‪ F‬ﻗﺎﺑﻠﺔ ﻟﻺﺷﺘﻘﺎق ﻋﻠﻰ اﻟﻴﻤﻴﻦ ﻓﻲ ‪ 0‬و ‪. Fd′ (0) = 0‬‬ ‫‪= lim+‬‬ ‫‪ lim+ f (2x ) = 0‬ﻓﺈن ‪= 0 :‬‬
‫‪x →0‬‬ ‫‪x −0‬‬ ‫‪x‬‬ ‫→‬ ‫‪0‬‬ ‫‪x‬‬ ‫‪x →0‬‬

‫وﺑﺎﻟﺘﺎﻟﻲ ﻓﺈن ‪ F‬ﻗﺎﺑﻠﺔ ﻟﻺﺷﺘﻘﺎق ﻋﻠﻰ اﻟﻤﺠﺎل [∞‪. [ 0, +‬‬


‫ب( ﺣﺴﺐ )أ( ﻧﻌﻠﻢ أن ‪ . Fd′ (0) = 0‬ﻟﻴﻜﻦ ‪ . x > 0‬ﻟﺪﻳﻨﺎ ‪:‬‬
‫‪1‬‬ ‫‪2‬‬
‫‪F ′(x ) = (ψ (2x ) −ψ (x ) )′ = ( 2x )′ ψ ′(2x ) −ψ ′(x ) = 2f (2x ) − f (x ) = 2 ( 2x + 2 ) e x − ( x + 2 ) e x‬‬
‫‪−‬‬ ‫‪−‬‬

‫‪−‬‬
‫‪2‬‬
‫⎡‬ ‫‪1‬‬
‫⎤‬ ‫‪−‬‬
‫‪2‬‬
‫⎡‬ ‫‪⎛ x1‬‬ ‫⎞‬ ‫‪1‬‬ ‫‪1‬‬
‫⎤‬
‫‪F ′(x ) = e‬‬ ‫‪x‬‬
‫⎢‬ ‫(‪4‬‬ ‫‪x‬‬ ‫‪+‬‬ ‫)‪1‬‬‫‪e‬‬ ‫‪x‬‬
‫‪−‬‬ ‫(‬ ‫‪x‬‬ ‫‪+‬‬ ‫)‪2‬‬ ‫⎥‬ ‫=‬ ‫‪e‬‬ ‫‪x‬‬
‫⎢‬ ‫(‬ ‫‪x‬‬ ‫‪+‬‬ ‫)‪2‬‬ ‫⎜‬ ‫‪e‬‬ ‫‪−‬‬ ‫‪1‬‬ ‫⎟‬ ‫‪−‬‬ ‫(‬ ‫‪x‬‬ ‫‪+‬‬ ‫)‪2‬‬‫‪e‬‬ ‫‪x‬‬
‫‪+‬‬ ‫(‪4‬‬ ‫‪x‬‬ ‫‪+‬‬ ‫)‪1‬‬‫‪e‬‬ ‫‪x‬‬
‫⎥‬
‫⎣‬ ‫⎦‬ ‫⎣⎢‬ ‫⎝‬ ‫⎠‬ ‫⎦⎥‬
‫‪−‬‬
‫‪2‬‬
‫⎡‬ ‫‪⎛ 1‬‬ ‫⎞‬ ‫‪1‬‬
‫⎤‬
‫‪. F ′(x ) = e‬‬ ‫‪x‬‬
‫⎥ ‪⎢(x + 2) ⎜ e x − 1⎟ + (3x + 2)e x‬‬ ‫وﻣﻨﻪ ﻓﺈن ‪:‬‬
‫⎢⎣‬ ‫⎝‬ ‫⎠‬ ‫⎥⎦‬
‫⎧‬ ‫⎡ ‪−‬‬
‫‪2‬‬
‫‪⎛ 1‬‬ ‫⎞‬ ‫‪1‬‬
‫⎤‬
‫‪⎪F ′(x ) = e x ⎢(x + 2) ⎜ e x − 1⎟ + (3x + 2)e x ⎥ ; x > 0‬‬
‫⎨‬ ‫⎢⎣‬ ‫⎝‬ ‫⎠‬ ‫⎥⎦‬ ‫وﺑﺎﻟﺘﺎﻟﻲ ﻓﺈن ‪:‬‬
‫⎪‬
‫‪⎩Fd′ ( 0 ) = 0‬‬
‫‪1‬‬ ‫‪1‬‬ ‫‪1‬‬
‫‪1‬‬
‫‪ ( 3x + 2 ) e > 0‬و ‪. x > 0 ⇒ x + 2 > 0‬‬ ‫‪ (3‬ﻟﺪﻳﻨﺎ ‪ x > 0 ⇒ > 0 ⇒ e > 1 ⇒ e − 1 > 0 :‬و‬
‫‪x‬‬ ‫‪x‬‬ ‫‪x‬‬
‫‪x‬‬
‫إذن ‪ ∀x > 0 : F ′(x ) > 0 :‬وﻣﻨﻪ ﻓﺈن ‪ F :‬ﺗﺰاﻳﺪﻳﺔ ﻗﻄﻌﺎ ﻋﻠﻰ اﻟﻤﺠﺎل [∞‪. [ 0, +‬‬
‫ﺟﺪول ﺗﻐﻴﺮات اﻟﺪاﻟﺔ ‪ F‬ﻋﻠﻰ اﻟﻤﺠﺎل [∞‪: [ 0, +‬‬
‫‪x‬‬ ‫‪0‬‬ ‫∞‪+‬‬
‫) ‪F ′(x‬‬ ‫‪0‬‬ ‫‪+‬‬
‫) ‪F (x‬‬ ‫∞‪+‬‬
‫‪0‬‬
‫‪iz − 1‬‬ ‫اﻟﺘﻤﺮﻳﻦ ‪: 4‬‬
‫= ) ‪∀z ∈ ^ − {−1} : f (z‬‬
‫‪(z + 1) 2‬‬
‫‪ (1‬أ( ﻟﻴﻜﻦ ‪ y‬ﻋﺪدا ﺣﻘﻴﻘﻴﺎ ‪ .‬ﻟﺪﻳﻨﺎ ‪:‬‬
‫‪− y −1‬‬
‫‪f (iy ) = iy‬‬ ‫⇔‬ ‫‪= iy‬‬
‫‪(iy + 1) 2‬‬
‫⇔‬ ‫) ‪− y − 1 = iy (− y 2 + 1 + 2iy‬‬
‫)‪⇔ − y − 1 = −2 y 2 + iy (− y 2 + 1‬‬
‫‪⎧− y − 1 = −2 y 2‬‬
‫⇔‬ ‫⎨‬
‫‪⎩ y (1 − y )(1 + y ) = 0‬‬

‫‪-6-‬‬
⎧⎪2 y 2 − y − 1 = 0
⇔ ⎨
⎪⎩( y = 0 ) ∨ ( y = 1) ∨ ( y = −1)
⇔ y =1
: ‫ب( ﻟﺪﻳﻨﺎ‬
iz − 1
f (z ) = z ⇔ =z
(z + 1) 2
⇔ iz − 1 = z (z 2 + 2z + 1)
⇔ iz − 1 = z 3 + 2z 2 + z
⇔ z 3 + 2z 2 + (1 − i )z + 1 = 0 : (*)
: ‫ آﻤﺎﻳﻠﻲ‬z − i ‫ ﻋﻠﻰ‬z 3 + 2z 2 + (1 − i )z + 1 ‫ ﻧﻨﺠﺰ اﻟﻘﺴﻤﺔ اﻷﻗﻠﻴﺪﻳﺔ ل‬. (*) ‫ ﺣﻞ ﻟﻠﻤﻌﺎدﻟﺔ‬i ‫ إذن‬f (i ) = i ‫ﻧﻌﻠﻢ أن‬
z 3 + 2z 2 + (1 − i )z + 1 z −i
: z 3 − iz 2
(2 + i )z 2 + (1 − i )z + 1 z 2 + (2 + i )z + i
: (2 + i )z 2 + (1 − 2i )z
iz + 1
iz + 1

00

f (z ) = z ⇔ ( z − i ) ( z 2 + (2 + i )z + i ) = 0
: ‫إذن‬
⇔ z = i ∨ z 2 + (2 + i )z + i = 0 : (**)
2
: ‫ إذن ﻟﻠﻤﻌﺎدﻟﺔ )**( ﺣﻠﻴﻦ هﻤﺎ‬. ∆ = (2 + i ) 2 − 4i = 7 − 1 + 4i − 4i = 3 = 3 ‫ ﻟﺪﻳﻨﺎ‬: (**) ‫ﻧﺤﻞ اﻟﻤﻌﺎدﻟﺔ‬
−(2 + i ) − 3 3 1 −(2 + i ) + 3 3 1
z = = −1 − − i ‫ أو‬z = = −1 + − i
2 2 2 2 2 2
⎧⎪ 3 1 3 1 ⎫⎪
. S = ⎨i , −1 + − i , −1 + − i ⎬ : ‫ ﻓﻲ ^ هﻲ‬f (z ) = z ‫وﺑﺎﻟﺘﺎﻟﻲ ﻓﺈن ﻣﺠﻤﻮﻋﺔ ﺣﻠﻮل اﻟﻤﻌﺎدﻟﺔ‬
⎩⎪ 2 2 2 2 ⎪⎭

3 1 3 1
. z 2 = −1 − − i ‫ و‬z 1 = −1 + − i ‫ ﻓﺈن‬ℜe (z 1 ) > ℜe (z 2 ) ‫ وﺑﻤﺎأن‬. z 0 = i ‫ ﻓﺈن‬, ℜe (z 0 ) = 0 ‫ﺑﻤﺎ أن‬
2 2 2 2
11π
3 1 ⎛π ⎞ ⎛π ⎞ ⎡ π ⎤ ⎡ π ⎤ ⎡ 11π ⎤ i
z 1 +1 = − i = cos ⎜ ⎟ − i sin ⎜ ⎟ = ⎢1, − ⎥ = ⎢1, − + 2π ⎥ = ⎢1, ⎥ = e 6
: ‫( أ( ﻟﺪﻳﻨﺎ‬2
2 2 ⎝6⎠ ⎝6⎠ ⎣ 6⎦ ⎣ 6 ⎦ ⎣ 6 ⎦

3 1 ⎛π ⎞ ⎛π ⎞ ⎡ π⎤ ⎡ π ⎤ ⎡ 7π ⎤ i
. z 2 +1 = − − i = − cos ⎜ ⎟ − i sin ⎜ ⎟ = − ⎢1, ⎥ = ⎢1, + π ⎥ = ⎢1, ⎥ = e 6 ‫و‬
2 2 ⎝6⎠ ⎝6⎠ ⎣ 6⎦ ⎣ 6 ⎦ ⎣ 6 ⎦
11π 11π
i i ⎛ − i 11π i
11π
⎞ 11π
⎛ 11π ⎞ i 12
π 11π
⎛ 11π ⎞ i 2 i 12 ⎛ 11π ⎞ i
17π
z 1 = −1 + e 6
= e 12 ⎜ −e 12 + e 12 ⎟ = 2i sin ⎜ ⎟ e = 2sin ⎜ ⎟ e e = 2sin ⎜ ⎟e
12
: ‫ب( ﻟﺪﻳﻨﺎ‬
⎝ ⎠ ⎝ 12 ⎠ ⎝ 12 ⎠ ⎝ 12 ⎠
7π 7π
i i ⎛ −i 712π i

⎞ ⎛ 7π

⎞ i 12 ⎛ 7π
π 7π
⎞ i 2 i 12 ⎛ 7π ⎞ i
13π
z 2 = −1 + e 6
=e 12
⎜ −e + e 12
⎟ = 2i sin ⎜ ⎟ e = 2sin ⎜ ⎟ e e = 2sin ⎜ ⎟e
12
‫و‬
⎝ ⎠ ⎝ 12 ⎠ ⎝ 12 ⎠ ⎝ 12 ⎠
π
i
: ‫ وﻟﺪﻳﻨﺎ‬. e 2
=i : ‫ﻷن‬
⎛ 7π ⎞ ⎛π π ⎞ ⎛π ⎞ ⎛π ⎞ ⎛π ⎞ ⎛π ⎞ 3 2 1 2 6+ 2
sin ⎜ ⎟ = sin ⎜ + ⎟ = sin ⎜ ⎟ cos ⎜ ⎟ + cos ⎜ ⎟ sin ⎜ ⎟ = + =
⎝ 12 ⎠ ⎝3 4⎠ ⎝3⎠ ⎝4⎠ ⎝3⎠ ⎝4⎠ 2 2 2 2 4
⎛ 11π ⎞ ⎛ π⎞ ⎛π ⎞ ⎛π π ⎞ ⎛π ⎞ ⎛π ⎞ ⎛π ⎞ ⎛π ⎞ 3 2 1 2 6− 2
sin ⎜ ⎟ = sin ⎜ π − ⎟ = sin ⎜ ⎟ = sin ⎜ − ⎟ = sin ⎜ ⎟ cos ⎜ ⎟ − cos ⎜ ⎟ sin ⎜ ⎟ = − =
⎝ 12 ⎠ ⎝ 12 ⎠ ⎝ 12 ⎠ ⎝ 3 4⎠ ⎝3⎠ ⎝4⎠ ⎝ 3⎠ ⎝ 4⎠ 2 2 2 2 4

-7-
6 + 2 i 1312π 6 + 2 i 1312π 6 − 2 i 1712π 6 − 2 i 1712π
. z2 =2 e = e ‫ و‬z1 = 2 e = e : ‫وﻣﻨﻪ ﻧﺠﺪ‬
4 2 4 2
. 0 ≤ α < π : ‫ ﺣﻴﺚ‬z = e i α ‫( ﻧﻀﻊ‬3

⎛ iz − 1 ⎞ −i z − 1 −iz 2 z − z 2 −iz − z 2 i +z 1
f (z ) = ⎜ 2 ⎟
= = = = −z : ‫ إذن‬. z = 1 ⇒ z z = 1 ⇒ z = : ‫أ( ﻟﺪﻳﻨﺎ‬
⎝ (z + 1) ⎠ (z + 1) ( )
(1 + z ) (1 + z )
2 2 2 2
z z +z z

iz − 1
. f (z ) = iz = izf (z ) : ‫وﻣﻨﻪ‬
(1 + z )
2

⎧z = i
⎧iz + 1 = 0 ⎪ ⎧z = i
f (z ) + f (z ) = 0 ⇔ izf (z ) + f (z ) = 0 ⇔ (iz + 1)f (z ) = 0 ⇔ ⎨ ⇔ ⎨ iz − 1 ⇔⎨ :‫ب( ﻟﺪﻳﻨﺎ‬
⎩f ( z ) = 0 ⎪ (z + 1) 2
= 0 ⎩ z = −i

⎧ π

⎪⎧e = i ⎪⎪
α ≡ [ 2π ] π
f (z ) + f (z ) = 0 ⇔ ⎨ i α ⇔⎨ 2 ⇔ α = ( 0 ≤ α < π ‫) ﻷن‬ : ‫إذن‬
⎪⎩e = −i π
⎪α ≡ − [ 2π ] 2
⎪⎩ 2
i ⎜ +α ⎟ ⎛ − i ⎜ +α ⎟ ⎞
1 ⎛π ⎞ 1 ⎛π ⎞ 1 ⎛π ⎞
i ⎜ +α ⎟
π e 2 ⎝2 ⎠
⎜e 2 ⎝2 ⎠
−e 2 ⎝2 ⎠


ie − 1
i
2 iα
e e −1 ⎜ ⎟

f (z ) = f (e ) = = = ⎝ ⎠ :‫ج( ﻟﺪﻳﻨﺎ‬
(e + 1) ⎛⎜e i α2 ⎛ e i α2 + e −i α2 ⎞ ⎞⎟
2 2 2

iα ⎛ ⎛ α ⎞⎞
⎜ ⎟⎟ e ⎜ 2cos ⎜ ⎟ ⎟
⎜ ⎝ ⎝ 2 ⎠⎠
⎝ ⎝ ⎠⎠
⎛1⎛π ⎞⎞ ⎛π α ⎞ ⎛π α ⎞
1 ⎛π ⎞ 2i sin ⎜ ⎜ + α ⎟ ⎟ sin ⎜ + ⎟ 1 i ⎛ π +π −α ⎞ sin ⎜ + ⎟ 3π − 2α
f (z ) = e
i ⎜ +α ⎟ −i α
2 ⎝2 ⎠ ⎝2⎝ 2 ⎠⎠ 1
= ⎝ 4 2 ⎠ e 2 ⎜⎝ 2 ⎟

=
1 ⎝ 4 2 ⎠ei 4
⎛α ⎞ 2 ⎛α ⎞ 2 ⎛α ⎞
4 cos 2 ⎜ ⎟ cos 2 ⎜ ⎟ cos 2 ⎜ ⎟
⎝2⎠ ⎝2⎠ ⎝2⎠
α π π α π π ⎛π α ⎞
‫ و‬0 ≤α <π ⇒ 0 ≤ < ⇒ ≤ + < ⇒ sin ⎜ + ⎟ > 0 :‫وذﻟﻚ ﻷن‬
2 2 4 2 4 2 ⎝4 2⎠

α π ⎛α ⎞
0 ≤α <π ⇒ 0 ≤ < ⇒ cos ⎜ ⎟ > 0
2 2 ⎝2⎠
⎧ z =1 ⎧z = e i α / 0 ≤ α < 2π
⎪ ⎪
: ‫ وﻟﺪﻳﻨﺎ‬. ⎨ 1⇔⎨ 1 : ‫( ﻟﺪﻳﻨﺎ‬4
⎪ℜe (f (z )) = ⎪ℜe (f (z )) =
⎩ 2 ⎩ 2
⎛π α ⎞
sin ⎜ + ⎟
1
ℜe (f (z )) = ⇔ ⎝ 4 2 ⎠ cos ⎛ 3π − 2α ⎞ = 1 ⇔ sin ⎛ π + α ⎞ cos ⎛ 3π − 2α ⎞ = cos 2 ⎛ α ⎞
⎜ ⎟ ⎜ ⎟ ⎜ ⎟ ⎜ ⎟
2 ⎛α ⎞ ⎝ 4 ⎠ 2 ⎝4 2⎠ ⎝ 4 ⎠ ⎝2⎠
2 cos 2 ⎜ ⎟
⎝2⎠
1 2⎛ ⎛α ⎞ ⎛α ⎞⎞ 2 ⎛ ⎛α ⎞ ⎛α ⎞⎞ ⎛α ⎞
ℜe (f (z )) = ⇔ ⎜ − cos ⎜ ⎟ + sin ⎜ ⎟ ⎟ ⎜ cos ⎜ ⎟ + sin ⎜ ⎟ ⎟ = cos 2 ⎜ ⎟
2 2 ⎝ ⎝2⎠ ⎝ 2 ⎠⎠ 2 ⎝ ⎝2⎠ ⎝ 2 ⎠⎠ ⎝2⎠
1 1⎛ ⎛α ⎞ ⎛α ⎞⎞ ⎛α ⎞ 1 ⎛α ⎞
ℜe (f (z )) = ⇔ − ⎜ cos 2 ⎜ ⎟ − sin 2 ⎜ ⎟ ⎟ = cos 2 ⎜ ⎟ ⇔ − cos (α ) = cos 2 ⎜ ⎟
2 2⎝ ⎝2⎠ ⎝ 2 ⎠⎠ ⎝2⎠ 2 ⎝2⎠

-8-
‫‪⎧α‬‬ ‫‪π‬‬
‫⎪‬ ‫] ‪≡ ± [ 2π‬‬
‫‪1‬‬ ‫⎛‪1‬‬ ‫⎞ ⎞ ‪⎛α‬‬ ‫⎞ ‪⎛α‬‬ ‫‪⎛α ⎞ 1‬‬ ‫⎞ ‪⎛α‬‬ ‫‪1‬‬ ‫‪⎪2‬‬ ‫‪3‬‬
‫⎨ ⇔ ‪ℜe (f (z )) = ⇔ − ⎜ 2 cos 2 ⎜ ⎟ − 1⎟ = cos 2 ⎜ ⎟ ⇔ cos 2 ⎜ ⎟ = ⇔ cos ⎜ ⎟ = ±‬‬
‫‪2‬‬ ‫⎝‪2‬‬ ‫⎠ ⎠‪⎝2‬‬ ‫⎠‪⎝2‬‬ ‫‪⎝2⎠ 4‬‬ ‫⎠‪⎝2‬‬ ‫‪2‬‬ ‫] ‪⎪α ≡ ± 2π [ 2π‬‬
‫‪⎪⎩ 2‬‬ ‫‪3‬‬
‫⎧‬ ‫‪2π‬‬
‫‪1‬‬ ‫⎪⎪‬ ‫‪α‬‬ ‫≡‬ ‫‪±‬‬ ‫] ‪[ 4π‬‬
‫‪3‬‬
‫⎨ ⇔ = )) ‪ℜe (f (z‬‬
‫‪2‬‬ ‫] ‪⎪α ≡ ± 4π [ 4π‬‬
‫⎩⎪‬ ‫‪3‬‬

‫‪2π‬‬ ‫‪2π‬‬
‫‪2π‬‬ ‫‪−i‬‬ ‫‪1‬‬ ‫‪3‬‬ ‫‪2π‬‬ ‫‪1‬‬ ‫‪3‬‬
‫‪α ≡ − [ 4π ] ⇒ z = e = e‬‬ ‫وﻟﺪﻳﻨﺎ ‪[ 4π ] ⇒ z = e = e 3 = − + i = j :‬‬
‫‪i‬‬
‫‪iα‬‬ ‫‪iα‬‬
‫‪3‬‬
‫‪= − −i‬‬ ‫≡‪ α‬و ‪=j‬‬
‫‪3‬‬ ‫‪2‬‬ ‫‪2‬‬ ‫‪3‬‬ ‫‪2‬‬ ‫‪2‬‬
‫‪4π‬‬ ‫‪2π‬‬ ‫‪4π‬‬ ‫‪2π‬‬
‫‪4π‬‬ ‫‪−i‬‬ ‫‪4π‬‬ ‫‪−i‬‬
‫≡ ‪ α‬و ‪α ≡ − [ 4π ] ⇒ z = e i α = e 3 = e 3 = j‬‬ ‫و ‪[ 4π ] ⇒ z = e i α = e 3 = e 3 = j‬‬
‫‪i‬‬ ‫‪i‬‬

‫‪3‬‬ ‫‪3‬‬
‫‪2π‬‬ ‫‪2π‬‬

‫} {‬ ‫‪1‬‬ ‫‪3‬‬ ‫‪1‬‬ ‫‪3‬‬


‫‪−i‬‬ ‫‪i‬‬
‫‪S = j, j‬‬ ‫‪. z = j = − −i‬‬ ‫‪ z = j = − + i‬أو ‪= e 3‬‬ ‫وﺑﺎﻟﺘﺎﻟﻲ ﻓﺈن ‪= e 3 :‬‬
‫‪2‬‬ ‫‪2‬‬ ‫‪2‬‬ ‫‪2‬‬
‫‪⎧ z =1‬‬
‫⎪‬
‫‪.‬‬ ‫⎨‬ ‫‪1‬‬ ‫ﻟﻨﺤﻞ اﻟﻨﻈﻤﺔ اﻟﺘﺎﻟﻴﺔ ‪:‬‬ ‫ﻃﺮﻳﻘﺔ أﺧﺮى ‪:‬‬
‫= ) ) ‪⎪ℜe ( f (z‬‬
‫⎩‬ ‫‪2‬‬
‫ﻟﺪﻳﻨﺎ ‪ . z = 1‬إذن ﺣﺴﺐ اﻟﺴﺆال ‪ -3‬أ ‪ -‬ﻟﺪﻳﻨﺎ ‪ f (z ) = izf (z ) :‬وﻣﻨﻪ ﻧﺴﺘﻨﺘﺞ أن ‪:‬‬
‫‪1‬‬
‫= ) ) ‪ℜe ( f ( z‬‬ ‫⇔‬ ‫‪f (z ) + f (z ) = 2ℜe ( f (z ) ) = 1‬‬
‫‪2‬‬
‫⇔‬ ‫‪f (z ) + izf (z ) = 1‬‬
‫⇔‬ ‫‪(iz + 1)f (z ) = 1‬‬
‫‪iz − 1‬‬
‫⇔‬ ‫)‪(iz + 1‬‬ ‫‪=1‬‬
‫‪(z + 1) 2‬‬
‫⇔‬ ‫‪(iz − 1)(iz + 1) = (z + 1) 2‬‬
‫⇔‬ ‫‪− z 2 − 1 = z 2 + 2z + 1‬‬
‫⇔‬ ‫‪2z 2 + 2z + 2 = 0‬‬
‫⇔‬ ‫‪z 2 + z +1 = 0‬‬
‫‪2π‬‬ ‫‪4π‬‬
‫⎛‬ ‫‪i‬‬ ‫⎞‬ ‫⎛‬ ‫‪i‬‬ ‫⎞‬
‫⇔‬ ‫⎜‬ ‫‪z‬‬ ‫=‬ ‫‪e‬‬ ‫‪3‬‬
‫⎟‬ ‫∨‬ ‫⎜‬ ‫‪z‬‬ ‫=‬ ‫‪e‬‬ ‫‪3‬‬
‫⎟‬
‫⎝‬ ‫⎠‬ ‫⎝‬ ‫⎠‬
‫⎡‬ ‫‪1‬‬ ‫⎤‪3‬‬ ‫⎡‬ ‫‪1‬‬ ‫⎤‪3‬‬
‫‪⇔ ⎢z = j = − + i‬‬ ‫‪⎥ ∨ ⎢z = j = j = − − i‬‬
‫‪2‬‬
‫⎥‬
‫⎣‬ ‫‪2‬‬ ‫⎦ ‪2‬‬ ‫⎣‬ ‫‪2‬‬ ‫⎦ ‪2‬‬

‫‪.‬‬ ‫‪S = j, j‬‬ ‫} {‬ ‫وﺑﺎﻟﺘﺎﻟﻲ ﻓﺈن ‪:‬‬


‫ﻃﺮﻳﻘﺔ أﺧﺮى ﻹﻧﺠﺎز اﻟﺴﺆال ‪ 1 – I‬ﻣﻦ اﻟﺘﻤﺮﻳﻦ اﻟﺜﺎﻧﻲ ‪:‬‬
‫ﻧﻀﻊ )‪. m = p ( p 2 − 1‬ﻟﺪﻳﻨﺎ‪ . m = p ( p − 1)( p + 1) :‬إذن ‪ m‬هﻮ ﺟﺪاء ﺛﻼﺛﺔ أﻋﺪاد ﻣﺘﺘﺎﺑﻌﺔ ؛ إذن ‪ 3 / m :‬وﻣﻨﻪ )‪3 / p ( p − 1‬‬
‫‪2‬‬

‫وﺑﻤﺎ أن ‪ p‬أوﻟﻲ و ‪ p ≥ 5‬ﻓﺈن ‪ p > 3‬و ‪ 3‬ﻻ ﻳﻘﺴﻢ ‪ . p‬إذن ‪. p ∧ 3 = 1 :‬‬


‫‪⎪⎧3 / p ( p − 1) Gauss‬‬
‫‪2‬‬

‫]‪. p 2 ≡ 1[3‬‬ ‫⎨ ‪ .‬وهﺬا ﻳﻌﻨﻲ أن ‪:‬‬ ‫⇒‬ ‫وﻋﻠﻴﻪ ﻓﺈن ‪3 / ( p 2 − 1) :‬‬


‫‪⎪⎩ p ∧ 3 = 1‬‬
‫ﻃﺮﻳﻘﺔ أﺧﺮى ﻹﻧﺠﺎز اﻟﺴﺆال ‪ 1 – II‬ﻣﻦ اﻟﺘﻤﺮﻳﻦ اﻟﺜﺎﻧﻲ ‪:‬‬

‫ﻟﺪﻳﻨﺎ ‪ a ∧ 24 = 1‬و ` ∈ ‪. a‬‬


‫‪α1‬‬ ‫‪α2‬‬ ‫‪αr‬‬
‫‪ a = p1 p 2 ... p r‬ﺣﻴﺚ ‪ p1‬و ‪ p 2‬و‪ ...‬و ‪ p r‬أﻋﺪاد ﺻﺤﻴﺤﺔ ﻃﺒﻴﻌﻴﺔ أوﻟﻴﺔ و ‪ α1‬و ‪ α 2‬و‪ ...‬و ‪α r‬‬ ‫ﻟﻨﻔﻜﻚ ‪ a‬إﻟﻰ ﺟﺪاء ﻋﻮاﻣﻞ أوﻟﻴﺔ ‪:‬‬
‫‪-9-‬‬
‫أﻋﺪاد ﺻﺤﻴﺤﺔ ﻃﺒﻴﻌﻴﺔ ﻏﻴﺮ ﻣﻨﻌﺪﻣﺔ ‪.‬‬
‫ﻧﻌﻠﻢ أن اﻟﻘﻮاﺳﻢ اﻷوﻟﻴﺔ ﻟﻠﻌﺪد ‪ 24‬هﻲ ‪ 2‬و ‪ . ( 24 = 23 × 3 ) 3‬وﺑﻤﺎ أن ‪ a ∧ 24 = 1‬ﻓﺈن ‪ 2‬و ‪ 3‬ﻻ ﻳﻘﺴﻤﺎن ‪ . a‬أي ‪ 2‬و ‪ 3‬ﻻ ﻳﻮﺟﺪان ﻓﻲ‬
‫ﺗﻔﻜﻴﻚ ‪ . a‬وﻣﻨﻪ ﻓﺈن اﻷﻋﺪاد ‪ p1‬و ‪ p 2‬و‪ ...‬و ‪ p r‬أﻋﺪاد أوﻟﻴﺔ أآﺒﺮ ﻗﻄﻌﺎ ﻣﻦ ‪ . 3‬أي أآﺒﺮ ﻣﻦ أو ﻳﺴﺎوي ‪ . 5‬وﺣﺴﺐ ‪ , 1 – I‬ﻟﺪﻳﻨﺎ ‪:‬‬
‫]‪ ∀i ∈ {1, 2,..., r } : p i ≡ 1[3‬وﻣﻨﻪ ﻓﺈن ‪:‬‬
‫‪a 2 = p12α1 p 22α 2 ... p rα r = ( p1‬‬
‫‪2 α1‬‬
‫) ‪) (p‬‬
‫‪2 α2‬‬
‫‪2‬‬ ‫‪... ( p r‬‬
‫‪2 αr‬‬
‫)‬ ‫]‪⇒ a 2 ≡ 1α1 × 1α 2 × ... × 1α r [3‬‬
‫]‪⇒ a 2 ≡ 1[3‬‬
‫اﻟﻌﺪد اﻟﻌﻘﺪي ‪ j‬ﻳﺴﻤﻰ ﻋﺪد ﺟﺎآﻮﺑﻲ ) ‪ ( Nombre de Jacobi‬وﻳﺤﻘﻖ ﻣﺎ ﻳﻠﻲ ‪:‬‬ ‫ﻧﺘﻴﺠﺔ ‪:‬‬
‫‪2π‬‬
‫‪1‬‬ ‫‪3‬‬ ‫‪i‬‬ ‫⎤ ‪⎡ 2π‬‬
‫‪j4 = j‬‬ ‫‪j = − +i‬‬ ‫و ⎥ ‪= e 3 = ⎢1,‬‬ ‫‪j2 = j‬‬ ‫و‬ ‫و ‪j 3 =1‬‬ ‫‪1+ j + j 2 = 0‬‬
‫‪2‬‬ ‫‪2‬‬ ‫⎦ ‪⎣ 3‬‬
‫‪∀z ∈ ^ : ∃!( a , b ) ∈ \ 2 / z = a + bj‬‬
‫اﻷﻋﺪاد اﻟﻌﻘﺪﻳﺔ ‪ 1‬و ‪ j‬و ‪ j‬هﻲ اﻟﺠﺬور اﻟﺜﺎﻟﺜﺔ ﻟﻠﻮﺣﺪة وﺻﻮرهﺎ ﻓﻲ اﻟﻤﺴﺘﻮى اﻟﻌﻘﺪي هﻲ رؤوس ﻣﺜﻠﺚ ﻣﺘﺴﺎوي اﻷﺿﻼع ‪.‬‬
‫*** اﻧﺘﻬﻰ ***‬

‫ﻟﺘﻜﻦ ‪ a‬و ‪ b‬و ‪ c‬أﻋﺪادا ﻣﻦ *` ﺑﺤﻴﺚ ‪ ab = c 2 :‬و ‪. a ∧ b = 1‬‬ ‫ﺗﻤﺮﻳﻦ ‪:‬‬


‫‪ b = n 2‬و ‪ ∃(m , n ) ∈ `* × `* / a = m 2‬؟‬ ‫ﺑﻴﻦ أن ‪:‬‬
‫اﻟﺠﻮاب ‪ :‬ﻧﻀﻊ ‪ . d = a ∧ c‬إذن ‪ m ∧ n = 1 :‬و ‪ c = dn‬و ‪. ∃(m , n ) ∈ `* × `* / a = dm‬‬
‫ﻟﺪﻳﻨﺎ ‪ . ab = c 2 ⇔ bdm = d 2 n 2 ⇔ bm = dn 2 :‬وﻟﺪﻳﻨﺎ ‪. m ∧ n = 1 ⇔ m ∧ n 2 = 1 :‬‬
‫‪⎧⎪n 2 / bm Gauss 2‬‬
‫⎨ ‪.‬‬ ‫ﺑﻤﺎ أن ‪ bm = dn 2 :‬ﻓﺈن ‪⇒ n / b : (i ) :‬‬
‫‪⎪⎩m ∧ n = 1‬‬
‫‪2‬‬

‫ﻳﻜﻔﻲ أن ﻧﺒﻴﻦ أن ‪ b / n 2 :‬؟‬


‫ﻟﺪﻳﻨﺎ ‪ bm = n 2d :‬إذن ‪ . b / n 2d :‬ﻟﻬﺬا ﻳﻜﻔﻲ أن ﻧﺒﻴﻦ أن ‪ b ∧ d = 1 :‬؟‬
‫‪⎧δ / b‬‬ ‫‪⎧δ / b‬‬
‫⎨ وﻣﻨﻪ ﻓﺈن‪ . δ / a ∧ b :‬وﺑﻤﺎ أن ‪ a ∧ b = 1‬ﻓﺈن‪ δ /1 :‬إذن‬ ‫⎨ إذن ‪ :‬و‬ ‫ﻧﻀﻊ ‪ . δ = b ∧ d‬ﻟﺪﻳﻨﺎ ‪ :‬و‬
‫‪⎩δ / dm = a‬‬ ‫‪⎩δ / d‬‬
‫‪⎧b / dn 2 Gauss‬‬
‫⎨ ‪.‬‬ ‫‪ . δ = 1‬وﻣﻨﻪ ﻓﺈن‪ . b ∧ d = 1 :‬وﻋﻠﻴﻪ ﻓﺈن ‪⇒ b / n 2 : (ii ) :‬‬
‫‪⎩b ∧ d = 1‬‬
‫ﻣﻦ ) ‪ (i‬و ) ‪ , (ii‬ﻧﺴﺘﻨﺘﺞ أن ‪. b = n 2 :‬‬
‫وﻟﺪﻳﻨﺎ ‪ . bm = dn 2 ⇒ n 2 m = dn 2 ⇒ m = d :‬وﺑﻤﺎ أن ‪ a = dm :‬ﻓﺈن ‪a = m 2 :‬‬
‫‪∃(m , n ) ∈ `* × `* / a = dm‬‬ ‫و‬ ‫‪c = dn‬‬ ‫و‬ ‫‪m ∧ n =1‬‬ ‫ﺧﻼﺻﺔ ‪:‬‬
‫ﻣﺒﺮهﻨﺔ ‪: Wilson‬‬
‫ﻟﻴﻜﻦ ‪ p‬ﻋﺪدا ﺻﺤﻴﺤﺎ ﻃﺒﻴﻌﻴﺎ ‪ .‬ﻟﺪﻳﻨﺎ ‪:‬‬
‫‪ p‬أوﻟﻲ ⇔ ] ‪( p − 1)!+ 1 ≡ 0 [ p‬‬
‫ﻣﺒﺮهﻨﺔ ‪: Fermat‬‬
‫أ( ﻟﻴﻜﻦ ‪ p‬ﻋﺪدا أوﻟﻴﺎ ﻣﻮﺟﺒﺎ وﻟﻴﻜﻦ ‪ n‬ﻋﺪدا ﺻﺤﻴﺤﺎ ﻃﺒﻴﻌﻴﺎ ﻏﻴﺮ ﻣﻨﻌﺪم ﺑﺤﻴﺚ ‪:‬‬
‫] ‪. n p −1 ≡ 1[ p‬‬ ‫‪ . n ∧ p = 1‬ﻟﺪﻳﻨﺎ ‪:‬‬
‫ب( ﻟﻜﻞ ﻋﺪد أوﻟﻲ ﻣﻮﺟﺐ ‪ p‬وﻟﻜﻞ ﻋﺪد ﺻﺤﻴﺢ ﻃﺒﻴﻌﻲ ﻏﻴﺮ ﻣﻨﻌﺪم ‪ , a‬ﻟﺪﻳﻨﺎ ‪:‬‬
‫]‪ap ≡ a[p‬‬

‫‪- 10 -‬‬
. (E ) : ( x + 1) = 9 + 5y : ¢2
2
(1
: . (E ) (x , y ) (
( x + 1) = 9 + 5y ⇒ ( x + 1) ≡ 4 [5]
2 2

⇒ ( x + 1) − 4 ≡ 0 [5]
2

⇒ ( x + 1) − 4 ≡ 0 [5]
2

⇒ ( x + 1 − 2 )( x + 1 + 2 ) ≡ 0 [5]
⇒ ( x − 1)( x + 3) ≡ 0 [5]
⇒ 5 / ( x − 1) 5 / ( x + 3)
⇒ x ≡ 1 [5 ] x ≡ −3 [5 ]
⇒ x ≡ 1 [5 ] x ≡ 2 [ 5]
: (E ) ¢2 (
: . x ≡2 [5 ] x ≡1 [5 ] : , (E ) (x , y )
x ≡1 [5] ⇒ ∃k ∈ ¢ : x = 1 + 5k
⇒ ∃k ∈ ¢ : ( 2 + 5k ) = 9 + 5 y ; ( x + 1 = 2 + 5k )
2

⇒ ∃k ∈ ¢ : 4 + 20k + 25k 2 = 9 + 5 y
⇒ ∃k ∈ ¢ : 5 y = −5 + 20k + 25k 2
⇒ ∃k ∈ ¢ : y = −1 + 4k + 5k 2

x ≡2 [5 ] ⇒ ∃k ∈ ¢ : x = 2 + 5k
⇒ ∃k ∈ ¢ : ( 3 + 5k ) = 9 + 5 y ; ( x + 1 = 3 + 5k )
2

⇒ ∃k ∈ ¢ : 9 + 30k + 25k 2 = 9 + 5 y
⇒ ∃k ∈ ¢ : 5 y = 30k + 25k 2
⇒ ∃k ∈ ¢ : y = 6 k + 5k 2
: (E ) , k ∈¢ ( 2 + 5k , 6k + 5k ) (1 + 5k , −1 + 4k + 5k )
2 2

S = {(1 + 5k , −1 + 4k + 5k ) ; ( 2 + 5k , 6k + 5k ) / k ∈ ¢}
2 2

: , k ∈¢ (2
5k + 4k − 1
2
5k + 1
. 5k + 4k − 1 = k ( 5k + 1) + ( 3k − 1) :
2
e 5k 2 + k
k
3k − 1
( 5k 2
+ 4k − 1) ∧ ( 5k + 1) = ( ( 5k 2
)
+ 4k − 1) − k ( 5k + 1) ∧ ( 5k + 1) :
= ( 3k − 1) ∧ ( 5k + 1)

-1-
= ( 3k − 1) ∧ ( ( 5k + 1) − ( 3k − 1) )
= ( 3k − 1) ∧ ( 2k + 2 )
= ( ( 3k − 1) − ( 2k + 2 ) ) ∧ ( 2k + 2 )
= ( k − 3) ∧ ( 2k + 2 )
= ( k − 3) ∧ ( ( 2k + 2 ) − 2 ( k − 3 ) )
( 5k 2
+ k − 1) ∧ ( 5k + 1) = ( k − 3) ∧ 8
∀k ∈ ¢ : ( 5k 2
+ 4k − 1) ∧ ( 5k + 1) = ( k − 3) ∧ 8 :
: ¥2 (3
121 (x )
= 59
(y )


(*) : x ∧ y = 8
 x ≡1 [5]

:
 x + 2x + 1
2
= 5y + 9

( *) ⇔  x ∧y = 8
 x ≡1 [5 ]

( x + 1)2 = 9 + 5y

⇔  x ∧y = 8
 x ≡1 [5]

(x , y ) = (1 + 5k , −1 + 4k + 5k 2 ) ou (x , y ) = ( 2 + 5k , 6k + 5k 2 ) / k ∈ ¢

⇔  x ∧y = 8
 x ≡1 [5]



 (x , y ) = (1 + 5k , −1 + 4k + 5k ) 2
/k ∈¢

⇔ (1 + 5k ) ∧ ( −1 + 4k + 5k ) =
2
8

 x ≡1 [5]
 (x , y ) =

(1 + 5k , −1 + 4k + 5k ) 2
/ k ∈¢
⇔ ( k − 3 ) ∧ 8 = 8
 x ≡1 [5]

 (x , y ) =

(1 + 5k , −1 + 4k + 5k ) 2
/ k ∈¢
⇔ 8 / ( k − 3)
 x ≡1 [5 ]

( x , y ) = (1 + 5k , −1 + 4k + 5k ) / k ∈¢
2

⇔ 
 ∃h ∈ ¢ / k = 3 + 8h
1 3
x ∈ ¥ ⇒ x ≥ 0 ⇒ 1 + 5k ≥ 0 ⇒ k ≥ − ⇒ k ≥ 0 ⇒ 3 + 8h ≥ 0 ⇒ h ≥ − ⇒ h ≥ 0 ⇒ h ∈ ¥ :
5 8
( *) (
⇔ ( x , y ) = 1 + 5 ( 3 + 8h ) , −1 + 4 ( 3 + 8h ) + 5 ( 3 + 8h )
2
) / h ∈¥ :

(*) ⇔ ( x , y ) = (16 + 40h , 56 + 272h + 320h 2 ) / h ∈ ¥

-2-
{
S = (16 + 40h , 56 + 272h + 320h 2 ) / h ∈ ¥ } : ¥2 ( *)

x2 y2
(C m ) : + = 1 ; m ∈ ¡ − {2,10}
10 − m 2 − m
. 10 − m > 0 ⇔ m < 10 2−m > 0 ⇔ m < 2 : (1 – I
2 2
x y
. (C m ) (C m ) : + =1 : . 2−m > 0 : , m <2
( ) ( )
2 2
10 − m 2−m
x2 y2
(C m ) : − =1 : : 10 − m > 0 2−m < 0 : , 2 < m < 10
( ) ( )
2 2
10 − m m −2
. (C m )
x2 y2
( ) 1= − <0 : 2 − m < 0 10 − m < 0 : , m > 10
( ) ( )
2 2
10 − m m −2
. (C m ) = ∅ :
x2 y2
( a >b ) b = 2−m a = 10 − m : (C m ) : + =1 : m <2 (2
a2 b 2
0  0   − 10 − m   10 − m   0
B ′  B  A′  A  O  : (C m )
− 2−m   2−m  0  0   0
 −2 2  2 2 
. F ′  F  (C m ) c = b 2 − a2 = (10 − m ) − ( 2 − m ) = 8=2 2
0  0 
x2 y2
. b = m −2 a = 10 − m : (C m ) :
− = 1: , 2 < m < 10
a2 b 2
 − 10 − m   10 − m   0
: A′  A  O  : (C m )
0  0   0
 −2 2  2 2 
: F ′  F  (C m ) , c = a2 + b 2 = ( m − 2 ) + (10 − m ) = 8=2 2
0  0 
m −2 m −2
. ( D )′ : y = − x ⇔ y = −
b b
x (D ) : y = x ⇔ y = x
a 10 − m a 10 − m
π
:α= (C 1 ) (3
4

A′ A

 −2 2  2 2 
F ′  F 
B′
0  0 

-3-
π
. 0 <α < : (E ) : z 2 − ( 6 cos(α ) ) z + 1 + 8 cos 2 (α ) : £ - II
2
: (E ) (1

∆′ = ( −3cos(α ) ) − (1 + 8cos2 (α ) ) = 9 cos2 (α ) − 1 − 8cos 2 (α ) = cos2 (α ) − 1 = − sin 2 (α ) = ( i sin(α ) )


2 2

. z = 3cos(α ) − i sin(α ) z = 3cos(α ) + i sin(α ) : (E )


z 1 = 3cos(α ) + i sin(α )  π
Im ( z 1 ) > 0 ⇒ : α ∈  0,  ⇒ [ cos(α ) > 0 sin(α ) > 0] :
z 2 = 3cos(α ) − i sin(α )  2
S = {3cos(α ) + i sin(α ),3cos(α ) − i sin(α )} :
. M 1 ( z 1 = 3cos(α ) + i sin(α ) ) ∈ (C 1 ) ( (2
: . y = ℑm ( z 1 ) = sin(α ) x = ℜe ( z 1 ) = 3cos(α ) :
x 2 y 2 ( 3cos(α ) ) ( sin(α ) )
2 2

. M 1 ∈ (C 1 ) : . 2 + 2 = + = cos 2 (α ) + sin 2 (α ) = 1
3 1 9 1
: (

(T1 )

P1 M1

(C 1 ) P2

(T 2 )
π
* α= *
4
x0 
: P (C 1 ) (T ) . ( x 0 , y 0 ) ∈ ¡2 (C 1 ) P 
y0
uuuuur  3cos(α )  r  −9y 0  xx 0 yy 0
: . OM 1  : (T ) u  +. = 1 ⇔ xx 0 + 9 yy 0 = 9
 sin(α )  x0  9 1
r uuuuur
u OM 1 ⇔ (OM 1 ) P (T )
uuuuur r
(
det OM 1 ,u = 0 ⇔ )
3cos(α ) −9 y 0
=0 ⇔
sin(α ) x0
3cos(α )x 0 + 9 sin(α ) y 0 = 0 ⇔
cos(α )x 0 + 3sin(α ) y 0 = 0 ⇔
cos(α )x 0 = −3 y 0 sin(α ) ⇔

-4-
sin(α )
x 0 = −3 y 0 ⇔
cos(α )
:
x0  x 2
y 2
P   ∈ (C 1 ) ⇔ + 0
2
=1 0
2
y0 3 1
⇔ x 02 + 9 y 02 = 9
sin 2 (α ) sin(α )
⇔ 9 y 02 + 9 y 02 = 9 ; x 0 = −3 y 0
cos (α )
2
cos(α )
sin(α )
⇔ sin 2 (α ) y 02 + cos 2 (α ) y 02 = cos2 (α ) ; x 0 = −3 y 0
cos(α )
sin(α )
⇔ y 02 = cos 2 (α ) ; x 0 = −3 y 0
cos(α )
sin(α )
⇔ y 0 = ± cos(α ) ; x 0 = −3 y 0 = m3sin(α )
cos(α )
 3sin(α )   −3sin(α ) 
P2   P1   :
 − cos(α )   cos(α ) 
 3sin(α )   −3sin(α ) 
(C 1 ) (T 2 ) (T1 ) (C 1 )P2   P1  
 − cos(α )   cos(α ) 
(T1 ) : −3sin(α )x + 9 cos(α ) y − 9 = 0 : (OM 1 ) ,
(T 2 ) : 3sin(α )x − 9 cos(α ) y −9 = 0
 −3sin(α ) 
P1  ; (T1 ) : − sin(α )x + 3cos(α ) y − 3 = 0
 cos(α ) 
(T1 ) P (T 2 ) P (OM 1 ) P2 ∈ (C 1 ) P1 ∈ (C 1 ) :
 3sin(α ) 
P2  ; (T 2 ) : sin(α )x − 3cos(α ) y − 3 = 0
 − cos(α ) 
.
uuuuur uuuuur
OM 2 ( 3cos(α ) − i sin(α ) ) OM 1 ( 3cos(α ) + i sin(α ) ) : (
uuur uuur
OP2 ( 3sin(α ) − i cos(α ) ) OP1 ( −3sin(α ) + i cos(α ) )
OM 12 + OP12 = 9 cos2 (α ) + sin 2 (α ) + 9sin 2 (α ) + cos 2 (α ) = 10 ( cos 2 (α ) + sin 2 (α ) ) = 10 :
OM 22 + OP22 = 9 cos2 (α ) + sin 2 (α ) + 9sin 2 (α ) + cos 2 (α ) = 10 ( cos 2 (α ) + sin 2 (α ) ) = 10 :
OM 12 + OP12 = OM 22 + OP22 :

. n ≥ 20 : n ∈¥
N B

n − 10 10

: , 0≤k ≤n . n .
. k = pk

-5-
k n −k n −k
 C 101   C 101 
k
 10   10 
: p k = C  1  1 − 1 
k
n = C   1 − 
k
n : . (1
 Cn   Cn  n   n 
n −k
 10   n − 10 
k

pk = C   k
n 
n   n 
p k +1
. ∀k ∈ {0,1,..., n − 1} : uk = : (2
pk
k +1 n − ( k +1)
k +1 10   n − 10  A nk +1
C    
p k +1 n
 n   n  C nk +1 10 1 ( k + 1)! 10
uk = = = k × × = × : (
pk k  10   n − 10 
k n −k
Cn n n − 10 A nk n − 10
Cn     n
n   n  k!
n × (n − 1) × ... × (n − k ) k! 10 n −k 10
= × × = ×
n × (n − 1) × ... × (n − k + 1) (k + 1)! n − 10 k + 1 n − 10
: (
n −k 10
uk ≥ 1 ⇔ × ≥1
k + 1 n − 10
⇔ 10 ( n − k ) ≥ ( k + 1)( n − 10 )
⇔ 10n − 10k/ ≥ nk − 10k/ + n − 10
⇔ 0 ≥ nk + n − 10 − 10n
⇔ 0 ≥ nk − 10 − 9n
⇔ 10 + 9n ≥ nk
10 + 9n 10
⇔ k ≤ = 9+
n n
1 1 1 10 1
: k ∈¥ : uk ≥ 1 ⇔ k ≤ 9 + : n ≥ 20 ⇒ ≤ ⇒ ≤ :
2 n 20 n 2
(i ) : u k ≥1 ⇔ 0 ≤ k ≤ 9
:
n −k 10
uk ≤ ⇔ × ≤1
k + 1 n − 10
⇔ 10n − 10k/ ≤ nk + n − 10k/ − 10
⇔ 9n ≤ nk − 10
⇔ 9n + 10 ≤ nk
10
⇔ 9+ ≤ k
n
⇔ 10 ≤ k ≤ n − 1
10 1
( n ≥ 20 ⇒ 0 < ≤ k ∈ {0,1,..., n − 1} k ∈ ¥ : )
n 2
( ii ) : u k ≤ 1 ⇔ 10 ≤ k ≤ n − 1 :
p1
u0 ≥ 1⇒ ≥ 1 ⇒ p1 ≥ p 0 : (
p0
p
u1 ≥ 1 ⇒ 2 ≥ 1 ⇒ p 2 ≥ p1
p1
M M M M M

-6-
p10
( i ) : p10 ≥ p 9 ≥ ... ≥ p 2 ≥ p1 ≥ p 0 : u9 ≥ 1 ⇒ ≥ 1 ⇒ p10 ≥ p 9
p9
p
u10 ≤ 1 ⇒ 11 ≤ 1 ⇒ p11 ≤ p10 :
p10
p
u11 ≤ 1 ⇒ 12 ≤ 1 ⇒ p12 ≤ p11
p11
M M M M M M M M
p
( ii ) : p n ≤ p n −1 ≤ ... ≤ p12 ≥ p11 ≥ p10 : u n −1 ≤ 1 ⇒ n ≤ 1 ⇒ p n ≤ p n −1
p n −1
. ∀k ∈ {0,1, 2,..., n } : p k ≤ p10 : ( ii ) ( i )
: {0,1, 2,..., n } k pk M
n −10
 10   n − 10  1010 (n − 10) n −10 n ! 1010 (n − 10)n −10
10
n!
M = p10 = C n10     = × 10 × = × ×
n   n  10!(n − 10)! n n n −10 n n 10! (n − 10)!

∀x ∈ ¡ : f (x ) = (1 + x )e −2 x
lim f (x ) = lim (1 + x )e −2 x = lim e −2 x + xe −2 x = 0 + 0 = 0 : ( (1 – I
x →+∞ x →+∞ x →+∞

1
lim xe 2 x = lim − te t = 0 x → +∞ ⇒ t → −∞ : t = −2 x
x →+∞ t →−∞ 2

: x → −∞ ⇒ t → +∞ : t = −2 x
 1 
. lim f (x ) = lim (1 + x )e −2 x = lim  1 − t  e t = −∞ × +∞ = −∞
x →−∞ x →−∞ t →+∞
 2 
. y =0 +∞ (C ) lim f (x ) = 0
x →+∞
–(
f (x ) 1   2 
lim = lim  + 1 e −2 x = lim  − + 1 e t = +∞ lim f (x ) = −∞ -
x →−∞ x 
x →−∞ x
 t →+∞
 t  x →−∞

. −∞ (C ) . t = −2x x →−∞


→ +∞ :
. f ′(x ) = (1 + x )′e −2 x + (1 + x )(−2)e −2 x = (1 − 2(1 + x ))e −2 x = −(1 + 2x )e −2 x : , x ∈¡ (2
: ¡ f . 1 + 2x ¡ f ′(x )

x 1
−∞ − +∞
2
f ′(x ) + 0 -
1
f (x ) e
2

−∞ 0
f ′′(x ) = −(1 + 2x )′e −2 x − (1 + 2x )(−2)e −2 x = (−2 + 2(1 + 2x ))e −2x = 4xe −2 x : , x ∈¡ ( (3
x −∞ 0 +∞
′′
f (x ) - 0 +

(C ) 0
(C ) A  (C )
1 
. A (0,1) (C )
-7-
r ur
(
: O ,i , j ) (C ) (

O y =0

(C )

: . f ′′(x ) = 4xe −2 x f ′(x ) = −(1 + 2x )e −2 x f (x ) = (1 + x )e −2 x : ( (4


f ′′(x ) + 3f ′(x ) + 2f (x ) = 4xe − 3(1 + 2x )e + 2(1 + x )e
−2 x −2 x −2 x
= [ 4x − 3(1 + 2x ) + 2(1 + x ) ]e −2 x

= [ 4x − 3 − 6x + 2 + 2x ) ]e −2 x = −e −2 x
. ( E ) : y ′′ + 3 y ′ + 2 y = −e −2 x : f
. ( E ′ ) : y ′′ + 3 y ′ + 2 y = 0 (
∆ = 32 − 4 ×1× 2 = 9 − 8 = 1 . ( F ) : r 2 + 3r + 2 = 0 : ( E ′)
−3 − 1 −3 + 1
. r2 = = −2 r1 = = −1 : (F )
2 2
. y = αe − x + β e −2 x /(α , β ) ∈ ¡ 2 : ( E ′)
: . y = αe − x + β e −2 x + f (x ) /(α , β ) ∈ ¡ 2 : (E )
y = αe − x + β e −2 x + (1 + x )e −2 x /(α , β ) ∈ ¡ 2
. n ∈ ¥* - II
. x =n (C ) : An
n
n n n n  1  1 1
A n = ∫ f (x )dx = ∫ (1 + x )e −2 x dx = ∫ e −2 x dx + ∫ xe −2 x dx =  − e −2 x  + I = − e −2 n + I : (1
0 0 0 0
 2 0 2 2
 1 −2 x
u (x ) = − e u ′(x ) = e −2 x n
: .  2 : .  : . I = ∫ xe −2 x dx :
v (x ) = x
v ′(x ) = 1
0

n
 1  1 n
I = ∫ u ′(x ).v (x )dx = [u (x ).v (x ) ]0 − ∫ u (x ).v ′( x )dx =  − xe −2 x  + ∫ e −2 x dx
n n n

0 0
 2 0 2 0
n
1 1 
= − ne −2 n +  − e −2 x  = − ne −2 n − (e −2 n − 1)
1 1 1
2 2 2 0 2 4
 1 1 1
− e − ne − (e − 1) =  − − n −  e −2 n + = − ( 3 + 2n ) e −2 n +
1 1 −2 n 1 −2 n 1 −2 n 3 1 3
An = :
2 2 2 4  2 2 4 4 4 4
: (2

-8-
1 3 13+m  3
. m = 2n 
n →+∞
→ +∞ : lim A n = lim − ( 3 + 2n ) e −2n + = mlim −  m +
n →+∞ n →+∞ 4 4 →+∞ 4 e  4
 
3 1 3 1  3 3
lim A n =
n →+∞
(u .a.) . lim A n = lim −  m + m  + =
n →+∞ m →+∞ 4 e
:
4  e  4

4
 m 
∀n ∈ ¥* : u n = n ∫ [ f (x )] dx
1 n
: – III
0

: . dt = ndx x =1⇔ t = n x =0⇔t =0 : . t = nx (1


n
n  
n
n   t 
n
  t  −2t
t
t −2
u n = ∫ f    dt = ∫  1 + ∫
n
e  dt = 1 +  e dt
n
0
  n  0
 n  
0
 n
1 1 − 2u + u 2 (u − 1)2 1
: . − (2 − u ) = = ≥0 : ≤1 :
. 1≤u ≤ 2 . u ∈ [1, 2] ( (2
u u u u
1 1
∀u ∈ [1, 2 ] : 2 −u ≤ ≤ 1 : . 2 −u ≤
u u
1
. u ∈ [1, 2] ⇔ t ∈ [ 0,1] : .u = t +1 t = u −1 : . ∀u ∈ [1, 2] : 2 − u ≤ ≤1 : (
u
1 1
∀t ∈ [ 0,1] : 1 − t ≤ ≤1 : . ∀t ∈ [ 0,1] : 2 − (t + 1) ≤ ≤1:
t +1 t +1
u
1  t2
∀u ∈ [ 0,1] : ∫ (1 − t )dt ≤ ∫ dt ≤ ∫ dt ⇒ ∀u ∈ [ 0,1] : t −  ≤ [ ln(t + 1)]0 ≤ u − 0 :
u u u u
0 0 t +1 0
 2 0
u2
⇒ ∀u ∈ [ 0,1] : u − ≤ ln(u + 1) ≤ u
2
u2 x x
: .u−
≤ ln(u + 1) ≤ u : . u = ∈ [ 0,1] x ∈ [ 0, n ] .u= :
2 n n
x2  x  x x2  x  x
∀n ∈ ¥* ; ∀x ∈ [ 0, n ] ; x − 2 ≤ n ln  1 +  ≤ x : − ≤ ln 1 + ≤
2n  n n 2n  n  n
n
 t 
. t ∈ [ 0, n ] . n ∈ ¥*
n
: . u n = ∫  1 +  e −2t dt : ( (3
0
 n
 t    t n 
n ln 1 +  ≤ t ⇒ ln   1 +   ≤ t
 n  n  
 
n
 t 
⇒ 1 +  ≤ e
t

 n
n
 t  −2t −t
⇒ 1 + n  e ≤ e
 
n
 t  −2t

n
1 +  e dt ≤ ∫0 e dt
n
−t

0
 n

u n ≤ ∫ e −t dt
n

0

n
∀n ∈ ¥* : u n ≤ ∫ e − x dx :
0

: . n ∈ ¥* (

-9-
t2 n
t2  t  t−  t 
t− ≤ n ln 1 +  ⇒ e 2n
≤ 1 + 
2n  n  n
t2 n
−t −  t 
⇒ e 2n
≤ 1 +  e −2t
 n
t2
−t −

n
⇒ e 2n
dt ≤ u n
0

t2
−t −
: ∀t ∈ [ 0, n ] : e 2n
≥0 : . n ≥ 1⇒ n 2 ≥ 1⇒ n3 ≥ n ⇒ n ≥ 3 n :
t2 t2
3
n −t − n −t −
: ∫
0
e 2n
dt ≤ ∫ e
0
2n
dt

0 ≤t ≤ 3 n ⇒ 0 ≤t 2 ≤ 3 n2
⇒ − 3 n 2 ≤ −t 2 ≤ 0
t2 1
⇒ − 3 ≤− ≤0
2 n 2n

1 t2

⇒ e 23 n
≤e 2n
≤1

1
−t t2
−t −
⇒ e 23 n
≤e 2n


1
−t t2
3 3
−t −

n
dt ≤ ∫ e
n
⇒ e 23 n 2n
dt
0 0


1 t2 t2
3 3
−t − −t −

n
e dt ≤ ∫ e
n
dt ≤ ∫ e
n
−t
⇒ e 23 n 2n 2n
dt ≤ u n
0 0 0
1
− 3

(*) : ∀n ∈ ¥* :
n n
e 23 n
∫ 0
e − x dx ≤ u n ≤ ∫ e − x dx
0
:
3 3

( *) ∫
n n

n n
e − x dx =  −e − x  = 1 − e − e − x dx =  −e − x  = 1 − e − n :
3
n
: . : . (
0 0 0 0

( ) (1 − e ) ≤ u
1 1
− −
1−e − =1 lim 1 − e − n = 1 : . ∀n ∈ ¥* : e −3 n
≤ 1 − e −n
3
23 n n 23 n
lim e n
n →+∞ n →+∞

lim u n = 1
n →+∞
: (u n ) : ,

. a ∈ ]0,1[ (4
( f (1) = 2e −2 ≥ 0 ) : . [ 0, +∞[ f (
∀x ∈ [a ,1] : a ≤ x ≤ 1 ⇒ f (1) ≤ f (x ) ≤ f (a )
⇒ 0 ≤ ( f (x ) ) ≤ ( f (a ) )
n n

∫ n ( f (x ) ) dx ≤ n ( f (a ) )
1 1
⇒ ∫ dx
n n
a a

∫ n ( f (x ) ) dx ≤ n (1 − a )( f (a ) )
1

n n
a

. ∀n ∈ ¥* : 0 ≤ ∫ n ( f (x ) ) dx ≤ n (1 − a )( f (a ) )
1 n n
: (
a

. lim n (1 − a )( f (a ) ) = lim (1 − a ) ne n ln ( f (a ))
n

n →+∞ n →+∞

. 0 < a < 1 ⇒ f (1) < f (a ) < f (0) ⇒ 0 < 2e −2


< f (a ) < 1 ⇒ ln ( f (a ) ) < 0 :
1− a 1− a
lim n (1 − a ) [ f (a ) ] = lim × n ln ( f (a ) ) e n ln ( f (a )) = lim × xe x = 0 :
n

n →+∞ n →+∞ ln ( f (a ) ) x →−∞ ln ( f (a ) )

- 10 -
. x 
n →+∞
→ −∞ : ln ( f (a ) ) < 0 x = n ln ( f (a ) ) :

∫ n [f (x )]
1
dx = 0 . lim 0 = 0 :
n
lim :
n →+∞ a n →+∞

: (

∫ n [f (x )]
1
lim u n = 1 ⇒ dx = 1
n
lim
n →+∞ n →+∞ 0

∫ n [ f (x ) ] dx + ∫ n [ f (x ) ] dx = 1
a 1

n n
lim
n →+∞ 0 a

∫ n [f (x )]
a
⇒ dx = 1
n
lim
n →+∞ 0

∫ n [ f (x )]
1
dx = 0 :
n
. lim
n →+∞ a

∀ a ∈ ]0,1[ : ∫ n [f ( x ) ] dx = 1
a n
lim :
n → +∞ 0

- 11 -
‫‪1/3‬‬ ‫اﻟﺼﻔـــــﺤﺔ‬ ‫اﻻﻣﺘﺤـــﺎن اﻟﻮﻃﻨﻲ اﻟﻤﻮﺣﺪ ﻟﻨﻴﻞ ﺷﻬﺎدة‬ ‫اﻟﻤﻤﻠﻜﺔ اﻟﻤﻐﺮﺑﻴﺔ‬
‫‪ 4‬ﺳﺎﻋﺎت‬ ‫ﻣﺪة اﻹﻧﺠﺎز‬ ‫اﻟﺒﻜﺎﻟـــــــــﻮرﻳﺎ‬ ‫وزارة اﻟﺘﺮﺑﻴﺔ اﻟﻮﻃﻨﻴﺔ‬
‫اﻟﺪورة اﻟﻌﺎدﻳﺔ‪ :‬ﻳﻮﻧﻴﻮ ‪2004‬‬ ‫واﻟﺸﺒﺎب‬
‫اﻟﻤﺎدة‪ :‬اﻟﺮﻳﺎﺿﻴﺎت‬
‫‪10‬‬ ‫اﻟﻤﻌـﺎﻣـــﻞ‬ ‫اﻟﺸﻌﺒﺔ‪ :‬ﻋﻠﻮم رﻳﺎﺿﻴﺔ)أ( و)ب(‬
‫ﻳﺴﻤﺢ ﺑﺎﺳﺘﻌﻤﺎل ﺣﺎﺳﺒﺔ ﻏﻴﺮ ﻗﺎﺑﻠﺔ ﻟﻠﺒﺮﻣﺠﺔ‬

‫اﻟﺘﻤﺮﻳﻦ‪3) 1‬ﻧﻘﻂ(‬
‫‪ -1‬ﻟﻴﻜﻦ ‪ n‬ﻋﺪدا ﺻﺤﻴﺤﺎ ﻃﺒﻴﻌﻴﺎ‬
‫أ( ﺑﻴﻦ أ ﻧﻪ اذا آﺎن ‪ n‬ﻓﺮدﻳﺎ ﻓﺎن ]‪n ≡ 1 [8‬‬
‫‪2‬‬

‫‪n2 ≡ 4‬‬ ‫]‪[ 8‬‬ ‫‪ n 2 ≡ 0‬أو‬ ‫] ‪[8‬‬ ‫ب( ﺑﻴﻦ أ ﻧﻪ اذا آﺎن ‪ n‬زوﺟﻴﺎ ﻓﺎن‬
‫‪ -2‬ﻟﻴﻜﻦ ‪ a‬و ‪ b‬و ‪ c‬أﻋﺪادا ﺻﺤﻴﺤﺔ ﻃﺒﻴﻌﻴﺔ ﻓﺮدﻳﺔ‪.‬‬
‫أ( ﺑﻴﻦ أن ‪ a 2 + b 2 + c 2‬ﻟﻴﺲ ﻣﺮﺑﻌﺎ آﺎﻣﻼ )أي ﻟﻴﺲ ﻣﺮﺑﻊ ﻟﻠﻌﺪد ﺻﺤﺒﺢ ﻃﺒﻴﻌﻲ(‬
‫ب( ﺑﻴﻦ أن ]‪2 ( ab + bc + ac ) ≡ 6 [8‬‬
‫)ﻳﻤﻜﻦ ﻣﻼﺣﻈﺔ ‪( ( a + b + c ) = a 2 + b 2 + c 2 + 2ab + abc + 2ac‬‬
‫‪2‬‬

‫ج( اﺳﺘﻨﺘﺞ أن ) ‪ 2 ( ab + bc + ac‬ﻟﻴﺲ ﻣﺮﺑﻌﺎ آﺎﻣﻼ‪.‬‬


‫د( ﺑﻴﻦ أن ‪ ab + bc + ac‬ﻟﻴﺲ ﻣﺮﺑﻌﺎ آﺎﻣﻼ‪.‬‬

‫اﻟﺘﻤﺮﻳﻦ‪3) 2‬ﻧﻘﻂ(‬
‫‪‬‬ ‫‪1 ‬‬ ‫‪1 ‬‬
‫‪a‬‬ ‫‪a − ‬‬
‫‪3‬‬ ‫‪a ‬‬
‫‪Ma = ‬‬ ‫ﻟﺘﻜﻦ ‪ E‬ﻣﺠﻤﻮﻋﺔ اﻟﻤﺼﻔﻮﻓﺎت اﻟﺘﻲ ﺗﻜﺘﺐ ﻋﻠﻰ ﺷﻜﻞ‬
‫‪‬‬ ‫‪1‬‬ ‫‪‬‬
‫‪0‬‬ ‫‪‬‬
‫‪‬‬ ‫‪a‬‬ ‫‪‬‬
‫‪‬‬ ‫‪1 ‬‬ ‫‪1 ‬‬
‫‪ a‬‬ ‫‪a − ‬‬
‫‪Na = ‬‬ ‫‪3‬‬ ‫و ‪ F‬ﻣﺠﻤﻮﻋﺔ اﻟﻤﺼﻔﻮﻓﺎت اﻟﺘﻲ ﺗﻜﺘﺐ ﻋﻠﻰ ﺷﻜﻞ ‪a  ‬‬
‫‪ −a 3‬‬ ‫‪−a‬‬ ‫‪‬‬
‫‪‬‬ ‫‪‬‬
‫ﺣﻴﺚ ∈ ‪. a‬‬
‫∈ ) ‪∀ ( a; b‬‬ ‫‪*2‬‬
‫‪M a × M b = M ab‬‬ ‫‪ -1‬أ( ﺑﻴﻦ أن‬
‫ﻧﺤﻮ ‪ E‬ﺑﺤﻴﺚ ‪ϕ ( a ) = M a‬‬ ‫*‬
‫ب( ﻟﻴﻜﻦ ‪ ϕ‬اﻟﺘﻄﺒﻴﻖ اﻟﻤﻌﺮف ﻣﻦ‬
‫)× ;‪( E‬‬ ‫ﻧﺤﻮ‬ ‫(‬ ‫*‬
‫)‬
‫ﺑﻴﻦ أن ‪ ϕ‬ﺗﺸﺎآﻞ ﻣﻦ ×;‬
‫)× ;‪( E‬‬ ‫اﺳﺘﻨﺘﺞ اﻟﺒﻨﻴﺔ اﻟﺠﺒﺮﻳﺔ ﻟـ‬
‫∈ ) ‪∀ ( a; b‬‬ ‫‪*2‬‬
‫‪ -2‬أ( ﺑﻴﻦ أن ‪N a × Nb = Mb‬‬
‫‪a‬‬
‫ب( ﻧﻀﻊ ‪ . G = E ∪ F‬ﺑﻴﻦ أن ) × ;‪ ( G‬زﻣﺮة‬
‫ج( هﻞ ) × ;‪ ( G‬زﻣﺮة ﺗﺒﺎدﻟﻴﺔ؟‬
‫اﻟﺘﻤﺮﻳﻦ‪ 3.50) 3‬ن(‬
‫‪2‬‬
‫اﻟﻤﻌﺎدﻟﺔ ‪z + z + 1 = 1‬‬ ‫‪ -1‬ﺣﻞ ﻓﻲ‬
‫‪2π‬‬ ‫‪2π‬‬
‫‪θ ≠−‬‬ ‫≠‪θ‬و‬ ‫‪ -2‬ﻟﻜﻞ ﻋﺪد ﻋﻘﺪي ‪ z‬ﺣﻴﺚ ) ‪ z = eiθ = cos (θ ) + i sin (θ‬ﻣﻊ ‪ −π ≤ θ ≤ π‬و‬
‫‪3‬‬ ‫‪3‬‬
‫‪1‬‬
‫‪z' = 2‬‬ ‫ﻧﻀﻊ‬
‫‪z + z +1‬‬
‫أ( ﺗﺤﻘﻖ أن ) ‪1 + z + z 2 = z (1 + z + z‬‬
‫ﺻﻔﺤﺔ ‪2/3‬‬
‫ب( أﺟﺴﺐ ﻣﻌﺒﺎر وﻋﻤﺪة ' ‪ z‬ﺑﺪﻻﻟﺔ ‪θ‬‬
‫∈ ) ‪( x; y‬‬ ‫‪2‬‬
‫ج( ﻧﻀﻊ ‪ z ' = x + iy‬ﺣﻴﺚ‬
‫ﺑﻴﻦ أن ) ‪x 2 + y 2 = (1 − 2 x‬‬
‫‪2‬‬

‫د( اﺳﺘﻨﺘﺞ أن اﻟﻨﻘﻄﺔ ‪ M‬ذات اﻟﻠﺤﻖ ' ‪ z‬ﺗﻨﺘﻤﻲ اﻟﻰ هﺬﻟﻮل ﻳﺘﻢ ﺗﺤﺪﻳﺪ ﻣﺮآﺰﻩ و رأﺳﻴﻪ و ﻣﻘﺎرﺑﻴﻪ‪.‬‬
‫اﻟﺘﻤﺮﻳﻦ‪10) 4‬ﻧﻘﻂ(‬
‫‪−x‬‬
‫‪e‬‬
‫= )‪f ( x‬‬ ‫‪ (I‬ﻧﻌﺘﺒﺮ ‪ f‬اﻟﺪاﻟﺔ اﻟﻤﻌﺮﻓﺔ ﻋﻠﻰ * ﺑـ‪:‬‬
‫‪x‬‬
‫‪ -1‬أﺣﺴﺐ ﻧﻬﺎﻳﺎت ‪ f‬ﻋﻨﺪ ﻣﺤﺪات ﻣﺠﻤﻮع ﺗﻌﺮﻳﻔﻬﺎ‬
‫‪ -2‬أدرس ﺗﻐﻴﺮات ‪f‬‬
‫‪ -3‬ﻟﻴﻜﻦ ) ‪ ( C‬اﻟﻤﻨﺤﻨﻰ اﻟﻤﻤﺜﻞ ﻟﻠﺪاﻟﺔ ‪ f‬ﻓﻲ ﻣﻌﻠﻢ ﻣﺘﻌﺎﻣﺪ ﻣﻤﻨﻈﻢ‬
‫) ‪(C‬‬ ‫أ( أدرس اﻟﻔﺮوع اﻟﻼﻧﻬﺎﺋﻴﺔ ﻟﻠﻤﻨﺤﻨﻰ‬
‫ج أﻧﺸﺊ ) ‪( C‬‬
‫) ∈ ‪( ∀n‬‬ ‫‪un+1 = un 2 f ( un ) = un e−un‬‬ ‫‪; u0 = 1‬‬ ‫‪ ( II‬ﻟﺘﻜﻦ ) ‪ ( un‬اﻟﻤﺘﺘﺎﻟﻴﺔ اﻟﻌﺪدﻳﺔ اﻟﻤﻌﺮﻓﺔ ﺑـ‪:‬‬
‫‪ -1‬ﺑﻴﻦ أن ‪( ∀x ∈ ) e x ≥ x + 1‬‬
‫‪x‬‬
‫‪∀x‬‬ ‫≤ ) ‪0 x2 f ( x‬‬ ‫‪ -2‬اﺳﺘﻨﺘﺞ أن‬
‫‪x +1‬‬
‫‪1‬‬
‫∈ ‪∀n‬‬ ‫≤ ‪0 ≺ un‬‬ ‫‪ -3‬أ( ﺑﺎﺳﺘﻌﻤﺎل اﻟﺒﺮهﺎن ﺑﺎﻟﺘﺮﺟﻊ ﺑﻴﻦ أن‬
‫‪n +1‬‬
‫ب( ﺑﻴﻦ أن ) ‪ ( un‬ﻣﺘﻘﺎرﺑﺔ وﺣﺪد ﻧﻬﺎﻳﺘﻬﺎ‬
‫‪n −1‬‬
‫= ‪vn‬‬ ‫‪∑ uk‬‬ ‫‪:‬‬ ‫*‬
‫‪ -4‬ﻧﻀﻊ ﻣﻦ أﺟﻞ آﻞ ‪ n‬ﻣﻦ‬
‫‪k =0‬‬

‫*‬ ‫‪ 1‬‬ ‫‪‬‬


‫∈ ‪∀n‬‬ ‫‪vn = ln ‬‬ ‫أ‪ -‬ﺑﻴﻦ أﻧﻪ ‪ :‬‬
‫‪ un‬‬ ‫‪‬‬
‫) ‪( vn‬‬ ‫ب‪ -‬ﺣﺪد ﻧﻬﺎﻳﺔ‬
‫‪ (III‬ﻧﻌﺘﺒﺮ اﻟﺪاﻟﺔ ‪ F‬اﻟﻤﻌﺮﻓﺔ ﻋﻠﻰ [∞‪ [ 0; +‬ﺑﻤﺎ ﻳﻠﻲ‪:‬‬
‫‪4 x2‬‬
‫= ) ‪ F ( x‬و ) ‪F ( 0 ) = 2 ln ( 2‬‬ ‫‪∫x‬‬ ‫‪2‬‬ ‫‪f ( t ) dt‬‬ ‫‪x‬‬ ‫‪0‬‬
‫‪4 x2 1‬‬
‫‪( ∀x‬‬ ‫)‪0‬‬ ‫‪ -1‬أ( ﺗﺤﻘﻖ أن ) ‪dt = 2 ln ( 2‬‬
‫‪∫x‬‬ ‫‪2‬‬
‫‪t‬‬
‫‪( ∀t‬‬ ‫‪0 ) −t ≤ e −t‬‬ ‫ب( ﺑﺎﺳﺘﻌﻤﺎل ﻧﺘﻴﺠﺔ ‪ ، (1 − II‬ﺑﻴﻦ أن ‪− 1 ≤ 0‬‬
‫‪∀x‬‬ ‫‪ -2‬أ( ﺑﻴﻦ أن ‪0 −3x 2 ≺ F ( x ) − 2 ln ( 2 ) ≤ 0‬‬
‫ب( اﺳﺘﻨﺘﺞ أن ‪ F‬ﻣﺘﺼﻠﺔ و ﻗﺎﺑﻠﺔ ﻟﻼﺷﺘﻘﺎق ﻋﻠﻰ اﻟﻴﻤﻴﻦ ﻓﻲ ‪0‬‬
‫‪ -3‬أ( ﺑﻴﻦ أن ‪∀t ≥ 1 f ( t ) ≺ e −t‬‬
‫ب( اﺳﺘﻨﺘﺞ ) ‪lim F ( x‬‬
‫∞‪x →+‬‬
‫‪ -4‬أ( ﺑﻴﻦ أن أ ‪ F‬ﻗﺎﺑﻠﺔ ﻟﻼﺷﺘﻘﺎق ﻋﻠﻰ [∞‪ ]0; +‬و أﺣﺴﺐ ) ‪F ' ( x‬‬
‫ب( أﻋﻂ ﺟﺪول ﺗﻐﻴﺮات ‪F‬‬
‫ج( أﻧﺸﺊ اﻟﻤﻨﺤﻨﻰ ‪ CF‬ﻓﻲ ﻣﻌﻠﻢ ﻣﺘﻌﺎﻣﺪ ﻣﻤﻨﻈﻢ‪.‬‬
‫‪4 x −t‬‬
‫∫ = )‪G ( x‬‬ ‫‪e‬‬ ‫‪ln ( t ) dt‬‬ ‫‪ -5‬ﻟﺘﻜﻦ ‪ G‬اﻟﺪاﻟﺔ اﻟﻤﻌﺮﻓﺔ ﻋﻠﻰ [∞‪ ]0; +‬ﺑـ‪:‬‬
‫‪x‬‬
3/3 ‫ﺻﻔﺤﺔ‬

∀x 0 G ( x) = F ( x ) − e−4 x ln ( 4 x ) + e− x ln ( x ) ‫أ( ﺑﻴﻦ أن‬


lim ( e − x − e −4 x ) ln ( x ) ‫ب( أﺣﺴﺐ‬
+
x →0
lim G ( x ) ‫ت( اﺳﺘﻨﺘﺞ‬
x →0 +
Exercice 1 (3 points)

1 Congruence de n
a. n impair implique n2 congru à 1 modulo 8
Soit n un entier impair,
2 2
Il existe donc un entier k tel que n = 2k +1 , d’où n = 4k + 4k + 1 = 4k ( k + 1) + 1 .
Si k est pair alors k + 1 est impair et si k est impair alors k + 1 est pair, donc dans tous les cas,
k (k + 1) est pair, c’est à dire qu’il existe un entier p tel que k (k + 1) = 2 p .
2 2
On en déduit alors que n = 8 p +1 , autrement dit n ≡ 1 [8] .

2
Finalement, si n est un entier naturel impair, alors n est congru à un modulo huit.

b. n pair implique n2 congru à 0 ou à 4 modulo 8


Soit n un entier pair.
2 2
Il existe alors un entier k tel que n = 2k , soit n = 4k . On distingue alors deux cas,

>> Premier cas


2 2 2
Si k est pair, alors il existe un entier p tel que k = 2 p , d’où n = 4(4 p ) = 16 p ⇒ n2 ≡ 0 [8]

>> Deuxième cas


Si k est impair, alors il existe un entier p tel que k = 2 p + 1 , d’où,

n 2 = 4(2 p + 1) 2 = 16k 2 + 16k + 4 = 16(k 2 + k ) + 4


2 2
Comme 16( k + k ) est divisible par 8 , alors n ≡ 4 [8] .

2
Finalement, si n est un entier naturel pair, alors n est congru à zéro ou à quatre modulo huit.

2 Un carré parfait
a. a2 + b2 + c2 n’est pas un carré parfait
a , b et c sont trois entiers naturels impairs.
2 2 2 2 2 2
D’après la question 1.a, on a a ≡ 1 [8] , b ≡ 1 [8] et c ≡ 1 [8] , d’où a + b + c ≡ 3 [8] .
D’après la question 1, un carré parfait est congru à zéro, à un ou à quatre modulo huit, donc
a 2 + b 2 + c 2 n’est pas un carré parfait.

a 2 + b 2 + c 2 n’est pas un carré parfait.

b. 2(ab+bc+ac) est congru à 6 modulo 8


2 2 2 2
On a ( a + b + c ) = a + b + c + 2( ab + ac + bc ) ,

2
2 2 2 2
D’où ( a + b + c ) − ( a + b + c ) = 2(ab + ac + bc ) .
Comme a , b et c sont trois entiers naturels impairs alors a + b + c est aussi impair.
2
D’après la question 1.a, on a ( a + b + c ) ≡ 1 [8] .
2 2 2
D’après la question 2.a, on a a + b + c ≡ 3 [8] .
2 2 2 2
Donc, ( a + b + c ) − ( a + b + c ) = 2( ab + ac + bc ) ≡ 1 − 3 [8] ≡ − 2 [8] ≡ 6 [8]

2(ab + ac + bc) est congru à 6 modulo 8.

c. 2(ab + bc + ac) n’est pas un carré parfait


d’après la question 1, un carré parfait est congru soit à zéro, soit à quatre modulo huit, comme
2(ab + ac + bc) ≡ 6 [8] alors 2(ab + ac + bc) n'est pas un carré parfait.

d. ab + bc + ac n’est pas un carré parfait :


D’après la question 2.b, on a 2( ab + ac + bc ) ≡ 6 [8] , donc il existe un entier naturel p tel que
2(ab + ac + bc) = 8 p + 6 , autrement dit, ab + ac + bc = 4 p + 3 .
D’où, ab + ac + bc ≡ 3 [4] .

Montrons maintenant qu’un carré parfait est congru soit à zéro soit à un modulo quatre.
Soit n un entier naturel,
>> Si n est impair, alors il existe un entier k tel que n = 2k +1 , d’où
n = 4k + 4k + 1 ⇒ n 2 ≡ 1 [4] .
2 2

2 2 2
>> Si n est pair, alors il existe un entier k tel que n = 2k , d’où n = 4k ⇒ n ≡ 0 [4] .

>> Conclusion
Un carré parfait est congru soit à zéro soit à un modulo quatre, comme ab + ac + bc ≡ 3 [4] alors
ab + ac + bc n’est pas un carré parfait.

3
Exercice 2 (3 points)

1 Une structure algébrique


a. Produit de deux matrices
Soient a et b deux réels non nuls, on a,
⎛ 1 ⎛ 1 ⎞ ⎞⎛ 1 ⎛ 1 ⎞⎞ ⎛ a ⎛ 1⎞ 1 ⎛ 1 ⎞⎞
⎜a ⎜ a − ⎟ ⎟⎜ b ⎜ b − ⎟ ⎟ ⎜ ab ⎜b − ⎟ + ⎜ a − ⎟⎟
3⎝ a ⎠ ⎟⎜ 3⎝ b ⎠⎟ ⎜ 3⎝ b⎠ b 3⎝ a ⎠⎟
M a × Mb = ⎜ =
⎜ 1 ⎟⎜ 1 ⎟ ⎜ 1 ⎟
⎜0 ⎟⎜ 0 ⎟ ⎜0 ⎟
⎝ a ⎠⎝ b ⎠ ⎝ ab ⎠
a ⎛ 1⎞ 1 ⎛ 1⎞ 1 ⎛ a a 1 ⎞ 1 ⎛ 1 ⎞
Or ⎜b − ⎟ + ⎜a − ⎟ = ⎜ ab − + − ⎟ = ⎜ ab − ⎟ , d’où,
3⎝ b⎠ b 3⎝ a⎠ 3⎝ b b ab ⎠ 3⎝ ab ⎠

⎛ 1 ⎛ 1 ⎞⎞
⎜ ab ⎜ ab − ⎟ ⎟
3⎝ ab ⎠ ⎟
M a × Mb = ⎜ = M ab .
⎜ 1 ⎟
⎜0 ⎟
⎝ ab ⎠

b. ϕ est un morphisme
*
ϕ est l’application définie de \ dans E par ϕ (a) = M a .
Montrer que ϕ (*
)
est un morphisme de \ ; × dans ( E ; ×) revient à montrer que ϕ (ab) = ϕ (a)ϕ (b) .
On a ϕ (ab) = M ab = M a × M b d’après la question précédente, donc ϕ (ab) = ϕ (a)ϕ (b) .

c. Structure algébrique de l’ensemble E


*
L’application ϕ est bijective, comme \ est un groupe multiplicatif alors E est aussi un groupe
multiplicatif.

2 Un groupe
a. Produit de deux matrices
Soient a et b deux réels non nuls, on a,
⎛ ⎛ 1⎞ a ⎛ 1⎞ b ⎛ 1 ⎞⎞
⎛ 1 ⎛ 1 ⎞⎞⎛ 1 ⎛ 1 ⎞ ⎞ ⎜ ab − b ⎜ a − ⎟ ⎜b − ⎟ − ⎜a − ⎟⎟
⎜ a ⎜ a − ⎟⎟⎜ b ⎜b − ⎟⎟ ⎜ ⎝ a⎠ 3⎝ b⎠ 3⎝ a ⎠⎟
N a × Nb = ⎜ 3⎝ a ⎠⎟⎜ 3⎝ b ⎠⎟ =
⎜ ⎟⎜ ⎟ ⎜⎜ − ab 3 + ab 3 ⎛ 1⎞
− a ⎜ b − ⎟ + ab

⎝ −a 3 −a ⎠ ⎝ −b 3 −b ⎠ b⎠

⎝ ⎝ ⎠

4
⎛b 1 ⎛ b a ⎞⎞
⎜ ⎜ − ⎟⎟
d’où, N a × Nb = ⎜ a 3 ⎝ a b ⎠⎟ = M .
b
⎜ a ⎟
⎜0 ⎟ a
⎝ b ⎠

b. G est un groupe
1/ G est stable par la loi × :

On a G = E ∪ F et on propose de montrer que G muni de la multiplication est un groupe.


Soient x et y deux éléments non nuls de G .
Si x et y sont dans E , comme E est un groupe (question 1.c) alors x × y ∈ E (produit de deux
matrices 2x2 donne une matrice 2x2) et à fortiori x × y ∈ G .
Si x et y sont dans F alors il existe deux réels non nuls a et b tels que x = N a et y = Nb et
donc x × y = N a × Nb = M b ∈ F , d’où x × y ∈ G .
a
Si x est dans F et y est dans E , alors il existe deux réels non nuls a et b tels que x = N a et
y = M b , d’où x × y = N a × M b = N a × M ab = N a × ( N a × N ab ) , d’après la question 2.a.
a
Alors, x × y = ( N a × N a ) × N ab , (le produit matriciel étant associatif).
⎛1 0⎞
Or N a × N a = M a = M 1 = I 2 , avec I 2 = ⎜ ⎟ (Matrice identité).
⎝ 0 1⎠
a
D’où x × y = I 2 × N ab = N ab ∈ F et à fortiori x × y ∈ G .
Si x est dans E et y est dans F , alors il existe deux réels non nuls a et b tels que x = M a et
y = Nb , d’où :
⎛ ⎞
x × y = M a × Nb = ⎜ N b × Nb ⎟ × Nb = N b × ( Nb × Nb ) = N b × M b = N b × M 1 = N b × I 2 = N b ∈ F
⎜ ⎟
⎝ a ⎠ a a b a a a

donc, x × y ∈ G .

2/ G admet un élément neutre :

⎧M a × I2 = I2 × M a = M a
De plus I 2 = M1 ∈ G et ⎨ , donc I 2 est l’élément neutre de G .
⎩ Na × I2 = I2 × Na = Na

3/ Les éléments de G sont inversibles :


−1
On a N a × N a = M 1 = I 2 donc l’inverse de N a par la loi × est N a ( N a × ( N a ) = I 2 ) et est donc
dans G .

5
De même, M a × M 1 = I 2 , donc l’inverse de M a par la loi × est M 1 et est donc dans G .
a a

c. Le groupe G est-il commutatif ?


Soient N a et Nb deux éléments de G et soient a et b deux réels non nuls.
Si le groupe G est commutatif alors N a × Nb = Nb × N a autrement dit M b = M a .
a b
Or en prenant a = 1 et b = 2 , on a :
⎛ 3 ⎞
⎜2 ⎟ ⎛ 1 −3 ⎞
M 2 = M2 = ⎜
2 3 ⎟ et M 1 = ⎜ 2 2 3 ⎟
⎜ 1 ⎟ ⎜ ⎟
0 ⎜0 2 ⎟
1 ⎜ ⎟ 2 ⎝ ⎠
⎝ 2 ⎠
donc M b ≠ M a ⇒ N a × Nb ≠ Nb × N a .
a b

G n’est pas un groupe commutatif.

6
Exercice 3 (3 points et ½)

1 Résolution d’une équation


2 2
Le discriminant du polynôme z + z + 1 et négatif et vaut −3 , l’équation z + z + 1 = 0 admet donc deux
racines complexes conjugués notés r 1 et r 2 avec :

−1 − i 3 −1 + i 3
r1 = et r 1 = .
2 2

2 Un ensemble de points
a. Une égalité
Soit z ≠ 0 ,
2 ⎛ 1⎞ 1 a −ib
On a z + z + 1 = z ⎜ 1 + z + ⎟ , comme z = a + i b , alors = .
⎝ z⎠ z 2
|z|
1
Or dans le cadre de cet exercice, on a | z |= 1 donc = a − ib = z d’où
z
2
l’égalité z + z + 1 = z (1 + z + z ) .

b. Le module et un argument de z’
' 1 1
On a z = =
2 z (1 + z + z )
1+ z + z
iθ 1 − iθ
Comme z = e alors = e , de plus on a z + z = 2 cos(θ ) d’où,
z
1 1 1 1
| z ' |= = × =
z (1 + z + z ) z 1 + z + z 1 + 2 cos(θ )
' ⎛1⎞ ⎛ 1 ⎞ ⎛1⎞ ⎛ 1 ⎞
et arg( z ) = arg ⎜ ⎟ + arg ⎜ ⎟ = arg ⎜ ⎟ + arg ⎜ ⎟ , on distingue alors deux cas,
⎝z⎠ ⎝ 1+ z + z ⎠ ⎝z⎠ ⎝ 1 + 2 cos(θ ) ⎠
>> Premier cas
⎛1⎞ ⎛ 1 ⎞
1 + 2 cos(θ ) > 0 , alors arg( z ' ) = arg ⎜ ⎟ + arg ⎜ ⎟ = −θ ≡ [π ] .
⎝z⎠ ⎝ 1 + 2 cos(θ ) ⎠
>> Deuxième cas
⎛1⎞ ⎛ −1 ⎞
1 + 2 cos(θ ) < 0 , alors arg( z ' ) = arg ⎜ ⎟ + arg ⎜ ⎟ = −θ − π ≡ [π ] .
⎝z⎠ ⎝ 1 + 2 cos(θ ) ⎠

c. Une égalité
' 2 2 2 2
On pose z = x + i y avec ( x ; y ) ∈ \ et on propose de montrer que x + y = (1 − 2 x) .

7
' 1 1 1 cos(θ ) − i sin(θ ) cos(θ ) − i sin(θ )
On a z = × = (cos(−θ ) + i sin(−θ )) × = =
z 1+ z + z 1 + 2 cos(θ ) 1 + 2 cos(θ ) 1 + 2 cos(θ )
' cos(θ ) − sin(θ )
Comme z = x + i y , alors x = et y = .
1 + 2 cos(θ ) 1 + 2 cos(θ )
2 2
2 2 cos (θ ) + sin (θ ) 1
D’où, x + y = = ,
2
(1 + 2 cos(θ )) (1 + 2 cos(θ ))2
2 1
Il reste à montrer que (1 − 2 x ) = ;-)
(1 + 2 cos(θ )) 2
2 2
2⎛ 2 cos(θ ) ⎞ ⎛ 1 ⎞
On a (1 − 2 x ) = ⎜ 1 − ⎟ =⎜ ⎟ , CQFD.
⎝ 1 + 2 cos(θ ) ⎠ ⎝ 1 + 2 cos(θ ) ⎠

2 2
Finalement, x + y = (1 − 2 x) 2 .

d. L’ensemble des points M


'
Le point M est le point d’affixe z , il s’agit de montrer que cet ensemble de points appartient à une
hyperbole que l’on caractérisera.
2 2
On a d’après la question précédente, x + y = (1 − 2 x) 2 , soit,
⎛ 4x 1 ⎞
y 2 = (1 − 2 x)2 − x 2 = 3 x 2 − 4 x + 1 = 3 ⎜ x 2 − + ⎟,
⎝ 3 3⎠
2 2
y2 ⎛ 2⎞ 4 3 ⎛ 2⎞ 1
=⎜x− ⎟ − + =⎜x− ⎟ − ,
3 ⎝ 3⎠ 9 9 ⎝ 3⎠ 9
2
⎛ 2⎞ y2 1
⎜ x − ⎟ − =
⎝ 3⎠ 3 9
2
⎛ 2⎞
9 ⎜ x − ⎟ − 3 y2 = 1
⎝ 3⎠
2
⎛ 2⎞
⎜x− ⎟ y2
⎝ 3⎠
− = 1.
2 2
⎛1⎞ ⎛ 1 ⎞
⎜ ⎟ ⎜ ⎟
⎝3⎠ ⎝ 3⎠

⎛2 ⎞
On reconnaît alors l’équation d’une hyperbole dont le foyer est le point de coordonnées ⎜ , 0 ⎟ , de demi
⎝3 ⎠
1 1
axe transverse et de demi axe non transverse .
3 3

8
Exercice 4 (10 points et ½)

I Etude de ƒ
e− x
f est la fonction définie sur \* par f ( x) = .
x

1 Limites de ƒ aux bornes de son ensemble de définition


>> Limite de f au voisinage de − ∞
Dans un soucis de clarté, on pose y = − x , on a alors ,

e− x ey ey
lim f ( x) = lim = lim = − lim = −∞ ,
x→−∞ x→−∞ x y→+∞ −y y→+∞ y

d’où, lim f ( x) = − ∞ .
x→−∞
>> Limite de f à gauche de zéro

e− x −x
lim − f ( x) = lim − , comme lim e = 1 et lim x = 0− ,
x→0 x→0 x x → 0− x → 0−

Alors, lim f ( x) = −∞ .
x → 0−
>> Limite de f à droite de zéro

e− x −x
lim + f ( x) = lim + , comme lim e = 1 et lim x = 0+ ,
x→0 x→0 x x→0 +
x → 0+

alors lim f ( x) = +∞ .
x → 0+
>> Limite de f au voisinage de + ∞

e− x 1
lim f ( x) = lim = lim = 0.
x→+∞ x→+∞ x x → + ∞ x ex

lim f ( x) = 0 .
x→+∞

2 Variations de ƒ
>> Expression de la dérivée
−x *
La fonction x 6 e est dérivable sur \ .
1 *
La fonction x 6 est également dérivable sur \ .
x
9
−x
* ' − xe − e− x −e− x ( x + 1) *
Donc f est dérivable sur \ et f ( x ) = = , ∀x ∈ \ .
2 2
x x
>> Signe de la dérivée
* 2 −x
Comme pour tout réel x dans \ , x > 0 et e > 0 , alors le signe de f ' ne dépend que de celui de
−( x + 1) , c’est à dire que f ' ( x) < 0 sur ]−1, + ∞[ , f ' ( x) > 0 sur ]− ∞ , − 1 [ et f ' (−1) = 0 .

>> Tableau de variations

x −∞ −1 0 +∞
f ' ( x) + - -
1 +∞

f ( x) e

−∞ 0
−∞

3 Branches infinies et construction de C


a. Nature des branches infinies de C

>> Puisque lim f ( x) = 0 , alors la courbe C admet l’axe des abscisses comme asymptote
x→+∞
horizontale au voisinage de + l’infini.

>> Puisque lim f ( x) = − ∞ , alors la courbe C admet l’axe des ordonnées comme asymptote
x → 0−
verticale.

>> Puisque lim f ( x) = + ∞ , alors la courbe C admet l’axe des ordonnées comme asymptote
x → 0+
verticale.

f ( x)
>> Comme lim f ( x) = + ∞ , il faut calculer lim .
x→−∞ x→−∞ x

f ( x) e− x
On a lim = lim = + ∞ , donc la courbe C admet au voisinage de – l’infini une branche
x→−∞ x x → − ∞ x2
parabolique.

10
b. Représentation graphique de la courbe C

Cf

II Etude de la suite (un)


1 Une inégalité
x
Il s’agit de montrer que pour tout réel x on a e ≥ x + 1 , pour cela il suffit d’étudier le signe de la
différence entre les deux membres de cette inégalité.
x
Posons d ( x ) = e − x − 1 et montons que ∀x ∈ \ , d ( x) ≥ 0 .

d est dérivable sur \ et d ' ( x) = e x − 1 , on en déduit facilement que :


d ' s’annule pour x = 0 .
d ' ≥ 0 sur l’intervalle [ 0, + ∞ [ .
d ' ≤ 0 sur l’intervalle ]− ∞ , 0 ] .

11
>> D’où le tableau de variations de la fonction d

x −∞ 0 +∞
d ' ( x) - +

d ( x)
0

On voit d’après le tableau de variations ci-dessus que d admet un minima en x = 0 et que d (0) = 0 .

x x
Conclusion : Pour tout réel x on a d ( x ) = e − ( x + 1) ≥ 0 d’où e ≥ x + 1 .

>> Nota

On peut montrer cette inégalité de la manière suivante :


x x

∫ ∫
x
x
∀x ≥ 0 , on a e ≥ 1 ⇒ t
e dt ≥ dt ⇒ ⎡et ⎤ ≥ x ⇒ e x − 1 ≥ x ⇒ e x ≥ x + 1 ;-)
⎣ ⎦0
0 0

2 Une autre inégalité


2 x
On propose de montrer que pour tout réel x strictement positif on a x f ( x ) ≤ , pour cela et comme
x +1
indiqué dans l’énoncé, on utilise l’inégalité établie à la question précédente, c’est à dire,
11 1 1
ex ≥ x +1 ⇒ , pour tout x > 0 ⇒ e ≤
≤ −x
car = e− x .
e x x +1 x +1 x
e
On divise les deux côtés de cette inégalité par x pour faire apparaître l’expression de f ( x ) ( x étant
strictement positif --> division autorisée + pas de changement de sens pour l’inégalité).
e− x 1
Il vient, ≤ , ∀x > 0
x x ( x + 1)

2 2 x
On multiplie finalement les deux côtés de cette inégalité par x et on obtient x f ( x ) ≤ .
x +1

3 Limite de (un)
a. Encadrement de un
1
Il s’agit de prouver par récurrence que pour tout entier naturel n , on a 0 < un ≤ .
n +1
>> un > 0
Pour n = 0 , on a u0 = 1 > 0 , donc la relation est vraie à l’ordre zéro.
On suppose que la relation est vraie à l’ordre n , c’est à dire que un > 0 et on s’intéresse à l’ordre n + 1 .

12
− un − un
On a un + 1 = un e , d’après l’hypothèse de récurrence on sait que un > 0 , de plus e >0
(l’exponentielle étant toujours positive), on en déduit alors que un + 1 > 0 , La relation est vraie à l’ordre
n + 1 . Elle est donc héréditaire.

1
>> un ≤
n +1
1
Pour n = 0 , on a u0 = 1 et = 1 donc la relation est vraie à l’ordre zéro.
0 +1
1
On suppose que la relation est vraie à l’ordre n , c’est à dire que un ≤ et on s’intéresse à l’ordre
n +1
n + 1.
2
On a un + 1 = un f (un ) , or nous avons montré à la question précédente que pour tout réel x strictement

2 x 2 un
positif, on a x f ( x ) ≤ et comme un > 0 alors un f (un ) ≤ .
x +1 un + 1
1 x
D’après l’hypothèse de récurrence on a un ≤ , de plus la fonction x 6 est croissante sur
n +1 x +1
]0, + ∞[ (pas de changement de sens pour l’inégalité).
2 un
L’inégalité un f (un ) ≤ devient alors,
un + 1
1
1
un 2 f (un ) ≤ n + 1 ⇒ un 2 f (un ) ≤ , la relation est vraie à l’ordre n + 1 , elle est donc
1 n + 2
+1
n +1
héréditaire.

1
Finalement, pour tout entier naturel n , on a 0 < un ≤ .
n +1

b. Convergence et limite de la suite (un)


1
On utilise l’encadrement de un démontré à la question précédente, c’est à dire 0 < un ≤ et on
n +1
passe à la limite (notons que le passage à la limite fait changer les inégalités strictes en inégalités larges),
1 1
On a, 0 ≤ lim un ≤ lim , comme lim = 0 alors d’après le théorème des gendarmes,
n →+ ∞ n →+ ∞ n + 1 n → + ∞ n +1
on a lim un = 0 .
n→+∞

La suite (un ) est donc convergente et sa limite vaut zéro.

13
4 Etude de la suite (vn)
a. Une autre expression de vn
n −1

La suite (vn ) est définie pour tout entier naturel n non nul par vn =
∑u
k =0
k , on propose de montrer que

⎛ 1 ⎞
vn = ln ⎜ ⎟ .
⎝ un ⎠
− un − 1
On part de l’expression de (un ) , c’est à dire un = un − 1 e et on « compose avec le LN » des deux

côtés de l’égalité, (notons que d’après la question II.3.a on a un > 0 ).


−u −u
ln(un ) = ln(un − 1 e n − 1 ) = ln(un − 1 ) + ln(e n − 1 ) = ln(un − 1 ) − un −1 .
On passe à la somme, terme à terme,
n n n

∑k =1
ln(uk ) =

k =1
ln(uk −1) −

k =1
uk − 1

n
ln(u1) + ln(u2 ) + ln(u3 ) + ... + ln(un ) = ln(u0 ) + ln(u1) + ln(u2 ) + ... + ln(un −1) −
∑k =1
uk − 1 .

Après simplification, il reste,


n
ln(un ) = ln(u0 ) −
∑ k =1
uk − 1

n n −1
Comme u0 = 1 , alors ln(u0 ) = 0 , de plus
∑k =1
uk −1 = u0 + u1 + u2 + ... + un −1 =

k =0
uk = vn

(décalage d’indice).

⎛ 1 ⎞
Alors, vn = − ln(un ) = ln ⎜ ⎟ , un étant strictement positif.
⎝ un ⎠

b. Limite de (vn)
1
Nous savons d’après la question II.3.a que lim un = +∞ , d’où lim =0.
n→+∞ n → + ∞ un
⎛ 1 ⎞ ⎛ 1 ⎞
Comme vn = ln ⎜ ⎟ alors lim vn = lim ln ⎜ ⎟ = − ∞ .
⎝ un ⎠ n→+∞ n → + ∞ ⎝ un ⎠

La suite (vn ) est divergente et lim vn = − ∞ .


n→+∞

14
III Etude de F
⎧ 4 x2
⎪ F ( x) =
F est la fonction définie sur l’intervalle [ 0, + ∞ [ par ⎨

x 2
f (t ) dt x > 0
. ∫
⎩ F (0) = 2ln(2)

1 Un encadrement
a. Valeur d’une intégrale
4 x2

∫ ( ) ( ) ( ) ( ) ( )
dt 4 x2
On a = ⎣⎡ ln t ⎦⎤ 2 = ln 4 x 2 − ln x 2 = ln ( 4 ) + ln x 2 − ln x 2 = ln(4) = ln 22 = 2ln(2) .
t x
x2

b. Encadrement de e- t – 1
−t
>> e − 1 ≥ −t
x
A la question II.1 on a montré que pour tout réel x , on a e ≥ x + 1 .
−t
Posons x = −t , l’inégalité devient alors e ≥ −t + 1 , autrement dit e −t − 1 ≥ −t .

−t
>> e −1 ≤ 0
−t
On a t > 0 ⇒ −t < 0 ⇒ e < 1 ⇒ e− t − 1 < 0 .

−t
Finalement, pour tout réel t strictement positif, on a −t ≤ e −1 ≤ 0 .

2 Continuité et dérivabilité de F à droite de zéro


a. Encadrement de F(x) – 2ln(2)
2
Il s’agit de montrer que pour tout réel x strictement positif, on a −3 x ≤ F ( x) − 2 ln(2) ≤ 0 , pour cela on
utilise l’encadrement établit à la question précédente :
∀t > 0 , on a −t ≤ e −t − 1 ≤ 0 .
On divise ensuite les membres de cette inégalité par t ( t est strictement positif) ce qui donne :
e −t 1
−1 ≤ − ≤ 0.
t t
On intègre ensuite cette inégalité sur l’intervalle ⎡ x , 4 x ⎤ , il vient :
2 2
⎣ ⎦
2 2 2
4x 4x 4x 4 x2 4 x2
e −t
∫ ∫ ∫ ∫ ∫
dt dt
− dt ≤ dt − ≤ 0 , autrement dit − dt ≤ F ( x) − ≤0
x2 x2 t x2 t x2 x2 t
4 x2 4 x2

∫ ∫
dt
On a vu à la question III.1.a que = 2ln(2) , de plus − dt = −3x 2 , d’où,
x2 t x2
−3 x 2 ≤ F ( x) − 2 ln(2) ≤ 0 .

15
2
Finalement, −3 x ≤ F ( x) − 2 ln(2) ≤ 0 .

b. Continuité et dérivabilité de F à droite de zéro


>> Continuité de F à droite de zéro

Il s’agit de montrer que lim F ( x) = F (0) , pour cela on utilise l’encadrement de la question
x → 0+
précédente.
2
On a −3 x < F ( x) − 2 ln(2) ≤ 0
On passe à la limite (notons que le passage à la limite fait passer les inégalités strictes en inégalités
larges).
lim − 3 x 2 ≤ lim ( F ( x) − 2 ln(2) ) ≤ 0
x → 0+ x → 0+

Comme lim − 3x 2 = 0 alors d’après les théorème des gendarmes, on a lim ( F ( x) − 2 ln(2) ) = 0 ,
x → 0+ x → 0+
d’où lim F ( x ) = 2 ln(2) = F (0) .
x → 0+
F est donc continue à droite de zéro.

>> Dérivabilité de F à droite de zéro


On part également de l’encadrement −3 x 2 < F ( x) − 2 ln(2) ≤ 0 et on fait apparaître le taux
d’accroissement de F à droite de zéro.
∀x > 0 , on a,
F ( x) − 2 ln(2) F ( x) − F (0) F ( x) − F (0)
−3 x < ≤ 0 ⇒ − 3x < ≤ 0 ⇒ lim − 3x ≤ lim ≤0
x x−0 x → 0+ x → 0+ x−0
F ( x) − F (0)
Comme lim − 3 x = 0 et lim = F ' (0) alors d’après les gendarmes :
x→0 +
x→0 + x−0

F est dérivable à droite de zéro et F ' (0) = 0 .

3 Limite de F au voisinage de + l’infini


a. Majoration de ƒ(t)
−t
Pour tout réel t ≥ 1 , f (t ) est majorée par e :
−t
1 e
t ≥1 ⇒ ≤1 ⇒ ≤ e − t , notons que l’exponentielle est positive donc l’inégalité ne change pas de
t t
sens ;-)
−t
Finalement, t ≥ 1 ⇒ f (t ) ≤ e .

16
b. Limite de ƒ au voisinage de + l’infini
−t e− t
Nous venons de montrer que pour tout réel t ≥ 1 , on a f (t ) < e , de plus f (t ) = > 0 , on en déduit
t
−t
alors que 0 < f (t ) < e .

On intègre ensuite cette inégalité sur l’intervalle ⎡ x , 4 x ⎤ , il vient,


2 2
⎣ ⎦
2 2 2
4x 4x 4x
0<
∫ x 2
f (t ) dt <
2
∫ x 2
−t
e dt ⇒ 0 < F ( x) <
∫ x 2
e−t dt .

( ) ( )
4x


2 2 2 2 2
Par ailleurs, e −t dt = [−e− t ] 42x = − e− 4 x − e− x , d’où 0 < F ( x) < − e− 4 x − e− x .
2 x
x

On passe à la limite, 0 ≤ lim F ( x ) ≤ lim − e


x→+∞ x→+∞
( − 4 x2
− e− x
2
) .

Comme
x→+∞
(
lim − e − 4 x − e − x
2 2
)=0 alors d’après les gendarmes, on a :

lim F ( x) = 0 .
x→+∞

4 Variations de F
a. La dérivée de F
On commence par justifier la dérivabilité de F sur l’intervalle [0, + ∞ [ et on donnera ensuite
'
l’expression de F ( x ) .
D’après la question III.2.b, on a F dérivable à droite de zéro.
e− t
Notons H la primitive de la fonction f : t 6 sur l’intervalle 0, + ∞ ] [ s’annulant en x = 1 , donc :
t
x −t


e
H ( x) = dt .
1 t
2 2
Nous avons F ( x ) = H (4 x ) − H ( x ) donc F est dérivable sur 0, + ∞ ] [ (car composée de fonctions
2 2
dérivables : H , x 6 4 x et x 6 x ).

>> Conclusion

F est dérivable sur [ 0, + ∞ [ et

F ' ( x) = 8 x ×
e − 4 x2
− 2x ×
e − x2
=
2e − 4 x2

2e − x2
=
( 2
2 e− 4 x − e− x
2
) .
4 x2 x2 x x x

17
b. Variations de F
'
>> Signe de F
− 4x 2 2
' *
Le dénominateur de F étant positif sur \ + , le signe de dérivée est celui de x 6 e − e− x .
−4 x 2 2
2 2 2 2
Or pour tout réel x ≥ 0 , on a 4 x ≥ x ⇒ − 4 x ≤ − x ⇒ e ≤ e− x car x 6 e x est croissante.
−4 x 2 2
D’où e − e− x ≤ 0 ⇒ F ' ( x) ≤ 0 ⇒ F est décroissante.

>> Tableau de variations de F

x 0 +∞
F ' ( x) -
2 ln(2)
F ( x)

c. Représentation graphique de CF

CF

5 Limite de G
4x
G est la fonction définie sur ]0, + ∞ [ par G ( x) =
∫ x
e−t ln(t ) dt .

a. Une autre expression de G


4x
On intègre par parties l’expression
∫x
e −t ln(t ) dt en posant :

18
⎧ ' 1
⎧⎪u (t ) = ln(t ) ⎪u (t ) =
⎨ ' −t
⇒ ⎨ t ,
⎩⎪v (t ) = e ⎪v(t ) = −e − t

4 x −t
Ce qui donne G ( x ) = ⎡ − ln t e

−t ⎤ 4 x
⎦x
+
∫ x
e
t
( ) ( x ) pour tout
dt = − ln(4 x)e− 4 x + ln( x)e− x + F

x >0.

Finalement, pour tout réel x strictement positif, on a bien G ( x ) = F ( x ) − e−4 x ln(4 x) + e− x ln( x) .
b. Une limite
Valeur de lim
x→0 + ( e− x − e− 4 x ) ln( x) .
−x − 4x
Pour tout x strictement positif on a e ( )
− x − e− 4 x ln( x) = e − e × x ln( x) .
x
e− x − e− 4 x
On sait que lim + x ln( x) = 0 et on calcule la valeur de lim .
x→0 x → 0+ x
−x
On pose l ( x ) = e − e−4 x , notons que l (0) = 0 et que l est dérivable, donc,
e− x − e− 4 x l ( x) − l (0)
lim = lim = l ' (0) , par ailleurs l ' ( x) = −e− x + 4e − 4 x , d’où l ' (0) = 3 .
x→0 + x x→0 + x−0

⎛ e− x − e− 4 x ⎞
Finalement, lim
x→0 +
( )
e − x − e− 4 x ln( x) = lim ⎜
x →0 + ⎜
⎝ x
× x ln( x) ⎟ = 3 × 0 = 0 .

c. Limite de G(x) à droite de zéro


On a montré à la question III.5.a que pour tout x > 0 on a G ( x ) = F ( x ) − e−4 x ln(4 x) + e− x ln( x) ,
autrement dit,

( x ) − e−4 x ( ln(4) + ln( x) ) + e− x ln( x) = F ( x ) − ln(4)e−4 x + ln( x) ( e− x − e−4 x ) .


G ( x) = F

Soit lim G ( x ) = lim F ( x ) − lim ln(4)e


−4 x
+ lim ln( x) ( e− x − e−4 x ) .
+ + + +
x→0 x→0 x→0 x→0

On sait que lim F ( x ) = F (0) = 2 ln(2) , on sait également d’après la question précédente que
+
x→0

( )
lim e − x − e− 4 x ln( x) = 0 .
x→0 +
D’où, lim G ( x) = 2 ln(2) − ln(4) = 2 ln(2) − 2 ln(2) = 0 .
x → 0+

FIN

19
‫      !  ‬ ‫  ‬
‫  ‪ 4 :‬‬ ‫      ‬ ‫    ‬
‫‪10‬‬ ‫ ‪:‬‬ ‫ ‬ ‫ ‬
‫ ‪2004  :‬‬
‫)    (‬

‫ا ا ول )  ن و (‬

‫ي آ  ‪10‬آات ء و ‪10‬آات اء   ا    ‪.‬‬
‫" ' & ا‪ %‬آة ‪ #‬ا ‪ .‬إذا آ"‪ .‬اة ا ‪ -‬اء "‪+,‬ه إ ا وإذا‬
‫آ"‪ .‬ء "‪ /0 + 1‬ا ‪3‬آات اء ‪ ' " 34‬آة ‪ #‬ا ‪.‬‬
‫‪ (1‬أ ' ا ل ‪ 8 /‬ن ا‪8‬ن ا  ن او  ‪.‬‬ ‫‪0,5‬‬
‫‪ (2‬أ ' ا ل ‪ 8 /‬ن ا‪8‬ن ا  ن و  ‪.‬‬ ‫‪0,5‬‬
‫‪ (3‬أ ' ا ل ‪ 8 /‬ن ا‪8‬ن ا  ن ‪. < ;# "  #‬‬ ‫‪0,5‬‬
‫‪ (4‬أ ' ا ل ‪ 8 /‬ن اة ا@و ا ‪ -‬ء  ?ن اة ا>"‪-‬‬ ‫‪1‬‬
‫ا ‪ -‬ء‬

‫ا ا ) ‪3‬ﻁ(‬


‫ا( ‪  ×  /0 A‬ا‪,‬د‪(E ): 3x − 2 y = 1 : -‬‬ ‫‪0,5‬‬
‫‪+ n  (2‬دا ‪+,# E ,FG   H‬م ‪.‬‬
‫أ‪  -‬أن اوج )‪, A (14n + 3 , 21n + 4‬د‪. (E ) -‬‬ ‫‪0,25‬‬
‫‪0,25‬‬
‫ب ‪ -‬ا‪ L  M‬أن ا‪+,‬د  ‪ 14n + 3‬و ‪ 21n + 4‬أون ‪.  0‬‬
‫‪ d  (3‬ه ا‪ 3MO‬ا& ك ا@آ‪+, F‬د  ‪ 2n + 1‬و ‪. 21n + 4‬‬
‫‪0,5‬‬
‫أ‪  -‬أن ‪ d = 1‬أو ‪. d = 13‬‬
‫ب ‪  -‬أن ‪d = 13 ⇔ n ≡ 6 [13] :‬‬ ‫‪0,5‬‬
‫‪ # (4‬أ‪ AQ‬آ‪+ A‬د ‪ H‬ﺡ ‪: 1" n ≥ 2 R  n /,FG‬‬
‫‪ A = 21n 2 − 17n − 4‬و ‪B = 28n 3 − 8n 2 − 17n − 3‬‬
‫أ‪  -‬أن ا‪+,‬د  ‪ A‬و ‪ /0 n − 1  -O S B‬ا‪.  - T‬‬ ‫‪0,5‬‬
‫ب ‪ n 3S ' + -‬ا‪ 3MO‬ا& ك ا@آ‪+, F‬د  ‪ A‬و ‪. B‬‬
‫‪0,5‬‬

‫ا ا ) ‪ 4‬ﻁ (‬


‫ ‬
‫ا ى ا‪+O,‬ي ‪ #‬ب إ ‪(O ,u ,v ) 3V# +#, # 3,#‬‬
‫ ‪+ a‬دا ‪+,# E  +O‬م ‪X‬ﻩ ا‪FT‬ي ه ‪. a = α + i β :‬‬
‫‪. z − (z ) = a − (a ) 2 : ZO‬‬
‫‪2‬‬ ‫‪2‬‬ ‫‪2‬‬
‫‪ - T# (H )   (1‬ا‪O‬ﻁ ‪ M‬ا ‪z O  /‬‬
‫أ‪+ -‬د ‪. (H ) -,FG‬‬ ‫‪0,5‬‬
‫‪0,5‬‬
‫ب ‪ -‬أ"&[ ) ‪ /0 (H‬ا ‪. a = 1 + i : -‬‬
‫‪. (z − a )(z − a ) = 4aa : ZO‬‬ ‫‪ - T# (C )   (2‬ا‪O‬ﻁ ‪ M‬ا ‪z O  /‬‬
‫أ‪. (C ) -,FG + -‬‬ ‫‪0,5‬‬
‫ب ‪ -‬أ"&[ ) ‪ /0 (C‬ا ‪. a = 1 + i : -‬‬ ‫‪0,5‬‬
‫‪z 2 − (z ) 2 = a 2 − (a ) 2‬‬
‫‪(S ) : ‬‬ ‫‪ 0 F ," (3‬ا‪ ،  - T‬ا‪: -V‬‬
‫‪ (z − a )(z − a ) = 4aa‬‬
‫و"‪u = z − a 1‬‬
u u = 4aa
(S ') :  -V‫[ ا‬08 (S ) -V‫  أن ا‬-‫أ‬ 0,5
(u + 2a )(u − 8a (a ) ) = 0
3 2

. −π < θ ≤ π ‫ و‬r > 0 R a = re i θ 1" - ‫ب‬ 0,75


. (H ) ‫( و‬C ) 1GO8 ‫ﻁ‬O" ‫ أ ق‬θ ‫ و‬r -+ ‫د‬+
. ‫_ع‬c@‫ وي ا‬# R> ‫ رؤوس‬/‫ﻁ ه‬O" ‫_ث‬4  (H ) ‫( و‬C ) 1GO8 ‫ أن‬L  M‫ ا‬- ‫ج‬ 0,75

(  ‫ ﻁ و‬10 ) ‫ا اا‬

:/   0,‫د  ا‬+,‫ا  ا‬+‫ ا‬g ‫ و‬f   - I


ln(2x )
g (x ) = ‫ و‬f (x ) = 4xe − x ln 2 − 2
x
+#, # 3,# /0 /‫  ا ا‬g ‫ و‬f  ‫ا‬+ >‫( ا  ا‬Γ) ‫( و‬C ) ‫و‬
 
(O , i , j ) 3V#
 
. ( i = j = 4cm : ‫ة‬+ ‫) ا‬
. +∞ ‫ و‬−∞ + f / " ' ‫ أ‬-‫(أ‬1 0,5
. (C )   %"_‫د ا< ا‬+ - ‫ب‬ 0,5
0,5
(∀x ∈ IR ) : f '(x ) = 4(1 − x ln 2)e − x ln 2 : ‫  أن‬-‫( أ‬2
. f -‫ا‬+‫ات ا‬g8 ‫ول‬+Q ‫ أﻁ‬- ‫ب‬ 0,25
. f (x ) = 0 -‫د‬, ‫ان‬+ ‫ ه ا  ا‬2 ‫ و‬1  ‫د‬+,‫  أن ا‬- ‫ج‬ 0,75
. ‫ات‬g ‫ ا‬، -%"_‫ ا<وع ا‬،‫ ا ت‬: g -‫ا‬+‫( أدرس ا‬3 1
. 3,‫ "< ا‬/0 (Γ) ‫( و‬C )  ‫ ا‬3M‫( أر‬4 1
1 1
( = 0, 4 ‫؛‬ = 1, 4 ‫؛‬ e = 2, 7 ‫ ؛‬ln 2 = 0, 7 : jk?" - ‫ ب‬h# E ‫ف‬h,"‫ﻁ ا‬O" + + 8 )
e ln 2

2
. 0<k < : R  OO ‫دا‬+ k  - II
e
1 0,5
. < α < β R  β ‫ و‬α ‫; < ل‬#  AFO8 g (x ) = k -‫د‬,‫" أن ا‬F# ZO 8 -‫(أ‬1
2
. f (x ) = 0 : -‫د‬,‫ ه _ ا‬β ‫ و‬α ‫دان‬+,‫  ن ا‬R  k ‫د‬+,‫ ا‬-S ‫د‬+ - ‫ب‬ 0,25

f k (x ) = 4xe − kx − 2 : /  IR  -0,‫ ا‬f k - ‫د‬+,‫ ا‬-‫ا‬+‫ ا‬F ,"


. (∀x ∈ IR ) : f k '(x ) = 4(1 − kx )e − kx : ‫ أن‬# +‫?آ‬8 -‫( أ‬2 0,25
. f k ‫ات‬g8 ‫ول‬+Q ‫ أﻁ‬- ‫ب‬ 0,5
: R  b ‫ و‬a < ;#  ‫ﻁ‬F AFO8 f k (x ) = 0 -‫د‬,‫ أن ا‬L  M‫ ا‬-‫( أ‬3
0,5
1
a < <b
k
. b = β ‫ و‬a = α : ‫  أن‬- ‫ب‬ 1
t 1
(∀t ∈ IR ) : ∫ xe − kx dx = 2 (1 − kte − kt − e − kt ) : ‫اء  أن‬Q@ -## ‫ل‬, M -‫( أ‬4 0,5
0 k
β
. β ‫ و‬α -+ I k = ∫ f k (x ) dx A# ‫ أ ' ا‬-‫ب‬ 0,5
α

. ln(2α ).ln(2 β ) ≤ 1 : ‫ أن‬L  M‫ ا‬-‫ج‬


0,75
ln(u ) ln(v )
= : R  ,hS FQ # < ;# OO  ‫د‬+ v ‫ و‬u ‫(  أ"ﻩ إذا آن‬5 0,75
u v
. ln(u ).ln(v ) ≤ 1 ‫ن‬n0
‫اﻟﺸﻌﺒﺔ‪ :‬ﻋﻠﻮم رﻳﺎﺿﻴﺔ‬ ‫اﻻﻡﺘﺤﺎن اﻟﻮﻃﻨﻲ اﻟﻤﻮﺣﺪ ﻟﻨﻴﻞ ﺵﻬﺎدة اﻟﺒﻜﺎﻟﻮرﻳﺎ‬ ‫اﻟﻤﻤﻠﻜﺔ اﻟﻤﻐﺮﺏﻴﺔ‬
‫اﻟﻤﺪة‪ 4 :‬ﺳﺎﻋﺎت‬ ‫دورة ﻳﻮﻥﻴﻮ‪2003‬‬ ‫وزارة اﻟﺘﺮﺏﻴﺔ اﻟﻮﻃﻨﻴﺔ و اﻟﺸﺒﺎب‬

‫‪ 03pts‬ﺕﻤﺮﻳﻦ‪1‬‬

‫‪(E):‬‬ ‫(‬ ‫)‬


‫) ‪x2 x2 + 7 = y ( 2 x + 7‬‬ ‫اﻟﻤﻌﺎدﻟﺔ اﻟﺘﺎﻟﻴﺔ ‪:‬‬ ‫‪*2‬‬
‫ﻥﻌﺘﺒﺮ ﻓﻲ‬

‫‪*2‬‬
‫∈ ) ‪ ( x; y‬وﻟﻴﻜﻦ ‪ δ‬اﻟﻘﺎﺳﻢ اﻟﻤﺸﺘﺮك اﻷآﺒﺮ ﻟﻠﻌﺪدﻳﻦ ‪ x‬و ‪. y‬‬ ‫ﻟﻴﻜﻦ‬

‫‪ x =δa‬و ‪y =δb‬‬ ‫ﻥﻀﻊ‬

‫‪ -1‬ﻟﻨﻔﺘﺮض أن ) ‪ ( x; y‬ﺣﻞ ﻟﻠﻤﻌﺎدﻟﺔ ) ‪. ( E‬‬

‫(‬ ‫)‬
‫أ( ﺕﺤﻘﻖ أن‪a 2 δ 2 a 2 + 7 = b ( 2a + b ) :‬‬ ‫‪0,50‬‬

‫ب( اﺳﺘﻨﺘﺞ أﻥﻪ‪ :‬ﻳﻮﺟﺪ ﻋﺪد ﺻﺤﻴﺢ ﻃﺒﻴﻌﻲ ‪ k‬ﺣﻴﺚ ‪ δ 2 a 2 + 7 = kb‬و ‪2a + b = ka 2‬‬ ‫‪0,50‬‬

‫ت( ﺏﻴﻦ أن ‪. a = 1‬‬ ‫‪0,50‬‬

‫ث( اﺳﺘﻨﺘﺞ أن ‪( b + 1)2 = δ 2 + 8‬‬ ‫‪0,75‬‬

‫‪*2‬‬
‫اﻟﻤﻌﺎدﻟﺔ ) ‪. ( E‬‬ ‫‪ -2‬ﺣﻞ ﻓﻲ‬ ‫‪0,75‬‬

‫ﺕﻤﺮﻳﻦ‪2‬‬ ‫‪3pts‬‬
‫اﻟﻤﺴﺘﻮى ﻡﻨﺴﻮب إﻟﻰ ﻡﻌﻠﻢ ﻡﺘﻌﺎﻡﺪ ﻡﻤﻨﻈﻢ ) ‪. ( O; i ; j‬‬

‫‪3‬‬
‫=‪y‬‬ ‫ﻥﻌﺘﺒﺮ اﻟﻤﻨﺤﻨﻰ ) ‪ ( E‬اﻟﺬي ﻡﻌﺎدﻟﺘﻪ ‪16 − x 2‬‬
‫‪4‬‬
‫‪ -1‬أ( ﺏﻴﻦ أن ) ‪ ( E‬ﺟﺰء ﻡﻦ اهﻠﻴﺞ ﻳﺘﻢ ﺕﺤﺪﻳﺪﻩ‪.‬‬ ‫‪0,50‬‬

‫ب( أرﺳﻢ اﻟﻤﻨﺤﻨﻰ ) ‪. ( E‬‬ ‫‪0,50‬‬


‫‪ -2‬ﻟﺘﻜﻦ ) ‪ A ( 4;0‬و )‪ B ( 0;3‬ﻥﻘﻄﺘﻴﻦ‪.‬‬

‫ﻥﻌﺘﺒﺮ اﻟﻨﻘﻄﺔ ‪ M1‬ﻡﻦ ) ‪ ( E‬اﻟﺘﻲ أﻓﺼﻮﻟﻬﺎ ‪ x1‬ﺣﻴﺚ ]‪. x1 ∈ [ 0;4‬‬

‫‪π‬‬
‫≤ ‪. 0 ≤ t1‬‬ ‫ﻥﻀﻊ ‪ x1 = 4cos t1‬ﺣﻴﺚ‬
‫‪2‬‬
‫‪3 4‬‬
‫= ) ‪. I ( x1‬‬ ‫∫‬‫‪x1‬‬
‫و ﻥﻌﺘﺒﺮ اﻟﺘﻜﺎﻡﻞ اﻵﺕﻲ ‪16 − x 2 dx :‬‬
‫‪4‬‬
‫‪π‬‬
‫≤ ‪ ، 0 ≤ t‬ﺏﻴﻦ أن‪:‬‬ ‫أ( ﺏﺎﺳﺘﻌﻤﺎل ﻡﻜﺎﻡﻠﺔ ﺏﺘﻐﻴﻴﺮ اﻟﻤﺘﻐﻴﺮ و ذﻟﻚ ﺏﻮﺿﻊ ‪ x = 4cos t‬ﺣﻴﺚ‬ ‫‪1,25‬‬
‫‪2‬‬
‫) ‪I ( x1 ) = 6t1 − 3sin ( 2t1‬‬
‫ب( ﻟﺘﻜﻦ ) ‪ S ( x1‬ﻡﺴﺎﺣﺔ اﻟﺴﻄﺢ اﻟﻤﺤﺼﻮر ﺏﻴﻦ اﻟﻤﺴﺘﻘﻴﻤﻴﻦ ) ‪ ( OA‬و ) ‪ ( OM 1‬واﻟﻤﻨﺤﻨﻰ ) ‪. ( E‬‬

‫وﻟﺘﻜﻦ ‪ S‬ﻡﺴﺎﺣﺔ اﻟﺴﻄﺢ اﻟﻤﺤﺼﻮر ﺏﻴﻦ اﻟﻤﺴﺘﻘﻴﻤﻴﻦ ) ‪ ( OA‬و ) ‪ ( OB‬واﻟﻤﻨﺤﻨﻰ ) ‪. ( E‬‬

‫• ﺕﺤﻘﻖ أن أرﺕﻮب اﻟﻨﻘﻄﺔ ‪ M 1‬هﻮ ‪3sin t1‬‬ ‫‪0,25‬‬

‫• أﺣﺴﺐ ) ‪ S ( x1‬ﺏﺪﻻﻟﺔ ‪. t1‬‬ ‫‪0,25‬‬


‫• اﺳﺘﻨﺘﺞ ﻗﻴﻤﺔ ‪. S‬‬ ‫‪0,25‬‬
‫‪1‬‬ ‫‪π‬‬
‫‪ * -3‬ﺏﻴﻦ أن = ‪. S ( x1 ) = S ⇔ t1‬‬ ‫‪0,25‬‬
‫‪2‬‬ ‫‪4‬‬
‫‪π‬‬
‫= ‪. t1‬‬
‫‪4‬‬
‫(‬ ‫)‬
‫* ﺣﺪد إﺣﺪاﺛﻴﺘﻲ ‪ M1‬ﻓﻲ اﻟﻨﻌﻠﻢ ‪ O; OA; OB‬ﻓﻲ ﺣﺎﻟﺔ‬
‫‪0,25‬‬

‫ﺕﻤﺮﻳﻦ ‪3‬‬ ‫‪4,5pts‬‬


‫‪ a + b −b ‬‬
‫‪M ( a;b ) = ‬‬ ‫‪ ،‬ﻥﻌﺘﺒﺮ اﻟﻤﺼﻔﻮﻓﺔ‬ ‫‪2‬‬
‫‪ -I‬ﻟﻜﻞ ) ‪ ( a; b‬ﻡﻦ‬
‫‪ b‬‬ ‫‪a ‬‬

‫{‬
‫∈ ) ‪. ( E ) = M ( a ;b ) / ( a; b‬‬ ‫‪2‬‬
‫‪، M‬ﻟﺘﻜﻦ ) ‪ ( E‬ﻡﺠﻤﻮﻋﺔ اﻟﻤﺼﻔﻮﻓﺎت اﻻﺕﻴﺔ‪} :‬‬ ‫‪2‬‬ ‫) (‬ ‫ﻓﻲ‬

‫( ‪ ( M 2‬ﺣﻠﻘﺔ ﺕﺒﺎدﻟﻴﺔ واﺣﺪﻳﺔ‪.‬‬ ‫) × ;‪) ; +‬‬ ‫ﻥﺬآﺮ أن‬

‫( ‪.(M2‬‬ ‫ﺏﻴﻦ أن ) ‪ ( E‬ﺟﺰء ﻡﺴﺘﻘﺮ ﻡﻦ ) ‪ ( M 2 ( ) ; +‬و ﻡﻦ )×; )‬ ‫‪-1‬‬ ‫‪0,75‬‬

‫ﺣﻠﻘﺔ ﺕﺒﺎدﻟﻴﺔ واﺣﺪﻳﺔ‪.‬‬ ‫ﺏﻴﻦ أن )×;‪( ( E ) ; +‬‬ ‫‪-2‬‬ ‫‪0,25‬‬

‫‪ -3‬أ( ﺏﻴﻦ أن ﻟﻜﻞ ﻋﺪدﻳﻦ ﺣﻘﻴﻘﻴﻴﻦ ‪ x‬و ‪ ، y‬ﻟﺪﻳﻨﺎ‪x 2 + xy + y 2 = 0 ⇔ x = y = 0 :‬‬ ‫‪0,50‬‬


‫ب( ﺣﺪد اﻟﻌﻨﺎﺻﺮ اﻟﺘﻲ ﺕﻘﺒﻞ ﻡﻘﻠﻮﺏﺎ ﻓﻲ اﻟﺤﻠﻘﺔ ) ×;‪. ( ( E ) ; +‬‬ ‫‪0,25‬‬
‫ﺟﺴﻢ ﺕﺒﺎدﻟﻲ‪.‬‬ ‫ج( اﺳﺘﻨﺘﺞ أن )×;‪( ( E ) ; +‬‬ ‫‪0,50‬‬
‫‪.‬‬ ‫‪ -II‬ﻟﻴﻜﻦ ‪ σ‬ﻋﺪدا ﻋﻘﺪﻳﺎ ﻻ ﻳﻨﺘﻤﻲ اﻟﻰ‬
‫‪ -1‬ﺏﻴﻦ أن ) ‪ (1;σ‬أﺳﺎس ﻟﻠﻔﻀﺎء اﻟﻤﺘﺠﻬﻲ اﻟﺤﻘﻴﻘﻲ ) ×;‪. ( ; +‬‬ ‫‪0,25‬‬

‫اﻟﻤﻌﺮف ﺏﻤﺎ ﻳﻠﻲ ‪.‬‬ ‫‪ -2‬ﻥﻌﺘﺒﺮ اﻟﺘﻄﺒﻴﻖ ‪ ψ‬ﻡﻦ ) ‪ ( E‬ﻥﺤﻮ‬ ‫‪0,75‬‬

‫‪ψ:‬‬ ‫→ )‪(E‬‬
‫‪M ( a ;b ) → a + σ b‬‬

‫(‬ ‫ﻥﺤﻮ ) ‪;+‬‬ ‫ﺕﻘﺎﺏﻠﻲ ﻡﻦ ) ‪( ( E ) ; +‬‬ ‫ﺏﻴﻦ أن ‪ ψ‬ﺕﺸﺎآﻞ‬

‫اﻟﻤﻌﺎدﻟﺔ ‪z 2 − z + 1 = 0‬‬ ‫‪ -3‬ﻥﻌﺘﺒﺮ ﻓﻲ‬ ‫‪0,75‬‬


‫هﺬﻩ اﻟﻤﻌﺎدﻟﺔ و أآﺘﺐ ﺣﻠﻴﻬﺎ ﻋﻠﻰ اﻟﺸﻜﻞ اﻟﻤﺜﻠﺜﻲ‪.‬‬ ‫ﺣﻞ ﻓﻲ‬
‫‪1‬‬ ‫‪3‬‬
‫=‪σ‬‬ ‫‪+i‬‬ ‫‪ -4‬ﻥﻔﺘﺮض ﻓﻲ هﺬا اﻟﺴﺆال أن‬
‫‪2‬‬ ‫‪2‬‬ ‫‪0,50‬‬
‫(‬ ‫ﻥﺤﻮ ) ×;‬ ‫‪ ψ‬ﺕﺸﺎآﻞ ﻡﻦ )×; ) ‪( ( E‬‬ ‫ﺏﻴﻦ أن‬

‫ﺕﻤﺮﻳﻦ‪4‬‬ ‫‪9pts‬‬
‫‪ln x 1‬‬
‫‪ ، f ( x ) = 4‬و ﻟﻴﻜﻦ ) ‪ ( C‬ﻡﻨﺤﻨﻰ اﻟﺪاﻟﺔ‬ ‫‪ -I‬ﻟﺘﻜﻦ ‪ f‬اﻟﺪاﻟﺔ اﻟﻌﺪدﻳﺔ اﻟﻤﻌﺮﻓﺔ [∞‪ ]0;+‬ﺏﻤﺎ ﻳﻠﻲ ‪−‬‬
‫‪x2 2‬‬
‫‪ f‬ﻓﻲ ﻡﻌﻠﻢ ﻡﺘﻌﺎﻡﺪ ﻡﻤﻨﻈﻢ ) ‪ ( O; i ; j‬و ﺣﺪﺕﻪ ‪. i = j = 2cm‬‬

‫‪ -1‬أﺣﺴﺐ ) ‪ lim+ f ( x‬و ) ‪ ، lim f ( x‬ﺛﻢ ﺣﺪد اﻟﻔﺮﻋﻴﻦ اﻟﻼﻥﻬﺎﺋﻴﻴﻦ ﻟﻠﻤﻨﺤﻨﻰ ) ‪. ( C‬‬ ‫‪0,50‬‬
‫∞‪x→+‬‬ ‫‪x→0‬‬

‫‪ 1 − 2ln x ‬‬


‫[∞‪. ∀x ∈ ]0; +‬‬ ‫‪f '( x ) = 4 ‬‬ ‫‪3‬‬ ‫‪ -2‬أ( ﺏﻴﻦ أن ‪‬‬ ‫‪0,25‬‬
‫‪ x‬‬ ‫‪‬‬
‫ب( أﻋﻂ ﺟﺪول ﺕﻐﻴﺮات ‪. f‬‬ ‫‪0,75‬‬

‫‪ -3‬ﺏﻴﻦ أن اﻟﻤﻌﺎدﻟﺔ ‪ f ( x ) = 0‬ﺕﻘﺒﻞ ﺏﺎﻟﻀﺒﻂ ﺣﻠﻴﻦ ﻡﺨﺘﻠﻔﻴﻦ ‪ β‬و ‪ α‬ﺏﺤﻴﺚ‬


‫‪0,75‬‬
‫) ﻥﻌﻄﻲ ‪(1 ≺ ln 3 ≺ 1,1‬‬ ‫‪1≺ α ≺ e ≺ β ≺ 3‬‬
‫‪ -4‬ﺣﺪد ﻡﻌﺎدﻟﺔ اﻟﻤﻤﺎس ) ‪ (T‬ﻟﻠﻤﻨﺤﻨﻰ ) ‪ ( C‬ﻓﻲ اﻟﻨﻘﻄﺔ اﻟﺘﻲ أﻓﺼﻮﻟﻬﺎ ‪.1‬‬ ‫‪0,50‬‬

‫‪ -5‬أرﺳﻢ اﻟﻤﻨﺤﻨﻰ ) ‪. ( C‬‬ ‫‪0,75‬‬


‫‪ -III‬ﻟﻜﻞ ﻋﺪد ﺻﺤﻴﺢ ‪ n‬ﺏﺤﻴﺚ ‪ ، n ≥ 4‬ﻥﻌﺘﺒﺮ اﻟﺪاﻟﺔ ‪ f n‬اﻟﻤﻌﺮﻓﺔ ﻋﻠﻰ [∞‪ ]0; +‬ﺏﻤﺎ ﻳﻠﻲ‪:‬‬

‫‪ln x 1‬‬
‫‪fn ( x ) = n‬‬ ‫‪−‬‬
‫‪x2 2‬‬
‫و ﻟﻴﻜﻦ ) ‪ ( Cn‬اﻟﻤﻨﺤﻨﻰ اﻟﻤﻤﺜﻞ ﻟﺪاﻟﺔ ‪ f n‬ﻓﻲ ﻡﻌﻠﻢ ﻡﺘﻌﺎﻡﺪ ﻡﻤﻨﻈﻢ‪.‬‬

‫‪ -1‬أدرس ﺕﻐﻴﺮات اﻟﺪاﻟﺔ ‪. f n‬‬ ‫‪0,50‬‬


‫‪5‬‬
‫‪. e6‬‬ ‫‪ -2‬أدرس ﺕﻘﻌﺮ ) ‪ ( Cn‬و ﺏﻴﻦ أﻥﻪ ﻳﻘﺒﻞ ﻥﻘﻄﺔ اﻥﻌﻄﺎف أﻓﺼﻮﻟﻬﺎ‬ ‫‪0,50‬‬
‫‪ -3‬أ( ﻗﺎرن ) ‪ f n ( x‬و ) ‪ f n+1 ( x‬ﺣﺴﺐ ﻗﺒﻢ ‪. x‬‬ ‫‪0,25‬‬

‫ب( اﺳﺘﻨﺞ اﻟﻮﺿﻊ اﻟﻨﺴﺒﻲ ﻟﻠﻤﻨﺤﻨﻴﻴﻦ ) ‪ ( Cn‬و ) ‪. ( Cn+1‬‬ ‫‪0,25‬‬

‫‪ -4‬ﺏﻴﻦ أن اﻟﻤﻌﺎدﻟﺔ ‪ f n ( x ) = 0‬ﺕﻘﺒﻞ ﺏﺎﻟﻀﺒﻂ ﺣﻠﻴﻦ ﻡﺨﺘﻠﻔﻴﻦ ‪ un‬و ‪ vn‬ﺏﺤﻴﺚ ‪1 ≺ un ≺ e ≺ vn‬‬ ‫‪0,50‬‬

‫‪ -5‬ﺏﻴﻦ أن ‪ ( un )n≥4‬ﻡﺘﺘﺎﻟﻴﺔ ﺕﻨﺎﻗﺼﻴﺔ ﻗﻄﻌﺎ )ﻳﻤﻜﻦ اﺳﺘﻌﻤﺎل ﻥﺘﻴﺠﺔ اﻟﺴﺆال ‪.(3-III‬‬ ‫‪050‬‬
‫‪ -6‬أ( ﺏﺎﺳﺘﻌﻤﺎل ﻥﺘﻴﺠﺔ اﻟﺴﺆال ‪ ،(2-II‬ﺏﻴﻦ أن ‪:‬‬ ‫‪0,25‬‬
‫) ‪( un − 1)( 3 − un‬‬
‫‪∀n ≥ 4‬‬ ‫‪≤ ln ( un ) ≤ un − 1‬‬
‫‪2‬‬
‫‪( un ) 2‬‬ ‫‪( un ) 2‬‬
‫‪∀n ≥ 4‬‬ ‫≤ ‪≤ un − 1‬‬ ‫ب( اﺳﺘﻨﺘﺞ أن‪:‬‬
‫‪2n‬‬ ‫) ‪n ( 3 − un‬‬ ‫‪0 ,25‬‬

‫‪1‬‬ ‫‪e‬‬
‫‪∀n ≥ 4‬‬ ‫ج( ﺏﻴﻦ أن ≤ ‪≤ un − 1‬‬ ‫‪0,25‬‬
‫‪2n‬‬ ‫‪n‬‬
‫د( اﺳﺘﻨﺘﺞ أن ‪ ( un )n≥4‬ﻡﺘﻘﺎرﺏﺔ ﻡﺤﺪدا ﻥﻬﺎﻳﺘﻬﺎ‪.‬‬ ‫‪0,50‬‬

‫‪5‬‬ ‫‪5‬‬
‫‪.‬‬ ‫‪( e6‬‬ ‫‪ ) ∀n ≥ 4‬ﻥﻌﻄﻲ ‪≺ 5,3‬‬ ‫‪e6‬‬ ‫‪ -7‬أ( ﺏﻴﻦ أن ‪≺ vn‬‬ ‫‪0,50‬‬
‫ب( اﺳﺘﻨﺘﺞ أن ∞‪lim vn = +‬‬ ‫‪0,50‬‬
‫∞‪n→+‬‬
Corrigé

Exercice 1
1. (a) On remplace x = da et y = db dans (E) :

d2 a2 (d2 a2 + 7) = db(2da + db)


⇔ d2 a2 (d2 a2 + 7) = d2 b(2a + b)

⇔ a2 (d2 a2 + 7) = b(2a + b)

(b) D’après (a), b|a2 (d2 a2 +7). Or comme x∧y = d, a∧b = 1, d’où a2 , b∧ = 1. Par le théorème
de Gauss, b divise donc d2 a2 + 7.
Par conséquent, il existe k ∈ N tel que

d2 a2 + 7 = kb

En remplaçant dans l’égalité du (a), il vient :

a2 kb = b(2a + b)

et comme b 6= 0,
a2 k = 2a + b
(c)
a2 k = 2a + b ⇒ b = a(ka − 2) ⇒ a|b ⇒ a=a∧b ⇒ a=1
(d) On a alors :

(E) ⇔ d2 + 7 = b(2 + b)
(E) ⇔ b2 + 2b = d2 + 7
(E) ⇔ b2 + 2b + 1 = d2 + 8

(E) ⇔ (b + 1)2 = d2 + 8

2. D’après (1), si (x, y) est solution de (E) alors x = x ∧ y, c’est-à-dire x|y, d’où y = bx, et alors :

(b + 1)2 = x2 + 8
⇔ (b + 1)2 − x2 = 8
⇔ (b + 1 + x)(b + 1 − x) = 8 = 23

Or b + 1 + x > b + 1 − x, et b + 1 + x + b + 1 − x = 2(b + 1) qui est paire, donc b + 1 + x et


b + 1 − x sont de même parité, soit pairs, comme leur produit est pair.
On a alors : b + 1 + x = 4 et b + 1 − x = 2 qui sont les seules solutions. On trouve alors le
système suivant :
( (
b+1+x=4 b+x=3

b+1−x=2 b−x=1
(
b=2

x=1

Or y = bx = 2.
Réciproquement, (1, 2) est solution de (E). Donc

S = {(1, 2)}

Exercice 2
1. (a)
9
(E) ⇒ y2 = (16 − x2 )
16
16y 2
⇒ + x2 = 16
9

x2 y 2
⇒ + =1
42 32
On retrouve l’équation d’une ellipse centrée en O, d’axe (Ox), de demi-grand axe a = 4
et de demi-petit axe b = 3.
(E) est la partie de cette ellipse située au-dessus de l’axe (Ox).
(b)
2. (a) Avec le changement de variable x = 4 cos t, on trouve :
3 0√
Z
I(x1 ) = 16 − 16 cos2 t × (−4 sin t) dt
4 t1
3 0
Z √
= −16 1 − cos2 t sin t dt
4 t1
Z t1
= 12 sin2 t dt
Z0 t1
1 − cos 2t
= 12 dt
0 2
 t
1 sin 2t 1
= 12 × (t1 − 0) − 6
2 2 0

I(x1 ) = 6t1 − 3 sin 2t1


(b) – M1 (x1 , y1 ) appartient à (E), donc
3
q
y1 = 16 − x21
4
Or x1 = 4 cos t1 , d’où :
3p
y1 = 16 − 16 cos2 t1
4p
= 3 1 − cos2 t1

y1 = 3 sin t1
– En faisant une figure, on voit facilement que
x1 × y1
S(x1 ) = I(x1 ) +
2
1
= I(x1 ) + × 4 cos t1 × 3 sin t1
2
= I(x1 ) + 3 × 2 sin t1 cos t1
= I(x1 ) + 3 sin 2t1
= 6t1 − 3 sin 2t1 + 3 sin t1

S(x1 ) = 6t1
– Clairement,
π π
S = S(0) = S(4 cos ) = 6 × = 3π
2 2
– On a l’équation :
1 3π π
S(x1 ) = S ⇔ 6t1 = ⇔ t1 =
2 2 4

– M1 a pour coordonnées (4 cos t1 , 3 sin t1 ), donc


−−−→
OM1 = 4 cos t1~ı + 3 sin t1~
= cos t1 × 4~ı + sin t1 × 3~
−→ −−→
= cos t1 OA + sin t1 OB
π
D’où, pour t1 = , M1 a pour coordonnées
4
√ √ !
2 2
M1 ,
2 2

Exercice 3

I.

1. Soit M(a1 ,b1 ) , M(a2 ,b2 ) ∈ E :


   
a1 + b1 −b1 a2 + b2 −b2
M(a1 ,b1 ) = M(a2 ,b2 ) =
b1 a1 b2 a2

Pour la somme :
 
(a1 + a2 ) + (b1 + b2 ) −(b1 + b2 )
M(a1 ,b1 ) + M(a2 ,b2 ) = = M(a1 +a2 ,b1 +b2 ) ∈ E
(b1 + b2 ) (a1 + a2 )

Donc E est stable pour la loi +.


Pour le produit :
 
(a1 + b1 )(a2 + b2 ) − b1 b2 −(a1 + b1 )b− b1 a2
M(a1 ,b1 ) M(a2 ,b2 ) =
b1 (a2 + b2 ) + a1 b2 −b1 b2 + a1 a2
 
a1 a2 + a1 b2 + b1 a2 + b1 b2 − b1 b2 −a1 b2 − b1 b2 − b1 a2
=
b 1 a2 + b 1 b 2 + a1 b 2 −b1 b2 + a1 a2
 
(a1 a2 − b1 b2 ) + (a1 b2 + b1 b2 + b1 a2 ) −(a1 b2 + b1 b2 + b1 a2 )
=
a1 b 2 + b 1 b 2 + b 1 a2 a1 a2 − b1 b2

M(a1 ,b1 ) M(a2 ,b2 ) = M(a1 a2 −b1 b2 ,a1 b2 +b1 b2 +b1 a2 ) ∈ E

Donc E est stable pour la loi ×.


E est donc une partie stable de (M2 (R), +) et de (M2 (R), ×).
2. (E, +) est un sous-groupe de M2 (R) car E est non vide et E est stable par +, et pour tout
M(a,b) ∈ E, on a −M(a,b) = M(−a,−b) ∈ E.
En outre, la stabilité de E par × suffit pour conclure que (E, +, ×) est un sous-anneau de
l’anneau (M2 (R), +, ×).
Pour tout (a1 , b1 ) et (a2 , b2 ) de R2 , on a :
 
(a2 + b2 )(a1 + b1 ) − b1 b2 −(a2 + b2 )b1 − b2 a1
M(a2 ,b2 ) M(a1 ,b1 ) =
(a1 + b1 )b2 + a2 b1 −b1 b2 + a1 a1
 
(a1 a2 − b1 b2 ) + (a1 b2 + b1 b2 + b1 a2 ) −(a1 b2 + b1 b2 + b1 a2 )
=
a 1 b 2 + b 1 b 2 + b 1 a2 a 1 a2 − b 1 b 2
= M(a1 a2 −b1 b2 ,a1 b2 +b1 b2 +b1 a2 )

M(a2 ,b2 ) M(a1 ,b1 ) = M(a1 ,b1 ) M(a2 ,b2 )

Donc (E, +, ×) est un anneau commutatif.


Puisque I = M(1,0) ∈ E, E est unitaire et d’unité la matrice unité I.
3. (a) Résolvons l’équation du second degré x2 +xy+y 2 = 0 en x. L’équation a pour discriminant :

∆ = y 2 − 4y 2 = −3y 2 6 0

L’équation admet une solution réelle uniquement pour y = 0. Cette solution est alors :
y
x=− =0
2
Ainsi,
x2 + xy + y 2 = 0 ⇔ x=y=0.
(b) Un élément M(a,b) de E est inversible si et seulement si det M(a,b) 6= 0.
Or
det M(a,b) = a(a + b) + b2 = a2 + ab + b2
Le déterminant est donc nul uniquement si a = b = 0, soit pour M(0,0) .
Ainsi, tous les éléments de E sauf M(0,0) sont inversibles.
(c) (E, +, ×) est un anneau commutatif non réduit à {0}, et tout élément non nul de E est
inversible pour le produit.
Par conséquent, (E, +, ×) est un corps commutatif.

II.
1. Posons u = x + iy, avec x ∈ R et y ∈ R∗ (u n’est pas réel). Soit (a, b) ∈ R2 , montrons que
a + ub = 0 ⇒ a = b = 0.
(
a + xb = 0
a + ub = 0 ⇔ a + xb + iyb = 0 ⇔
yb = 0
Or y 6= 0, donc on a b = 0, ce qui donne aussi a = 0. Ainsi,

∀(a, b) ∈ R2 , a + ub = 0 ⇒ a = b = 0

Donc (1, u) est une famille libre.


Montrons que (1, u) est génératrice. Prenons alors v ∈ C, et montrons qu’il existe (a, b) ∈ R2
tels que v = a + bu.
On a v = v1 + iv2 , avec (v1 , v2 ) ∈ R2 , et ∀(a, b) ∈ R2 , a + bu = a + xb + iyb.
v2 v2 x
Soit, en posant b = et a = v1 − bx = v1 − , on a bien (a, b) ∈ R2 tels que
y y
v2 x v2 v2
a + bu = v1 − + x + i y = v1 + iv2 = v
y y y
Par conséquent,
∀v ∈ C, ∃(a, b) ∈ R2 , v = a + bu
Donc (1, u) est une famille génératrice.
Par conséquent, (1, u) est une base de l’espace vectoriel réel (C, +, ·).
2. Soit (M(a1 ,b1 ) , M(a2 ,b2 ) ) ∈ E 2 . Alors :
ψ(M(a1 ,b1 ) + M(a2 ,b2 ) ) = ψ(M(a1 +a2 ,b1 +b2 ) )
= a1 + a2 + (b1 + b2 )u
= (a1 + b1 u) + (a2 + b2 u)

ψ(M(a1 ,b1 ) + M(a2 ,b2 ) ) = ψ(M(a1 ,b1 ) ) + ψ(M(a2 ,b2 ) )


ψ est donc un morphisme de (E, +) dans (C, +). Montrons que ψ est bijective.
On voit clairement qu’à chaque matrice M(a,b) ∈ E correspond un unique couple (a, b) ∈ R2
et réciproquement. De plus, comme (1, u) est une base de (C, +, ·), à chaque z ∈ C correspond
un unique couple (a, b) ∈ R2 tel que z = a + bu, et réciproquement.
Par conséquent, à chaque matrice M(a,b) ∈ E correspond un unique z ∈ C et réciproquement,
donc ψ est bijective.
On a donc montré que ψ est un isomorphisme de groupe de (E, +) sur (C, +).
3. L’équation dans C z 2 − z + 1 = 0 a pour discriminant :
∆ = 1 − 4 = −3
Elle admet donc deux solutions complexes conjuguées, à savoir :

1±i 3
z=
2
Donc ( √ √ )
1+i 3 1−i 3
S= ,
2 2

Sous forme trigonométrique, cela nous donne :


n π π  π  π o
S = cos + i sin , cos − + i sin −
3 3 3 3

4. Remarquons premièrement que u est solution de l’équation z 2 − z + 1 = 0, donc u2 = u − 1.


On a :
ψ(M(a1 ,b1 ) × M(a2 ,b2 ) ) = ψ(M(a1 a2 −b1 b2 ,a1 b2 +b1 b2 +b1 a2 ) )
= a1 a2 − b1 b2 + (a1 b2 + b1 b2 + b1 a2 )u
et
ψ(M(a1 ,b1 ) ) × ψ(M(a2 ,b2 ) ) = (a1 + b1 u)(a2 + b2 u)
= a1 a2 + (a1 b2 + b1 a2 )u + b1 b2 u2
= a1 a2 + (a1 b2 + b1 a2 )u + b1 b2 (u − 1)
= a1 a2 − b1 b2 + (a1 b2 + b1 b2 + b1 a2 )u
ψ(M(a1 ,b1 ) × M(a2 ,b2 ) ) = ψ(M(a1 ,b1 ) ) × ψ(M(a2 ,b2 ) )

D’où ψ est un morphisme de (E, ×) vers (C, ×). On a montré auparavant que ψ est bijective,
donc ψ est un isomorphisme de (E, ×) sur (C, ×).

Exercice 4

I.
1. En 0+ :
lim ln x = −∞
x→0+

et
4
lim+ = +∞
x→0 x2
d’où :
lim f (x) = −∞
x→0+

En +∞ :
ln x
lim =0
x→+∞ x2

donc
1
lim f (x) = −
x→+∞ 2
1
Donc (C) admet une asymptote horizontale en +∞ d’équation y = − .
2
2. (a) f est dérivable sur ]0, +∞[ comme quotient et somme de fonctions dérivables sur cet
intervalle. Pour tout x de ]0, +∞[ :

x2 /x − 2x ln x
f 0 (x) = 4
x4

1 − 2 ln x
f 0 (x) = 4
x3
(b) Pour tout x > 0, x3 > 0, donc f 0 (x) est du signe de 1 − 2 ln x. x 7→ 1 − 2 ln x est
strictement décroissante
√ (car ln est strictement croissante) sur ]0, +∞[, et s’annule pour
ln x = 1/2 ⇔ x = e. On a donc le tableau de variations suivant pour f :

x 0 e +∞
f0 + −
2e−1 − 1/2
f % &
−∞ −1/2
3. f est dérivable donc continue sur ]0, +∞[.
√ √
f est continue √ et strictement croissante sur ]0, e], donc la restriction de f à ]0, e] est une
bijection de ]0, e] sur ] − ∞, 2e−1 −1
√ − 1/2]. Ainsi, tout−1élément de ] − ∞, 2e − 1/2] admet un
√ e[. Or 0 ∈] − ∞, 2e − 1/2], donc l’équation f (x) = 0 admet
unique antécédent par f dans ]0,
une unique solution a dans ]0, e].

De
√ même, f est continue et strictement
√ décroissante sur [ e, +∞[, donc la restriction de f à
[ e, +∞[ est une bijection de [ e, +∞[ sur [2e−1 − 1/2, −1/2]. Ainsi, tout élément de [2e−1 −

1/2, −1/2] admet un unique antécédent par f dans [ e,√+∞[. Or 0 ∈ [2e−1 − 1/2, −1/2], donc
l’équation f (x) = 0 admet une unique solution b dans [ e, +∞[.
De plus, on a √
f (1) = −1/2 < f (a) = 0 < f ( e) = 2e−1 − 1/2

et f est strictement croissante sur ]0, e], donc

1<a< e

De même : √
f ( e) > f (b) = 0 > f (3)
car
4 ln 3 4, 4 1 1
1 < ln 3 < 1, 1 ⇒ 4/9 < 2 < ⇒ − < f (3) < − < 0
3 9 18 90

et f est strictement décroissante sur [ e, +∞[, donc

e<b<3

Par conséquent, l’équation f (x) = 0 admet exactement deux solutions a et b vérifiant :



1<a< e<b<3

4. On a f (1) = −1/2 et f 0 (1) = 4, d’où l’équation de (T ), tangente à (C) au point d’abscisse 1,


a pour équation :
(T ) : y = f 0 (1)(x − 1) + f (1)
9
y = 4x −
2
5.

II.
1. Pour tout t > 0 :
1+t>1
1
⇔ 61
1+t
(car x 7→ 1/x strictement décroissante sur ]0, +∞[).
De plus,
1 − t2 6 1
⇔ (1 − t)(1 + t) 6 1
1
⇔ 1−t6
1+t
car 1 + t > 0.
D’où on trouve la double inégalité cherchée : pour tout t de [0, +∞[,

1
1−t6 61
1+t

2. En intégrant la relation précédente entre 0 et a :


Z a Z a Z a
dt
(1 − t) dt 6 6 dt
0 0 1+t 0

a2
⇔ a− 6 ln(1 + t) 6 a
2
III.
1. fn est dérivable sur ]0, +∞[ comme quotient de fonctions dérivables. Pour tout x de ]0, +∞[,

1 − 2 ln x
fn0 (x) = n
x3

Les variations sont les mêmes que pour f ...


2. fn0 est aussi dérivable sur R∗+ et pour tout x de R∗+ ,

−2x3 /x − 3x2 (1 − 2 ln x)
fn00 (x) = n
x6
6 ln x − 5
fn00 (x) = n
x4
On a n > 0, x4 > 0, donc fn00 est du signe de x 7→ 6 ln x − 5, qui est strictement croissante sur
]0, +∞[, et s’annule pour x = e5/6 .
Ainsi, fn00 est strictement négative sur ]0, e5/6 [, s’annule en e5/6 , et est strictement positive sur
]e5/6 , +∞[.
D’où, (Cn ) est concave sur ]0, e5/6 [, convexe sur ]e5/6 , +∞[, et admet un point d’inflexion d’abs-
cisse e5/6 .
3. (a)
ln x
fn+1 (x) − fn (x) =
x2
et pour tout x de ]0, +∞[, x2 > 0, donc : pour 0 < x < 1, fn+1 (x) < fn (x) ; pour x > 1,
fn+1 (x) > fn (x). En x = 1, fn+1 (x) = fn (x).
(b) Il en découle que sur ]0, 1[, (Cn+1 ) est en dessous de (Cn ), et sur ]1, +∞[, (Cn ) est en-
dessous de (Cn+1 ). Les deux courbes se coupent au point d’abscisse 1.
4. On prend exactement la même démonstration qu’au I-3, en remplaçant a et b par un et vn .
5. On sait que pour tout x de ]1, +∞[, fn+1 (x) > fn (x). Or un ∈]1, +∞[, d’où

fn+1 (un ) > fn (un ) = 0

Or fn+1 (un+1 ) = 0, d’où :


fn+1 (un ) > fn+1 (un+1 )
√ √
fn+1 est strictement croissante sur ]1, e[, et un , un+1 ∈]1, e[, donc :

un > un+1

La suite (un )n>4 est donc strictement décroissante.


6. (a) Pour tout n > 4, un > 1, donc un − 1 > 0. En appliquant la formule de II-2, on trouve
alors :
(un − 1)2
un − 1 − 6 ln un 6 un − 1
2
(un − 1)(2 − un + 1)
⇔ 6 ln un 6 un − 1
2
(un − 1)(3 − un )
⇔ 6 ln un 6 un − 1
2
(b) Partons du fait que fn (un ) = 0 :

n ln un 1
fn (un ) = − =0
u2n 2

u2n
⇔ ln un =
2n
Or d’après la question précédente,

u2n
ln un = 6 un − 1.
2n
De plus,
(un − 1)(3 − un ) u2
6 ln un = n
2 2n
2u2n
⇔ un − 1 6
2n(3 − un )

(la division par 3 − un ne change pas le sens de l’inégalité car un < e < 3.
Ainsi, on trouve l’inégalité cherchée : pour tout n > 4,

u2n u2n
6 un − 1 6
2n n(3 − un )

√ 1 u2 u2n e
(c) Par ailleurs, on sait que 1 < un < e, donc 6 n , et 6 .
2n 2n n(3 − un ) n(3 − un )
De plus,

un 6 e ≈ 1, 65 6 2
⇔ un + 1 6 3
⇔ 3 − un > 1
1
⇔ 61
3 − un
e e
⇔ 6
n(3 − un ) n

Ainsi, on trouve :
1 e
6 un − 1 6
2n n
(d) Clairement,
1 e
lim = lim =0
n→+∞ 2n n→+∞ n

Donc, d’après le théorème des gendarmes,

lim un − 1 = 0
n→+∞

d’où :
lim un = 1
n→+∞

7. (a) Calculons fn (e5/6 ) pour tout n > 4 :


5n 1
fn (e5/6 ) = 5/3

6e 2
5×4 1
> −
6 × 5, 3 2
> 0, 12
>0

Or fn (vn ) = 0, donc fn (e5/6 ) > fn (vn ), d’où comme vn , e5/6 ∈ [ e, +∞[ et fn est stricte-
ment décroissante sur cet intervalle,

vn > e5/6 .

(b) Pour tout n > 4, par définition fn (vn ) = 0 donne vn2 = 2n ln(vn ).
Le (7a) donne ln vn > 5/6.
D’où
5
vn2 > n
3
r
5
vn > n
3
r
5
Et comme lim n = +∞, on a
n→+∞ 3

lim vn = +∞ .
n→+∞
‫اﻟﺸﻌﺒﺔ‪ :‬ﻋﻠﻮم رﻳﺎﺿﻴﺔ‬ ‫اﻻﻡﺘﺤﺎن اﻟﻮﻃﻨﻲ اﻟﻤﻮﺣﺪ ﻟﻨﻴﻞ ﺵﻬﺎدة اﻟﺒﻜﺎﻟﻮرﻳﺎ‬ ‫اﻟﻤﻤﻠﻜﺔ اﻟﻤﻐﺮﺏﻴﺔ‬
‫اﻟﻤﺪة‪ 4 :‬ﺳﺎﻋﺎت‬ ‫اﻟﺪورةاﻻﺳﺘﺪراآﻴﺔ‪ :‬ﻳﻮﻟﻴﻮز‪2003‬‬ ‫وزارة اﻟﺘﺮﺏﻴﺔ اﻟﻮﻃﻨﻴﺔ و اﻟﺸﺒﺎب‬

‫‪ 03pts‬ﺕﻤﺮﻳﻦ‪1‬‬
‫ﻟﺪﻳﻨﺎ ﺻﻨﺪوﻗﺎن ‪ U‬و‪ . V‬اﻟﺼﻨﺪوق ‪ U‬ﻳﺤﺘﻮي ﻋﻠﻰ ‪ 4‬آﺮات ﺣﻤﺮاء و ‪ 4‬آﺮات زرﻗﺎء‬
‫اﻟﺼﻨﺪوق ‪ V‬ﻳﺤﺘﻮي ﻋﻠﻰ آﺮﺕﻴﻦ ﺣﻤﺮاوﻳﻴﻦ و ‪ 4‬آﺮات زرﻗﺎء‬
‫ﻥﻌﺘﺒﺮ اﻟﺘﺠﺮﺏﺔ اﻻﺕﻴﺔ‪:‬‬
‫ﻥﺴﺤﺐ ﻋﺸﻮاﺋﻴﺎ آﺮة ﻡﻦ اﻟﺼﻨﺪوق ‪ ،U‬اذا آﺎﻥﺖ ﺣﻤﺮاء ﻥﻀﻌﻬﺎ ﻓﻲ اﻟﺼﻨﺪوق ‪ V‬ﺛﻢ ﻥﺴﺤﺐ ﻋﺸﻮاﺋﻴﺎ‬
‫آﺮة ﻡﻦ اﻟﺼﻨﺪوق ‪ ،V‬و اذا آﺎﻥﺖ زرﻗﺎء ﻥﻀﻌﻬﺎ ﺝﺎﻥﺒﺎ ﺛﻢ ﻥﺴﺤﺐ ﻋﺸﻮاﺋﻴﺎ آﺮة ﻡﻦ اﻟﺼﻨﺪوق ‪V‬‬
‫وﻟﺘﻜﻦ اﻷﺣﺪاث اﻟﺘﺎﻟﻴﺔ‪:‬‬
‫‪ " R1‬اﻟﻜﺮة اﻟﻤﺴﺤﻮﺏﺔ ﻡﻦ ‪ U‬ﺣﻤﺮاء"‬
‫‪ " B1‬اﻟﻜﺮة اﻟﻤﺴﺤﻮﺏﺔ ﻡﻦ ‪ U‬زرﻗﺎء"‬
‫‪ " R2‬اﻟﻜﺮة اﻟﻤﺴﺤﻮﺏﺔ ﻡﻦ ‪V‬ﺣﻤﺮاء"‬
‫‪ " B2‬اﻟﻜﺮة اﻟﻤﺴﺤﻮﺏﺔ ﻡﻦ ‪V‬ﺣﻤﺮاء"‬
‫‪ -1‬أﺣﺴﺐ اﺣﺘﻤﺎل ‪ R1‬و ‪B1‬‬ ‫‪1‬‬

‫‪ -2‬أﺣﺴﺐ اﺣﺘﻤﺎل " ‪ B2‬ﻋﻠﻤﺎ أن ‪ R1‬ﻡﺤﻘﻖ" و اﺣﺘﻤﺎل " ‪ B2‬ﻋﻠﻤﺎ أن ‪ B1‬ﻡﺤﻘﻖ"‬ ‫‪1‬‬
‫‪13‬‬
‫= ) ‪p ( B2‬‬ ‫‪ -3‬ﺏﻴﻦ أن‬ ‫‪0,50‬‬
‫‪21‬‬
‫‪ -4‬اﺳﺘﻨﺘﺞ ) ‪p ( R2‬‬ ‫‪0,50‬‬

‫ﺕﻤﺮﻳﻦ‪2‬‬ ‫‪4,50‬‬
‫ﻟﻴﻜﻦ ‪ θ‬ﻋﺪدا ﺣﻘﻴﻘﻴﺎ ﺏﺤﻴﺚ ‪ 0 ≤ θ ≺ 2π‬ﻥﻀﻊ ‪. p = 5cosθ + 3i sin θ‬‬

‫‪(E) :‬‬ ‫اﻟﻤﻌﺎدﻟﺔ ) ‪ ( E‬اﻟﺘﺎﻟﻴﺔ ‪z 2 − 2 pz + 16 = 0‬‬ ‫ﻥﻌﺘﺒﺮ ﻓﻲ‬

‫‪ -1‬أ( ﺕﺤﻘﻖ ان ‪p 2 − ( 3cos θ + 5i sin θ ) = 16‬‬ ‫‪0,50‬‬


‫‪2‬‬

‫اﻟﻤﻌﺎدﻟﺔ ) ‪ : ( E‬ﻥﺮﻡﺰ ﺏـ ‪ z1‬و ‪ z2‬ﻟﺤﻠﻲ اﻟﻤﻌﺎدﻟﺔ ) ‪ ( E‬ﺣﻴﺚ ‪z1 ≺ z2‬‬ ‫ب( ﺣﻞ ﻓﻲ‬ ‫‪0,50‬‬
‫‪ -2‬اﻟﻤﺴﺘﻮى اﻟﻌﻘﺪي ﻡﻨﺴﻮب اﻟﻰ ﻡﻌﻠﻢ ﻡﺘﻌﺎﻡﺪ ﻡﻤﻨﻈﻢ ﻡﺒﺎﺵﺮ ) ‪ ، ( O; u ; v‬ﻥﻌﺘﺒﺮ اﻟﻨﻘﻄﺘﻴﻦ ‪ M1‬و ‪ M 2‬اﻟﻠﺘﻴﻦ‬

‫ﻟﺤﻘﺎهﻤﺎ ﻋﻠﻰ اﻟﺘﻮاﻟﻲ ‪ z1‬و ‪. z2‬‬


‫أ( ﺏﻴﻦ أﻥﻪ ﻋﻨﺪﻡﺎ ﻳﺘﻐﻴﺮ ‪ θ‬ﻓﻲ [ ‪ [ 0; 2π‬ﻓﺎن اﻟﻨﻘﻄﺔ ‪ M1‬ﺕﺘﻐﻴﺮ ﻋﻠﻰ داﺋﺮة ) ‪ ( C‬ﻳﻨﺒﻐﻲ ﺕﺤﺪﻳﺪ ﻡﻌﺎدﻟﺘﻬﺎ‬ ‫‪0,50‬‬

‫ب( ﻟﺘﻜﻦ ‪ P‬ﻡﻨﺘﺼﻒ ] ‪. [ M1M 2‬‬


‫‪0,50‬‬
‫‪ θ‬ﻓﻲ [ ‪[0; 2π‬‬ ‫و ﻟﺘﻜﻦ ) ‪ ( Γ‬ﻡﺠﻤﻮﻋﺔ اﻟﻨﻘﻂ ‪ P‬ﻋﻨﺪﻡﺎ ﻳﺘﻐﻴﺮ‬
‫ﺏﻴﻦ أن ) ‪ ( Γ‬اهﻠﻴﺞ ﺏﺆرﺕﺎﻩ اﻟﻨﻘﻄﺘﺎن ‪ F‬و ' ‪ F‬اﻟﻠﺘﻴﻦ ﻟﺤﻘﺎهﻤﺎ ‪ 4‬و ‪.-4‬‬

‫‪b+4‬‬ ‫‪a+4‬‬ ‫‪0,50‬‬


‫‪‬‬ ‫‪=−‬‬ ‫ﻟﺪﻳﻨﺎ‪ ⇔ ( ab = 16 ) :‬‬ ‫‪-3‬أ( ﺏﻴﻦ أﻥﻪ ﻟﻜﻞ ﻋﺪدﻳﻦ ﻋﻘﺪﻳﻴﻦ ‪ a‬و ‪ b‬ﻡﻦ }‪− {4‬‬
‫‪b−4‬‬ ‫‪a−4‬‬
‫‪z2 + 4‬‬ ‫‪z +4‬‬
‫‪=− 1‬‬ ‫ب( اﺳﺘﻨﺘﺞ أن‬ ‫‪0,50‬‬
‫‪z2 − 4‬‬ ‫‪z1 − 4‬‬

‫‪( M F ; M F ') ≡ π + ( M‬‬


‫‪1‬‬ ‫‪1‬‬ ‫' ‪2F; M 2F‬‬ ‫)‬ ‫] ‪[ 2π‬‬ ‫ج( ﺏﻴﻦ أن‬ ‫‪0,50‬‬

‫‪ -4‬أ( ﺏﻴﻦ أن ﻡﻌﺎدﻟﺔ اﻟﻤﻤﺎس ) ‪ (T‬ﻟﻠﻤﻨﺤﻨﻰ ) ‪ ( Γ‬ﻓﻲ اﻟﻨﻘﻄﺔ ‪ P‬هﻲ ‪3 x cos θ + 5 y sin θ = 15‬‬ ‫‪0,50‬‬

‫ب( ﺏﻴﻦ أن اﻟﻤﻤﺎس ) ‪ (T‬ﻋﻤﻮدي ﻋﻠﻰ ) ‪. ( M1M 2‬‬ ‫‪0,50‬‬

‫ﺕﻤﺮﻳﻦ ‪3‬‬ ‫‪03‬‬


‫‪ a‬‬ ‫‪b 2‬‬
‫‪M ( a;b ) = ‬‬ ‫‪ ،‬ﻥﻌﺘﺒﺮ اﻟﻤﺼﻔﻮﻓﺔ ‪‬‬ ‫‪2‬‬
‫‪ -I‬ﻟﻜﻞ ) ‪ ( a; b‬ﻡﻦ‬
‫‪b 2‬‬ ‫‪a‬‬ ‫‪‬‬
‫‪‬‬ ‫‪‬‬

‫{‬ ‫}‬
‫( ‪، M 2‬ﻟﺘﻜﻦ ) ‪ ( E‬ﻡﺠﻤﻮﻋﺔ اﻟﻤﺼﻔﻮﻓﺎت اﻻﺕﻴﺔ‪. ( E ) = M ( a;b ) / a 2 − 2b 2 = 1 :‬‬ ‫)‬ ‫ﻓﻲ‬

‫‪ 3‬‬ ‫‪2 2‬‬


‫‪ ، A = ‬ﺕﺤﻘﻖ أن ) ‪A ∈ ( E‬‬ ‫‪ -1‬ﻥﻀﻊ ‪‬‬ ‫‪0,25‬‬
‫‪2 2‬‬ ‫‪3 ‬‬
‫‪‬‬

‫أ‪ -‬ﺏﻴﻦ أن ) ‪ ( E‬ﺝﺰء ﻡﺴﺘﻘﺮ ﻓﻲ )×; ) ( ‪ ( M 2‬و أن اﻟﻘﺎﻥﻮن × ﺕﺒﺎدﻟﻲ ﻓﻲ ) ‪( E‬‬ ‫‪-2‬‬ ‫‪0,50‬‬

‫ب‪ -‬ﺏﻴﻦ أن ﺝﻤﻴﻊ ﻋﻨﺎﺻﺮ ) ‪ ( E‬ﺕﻘﺒﻞ ﻡﻘﻠﻮﺏﺎ ﻓﻲ ) ‪ ( E‬ﺏﺎﻟﻨﺴﺒﺔ ﻟﻘﺎﻥﻮن اﻟﺘﺮآﻴﺐ اﻟﺪاﺧﻠﻲ ×‬ ‫‪0,50‬‬

‫زﻡﺮة ﺕﺒﺎدﻟﻴﺔ‪.‬‬ ‫ج‪ -‬ﺏﻴﻦ أن )×; ) ‪( ( E‬‬ ‫‪0,50‬‬

‫‪1 0‬‬
‫{‬
‫∈ ‪G = An / n‬‬ ‫‪ . A‬ﻥﻌﺘﺒﺮ اﻟﻤﺠﻤﻮﻋﺔ }‬ ‫‪n +1‬‬
‫‪= An × A‬‬ ‫‪ A0 = ‬و ﻟﻜﻞ ‪ n‬ﻡﻦ‬ ‫‪ -3‬ﻥﻀﻊ ‪‬‬
‫‪0 1‬‬
‫أ( ﺕﺤﻘﻖ أن ) ‪G ⊂ ( E‬‬ ‫‪0,25‬‬

‫× ﻓﻲ ) ‪( E‬‬ ‫ب( ﻟﺘﻜﻦ ‪ H‬ﻡﺠﻤﻮﻋﺔ ﻡﻤﺎﺛﻼت ﻡﺼﻔﻮﻓﺎت ‪ G‬ﺏﺎﻟﻨﺴﺒﺔ ﻟﻠﻌﻤﻠﻴﺔ‬ ‫‪0,50‬‬


‫‪ 3‬‬ ‫‪−2 2 ‬‬
‫‪B‬‬
‫‪ −2 2‬‬ ‫‪3 ‬‬
‫{‬
‫∈ ‪ H = B n / n‬ﺣﻴﺚ ‪‬‬ ‫ﺏﻴﻦ أن }‬
‫‪‬‬

‫‪ G ∪ H‬زﻡﺮة ﺝﺰﺋﻴﺔ ﻡﻦ )×; ) ‪( ( E‬‬ ‫ج( ﺏﻴﻦ أن‬ ‫‪0,50‬‬


‫ﺕﻤﺮﻳﻦ‪4‬‬ ‫‪9,50‬‬
‫*‬
‫∈‪.n‬‬ ‫‪ -I‬ﻟﻴﻜﻦ‬
‫ﺏﻤﺎ ﻳﻠﻲ ‪ ، g n ( x ) = x + e − nx‬و ﻟﻴﻜﻦ ) ‪ ( Cn‬ﻡﻨﺤﻨﻰ اﻟﺪاﻟﺔ‬ ‫ﻥﻌﺘﺒﺮ ‪ g n‬اﻟﺪاﻟﺔ اﻟﻌﺪدﻳﺔ اﻟﻤﻌﺮﻓﺔ‬

‫‪ gn‬ﻓﻲ ﻡﻌﻠﻢ ﻡﺘﻌﺎﻡﺪ ﻡﻤﻨﻈﻢ ) ‪( O; i ; j‬‬


‫‪ -1‬أ‪ -‬أدرس ﺕﻐﻴﺮات ‪g n‬‬ ‫‪0,50‬‬
‫ب‪ -‬ﺏﻴﻦ أن ‪ g n‬ﺕﻘﺒﻞ ﻗﻴﻤﺔ دﻥﻴﺎ ﻋﻨﺪ ﻋﺪد ﺣﻘﻴﻘﻲ ‪ un‬ﻳﺘﻢ ﺕﺤﺪﻳﺪﻩ ﺏﺪﻻﻟﺔ ‪n‬‬ ‫‪0,50‬‬

‫‪ -2‬أ( أﺣﺴﺐ ) ‪ lim g n ( x‬و ) ‪، lim g n ( x‬‬ ‫‪0,50‬‬


‫∞‪x →−‬‬ ‫∞‪x →+‬‬

‫ب( ﺣﺪد اﻟﻔﺮﻋﻴﻦ اﻟﻼﻥﻬﺎﺋﻴﻴﻦ ﻟﻠﻤﻨﺤﻨﻰ ) ‪( Cn‬‬ ‫‪0,50‬‬

‫‪ -3‬أ( أدرس اﻟﻮﺿﻊ اﻟﻨﺴﺒﻲ ﻟﻠﻤﻨﺤﻨﻴﻴﻦ ) ‪ ( C1‬و ) ‪ ( C2‬اﻟﻤﺜﻠﻴﻦ ‪ g1‬و ‪g 2‬‬ ‫‪0,50‬‬

‫; ‪( ln 2 ≈ 0;7‬‬ ‫)ﻥﺄﺧﺪ ‪i = j = 2cm‬‬ ‫) ‪ ( C1‬و ) ‪( C2‬‬ ‫ب( أﻥﺸﺊ‬ ‫‪0,50‬‬

‫‪x‬‬
‫‪ -4‬أ( ﺏﺎﺳﺘﻌﻤﺎل ﻡﻜﺎﻡﻠﺔ ﺏﺎﻷﺝﺰاء أﺣﺴﺐ ﺏﺪﻻﻟﺔ ‪ x‬اﻟﺘﻜﺎﻡﻞ‪I ( x ) = ∫ te −2t dt :‬‬ ‫‪0,50‬‬
‫‪0‬‬

‫ب( ﻟﺘﻜﻦ ‪ h2‬ﻗﺼﻮر اﻟﺪاﻟﺔ ‪ g 2‬ﻋﻠﻰ ]‪. [ 0;ln 2‬‬ ‫‪0,50‬‬

‫أﺣﺴﺐ ﺣﺠﻢ ﻡﺠﺴﻢ اﻟﺪوران اﻟﻤﻮﻟﺪ ﻡﻦ دوران اﻟﺘﻤﺜﻴﻞ اﻟﻤﺒﻴﺎﻥﻲ ﻟـ ‪ h2‬ﺣﻮل ﻡﺤﻮر اﻷﻓﺎﺻﻴﻞ‪.‬‬
‫‪ -5‬ﻥﻀﻊ ) ‪vn = g n ( un‬‬
‫‪1‬‬
‫∈‪ ( vn )n‬ﻡﺘﻘﺎرﺏﺘﺎن و ﺣﺪد ﻥﻬﺎﻳﺘﻬﻤﺎ‪.‬‬ ‫*‬ ‫∈‪ ( un )n‬و‬ ‫*‬ ‫ﺏﻴﻦ أن اﻟﻤﺘﺘﺎﻟﻴﺘﻴﻦ‬

‫‪f n ( x ) = x + e nx‬‬ ‫ﺏﻤﺎ ﻳﻠﻲ‪:‬‬ ‫‪II‬ﻥﻌﺘﺒﺮ اﻟﺪاﻟﺔ ‪ f n‬اﻟﻤﻌﺮﻓﺔ ﻋﻠﻰ‬

‫و ﻟﻴﻜﻦ ) ‪ ( Γ n‬اﻟﻤﻨﺤﻨﻰ اﻟﻤﻤﺜﻞ ﻟﺪاﻟﺔ ‪ f n‬ﻓﻲ ﻡﻌﻠﻢ ﻡﺘﻌﺎﻡﺪ ﻡﻤﻨﻈﻢ ) ‪. ( O; u ; v‬‬

‫‪ -1‬أدرس ﺕﻐﻴﺮات اﻟﺪاﻟﺔ ‪. f n‬‬ ‫‪0,50‬‬

‫‪-2‬اﺳﺘﻨﺘﺞ أن اﻟﻤﻌﺎدﻟﺔ ‪ f n ( x ) = 0‬ﺕﻘﺒﻞ ﺣﻼ وﺣﻴﺪا ‪. α n‬‬ ‫‪0,50‬‬

‫‪‬‬ ‫‪1‬‬
‫‪ -3‬أ( ﺏﻴﻦ أن ‪α1 ∈  − ln 2; − ‬‬ ‫‪0,25‬‬
‫‪‬‬ ‫‪2‬‬

‫ب( ﺏﻴﻦ أن ‪ x − α1‬و ‪ e x + α1‬ﻟﻬﻤﺎ ﻥﻔﺲ اﻹﺵﺎرة‪.‬‬ ‫‪0,50‬‬

‫‪1‬‬ ‫‪‬‬ ‫‪1‬‬


‫‪ϕ ( x) = ex −‬‬ ‫ﺏﻤﺎ ﻳﻠﻲ ‪x‬‬ ‫‪ -4‬أ( ﻟﺘﻜﻦ ‪ ϕ‬اﻟﺪاﻟﺔ اﻟﻤﻌﺮﻓﺔ ﻋﻠﻰ ‪ −∞; − 2 ‬‬
‫‪e‬‬
‫‪‬‬ ‫‪1‬‬
‫ﺏﻴﻦ أن ‪ ϕ‬ﺕﻨﺎﻗﺼﻴﺔ ﻋﻠﻰ ‪.  −∞; − ‬‬
‫‪‬‬ ‫‪2‬‬ ‫‪0,50‬‬
‫‪1‬‬ ‫‪0,50‬‬
‫≤ ‪e x + α1‬‬ ‫ب( اﺳﺘﻨﺘﺞ أن ‪x − α1‬‬
‫‪e‬‬
‫‪1‬‬
‫‪ β 0 = −‬و ﻟﻜﻞ ﻋﺪد ﺻﺤﻴﺢ ﻃﺒﻴﻌﻲ ‪β n+1 = e− βn : n‬‬ ‫‪ -5‬ﻥﻀﻊ‬
‫‪2‬‬
‫∈ ‪. ∀n‬‬ ‫‪β n+1 α−1‬‬ ‫‪a β≤n‬‬ ‫‪1‬‬ ‫أ( ﺏﻴﻦ أﻥﻪ ﻳﻮﺝﺪ ﻋﺪد ‪α‬‬
‫ﺣﻘﻴﻘﻲ ‪− a‬‬
‫ﺣﻴﺚ‬ ‫‪0 ,50‬‬

‫ب( ﺏﻴﻦ أن اﻟﻤﺘﺘﺎﻟﻴﺔ ) ‪ ( β n‬ﻡﺘﻘﺎرﺏﺔ وﺣﺪد ﻥﻬﺎﻳﺘﻬﺎ‪.‬‬ ‫‪0,50‬‬

‫‪http://mathkas.ici.ma‬‬

‫‪http://mathkas.9e.cc‬‬

You might also like